Test 2

Pataasin ang iyong marka sa homework at exams ngayon gamit ang Quizwiz!

The home care nurse visits a 34-yr-old woman receiving peritoneal dialysis. Which statement indicates a need for immediate follow-up by the nurse? "Drain time is faster if I rub my abdomen." "The fluid draining from the catheter is cloudy." "The drainage is bloody when I have my period." "I wash around the catheter with soap and water."

"The fluid draining from the catheter is cloudy."

After a transurethral prostatectomy a patient returns to his room with a triple-lumen indwelling catheter and continuous bladder irrigation. The irrigation is normal saline at 150 mL/hr. The nurse empties the drainage bag for a total of 2520 mL after an 8-hour period. How much of the total is urine output?

1320

The RN directs the LPN/LVN to remove a Foley catheter at 1300. The nurse would check if the patient has voided by: A) 1400. B) 1600 C) 1700. D) 2300.

1700.

The nurse is assessing a client in an outpatient clinic. Which assessment data are a risk factor for developing pheochromocytoma? 1. A history of skin cancer. 2. A history of high blood pressure. 3. A family history of adrenal tumors. 4. A family history of migraine headaches.

3. There is a high incidence of pheochromocytomas in family members with adrenal tumors,

The nurse is preparing to administer the following medications. Which medication should the nurse question administering? 1. The thyroid hormone to the client who does not have a T3, T4 level. 2. The regular insulin to the client with a blood glucose level of 210 mg/dL. 3. The loop diuretic to the client with a potassium level of 3.3 mEq/L. 4. The cardiac glycoside to the client who has a digoxin level of 1.4 mg/dL.

3. This potassium level is below normal, which is 3.5 to 5.5 mEq/L.

A patient complains of nausea. When administering a dose of metoclopramide (Reglan), the nurse should teach the patient to report which potential adverse effect? a) Tremors b) Constipation c) Double vision d) Numbness in fingers and toes

A Extrapyramidal side effects, including tremors and tardive dyskinesias, may occur as a result of metoclopramide (Reglan) administration. Constipation, double vision, and numbness in fingers and toes are not adverse effects of metoclopramide.

The patient is admitted to the hospital with a severe exacerbation of ulcerative colitis. What finding is most important for you to act on? A. Blood urea nitrogen (BUN): 50 mg/dL B. Hemoglobin (Hb): 12 g/dL C. White blood cells (WBC): 11,000/μL D. Sodium (Na+): 148 mEq/L

A Rationale Patients with severe ulcerative colitis frequently have bloody diarrhea. Dehydration is present as evidenced by the high BUN. This must be treated first before the mild anemia and mild inflammation are addressed. Hypernatremia is usually not treated until the sodium level is above 150 mEq/L, and it usually responds to fluid replacement. Reference: 1024

After neck surgery, a patient develops hypoparathyroidism. The nurse should plan to teach the patient about a. calcium supplementation to normalize serum calcium levels. b. including whole grains in the diet to prevent constipation. c. use of bisphosphonates to reduce bone demineralization. d. having a high fluid intake to decrease risk for nephrolithiasis.

A R: Oral calcium supplements are used to maintain the serum calcium in normal range and prevent the complications of hypocalcemia. Whole-grain foods decrease calcium absorption and will not be recommended. Bisphosphonates will lower serum calcium level further by preventing calcium from being reabsorbed from bone. Kidney stones are not a complication of hypoparathyroidism and low calcium levels.

Cardiac monitoring is initiated for a patient in diabetic ketoacidosis (DKA). The nurse recognizes that this measure is important to identify a. electrocardiographic (ECG) changes and dysrhythmias related to hypokalemia. b. fluid overload resulting from aggressive fluid replacement. c. the presence of hypovolemic shock related to osmotic diuresis. d. cardiovascular collapse resulting from the effects of hyperglycemia.

A Rationale: The hypokalemia associated with metabolic acidosis can lead to potentially fatal dysrhythmias such as ventricular tachycardia and ventricular fibrillation, which would be detected with ECG monitoring. Fluid overload, hypovolemia, and cardiovascular collapse are possible complications of DKA, but cardiac monitoring would not detect theses. Cognitive Level: Application Text Reference: p. 1281 Nursing Process: Assessment NCLEX: Physiological Integrity

Which patient has the most significant risk factors for CKD? A 50-yr-old white woman with hypertension A 61-yr-old Native American man with diabetes A 40-yr-old Hispanic woman with cardiovascular disease A 28-yr-old African American woman with a urinary tract infection

A 61-yr-old Native American man with diabetes

A male patient returned from the operating room 6 hours ago with a cast on his right arm. He has not yet voided. Which action would be the most beneficial in assisting the patient to void? A) Suggest he stand at the bedside B) Stay with the patient C) Give him the urinal to use in bed D) Tell him that, if he doesn't urinate, he will be catheterized

A) Suggest he stand at the bedside

When administered a dose of metoclopramide (Reglan), a patient complains of nausea. The nurse would teach the patient to report which of the following potential adverse effects? A) Tremors B) Constipation C) Double vision D) Numbness in the fingers and dose

A) Tremors (Extrapyramidal side effects, including tremors and dyskinesias, may occur as a result of metoclopramide (Reglan) administration.)

The patient with cirrhosis has an increased abdominal girth from ascites. The nurse should know that this fluid gathers in the abdomen for which reasons (select all that apply)? A) There is decreased colloid oncotic pressure from the liver's inability to synthesize albumin. B) Hyperaldosteronism related to damaged hepatocytes increases sodium and fluid retention. C) Portal hypertension pushes proteins from the blood vessels, causing leaking into the peritoneal cavity. D) Osmoreceptors in the hypothalamus stimulate thirst, which causes the stimulation to take in fluids orally. E) Overactivity of the enlarged spleen results in increased removal of blood cells from the circulation, which decreases the vascular pressure.

A,B.C) The ascites related to cirrhosis are caused by decreased colloid oncotic pressure from the lack of albumin from liver inability to synthesize it and the portal hypertension that shifts the protein from the blood vessels to the peritoneal cavity, and hyperaldosteronism which increases sodium and fluid retention. The intake of fluids orally and the removal of blood cells by the spleen do not directly contribute to ascites.

The patient with type 2 diabetes has a second UTI within one month of being treated for a previous UTI. Which medication should the nurse expect to teach the patient about taking for this infection? A. Ciprofloxacin (Cipro) B. Fosfomycin (Monurol) C. Nitrofurantoin (Macrodantin) D. Trimethoprim/sulfamethoxazole (Bactrim)

A. Ciprofloxacin (Cipro) This UTI is a complicated UTI because the patient has type 2 diabetes and the UTI is recurrent. Ciprofloxacin (Cipro) would be used for a complicated UTI. Fosfomycin (Monurol), nitrofurantoin (Macrodantin), and trimethoprim/sulfamethoxazole (Bactrim) should be used for uncomplicated UTIs.

A nurse is caring for a client for whom a tap water enema is prescribed, to be repeated until the return is clear. Which of the following actions should the nurse take? A. Clarify the order with the provider. B. Explain the procedure to the client. C. Ensure that the tap water is not too hot. D. Keep the amount per enema to less than 1,000 mL.

A. Clarify the order with the provider. Tap water is a hypotonic solution that can cause water toxicity. It should not be repeated. The nurse should clarify the order with the provider. Explaining the procedure to the client, ensuring that the tap water is not too hot, and keeping the amount to less than 1,000 mL are not pertinent if the enema should not be repeated. If you got this question wrong, I am judging you...just kidding :)

The nurse knows the patient with AKI has entered the diuretic phase when what assessments occur (select all that apply)? A. Dehydration B. Hypokalemia C. Hypernatremia D. BUN increases E. Serum creatinine increases

A. Dehydration B. Hypokalemia Dehydration, hypokalemia, and hyponatremia occur in the diuretic phase of AKI because the nephrons can excrete wastes but not concentrate urine. Therefore the serum BUN and serum creatinine levels also begin to decrease.

When a patient is hospitalized with acute adrenal insufficiency, which assessment finding by the nurse indicates that the prescribed therapies are effective? A. Increasing serum sodium levels B. Decreasing blood glucose levels C. Decreasing serum chloride levels D. Increasing serum potassium levels

A. Increasing serum sodium levels

The patient has a form of glomerular inflammation that is progressing rapidly. She is gaining weight, and the urine output is steadily declining. What is the priority nursing intervention? A. Monitor the patient's cardiac status. B. Teach the patient about hand washing. C. Obtain a serum specimen for electrolytes. D. Increase direct observation of the patient.

A. Monitor the patient's cardiac status The nurse's priority is to monitor the patient's cardiac status. With the rapidly progressing glomerulonephritis, renal function begins to fail and fluid, potassium, and hydrogen retention lead to hypervolemia, hyperkalemia, and metabolic acidosis. Excess fluid increases the workload of the heart, and hyperkalemia can lead to life-threatening dysrhythmias. Teaching about hand washing and observation of the patient are important nursing interventions but are not the priority. Electrolyte measurement is a collaborative intervention that will be done as ordered by the health care provider.

In addition to urine function, the nurse recognizes that the kidneys perform numerous other functions important to the maintenance of homeostasis. Which physiologic processes are performed by the kidneys (select all that apply)? A. Production of renin B. Activation of vitamin D C. Carbohydrate metabolism D. Erythropoietin production E. Hemolysis of old red blood cells (RBCs)

A. Production of renin B. Activation of vitamin D D. Erythropoietin production In addition to urine formation, the kidneys release renin to maintain blood pressure, activate vitamin D to maintain calcium levels, and produce erythropoietin to stimulate RBC production. Carbohydrate metabolism and hemolysis of old RBCs are not physiologic functions that are performed by the kidneys.

The nurse is caring for a 56-year-old man receiving high-dose oral corticosteroid therapy to prevent organ rejection after a kidney transplant. What is most important for the nurse to observe related to this medication? A. Signs of infection B. Low blood pressure C. Increased urine output D. Decreased blood glucose

A. Signs of infection Side effects of corticosteroid therapy include increased susceptibility to infection, edema related to sodium and water retention (decreased urine output), hypertension, and hyperglycemia.

The patient had surgery and a urinary catheter. Eight hours after catheter removal and drinking fluids, the patient has not been able to void. What should the nurse do first to assess for urinary retention? A. Bladder scan B.Cystometrogram C. Residual urine test D. Kidneys, ureters, bladder (KUB) x-ray

A.Bladder scan If the patient is unable to void, the bladder may be palpated for distention, percussed for dullness if it is full, or a bladder scan may be done to determine the approximate amount of urine in the bladder. A cystometrogram visualizes the bladder and evaluates vesicoureteral reflux. A KUB x-ray delineates size, shape, and positions of kidneys and possibly a full bladder. Neither of these would be useful in this situation. A residual urine test requires urination before catheterizing the patient to determine the amount of urine left in the bladder, so this assessment would not be helpful for this patient.

Vasopressin (Pitressin) 0.2 units/min infusion is prescribed for a patient with acute arterial gastrointestinal (GI) bleeding. The vasopressin label states vasopressin 20 units/50 mL normal saline. How many mL/min will the nurse infuse? ____________________

ANS: 0.5 There are 0.4 units/1 mL. An infusion of 0.5 mL/min will result in the patient receiving 0.2 mL/min as prescribed.

When teaching a patient about testing to diagnose metabolic syndrome, which topic would the nurse include? a. Blood glucose test b. Cardiac enzyme tests c. Postural blood pressures d. Resting electrocardiogram

ANS: A A fasting blood glucose test >100 mg/dL is one of the diagnostic criteria for metabolic syndrome. The other tests are not used to diagnose metabolic syndrome although they may be used to check for cardiovascular complications of the disorder

Which nursing action is of highest priority for a 68-year-old patient with renal calculi who is being admitted to the hospital with gross hematuria and severe colicky left flank pain? a. Administer prescribed analgesics. b. Monitor temperature every 4 hours. c. Encourage increased oral fluid intake. d. Give antiemetics as needed for nausea.

ANS: A Although all of the nursing actions may be used for patients with renal lithiasis, the patient's presentation indicates that management of pain is the highest priority action. If the patient has urinary obstruction, increasing oral fluids may increase the symptoms. There is no evidence of infection or nausea.

A 70-year-old patient who has had a transurethral resection of the prostate (TURP) for benign prostatic hyperplasia (BPH) is being discharged from the hospital today, The nurse determines that additional instruction is needed when the patient says which of the following? a. "I should call the doctor if I have incontinence at home." b. "I will avoid driving until I get approval from my doctor." c. "I will increase fiber and fluids in my diet to prevent constipation." d. "I should continue to schedule yearly appointments for prostate exams."

ANS: A Because incontinence is common for several weeks after a TURP, the patient does not need to call the health care provider if this occurs. The other patient statements indicate that the patient has a good understanding of post-TURP instructions.

A patient with urinary obstruction from benign prostatic hyperplasia (BPH) tells the nurse, "My symptoms are much worse this week." Which response by the nurse is most appropriate? a. "Have you been taking any over-the-counter (OTC) medications recently?" b. "I will talk to the doctor about ordering a prostate specific antigen (PSA) test." c. "Have you talked to the doctor about surgery such as transurethral resection of the prostate (TURP)?" d. "The prostate gland changes in size from day to day, and this may be making your symptoms worse."

ANS: A Because the patient's increase in symptoms has occurred abruptly, the nurse should ask about OTC medications that might cause contraction of the smooth muscle in the prostate and worsen obstruction. The prostate gland does not vary in size from day to day. A TURP may be needed, but more assessment about possible reasons for the sudden symptom change is a more appropriate first response by the nurse. PSA testing is done to differentiate BPH from prostatic cancer.

Which finding indicates to the nurse that the current therapies are effective for a patient with acute adrenal insufficiency? a. Increasing serum sodium levels b. Decreasing blood glucose levels c. Decreasing serum chloride levels d. Increasing serum potassium levels

ANS: A Clinical manifestations of Addison's disease include hyponatremia and an increase in sodium level indicates improvement. The other values indicate that treatment has not been effective.

A 56-year-old female patient has an adrenocortical adenoma, causing hyperaldosteronism. The nurse providing care should a. monitor the blood pressure every 4 hours. b. elevate the patient's legs to relieve edema. c. monitor blood glucose level every 4 hours. d. order the patient a potassium-restricted diet.

ANS: A Hypertension caused by sodium retention is a common complication of hyperaldosteronism. Hyperaldosteronism does not cause an elevation in blood glucose. The patient will be hypokalemic and require potassium supplementation before surgery. Edema does not usually occur with hyperaldosteronism.

A 51-year-old woman with Crohn's disease who is taking infliximab (Remicade) calls the nurse in the outpatient clinic about new symptoms. Which symptom is most important to communicate to the health care provider? a. Fever b. Nausea c. Joint pain d. Headache

ANS: A Since infliximab suppresses the immune response, rapid treatment of infection is essential. The other patient complaints are common side effects of the medication, but they do not indicate any potentially life-threatening complications

The nurse will plan to monitor a patient with an obstructed common bile duct for a. melena. b. steatorrhea. c. decreased serum cholesterol levels. d. increased serum indirect bilirubin levels.

ANS: B A common bile duct obstruction will reduce the absorption of fat in the small intestine, leading to fatty stools. Gastrointestinal (GI) bleeding is not caused by common bile duct obstruction. Serum cholesterol levels are increased with biliary obstruction. Direct bilirubin level is increased with biliary obstruction

The nurse is caring for a 45-year-old male patient during a water deprivation test. Which finding is most important for the nurse to communicate to the health care provider? a. The patient complains of intense thirst. b. The patient has a 5-lb (2.3 kg) weight loss. c. The patient's urine osmolality does not increase. d. The patient feels dizzy when sitting on the edge of the bed.

ANS: B A drop in the weight of more than 2 kg indicates severe dehydration, and the test should be discontinued. The other assessment data are not unusual with this test.

A 38-year-old patient who had a kidney transplant 8 years ago is receiving the immunosuppressants tacrolimus (Prograf), cyclosporine (Sandimmune), and prednisone (Deltasone). Which assessment data will be of most concern to the nurse? a. The blood glucose is 144 mg/dL. b. There is a nontender axillary lump. c. The patient's skin is thin and fragile. d. The patient's blood pressure is 150/92.

ANS: B A nontender lump suggests a malignancy such as a lymphoma, which could occur as a result of chronic immunosuppressive therapy. The elevated glucose, skin change, and hypertension are possible side effects of the prednisone and should be addressed, but they are not as great a concern as the possibility of a malignancy.

A patient with renal calculi is hospitalized with gross hematuria and severe colicky left flank pain. Which nursing action will be of highest priority at this time? a. Encourage oral fluid intake. b. Administer prescribed analgesics. c. Monitor temperature every 4 hours. d. Give antiemetics as needed for nausea.

ANS: B Although all of the nursing actions may be used for patients with renal lithiasis, the patient's presentation indicates that management of pain is the highest priority action. If the patient has urinary obstruction, increasing oral fluids may increase the symptoms. There is no evidence of infection or nausea. DIF: Cognitive Level: Application REF: 1137-1138 | 1139-1141 | 1140

The nurse is admitting a 67-year-old patient with new-onset steatorrhea. Which question is most important for the nurse to ask? a. "How much milk do you usually drink?" b. "Have you noticed a recent weight loss?" c. "What time of day do your bowels move?" d. "Do you eat meat or other animal products?"

ANS: B Although all of the questions provide useful information, it is most important to determine if the patient has an imbalance in nutrition because of the steatorrhea

A patient being admitted with an acute exacerbation of ulcerative colitis reports crampy abdominal pain and passing 15 or more bloody stools a day. The nurse will plan to a. administer IV metoclopramide (Reglan). b. discontinue the patient's oral food intake. c. administer cobalamin (vitamin B12) injections. d. teach the patient about total colectomy surgery.

ANS: B An initial therapy for an acute exacerbation of inflammatory bowel disease (IBD) is to rest the bowel by making the patient NPO. Metoclopramide increases peristalsis and will worsen symptoms. Cobalamin (vitamin B12) is absorbed in the ileum, which is not affected by ulcerative colitis. Although total colectomy is needed for some patients, there is no indication that this patient is a candidate

Which nursing action will the nurse include in the plan of care for a 35-year-old male patient admitted with an exacerbation of inflammatory bowel disease (IBD)? a. Restrict oral fluid intake. b. Monitor stools for blood. c. Ambulate four times daily. d. Increase dietary fiber intake.

ANS: B Because anemia or hemorrhage may occur with IBD, stools should be assessed for the presence of blood. The other actions would not be appropriate for the patient with IBD. Because dietary fiber may increase gastrointestinal (GI) motility and exacerbate the diarrhea, severe fatigue is common with IBD exacerbations, and dehydration may occur.

A 42-year-old patient admitted with acute kidney injury due to dehydration has oliguria, anemia, and hyperkalemia. Which prescribed actions should the nurse take first? a. Insert a urinary retention catheter. b. Place the patient on a cardiac monitor. c. Administer epoetin alfa (Epogen, Procrit). d. Give sodium polystyrene sulfonate (Kayexalate).

ANS: B Because hyperkalemia can cause fatal cardiac dysrhythmias, the initial action should be to monitor the cardiac rhythm. Kayexalate and Epogen will take time to correct the hyperkalemia and anemia. The catheter allows monitoring of the urine output but does not correct the cause of the renal failure.

The nurse will plan to teach the patient diagnosed with acute hepatitis B about a. side effects of nucleotide analogs. b. measures for improving the appetite. c. ways to increase activity and exercise. d. administering a-interferon (Intron A).

ANS: B Maintaining adequate nutritional intake is important for regeneration of hepatocytes. Interferon and antivirals may be used for chronic hepatitis B, but they are not prescribed for acute hepatitis B infection. Rest is recommended

Which question will the nurse in the endocrine clinic ask to help determine a patient's risk factors for goiter? a. "How much milk do you drink?" b. "What medications are you taking?" c. "Are your immunizations up to date?" d. "Have you had any recent neck injuries?"

ANS: B Medications that contain thyroid-inhibiting substances can cause goiter. Milk intake, neck injury, and immunization history are not risk factors for goiter.

To prevent recurrence of uric acid renal calculi, the nurse teaches the patient to avoid eating a. milk and cheese. b. sardines and liver. c. legumes and dried fruit. d. spinach, chocolate, and tea.

ANS: B Organ meats and fish such as sardines increase purine levels and uric acid. Spinach, chocolate, and tomatoes should be avoided in patients who have oxalate stones. Milk, dairy products, legumes, and dried fruits may increase the incidence of calcium-containing stones.

Which action will the nurse in the gastrointestinal clinic include in the plan of care? a. Obtain blood samples for DNA analysis. b. Schedule the patient for yearly colonoscopy. c. Provide preoperative teaching about total colectomy. d. Discuss lifestyle modifications to decrease cancer risk.

ANS: B Patients with FAP should have annual colonoscopy starting at age 16 and usually have total colectomy by age 25 to avoid developing colorectal cancer. DNA analysis is used to make the diagnosis, but is not needed now for this patient. Lifestyle modifications will not decrease cancer risk for this patient

For a patient with cirrhosis, which of the following nursing actions can the registered nurse (RN) delegate to unlicensed assistive personnel (UAP)? a. Assessing the patient for jaundice b. Providing oral hygiene after a meal c. Palpating the abdomen for distention d. Assisting the patient to choose the diet

ANS: B Providing oral hygiene is within the scope of UAP. Assessments and assisting patients to choose therapeutic diets are nursing actions that require higher-level nursing education and scope of practice and would be delegated to licensed practical/vocational nurses (LPNs/LVNs) or RNs

Which information will the nurse include when teaching a patient with peptic ulcer disease about the effect of ranitidine (Zantac)? a. "Ranitidine absorbs the gastric acid." b. "Ranitidine decreases gastric acid secretion." c. "Ranitidine constricts the blood vessels near the ulcer." d. "Ranitidine covers the ulcer with a protective material."

ANS: B Ranitidine is a histamine-2 (H2) receptor blocker, which decreases the secretion of gastric acid. The response beginning, "Ranitidine constricts the blood vessels" describes the effect of vasopressin. The response "Ranitidine absorbs the gastric acid" describes the effect of antacids. The response beginning "Ranitidine covers the ulcer" describes the action of sucralfate (Carafate).

A 58-year-old man with blunt abdominal trauma from a motor vehicle crash undergoes peritoneal lavage. If the lavage returns brown fecal drainage, which action will the nurse plan to take next? a. Auscultate the bowel sounds. b. Prepare the patient for surgery. c. Check the patient's oral temperature. d. Obtain information about the accident.

ANS: B Return of brown drainage and fecal material suggests perforation of the bowel and the need for immediate surgery. Auscultation of bowel sounds, checking the temperature, and obtaining information about the accident are appropriate actions, but the priority is to prepare to send the patient for emergency surgery

A 53-year-old patient is being treated for bleeding esophageal varices with balloon tamponade. Which nursing action will be included in the plan of care? a. Instruct the patient to cough every hour. b. Monitor the patient for shortness of breath. c. Verify the position of the balloon every 4 hours. d. Deflate the gastric balloon if the patient reports nausea.

ANS: B The most common complication of balloon tamponade is aspiration pneumonia. In addition, if the gastric balloon ruptures, the esophageal balloon may slip upward and occlude the airway. Coughing increases the pressure on the varices and increases the risk for bleeding. Balloon position is verified after insertion and does not require further verification. The esophageal balloon is deflated every 8 to 12 hours to avoid necrosis, but if the gastric balloon is deflated, the esophageal balloon may occlude the airway

A 71-year-old patient who has benign prostatic hyperplasia (BPH) with urinary retention is admitted to the hospital with elevated blood urea nitrogen (BUN) and creatinine. Which prescribed therapy should the nurse implement first? a. Infuse normal saline at 50 mL/hr. b. Insert a urinary retention catheter. c. Draw blood for a complete blood count. d. Schedule a pelvic computed tomography (CT) scan.

ANS: B The patient data indicate that the patient may have acute kidney injury caused by the BPH. The initial therapy will be to insert a catheter. The other actions are also appropriate, but they can be implemented after the acute urinary retention is resolved.

A 58-year-old patient has just been admitted to the emergency department with nausea and vomiting. Which information requires the most rapid intervention by the nurse? a. The patient has been vomiting for 4 days. b. The patient takes antacids 8 to 10 times a day. c. The patient is lethargic and difficult to arouse. d. The patient has undergone a small intestinal resection.

ANS: C A lethargic patient is at risk for aspiration, and the nurse will need to position the patient to decrease aspiration risk. The other information is also important to collect, but it does not require as quick action as the risk for aspiration.

A hospitalized patient with possible renal insufficiency after coronary artery bypass surgery is scheduled for a creatinine clearance test. Which equipment will the nurse need to obtain? a. Urinary catheter b. Cleaning towelettes c. Large container for urine d. Sterile urine specimen cup

ANS: C Because creatinine clearance testing involves a 24-hour urine specimen, the nurse should obtain a large container for the urine collection. Catheterization, cleaning of the perineum with antiseptic towelettes, and a sterile specimen cup are not needed for this test

Which question will provide the most useful information to a nurse who is interviewing a patient about a possible thyroid disorder? a. "What methods do you use to help cope with stress?" b. "Have you experienced any blurring or double vision?" c. "Have you had a recent unplanned weight gain or loss?" d. "Do you have to get up at night to empty your bladder?"

ANS: C Because thyroid function affects metabolic rate, changes in weight may indicate hyperfunction or hypofunction of the thyroid gland. Nocturia, visual difficulty, and changes in stress level are associated with other endocrine disorders.

A 38-year old woman receiving chemotherapy for breast cancer develops a Candida albicans oral infection. The nurse will anticipate the need for a. hydrogen peroxide rinses. b. the use of antiviral agents. c. administration of nystatin (Mycostatin) tablets. d. referral to a dentist for professional tooth cleaning.

ANS: C Candida albicans is treated with an antifungal such as nystatin. Oral saltwater rinses may be used but will not cure the infection. Antiviral agents are used for viral infections such as herpes simplex. Referral to a dentist is indicated for gingivitis but not for Candida infection

A 25-year-old woman who is scheduled for a routine gynecologic examination tells the nurse that she has had intercourse during the last year with several men. The nurse will plan to teach about the reason for a. contraceptive use. b. antibiotic therapy. c. Chlamydia testing. d. pregnancy testing.

ANS: C Chlamydia testing is recommended annually for women with multiple sex partners. There is no indication that the patient needs teaching about contraceptives, pregnancy testing, or antibiotic therapy

Which information from a patient who had a transurethral resection with fulguration for bladder cancer 3 days ago is most important to report to the health care provider? a. The patient is voiding every 4 hours. b. The patient is using opioids for pain. c. The patient has seen clots in the urine. d. The patient is anxious about the cancer.

ANS: C Clots in the urine are not expected and require further follow-up. Voiding every 4 hours, use of opioids for pain, and anxiety are typical after this procedure.

Which activity in the care of a 48-year-old female patient with a new colostomy could the nurse delegate to unlicensed assistive personnel (UAP)? a. Document the appearance of the stoma. b. Place a pouching system over the ostomy. c. Drain and measure the output from the ostomy. d. Check the skin around the stoma for breakdown.

ANS: C Draining and measuring the output from the ostomy is included in UAP education and scope of practice. The other actions should be implemented by LPNs or RNs.

A 46-year-old female with gastroesophageal reflux disease (GERD) is experiencing increasing discomfort. Which patient statement indicates that additional teaching about GERD is needed? a. "I take antacids between meals and at bedtime each night." b. "I sleep with the head of the bed elevated on 4-inch blocks." c. "I eat small meals during the day and have a bedtime snack." d. "I quit smoking several years ago, but I still chew a lot of gum."

ANS: C GERD is exacerbated by eating late at night, and the nurse should plan to teach the patient to avoid eating at bedtime. The other patient actions are appropriate to control symptoms of GERD

A 37-year-old female patient is hospitalized with acute kidney injury (AKI). Which information will be most useful to the nurse in evaluating improvement in kidney function? a. Urine volume b. Creatinine level c. Glomerular filtration rate (GFR) d. Blood urea nitrogen (BUN) level

ANS: C GFR is the preferred method for evaluating kidney function. BUN levels can fluctuate based on factors such as fluid volume status and protein intake. Urine output can be normal or high in patients with AKI and does not accurately reflect kidney function. Creatinine alone is not an accurate reflection of renal function.

A 55-year-old patient with end-stage kidney disease (ESKD) is scheduled to receive a prescribed dose of epoetin alfa (Procrit). Which information should the nurse report to the health care provider before giving the medication? a. Creatinine 1.6 mg/dL b. Oxygen saturation 89% c. Hemoglobin level 13 g/dL d. Blood pressure 98/56 mm Hg

ANS: C High hemoglobin levels are associated with a higher rate of thromboembolic events and increased risk of death from serious cardiovascular events (heart attack, heart failure, stroke) when erythropoietin (EPO) is administered to a target hemoglobin of >12 g/dL. Hemoglobin levels higher than 12 g/dL indicate a need for a decrease in epoetin alfa dose. The other information also will be reported to the health care provider but will not affect whether the medication is administered.

After change-of-shift report, which patient will the nurse assess first? a. 19-year-old with type 1 diabetes who was admitted with possible dawn phenomenon b. 35-year-old with type 1 diabetes whose most recent blood glucose reading was 230 mg/dL c. 60-year-old with hyperosmolar hyperglycemic syndrome who has poor skin turgor and dry oral mucosa d. 68-year-old with type 2 diabetes who has severe peripheral neuropathy and complains of burning foot pain

ANS: C The patient's diagnosis of HHS and signs of dehydration indicate that the nurse should rapidly assess for signs of shock and determine whether increased fluid infusion is needed. The other patients also need assessment and intervention but do not have life-threatening complications

A 42-year-old male patient has had a herniorrhaphy to repair an incarcerated inguinal hernia. Which patient teaching will the nurse provide before discharge? a. Soak in sitz baths several times each day. b. Cough 5 times each hour for the next 48 hours. c. Avoid use of acetaminophen (Tylenol) for pain. d. Apply a scrotal support and ice to reduce swelling.

ANS: D A scrotal support and ice are used to reduce edema and pain. Coughing will increase pressure on the incision. Sitz baths will not relieve pain and would not be of use after this surgery. Acetaminophen can be used for postoperative pain.

A patient who requires daily use of a nonsteroidal anti-inflammatory drug (NSAID) for management of severe rheumatoid arthritis has recently developed melena. The nurse will anticipate teaching the patient about a. substitution of acetaminophen (Tylenol) for the NSAID. b. use of enteric-coated NSAIDs to reduce gastric irritation. c. reasons for using corticosteroids to treat the rheumatoid arthritis. d. the benefits of misoprostol (Cytotec) in protecting the gastrointestinal (GI) mucosa.

ANS: D Misoprostol, a prostaglandin analog, reduces acid secretion and incidence of upper GI bleeding associated with NSAID use. Enteric coating of NSAIDs does not reduce the risk for GI bleeding. Corticosteroids increase the risk for ulcer development and will not be substituted for NSAIDs for this patient. Acetaminophen will not be effective in treating the patient's rheumatoid arthritis.

The client is scheduled to have a renogram (kidney scan). She is concerned about discomfort during the procedure. What is the nurse's best response? A. "Before the test you will be given a sedative to reduce any pain." B. "A local anesthetic agent will be used, so you might feel a little pressure but no pain." C. "Although this test is very sensitive, there is no more discomfort than you would have with an ordinary x-ray." D. "The only pain associated with this procedure is a small needle stick when you are given the radioisotope

ANS: D The test involves an intravenous injection of the radioisotope and the subsequent recording of the emission by a scintillator.

A 33-year-old male patient with a gunshot wound to the abdomen undergoes surgery, and a colostomy is formed as shown in the accompanying figure. Which information will be included in patient teaching? a. Stool will be expelled from both stomas. b. This type of colostomy is usually temporary. c. Soft, formed stool can be expected as drainage. d. Irrigations can regulate drainage from the stomas.

B

In caring for a patient with pelvic inflammatory disease the nurse should place her in the semi-fowlers position. The rationale for this measure is to: A. relieve severe pain B. promote drainage to prevent abscesses C. improve circulation and promote healing D. prevent complication of bowel obstruction

B

The nurse would increase the comfort of a patient with appendicitis by: A. having the patient lie prone B. flexing the patient's right knee C. sitting the patient upright in a chair D. turning the patient onto his left side

B

The patient's thick, white, and curdlike vaginal discharge and vulvar pruritus is most consistent with: A. trichomoniasis B. monilial vaginitis C. bacterial vaginosis D. chlamydial cervicitis

B

The results of a patient's recent endoscopy indicate the presence of peptic ulcer disease (PUD). Which teaching point should the nurse provide to the patient based on this new diagnosis? a) "You'll need to drink at least two to three glasses of milk daily." b) "It would likely be beneficial for you to eliminate drinking alcohol." c) "Many people find that a minced or pureed diet eases their symptoms of PUD." d) "Your medications should allow you to maintain your present diet while minimizing symptoms."

B Alcohol increases the amount of stomach acid produced so it should be avoided. Although there is no specific recommended dietary modification for PUD, most patients find it necessary to make some sort of dietary modifications to minimize symptoms. Milk may exacerbate PUD.

Which is the best category of food to encourage prevention of diverticulosis? A. High iron, such as organ meats B. Low gluten C. High fiber, such as raw vegetables D. No nuts or popcorn

C Rationale Foods high in fiber (bulk), such as fresh fruits and vegetables (along with decreased fat and red meat), help to prevent diverticulosis. Iron and gluten are not specific for diverticulosis. There is no evidence to support avoiding nuts and seeds to prevent diverticulitis, and they may have a protective effect. Reference: 1047

What is a nursing priority in the care of a patient with a diagnosis of hypothyroidism? A. Providing a dark, low-stimulation environment B. Closely monitoring the patient's intake and output C. Patient teaching related to levothyroxine (Synthroid) D. Patient teaching related to radioactive iodine therapy

C. Patient teaching related to levothyroxine (Synthroid) A euthyroid state is most often achieved in patients with hypothyroidism by the administration of levothyroxine (Synthroid). It is not necessary to carefully monitor intake and output, and low stimulation and radioactive iodine therapy are indicated in the treatment of hyperthyroidism.

A patient has been taking oral prednisone for the past several weeks after having a severe reaction to poison ivy. The nurse has explained the procedure for gradual reduction rather than sudden cessation of the drug. What is the rationale for this approach to drug administration? A. Prevention of hypothyroidism B. Prevention of diabetes insipid us C. Prevention of adrenal insufficiency D. Prevention of cardiovascular complications

C. Prevention of adrenal insufficiency Sudden cessation of corticosteroid therapy can precipitate life-threatening adrenal insufficiency. Diabetes insipidus, hypothyroidism, and cardiovascular complications are not common consequences of suddenly stopping corticosteroid therapy.

When providing postoperative care for a patient who had a bilateral adrenalectomy, which assessment information requires the most rapid action by the nurse? A. The blood glucose is 176 mg/dL. B. The lungs have bibasilar crackles. C. The patient's BP is 88/50 mm Hg. D. The patient has 5/10 incisional pain.

C. the pt's BP is 88/50

A patient with a 25-year history of type 1 diabetes mellitus is reporting fatigue, edema, and an irregular heartbeat. On assessment, the nurse notes newly developed hypertension and uncontrolled blood sugars. Which diagnostic study is most indicative of chronic kidney disease (CKD)? Serum creatinine Serum potassium Microalbuminuria Calculated glomerular filtration rate (GFR)

Calculated glomerular filtration rate (GFR)

The patient with suspected pancreatic cancer is having many diagnostic studies done. Which one can be used to establish the diagnosis of pancreatic adenocarcinoma and for monitoring the response to treatment? Spiral CT scan A PET/CT scan Abdominal ultrasound Cancer-associated antigen 19-9

Cancer-associated antigen 19-9 Correct The cancer-associated antigen 19-9 (CA 19-9) is the tumor marker used for the diagnosis of pancreatic adenocarcinoma and for monitoring the response to treatment. Although a spiral CT scan may be the initial study done and provides information on metastasis and vascular involvement, this test and the PET/CT scan or abdominal ultrasound do not provide additional information.

A 56-yr-old woman with type 2 diabetes mellitus and chronic kidney disease has a serum potassium level of 6.8 mEq/L. Which finding will the nurse monitor for? Fatigue Hypoglycemia Cardiac dysrhythmias Elevated triglycerides

Cardiac dysrhythmias

Since removal of the patient's Foley catheter, the patient has voided 50 to 100 mL every 2 to 3 hours. Which action should the nurse take first? A) Check for bladder distention B) Encourage fluid intake C) Obtain an order to recatheterize the patient D) Document the amount of each voiding for 24 hours

Check for bladder distention

The nurse notes that the patient's Foley catheter bag has been empty for 4 hours. The priority action would be to: A) Irrigate the Foley. B) Check for kinks in the tubing. C) Notify the health care provider. D) Assess the patient's intake.

Check for kinks in the tubing.

After administration of a dose of metoclopramide, which patient assessment finding would show the medication was effective? a. Decreased blood pressure b. Absence of muscle tremors c. Relief of nausea and vomiting d. No further episodes of diarrhea

Correct answer: C Metoclopramide is classified as a prokinetic and antiemetic medication. If it is effective, the patient's nausea and vomiting should resolve. Metoclopramide does not affect blood pressure, muscle tremors, or diarrhea.

The nurse teaches senior citizens at a community center how to prevent food poisoning at social events. Which community member statement reflects accurate understanding? a. "Pasteurized juices and milk are safe to drink." b. "Alfalfa sprouts are safe if rinsed before eating." c. "Fresh fruits do not need to be washed before eating." d. "Ground beef is safe to eat if cooked until it is brown."

Correct answer: a Drink only pasteurized milk, juice, or cider. Ground beef should be cooked thoroughly. Browned meat can still harbor live bacteria. Cook ground beef until a thermometer reads at least 160° F. If a thermometer is unavailable, decrease the risk of illness by cooking the ground beef until there is no pink color in the middle. Fruits and vegetables should be washed thoroughly, especially those that will not be cooked. Persons who are immunocompromised or older should avoid eating alfalfa sprouts until the safety of the sprouts can be ensured.

After a hypophysectomy for acromegaly, postoperative nursing care should focus on a. frequent monitoring of serum and urine osmolarity. b. parenteral administration of a GH-receptor antagonist. c. keeping the patient in a recumbent position at all times. d. patient teaching regarding the need for lifelong hormone therapy.

Correct answer: a Rationale: A possible postoperative complication after a hypophysectomy is transient diabetes insipidus (DI). It may occur because of the loss of antidiuretic hormone (ADH), which is stored in the posterior lobe of the pituitary gland, or because of cerebral edema related to manipulation of the pituitary gland during surgery. To assess for DI, urine output and serum and urine osmolarity should be monitored closely.

The nurse instructs an obese 22-year-old man with a sedentary job about the health benefits of an exercise program. The nurse evaluates that teaching is effective when the patient makes which statement? a. "The goal is to walk at least 10,000 steps every day of the week." b. "Weekend aerobics for 2 hours is better than exercising every day." c. "Aerobic exercise will increase my appetite and result in weight gain." d. "Exercise causes weight loss by decreasing my resting metabolic rate."

Correct answer: a Rationale: A realistic activity goal is to walk 10,000 steps a day. Increased activity does not promote an increase in appetite or lead to weight gain. Exercise should be done daily, preferably 30 minutes to an hour a day. Exercise increases metabolic rate.

To control the side effects of corticosteroid therapy, the nurse teaches the patient who is taking corticosteroids to a. increase calcium intake to 1500 mg/day. b. perform glucose monitoring for hypoglycemia. c. obtain immunizations due to high risk of infections. d. avoid abrupt position changes because of orthostatic hypotension.

Correct answer: a Rationale: Because patients often receive corticosteroid treatment for prolonged periods (more than 3 months), corticosteroid-induced osteoporosis is an important concern. Therapies to reduce the resorption of bone may include increased calcium intake, vitamin D supplementation, bisphosphonates (e.g., alendronate [Fosamax]), and institution of a low-impact exercise program.

A patient is jaundiced and her stools are clay colored (gray). This is most likely related to a. decreased bile flow into the intestine. b. increase production of urobilinogen. c. increased production of cholecystokinin. d. increased bile and bilirubin in the blood.

Correct answer: a Rationale: Bile is produced by the hepatocytes and is stored and concentrated in the gallbladder. When bile is released from the common bile duct, it enters the duodenum. In the intestines, bilirubin is reduced to stercobilinogen and urobilinogen by bacterial action. Stercobilinogen accounts for the brown color of stool. Stools may be clay-colored if bile is not released from the common bile duct into the duodenum. Jaundice may result if the bilirubin level in the blood is elevated.

What information should be included in the dietary teaching for the patient following a Roux-en-Y gastric bypass? a. Avoid sugary foods and limit fluids to prevent dumping syndrome. b. Gradually increase the amount of food ingested to preoperative levels. c. Maintain a long-term liquid diet to prevent damage to the surgical site. d. Consume foods high in complex carbohydrates, protein, and fiber to add bulk to contents.

Correct answer: a Rationale: Fluids and foods high in carbohydrates tend to promote diarrhea and symptoms of dumping syndrome in patients with gastric bypass surgery. The diet generally should be high in protein and low in carbohydrates, fat, and roughage and consists of six small feedings a day because of the small stomach size. Liquid diets are likely to be used longer for the patient with a gastroplasty.

The nurse teaches the diabetic patient who rides a bicycle to work every day to administer morning insulin into the a. thigh. b. buttock. c. arm. d. abdomen.

D Rationale: Patients should be taught not to administer insulin into a site that will be exercised because exercise will increase the rate of absorption. The thigh, buttock, and arm are all exercised by riding a bicycle. Cognitive Level: Application Text Reference: p. 1262 Nursing Process: Implementation NCLEX: Physiological Integrity

Which of these nursing actions in the plan of care for a patient who has diabetes insipidus will be most appropriate for the RN to delegate to an experienced LPN/LVN? A. Titrate the infusion of 5% dextrose in water. B. Teach patient how to use DDAVP nasal spray. C. Assess patient's hydration status every 8 hours. D. Administer subcutaneous desmopressin (DDAVP).

D. Administer subQ desmopressin (DDAVP)

The nurse is providing care for a patient who has been admitted to the hospital for the treatment of nephrotic syndrome. What are priority nursing assessments in the care of this patient? A. Assessment of pain and level of consciousness B. Assessment of serum calcium and phosphorus levels C. Blood pressure and assessment for orthostatic hypotension D. Daily weights and measurement of the patient's abdominal girth

D. Daily weights and measurement of the patient's abdominal girth Peripheral edema is characteristic of nephrotic syndrome, and a key nursing responsibility in the care of patients with the disease is close monitoring of abdominal girth, weights, and extremity size. Pain, level of consciousness, and orthostatic blood pressure are less important in the care of patients with nephrotic syndrome. Abnormal calcium and phosphorus levels are not commonly associated with the diagnosis of nephrotic syndrome.

The patient was diagnosed with prerenal AKI. The nurse should know that what is most likely the cause of the patient's diagnosis? A. IV tobramycin (Nebcin) B. Incompatible blood transfusion C. Poststreptococcal glomerulonephritis D. Dissecting abdominal aortic aneurysm

D. Dissecting abdominal aortic aneurysm A dissecting abdominal aortic aneurysm is a prerenal cause of AKI because it can decrease renal artery perfusion and therefore the glomerular filtrate rate. Aminoglycoside antibiotic administration, a hemolytic blood transfusion reaction, and poststretpcoccal glomerulonephritis are intrarenal causes of AKI.

The condition of a patient who has cirrhosis of the liver has deteriorated. Which diagnostic study would help determine if the patient has developed liver cancer? Serum α-fetoprotein level Ventilation/perfusion scan Hepatic structure ultrasound Abdominal girth measurement

Hepatic structure ultrasound Hepatic structure ultrasound, CT, and MRI are used to screen and diagnose liver cancer. Serum α-fetoprotein level may be elevated with liver cancer or other liver problems. Ventilation/perfusion scans do not diagnose liver cancer. Abdominal girth measurement would not differentiate between cirrhosis and liver cancer.

A patient with a Foley catheter carries the collection bag at waist level when ambulating. The nurse tells the patient that he or she is at risk for: (Select all that apply.) A) Infection. B) Retention. C) Stagnant urine. D) Reflux of urine.

Infection. Reflux of urine.

The patient is to have an intravenous pyelogram (IVP). Which of the following apply to this procedure? (Select all that apply.) A) Note any allergies. B) Monitor intake and output. C) Provide for perineal hygiene. D) Assess vital signs. E) Encourage fluids after the procedure.

Note any allergies. Encourage fluids after the procedure.

A patient with Graves' disease is admitted to the emergency department with thyroid storm. Which of these prescribed medications should the nurse administer first? A. Propranolol (Inderal) B. Propylthiouracil (PTU) C. Methimazole (Tapazole) D. Iodine (Lugol's solution)

Propranolol (Inderal) Feedback ANS: A -adrenergic blockers work rapidly to decrease the cardiovascular manifestations of thyroid storm. The other medications take days to weeks to have an impact on thyroid function.

The nurse is caring for a 68-yr-old man who had coronary artery bypass surgery 3 weeks ago. During the oliguric phase of acute kidney disease, which action would be appropriate to include in the plan of care? Provide foods high in potassium. Restrict fluids based on urine output. Monitor output from peritoneal dialysis. Offer high-protein snacks between meals.

Restrict fluids based on urine output.

Which assessment finding for a 24-year-old patient admitted with Graves' disease requires the most rapid intervention by the nurse? A. BP 166/100 mm Hg B. Bilateral exophthalmos C. Heart rate 136 beats/minute D. Temperature 104.8° F (40.4°

Temperature 104.8° F (40.4° Feedback ANS: D The patient's temperature indicates that the patient may have thyrotoxic crisis and that interventions to lower the temperature are needed immediately. The other findings also require intervention but do not indicate potentially life-threatening complications

The patient with a history of lung cancer and hepatitis C has developed liver failure and is considering liver transplantation. After the comprehensive evaluation, the nurse knows that which factor discovered may be a contraindication for liver transplantation? Has completed a college education Has been able to stop smoking cigarettes Has well-controlled type 1 diabetes mellitus The chest x-ray showed another lung cancer lesion.

The chest x-ray showed another lung cancer lesion. Correct Contraindications for liver transplant include severe extrahepatic disease, advanced hepatocellular carcinoma or other cancer, ongoing drug and/or alcohol abuse, and the inability to comprehend or comply with the rigorous post-transplant course.

When caring for a patient with a biliary obstruction, the nurse will anticipate administering which vitamin supplements (select all that apply)? Vitamin A Vitamin D Vitamin E Vitamin K Vitamin B

Vitamin A Correct Vitamin D Correct Vitamin E Correct Vitamin K Correct Biliary obstruction prevents bile from entering the small intestine and thus prevents the absorption of fat-soluble vitamins. Vitamins A, D, E, and K are all fat-soluble and thus would need to be supplemented in a patient with biliary obstruction.

What causes a pilonidal cyst? A. Infection of a congenital tract under the skin between the buttocks B. Fistula formed from large intestine C. Cellulitis most often caused by methicillin-resistant Staphylococcus (MRSA) D. Complication of exudates from hemorrhoids

a Rationale A pilonidal sinus is a small tract under the skin between the buttocks in the sacrococcygeal area, and it is thought to be congenital. It is lined with epithelium and hair, and with moisture collection and movement of the hair, it becomes infected, forming an abscess. Reference: 1054

The patient with Crohn's disease has had multiple intestinal resections. Which symptom indicates that short bowel syndrome has developed? A. Steatorrhea B. Constipation C. Hypercholesteremia D. Hypercalcemia

a Rationale The predominant manifestation is diarrhea or steatorrhea. Diarrhea, not constipation, is a concern because there is decreased intestinal surface to absorb fluid and nutrients. Decreased absorption of bile salts is the issue; increased cholesterol is not related to short bowel syndrome. The risk is deficiencies of cobalamin, zinc, and calcium. Reference: 1051

In planning care for a patient with metastatic liver cancer, the nurse should include interventions that a. focus primarily on symptomatic and comfort measures. b. reassure the patient that chemotherapy offers a good prognosis. c. promote the patient's confidence that surgical excision of the tumor will be successful. d. provide information necessary for the patient to make decisions regarding liver transplantation. (Lewis 1042)

a Rationale: Nursing intervention for a patient with liver cancer focuses on keeping the patient as comfortable as possible. The prognosis for patients with liver cancer is poor. The cancer grows rapidly, and death may occur within 4 to 7 months as a result of hepatic encephalopathy or massive blood loss from gastrointestinal (GI) bleeding.

what characterizes type 2 diabetes? select all that apply a. B cell exhaustion b. insulin resistance c. genetic predisposition d. altered production of adipokines e. inherited defect in insulin receptors f. inappropraite glucose production by the liver

a. B cell exhaustion b. insulin resistance c. genetic predisposition d. altered production of adipokines e. inherited defect in insulin receptors f. inappropraite glucose production by the liver

Antidiabetic drugs are designed to control signs and symptoms of diabetes mellitus. The nurse primarily expects a decrease in which? a. Blood glucose b. Fat metabolism c. Glycogen storage d. Protein mobilization

a. Blood glucose

A nurse who is teaching a client how to recognize symptoms of hypoglycemia should include which symptoms in the teaching? (Select all that apply.) a. Headache b. Nervousness c. Bradycardia d. Sweating e. Thirst f. Sweet breath odor

a. Headache b. Nervousness d. Sweating

Which urinary diversion is a continent diversion created by formation of an ileal pouch with a stoma for catheterization? a. Kock pouch c. Orthotopic neobladder b. Ileal conduit d. Cutaneous ureterostomy

a. The Kock pouch is a continent diversion created by formation of an ileal pouch with an external stoma requiring catheterization. Ileal conduit is the most common incontinent diversion using a stoma of resected ileum with implanted ureters. Orthotopic neobladder is a new bladder from a reshaped segment of intestine in the anatomic position of the bladder with urine discharged through the urethra. A cutaneous ureterostomy diverts the ureter from the bladder to the abdominal skin but there is frequent scarring and strictures of the ureters, so ileal conduits are used more often.

Nurses need to teach patients at risk for developing chronic kidney disease. Individuals considered to be at increased risk include (select all that apply) a. older African Americans. b. patients more than 60 years old. c. those with a history of pancreatitis. d. those with a history of hypertension. e. those with a history of type 2 diabetes.

a. older African Americans. b. patients more than 60 years old. d. those with a history of hypertension. e. those with a history of type 2 diabetes.

The nurse determines that the goals of dietary teaching have been met when the patient with celiac disease selects from the menu: a. scrambled eggs and sausage b. buckwheat pancakes with syrup c. oatmeal, skim milk, and orange juice d. yogurt, strawberries, and rye toast with butter

a. scrambled eggs and sausage Rationale: Celiac disease is treated with lifelong avoidance of dietary gluten. Wheat, barley, oats, and rye products must be avoided. Although pure oats do not contain gluten, oat products can become contaminated with wheat, rye, and barley during the milling process. Gluten is also found in some medications and in many food additives, preservatives, and stabilizers.

The nurse is developing a care plan for the client diagnosed with type 1 diabetes. The nurse identifies the problem "high risk for hyperglycemia related to noncompliance with the medication regimen." What statement is an appropriate short-term goal for the client a. the client will have a blood glucose level between 90 and 140 mg/dL b. the client will demonstrate appropriate insulin injection technique c. the nurse will monitor the clients blood glucose levels 4 time a day d. the client will maintain normal kidney function with 30 ml/hr urine output

a. the client will have a blood glucose level between 90 and 140 mg/dL

The nurse determines that the pt in acute adrenal insufficiency is responding favorably to trtmt when a. the patient appears alert and oriented b. the patient's urinary output has increased c. pulmonary edema is reduced as evidenced by clear lung sounds d. laboratory tests reveal serum elevations of K and glucose and a decrease in sodium

a. the patient appears alert and oriented (R- confusion, irritability, disorientation, or depressioni s often present in pt with Addison's dz, and a (+) response to therapy would be indicated by a return to alertness and orientation. Other indication of response to therapy would be a decreased urinary output, decreased serum potassium, and increased serum sodium and glucose. The pt with Addison's would be very dehydrated and volume-depleted and would not have pulmonary edema.)

medication used to treat ploynephritis

antispasmodics with anticholinergic properties MOA: reduce spasms and smooth muscle contractions by inhibiting the effects of acetylcholine, thereby increasing bladder capacity S.E: same as antispasmodics same nrsg considers as antispasmodics Rx: propantheline(Pro-Banthine); hyoscyamine(Lievsinex); tincture of belldonna

When considering the following causes of acute abdomen, the nurse should know that surgery would be indicated for (select all that apply)? a. pancreatitis b. acute ischemic bowel c. foreign-body perforation d. pelvic inflammatory disease e. ruptured ectopic pregnancy f. ruptured abdominal aneurysm

b, c, e, f. An immediate surgical consult is needed for acute ischemic bowel, foreign-body perforation, ruptured ectopic pregnancy, or ruptured abdominal aneurysm. A diagnostic laparoscopy may be done or a laparotomy may be done to repair a ruptured abdominal aneurysm or remove the appendix. Surgery is not needed for pancreatitis or pelvic inflammatory disease, as these can be diagnosed and treated without surgery.

In a patient with AKI, which laboratory urinalysis result indicates tubular damage? a. Hematuria b. Specific gravity fixed at 1.010 c. Urine sodium of 12 mEq/L (12 mmol/L) d. Osmolality of 1000 mOsm/kg (1000 mmol/kg)

b. A urine specific gravity that is consistently 1.010 and a urine osmolality of about 300 mOsm/kg is the same specific gravity and osmolality as plasma. This indicates that tubules are damaged and unable to concentrate urine. Hematuria is more common with postrenal damage. Tubular damage is associated with a high sodium concentration (greater than 40 mEq/L).

Analyze the following diagnostic findings for your patient with type 2 diabetes. Which result will need further assessment? a. BP 126/80 mm Hg b. A1C 9% c. FBG 130mg/dL (7.2 mmol/L) d. LDL cholesterol 100 mg/dL (2.6 mmol/L)

b. A1C 9%

A patient with AKI has a serum potassium level of 6.7 mEq/L (6.7 mmol/L) and the following arterial blood gas results: pH 7.28, PaCO2 30 mm Hg, PaO2 86 mm Hg, HCO3 − 18 mEq/L (18 mmol/L). The nurse recognizes that treatment of the acid-base problem with sodium bicarbonate would cause a decrease in which value? a. pH b. Potassium level c. Bicarbonate level d. Carbon dioxide level

b. During acidosis, potassium moves out of the cell in exchange for H+ ions, increasing the serum potassium level. Correction of the acidosis with sodium bicarbonate will help to shift the potassium back into the cells. A decrease in pH and the bicarbonate and PaCO2 levels would indicate worsening acidosis.

During the oliguric phase of AKI, the nurse monitors the patient for (select all that apply) a. hypotension. b. ECG changes. c. hypernatremia. d. pulmonary edema. e. urine with high specific gravity.

b. ECG changes. d. pulmonary edema.

The patient with CKD asks why she is receiving nifedipine (Procardia) and furosemide (Lasix). The nurse understands that these drugs are being used to treat the patient's a. anemia. b. hypertension. c. hyperkalemia. d. mineral and bone disorder.

b. Nifedipine (Procardia) is a calcium channel blocker and furosemide (Lasix) is a loop diuretic. Both are used to treat hypertension.

The following interventions are planned for a diabetic patient. Which intervention can the nurse delegate to nursing assistive personnel (NAP)? a. Discuss complications of diabetes. b. Check that the bath water is not too hot. c. Check the patient's technique for drawing up insulin. d. Teach the patient to use the glucometer for in-home glucose monitoring.

b. Rationale: Checking the temperature of the bath water is part of assisting with activities of daily living (ADLs) and within the scope of care for the nursing assistive personnel (NAP). Discussion of complications, teaching, and assessing learning are appropriate for RNs.

Which infection is asymptomatic in the male patient at first and then progresses to cystitis, frequent urination, burning on voiding, and epididymitis? a. Urosepsis c. Urethral diverticula b. Renal tuberculosis d. Goodpasture syndrome

b. The manifestations of renal tuberculosis are described. Urosepsis is when the UTI has spread systemically. Urethral diverticula are localized outpouching of the urethra and occur more often in women. Goodpasture syndrome manifests with flu-like symptoms with pulmonary symptoms that include cough, shortness of breath, and pulmonary insufficiency and renal manifestations that include hematuria, weakness, pallor, anemia, and renal failure.

with which diagnosis will the patient benefit from being taught to do self-catheterization? a. renal trauma b. urethral stricture c. renal artery stenosis d. accelerated nephrosclerosis

b. The patient with urethral stricture will benefit from being taught to dilate the urethra by self-catheterization every few days. Renal trauma is treated related to the severity of the injury with bed rest, fluids, and analgesia. Renal artery stenosis includes control of hypertension with possible surgical revascularization. Accelerated nephrosclerosis is associated with malignant hypertension that must be aggressively treated as well as monitoring kidney function.

A 18 year old female client, 5'4" tall, weighting 113 kg, comes to the clinic for a nonhealing wound on her lower leg, which she has had for two weeks. Which disease process should the nurse suspect the client has developed? a. Type 1 diabetes b. Type 2 diabetes c. gestational diabetes d. acanthosis nigricans

b. Type 2 diabetes

The male patient is Jewish, has a history of gout, and has been diagnosed with renal calculi. Which treatment will be used with this patient (select all that apply?) a. reduce dietary oxalate b. administer allopurinol c. administer x-penicillamin d. administer thiazide diuretics e. reduce animal protein intake f. reduce intake in milk products

b. administer allopurinol e. reduce animal protein intake

The nurse at a freestanding health care clinic is caring for a 56 year old male client who is homeless and is a type 2 diabetic controlled with insulin. Which action is an example of client advocacy? a. ask the client if he has somewhere he can go and live b. arrange for someone to give him insulin at a local homeless shelter c. notify adult protective services about the clients situation d. ask the HCP to take the client off insulin because he is homeless

b. arrange for someone to give him insulin at a local homeless shelter

Which class of oral glucose-lowering agents is most commonly used for people with type 2 diabetes because is reduces hepatic glucose production and enhances tissue uptake of glucose? a. insulin b. biguanide c. meglitinide d. sulfonylurea

b. biguanide

what disorders and diseases are related to macrovascular complications of diabetes? select all that apply a. chronic kidney disease b. coronary artery disease c. microaneurysms and destruction or retinal vessels d. ulceration and amputation of the lower extremities e. capillary and arteriole membrane thickening specific to diabetes

b. coronary artery disease d. ulceration and amputation of the lower extremities

The immunologic mechanisms involved in acute poststreptococal glomerulonephritis include:* a. tubular blocking by precipitates of bacteria and antibody reactions b. deposition of immune complexes and complement along the GBM c. thickening of the GBM from autoimmune microangiopathic changes d. destruction of glomeruli by proteolytic enzymes contained in the GBM

b. deposition of immune complexes and complement along the GBM

The most important nursing intervention during the medical and surgical treatment of the patient with a pheochromocytoma is a. administering IV fluids b. monitoring blood pressure c. monitoring I&O and daily weights d. administering B-adrenergic blocking agents

b. monitoring blood pressure (R- a pheochromocytoma is a catecholamine-producing tumor of the adrenal medulla, which may cause severe, episodic HTN; severe, pounding headache; and profuse sweating. Monitoring for dangerously high BP before surgery is critical, as is monitoring for BP fluctuation during medical and surgical tx.)

What should the goals of nutrition therapy for the patient with type two diabetes include? a. ideal body weight b. normal serum glucose and lipid levels c. a special diabetic diet using dietetic foods d. five small meals per day with a bedtime snack

b. normal serum glucose and lipid levels

In contrast to diverticulitis, the patient with diverticulosis: a. has rectal bleeding b. often has no symptoms c. has localized cramping pain d. frequently develops peritonitis

b. often has no symptoms Rationale: Many people with diverticulosis have no symptoms. Patients with diverticulitis have symptoms of inflammation. Diverticulitis can lead to obstruction or perforation.

When caring for a patient with nephrogenic diabetes insipidus, the nurse would expect treatment to include a. fluid restriction b. thiazide diuretics c. a high sodium diet d. chlorprpamide (Diabinese)

b. thiazide diuretics Rationale- in nephrogenic Di the kidney is unable to respond to ADH, so vasopressin or hormone analogs are not effective.

which treatment for BPH uses low-wave radiofreqency to precisely destroy prostate tissue? a. laster prostatectomy b. transurethral needle ablation (TUNA) c. tranurethral microwave thermotherapy (TUMT) d. Transurethral electrovaporization of prostate (TUVP)

b. transurethral needle ablation (TUNA)

The patient with chronic kidney disease is considering whether to use peritoneal dialysis (PD) or hemodialysis (HD). What are advantages of PD when compared to HD (select all that apply)? a. Less protein loss b. Rapid fluid removal c. Less cardiovascular stress d. Decreased hyperlipidemia e. Requires fewer dietary restrictions

c, e. Peritoneal dialysis is less stressful for the cardiovascular system and requires fewer dietary restrictions. Peritoneal dialysis actually contributes to more protein loss and increased hyperlipidemia. The fluid and creatinine removal are slower with peritoneal dialysis than hemodialysis.

A client is receiving a daily dose of Humulin N insulin at 7:30 am. The nurse expects the peak effect of this drug to occur at which time? a. 8:15 am b. 10:30 am c. 5:00 pm d. 11:00 pm

c. 5:00 pm

Which type of urinary tract calculi are the most common and frequently obstruct the ureter? a. Cystine c. Calcium oxalate b. Uric acid d. Calcium phosphate

c. Calcium oxalate calculi are most common and small enough to get trapped in the ureter.

What should the nurse teach the patient with diverticulosis to do? a. Use anticholinergic drugs routinely to prevent bowel spasm. b. Have an annual colonoscopy to detect malignant changes in the lesions. c. Maintain a high-fiber diet and use bulk laxatives to increase fecal volume. d. Exclude whole grain breads and cereals from the diet to prevent irritating the bowel.

c. Formation of diverticula is common when decreased bulk of stool, combined with a more narrowed lumen in the sigmoid colon, causes high intraluminal pressures that result in saccular dilation or outpouching of the mucosa through the muscle of the intestinal wall. To prevent the high intraluminal pressure, fecal volume should be increased with use of high-fiber diets and bulk laxatives, such as psyllium (Metamucil). Anticholinergic drugs are used only during an acute episode of diverticulitis and the lesions are not premalignant.

A 60-year-old African American patient is afraid she might have anal cancer. What assessment finding puts her at high risk for anal cancer? a. Alcohol use c. Human papillomavirus (HPV) b. Only one sexual partner d. Use of a condom with sexual intercourse

c. Human papillomavirus (HPV) is associated with about 80% of anal cancer cases. Other risk factors include multiple sexual partners, smoking, receptive anal sex, and HIV infection, as well as being female, age 60, and African American. The other options are not considered risk factors for anal cancer.

If a patient is in the diuretic phase of AKI, the nurse must monitor for which serum electrolyte imbalances?

c. Hypokalemia and hyponatremia

During the nursing assessment of the patient with renal insufficiency, the nurse asks the patient specifically about a history of a. angina. b. asthma. c. hypertension. d. rheumatoid arthritis.

c. The most common causes of CKD in the United States are diabetes mellitus and hypertension. The nurse should obtain information on long-term health problems that are related to kidney disease. The other disorders are not closely associated with renal disease.

A patient who is on corticosteroid therapy for treatment of an autoimmune disorder has the following additional drugs ordered. Which on is used to prevent corticosteroid-induced osteoporosis a. potassium b. fursemide (Lasix) c. alendronate (Foxamax) d. pantoprazole (Protonix)

c. alendronate (Foxamax)

the nurse is admitting a client diagnosed with primary adrenal cortex insuffeciency (Addisons disease). Which clinical manifestations should the nurse expect to assess? a. moon face, buffalo hump, and hyperglycemia b. hirsutism, fever, and irritability c. bronze pigmentation, hypotension, and anorexia d. tachycardia, bulging eyes, and goiter

c. bronze pigmentation, hypotension, and anorexia

A patient is admitted to the hospital with thyrotoxicosis. On physical assessment of the patient, what should the nurse expect to find? a. hoarsness and laryngeal stridor b. bulging eyeball and dysrhythmias c. elevated temp and signs of HF d. lethargy progressing suddenly to impairment of consciouness

c. elevated temp and signs of HF

To assess the patency of a newly placed arteriovenous graft for dialysis, the nurse should (select all that apply)

c. palpate the area of the graft to feel a normal thrill. d. listen with a stethoscope over the graft to detect a bruit. e. frequently monitor the pulses and neurovascular status distal to the graft.

You are answering a patient's questions about celiac or gluten-sensitive enteropathy disease. Which option is the correct information to provide? A. Celiac is also known as tropical sprue. B. Celiac is only seen in children. C. Its symptoms mimic inflammatory bowel disease (IBD). D. It is an autoimmune disease.

d Rationale Celiac is an autoimmune disease in people who have a genetic predisposition, consume gluten, and an immune-mediated response. Celiac disease is different from tropical sprue, which is a chronic disorder acquired in tropical areas and treated with folic acid and tetracycline. Celiac disease is a relatively common disease that occurs in all ages. The symptoms mimic irritable bowel syndrome (IBS). Reference: 1049

When replacement therapy is started for a patient with long-standing hypothyroidism, what is most important for the nurse to monitor the patient for? a. Insomnia b. Weight loss c. Nervousness d. Dysrhythmias

d. All these manifestations may occur with treatment of hypothyroidism. However, as a result of the effects of hypothyroidism on the cardiovascular system, when thyroid replacement therapy is started myocardial oxygen consumption is increased and the resultant oxygen demand may cause angina, cardiac dysrhythmias, and heart failure, so monitoring for dysrhythmias is most important.

Which serum laboratory value indicates to the nurse that the patient's CKD is getting worse? a. Decreased BUN b. Decreased sodium c. Decreased creatinine d. Decreased calculated glomerular filtration rate (GFR)

d. As GFR decreases, BUN and serum creatinine levels increase. Although elevated BUN and creatinine indicate that waste products are accumulating, the calculated GFR is considered a more accurate indicator of kidney function than BUN or serum creatinine.

Following the teaching of foot care to a diabetic patient, the nurse determines that additional instruction is needed when the patient says, a. "I should wash my feet daily with soap and warm water." b. "I should always wear shoes to protect my feet from injury." c. "If my feet are cold, I should wear socks instead of using a heating pad." d. "I'll know if I have sores or lesions on my feet because they will be painful."

d. Rationale: Complete or partial loss of sensitivity of the feet is common with peripheral neuropathy of diabetes, and diabetics may suffer foot injury and ulceration without ever having pain. Feet must be inspected during daily care for any cuts, blisters, swelling, or reddened areas.

What is the only type of insulin that may be administered IV? a. NPH b. Detemir c. Lantus d. Regular

d. Regular

In instituting a bowel training program for a patient with fecal incontinence, what should the nurse first plan to do? a. Teach the patient to use a perianal pouch. b. Insert a rectal suppository at the same time every morning. c. Place the patient on a bedpan 30 minutes before breakfast. d. Assist the patient to the bathroom at the time of the patient's normal defecation.

d. The first intervention to establish bowel regularity includes promoting bowel evacuation at a regular time each day, preferably by placing the patient on the bedpan, using a bedside commode, or walking the patient to the bathroom. To take advantage of the gastrocolic reflex, an appropriate time is 30 minutes after the first meal of the day or at the patient's usual individual time. Perianal pouches are used to protect the skin only when regularity cannot be established and evacuation suppositories are also used only if other techniques are not successful.

When comparing the pathophysiology of type 1 and type 2 diabetes, which statement would be correct for a patient with type 2 diabetes who was admitted to the hospital with pneumonia? a. The patient must receive insulin therapy to prevent the development of ketoacidosis. b. The patient has islet cell antibodies that have destroyed the ability of the pancreas to produce insulin. c. The patient has minimal or absent endogenous insulin secretion and requires daily insulin injections. d. The patient may have sufficient endogenous insulin secretion to prevent ketosis but is at risk for development of hyperosmolar hyperglycemic syndrome.

d. The patient may have sufficient endogenous insulin secretion to prevent ketosis but is at risk for development of hyperosmolar hyperglycemic syndrome.

A patient who recently had a calcium oxalate renal stone had a bone density study, which showed a decrease in her bone density. What endocrine problem could this patient have? a. SIADH b. Hypothyroidism c. Cushing syndrome d. Hyperparathyroidism

d. The patient with hyperparathyroidism may have calcium nephrolithiasis, skeletal pain, decreased bone density, psychomotor retardation, or cardiac dysrhythmias. The other endocrine problems would not be related to calcium kidney stones or decreased bone density.

When a patient returns to the clinical unit after an abdominal-perineal resection (APR), what should the nurse expect? a. An abdominal dressing c. A temporary colostomy and drains b. An abdominal wound and drains d. A perineal wound, drains, and a stoma

d. With an abdominal perineal-resection (APR), an abdominal incision is made and the proximal sigmoid colon is brought through the abdominal wall and formed into a permanent colostomy. The patient is repositioned, a perineal incision is made, and the distal sigmoid colon, rectum, and anus are removed through the perineal incision, which may be left open, packed, and have drains.

RIFLE defines three stages of AKI based on changes in a. blood pressure and urine osmolality. b. fractional excretion of urinary sodium. c. estimation of GFR with the MDRD equation. d. serum creatinine or urine output from baseline.

d. serum creatinine or urine output from baseline.

The nurse will inform a patient with cancer of the prostate that side effects of leuprolide (Lupron) may include a. flushing. b. dizziness. c. infection. d. incontinence.

ANS: A Hot flashes may occur with decreased testosterone production. Dizziness may occur with the alpha-blockers used for benign prostatic hyperplasia (BPH). Urinary incontinence may occur after prostate surgery, but it is not an expected side effect of medication. Risk for infection is increased in patients receiving chemotherapy.

A 90-year-old healthy man is suffering from dysphagia. The nurse explains what age-related change of the GI tract is the most likely cause of his difficulty? a. Xerostomia b. Esophageal cancer c. Decreased taste buds d. Thinner abdominal wall

Correct answer: a Rationale: Xerostomia, decreased volume of saliva, leads to dry oral mucosa and dysphagia. Esophageal cancer is not an age-related change. Decreased taste buds and a thinner abdominal wall do not contribute to difficulty swallowing.

The elderly male patient reports his stool is very narrow and thin like a pencil. What action should you take? A. Encourage more bulk in the diet. B. Stress the importance of adequate fluids. C. Assess for laxative abuse. D. Initiate evaluation for colorectal cancer.

D Rationale Left-sided colon cancerous lesions can change the stool caliber. This is the priority action. Reference: 1036

The nurse should monitor for increases in which laboratory value for the patient as a result of being treated with dexamethasone (Decadron)? A. Sodium B. Calcium C. Potassium D. Blood glucose

D. Blood glucose Hyperglycemia or increased blood glucose level is an adverse effect of corticosteroid therapy. Sodium, calcium, and potassium levels are not directly affected by dexamethasone.

When planning care for a patient with cirrhosis, the nurse will give highest priority to which nursing diagnosis? Impaired skin integrity related to edema, ascites, and pruritus Imbalanced nutrition: less than body requirements related to anorexia Excess fluid volume related to portal hypertension and hyperaldosteronism Ineffective breathing pattern related to pressure on diaphragm and reduced lung volume

Ineffective breathing pattern related to pressure on diaphragm and reduced lung volume Although all of these nursing diagnoses are appropriate and important in the care of a patient with cirrhosis, airway and breathing are always the highest priorities.

The nurse determines a patient has experienced the beneficial effects of therapy with famotidine when which symptom is relieved? Nausea Belching Epigastric pain Difficulty swallowing

correct answer c Famotidine is an H2-receptor antagonist that inhibits parietal cell output of HCl acid and minimizes damage to gastric mucosa related to hyperacidity, thus relieving epigastric pain. It is not indicated for nausea, belching, and dysphagia.

What is the rationale for rotation of insulin injection sites? a. prevents polyuria b. prevents rejection of insulin c. prevents an allergic reaction d. prevents lipodystrophy

d. prevents lipodystrophy

Prevention of calcium oxalate stones would include dietary restrictions of which food or drinks? a. milk and milk products b. dried beans and dried fruit c. liver, kidney, and sweetbreads d. spinach, cabbage, and tomatoes

d. spinach, cabbage, and tomatoes

What test is required for a diagnosis of pyelonephritis? a. renal biopsy b. blood culture c. intravenous pyelogram (IVP) d. urine for culture and sensitivity

d. urine for culture and sensitivity

Which classification of urinary tract infection (UTI) is described as infection of the renal parenchyma, renal pelvis, and ureters? a. Upper UTI c. Complicated UTI b. Lower UTI d. Uncomplicated UTI

a. An upper urinary tract infection (UTI) affects the renal parenchyma, renal pelvis, and ureters. A lower UTI is an infection of the bladder and/or urethra. A complicated UTI exists in the presence of obstruction, stones, or preexisting diseases. An uncomplicated UTI occurs in an otherwise normal urinary tract.

A 44-year-old patient is unable to void after having an open loop resection and fulguration of the bladder. Which nursing action should be implemented first? a. Assist the patient to soak in a 15-minute sitz bath. b. Insert a straight urethral catheter and drain the bladder. c. Encourage the patient to drink several glasses of water. d. Teach the patient how to do isometric perineal exercises.

a. Assist the patient to soak in a 15-minute sitz bath.

the home health nurse is completing the admission assessment for a 76 year old client diagnosed with type 2 diabetes controlled with 70/30 insulin. Which intervention should be included in the plan of care a. assess the clients ability to read small print b. monitor the clients serum pt level c. teach the client how to perform a hemoglobin A1C test daily d. instruct the client to check the feet weekly

a. assess the clients ability to read small print

The pt with diabetes has been diagnosed with autnomic neurupathy. What problems should the nurse expect to find in the patient (select all that apply) a. painless foot ulcers b. erectile dysfunction c. burning foot pain at night d. loss of fine motor control e. vomiting undigested food f. painless myocardial infarction

b. erectile dysfunction e. vomiting undigested food f. painless myocardial infarction

the client with type 2 diabetes controlled with biguanide oral diabetic medication is scheduled for a CT scan with contrast of the abdomen to evaluate pancreatic function. Which intervention should the nurse implement? a. provide a high-fat diet 24 hour prior to test b. hold the biguanide med for 48 hours prior to test c. obtain an informed consent form for the test d. administer pancreatic enzymes prior to the test

b. hold the biguanide med for 48 hours prior to test

An appropriate nursing intervention for the patient with hyperparathyroidism is to a. pad side rails as a seizure precaution b. increase fluid intake to 3000 to 4000ml/day c. maintain bed rest to prevent pathologic frxturs d. monitor the patient for Trousseau's phenomenon or Chvostek's sign

b. increase fluid intake to 3000 to 4000ml/day (R-high fluid intake is indicated in hyperparathyroidism to dilute hypercalcemia and flush the kidneys so that calcium stone formation is reduced.)

in providing care for the patient with adult-onset polycystic kidney disease, what should the nurse do? a. help the patient cope with the rapid progression of the disease. b. suggest genetic counseling resources for the children of the patient c. expect the pt to have polyuria and poor concentration ability of the kidneys d. implement measures for the pt deafness and blindness in addition to renal prboblems

b. suggest genetic counseling resources for the children of the patient

Which assessment data indicate the client diagnosed with DKA is responding to the medical treatment? a. the client has tended skin turgor and dry mucous membranes b. the client is alert and oriented times three c. the clients ABG results are pH 7.29, PaCO2 44, HCO3 15 d. the clients serum potassium level is 3.3 mEq/L

b. the client is alert and oriented times three

When caring for the patient with interstitial cystitis, what can the nurse teach the patient to do? a. avoid foods that make the urine more alkaline b. use high-potency vitamin therapy to decrease the autoimmune effects of the disorder c. always keep a voiding diary to document pain, voiding frequency, and patterns of nocturia d. use the dietary supplement calcium glycerophosphate (prelief) to decrease bladder irritation.

d. use the dietary supplement calcium glycerophosphate (prelief) to decrease bladder irritation.

When caring for a patient with metabolic syndrome, what should the nurse give the highest priority to teach the patient about? a. achieving a normal weight b. performing daily aerobic exercise c. eliminating red meat from the diet d. monitoring blood glucose periodically

a. achieving a normal weight

During change-of-shift report, the nurse learns about the following four patients. Which patient requires assessment first? a. 40-year-old with chronic pancreatitis who has gnawing abdominal pain b. 58-year-old who has compensated cirrhosis and is complaining of anorexia c. 55-year-old with cirrhosis and ascites who has an oral temperature of 102° F (38.8° C) d. 36-year-old recovering from a laparoscopic cholecystectomy who has severe shoulder pain

ANS: C This patient's history and fever suggest possible spontaneous bacterial peritonitis, which would require rapid assessment and interventions such as antibiotic therapy. The clinical manifestations for the other patients are consistent with their diagnoses and do not indicate complications are occurring

Which findings will the nurse expect when caring for a patient with chronic kidney disease (CKD) (select all that apply.)? Anemia Dehydration Hypertension Hypercalcemia Increased risk for fractures Elevated white blood cells

Anemia Hypertension Increased risk for fractures *When the kidney fails, erythropoietin in not excreted, so anemia is expected. Hypocalcemia from chronic renal disease stimulates the parathyroid to release parathyroid hormone, causing calcium liberation from bones increasing the risk of pathological fracture. Dehydration and hypercalcemia are not expected in chronic renal disease. Fluid volume overload and hypocalcemia are expected. Although impaired immune function should be expected, elevated white blood cells would indicate inflammation or infection not associated with chronic renal failure itself but a complication.

What is the first step in the identification of the cause of male infertility? A. Estrogen levels B. Progesterone levels C. Semen analysis D. Time to ejaculation

Answer: C The first test in an infertility study is a semen analysis. The test determines the sperm concentration, motility, and morphology

A diabetic patient has a new order for inhaled insulin (Exubera). Which information about the patient indicates that the nurse should contact the patient before administering the Exubera? a. The patient has a history of a recent myocardial infarction. b. The patient's blood glucose is 224 mg/dl. c. The patient uses a bronchodilator to treat emphysema. d. The patient's temperature is 101.4° F.

C Rationale: Exubera is not recommended for patients with emphysema. The other data do not indicate any contraindication to using Exubera. Cognitive Level: Application Text Reference: p. 1263 Nursing Process: Assessment NCLEX: Physiological Integrity

A diagnosis of hyperglycemic hyperosmolar nonketotic coma (HHNC) is made for a patient with type 2 diabetes who is brought to the emergency department in an unresponsive state. The nurse will anticipate the need to a. administer glargine (Lantus) insulin. b. initiate oxygen by nasal cannula. c. insert a large-bore IV catheter. d. give 50% dextrose as a bolus.

C Rationale: HHNC is initially treated with large volumes of IV fluids to correct hypovolemia. Regular insulin is administered, not a long-acting insulin. There is no indication that the patient requires oxygen. Dextrose solutions will increase the patient's blood glucose and would be contraindicated. Cognitive Level: Application Text Reference: p. 1281 Nursing Process: Planning NCLEX: Physiological Integrity

A patient who has just been diagnosed with type 2 diabetes is 5 ft 4 in (160 cm) tall and weighs 182 pounds (82 kg). A nursing diagnosis of imbalanced nutrition: more than body requirements is developed. Which patient outcome is most important for this patient? a. The patient will have a diet and exercise plan that results in weight loss. b. The patient will state the reasons for eliminating simple sugars in the diet. c. The patient will have a glycosylated hemoglobin level of less than 7%. d. The patient will choose a diet that distributes calories throughout the day.

C Rationale: The complications of diabetes are related to elevated blood glucose, and the most important patient outcome is the reduction of glucose to near-normal levels. The other outcomes are also appropriate but are not as high in priority. Cognitive Level: Application Text Reference: p. 1273 Nursing Process: Planning NCLEX: Physiological Integrity

A patient with Graves' disease has exophthalmos. Which nursing action will be included in the plan of care? A. Apply eye patches to protect the cornea from irritation. B. Place cold packs on the eyes to relieve pain and swelling. C. Elevate the head of the patient's bed to reduce periorbital fluid. D. Teach the patient to blink every few seconds to lubricate the cornea.

C. Elevate the head of the pt's bed to reduce periorbital fluid.

Which is the best indication that the patient is adjusting emotionally to having a colostomy? A. Indicates his spouse will be taking care of things B. Agrees to attend a future support group meeting C. Reads a brochure about colostomy care D. Participates in changing the drainage bag

D Rationale The best indication that a patient is adjusting emotionally is involvement in his or her care. Participation is a better indication than relying on others, promises of future activity, or only reading about the care. Reference: 1042

When teaching a patient newly diagnosed with Graves' disease about the disorder, the nurse explains that a. restriction of iodine intake is needed to reduce thyroid activity. b. exercise is contraindicated to avoid increasing metabolic rate. c. surgery will eventually be required to remove the thyroid gland. d. antithyroid medications may take several weeks to have an effect.

D R: Improvement usually begins in 1-2 wks w good results at 4-6 weeks. Large doses of iodine are used to inhibit the synthesis of thyroid hormones. Exercise using large muscle groups is encouraged to decrease irritability and hyperactivity associated with high levels of thyroid hormones. Radioactive iodine is the most common trtmt for Graves' disease, although surgery may be used.

Number the following in the order of the phases of exchange in PD. Begin with 1 and end with 3. a. Drain b. Dwell c. Inflow

a. 3; b. 2; c. 1

You are caring for a patient with newly diagnosed type 1 diabetes. What information is essential to include in your patient teaching before discharge from the hospital (select all that apply)? a. Insulin administration b. Elimination of sugar from diet c. Need to reduce physical activity d. Hypoglycemia prevention, symptoms, and treatment

a. Insulin administration d. Hypoglycemia prevention, symptoms, and treatment

What is the most serious electrolyte disorder associated with kidney disease? a. Hypocalcemia b. Hyperkalemia c. Hyponatremia d. Hypermagnesemia

b. Hyperkalemia can lead to life-threatening dysrhythmias. Hypocalcemia leads to an accelerated rate of bone remodeling and potentially to tetany. Hyponatremia may lead to confusion. Elevated sodium levels lead to edema, hypertension, and heart failure. Hypermagnesemia may decrease reflexes, mental status, and blood pressure.

Which patient is most likely to be diagnosed with short bowel syndrome? a. History of ulcerative colitis c. Diagnosed with irritable bowel syndrome b. Had extensive resection of the ileum d. Had colectomy performed for cancer of the bowel

b. Short bowel syndrome results from extensive resection of portions of the small bowel and would occur if a patient had an extensive resection of the ileum. The other conditions primarily affect the large colon and result in fewer and less severe symptoms.

which statement best describes atherosclerotic disease affected the cerebrovascular, cardiovascular, and peripheral vascular systems in patients with diabetes? a. it can be prevented by tight glucose control b. it occurs with a higher frequency and earlier onset than in the nondiabetic population c. it is caused by the hyperinsulinemia related to insulin resistance common in type 2 diabetes d. it cannot be modified by reduction of risk factors sch as smoking, obesity, and high fat intake

b. it occurs with a higher frequency and earlier onset than in the nondiabetic population

Which drugs are used to treat overflow incontinence (select all that apply)? a. Baclofen (Lioresal) b. Anticholinergic drugs c. α-Adrenergic blockers d. 5α-reductase inhibitors e. Bethanechol (Urecholine)

c, d, e. α-Adrenergic blockers block the stimulation of the smooth muscle of the bladder, 5α-reductase inhibitors decrease outlet resistance, and bethanechol enhances bladder contractions. Baclofen or diazepam is used to relax the external sphincter for reflex incontinence. Anticholinergics are used to relax bladder tone and increase sphincter tone with urge incontinence.

The medications prescribed for the patient with inflammatory bowel disease include cobalamin and iron injections. What is the rationale for using these drugs? a. Alleviate stress c. Correct malnutrition b. Combat infection d. Improve quality of life

c. Cobalamin and iron injections will help to correct malnutrition. Correcting malnutrition will also indirectly help to improve quality of life and fight infections.

A patient with DI is treated with nasal desmopression. The nurse recognize that the drug is not having an adequate therapeutic effect the the patient experiences a. headache and weight gain b. nasal irritation and nausea c. a urine specific gravity of 1.002 d. an oral intake greater than urinary output

c. a urine specific gravity of 1.002 (rationale- normal urine specific gravity is 1.003 to 1.030, and urine with a specific gravity of 1.002 is very dilute, indicating that there continues to be excessive loss of water and that treatment of DI is inadequate. H/A, weight gain, and oral intake greater the urinary output are signs of volume excess that occur with overmedication. Nasal irritation & nausea may also indicate overmedication.)

Which med order should the nurse question in the client diagnosed with untreated hypothyroidism a. thyroid hormones b. oxygen c. sedatives d. laxatives

c. sedatives

A patient asks the nurse what the difference is between BPH and prostate cancer. the best response by the nurse includes what information about BPH? a. BPH is a benign tumor that does not spread beyond the prostate gland. b. BPH is a precursor to prostate cancer but does not yet show any malignant changes c. BPH is an enlargement of the glandd caused by an increased in the size of existing cells d. BPH is a benign enlargement of the gland caused by an increase in the number of normal cells

d. BPH is a benign enlargement of the gland caused by an increase in the number of normal cells

The nurse recommends genetic counseling for the children of a patient with a.nephrotic syndrome b.chronic pyelonephritis c. malignant nephrosclerosis d.adult onset polycystic kidney disease

d.adult onset polycystic kidney disease

The nurse is providing education to a group of perimenopausal women. Which herbs and/or supplements would the nurse include in a discussion regarding effective alternative therapies for menopausal symptoms (select all that apply)? A. Soy B. Garlic C. Gingko D. Vitamin A E. Black cohosh

A, E A. Soy E. Black cohosh There is good scientific evidence that soy is useful in decreasing menopausal hot flashes and that black cohosh is safe to use for up to 6 months to decrease menopausal symptoms. Garlic, gingko, and vitamin A do not affect menopausal symptoms.

The nurse is caring for a 26-year-old patient who is being discharged after an induced abortion. Which statement should the nurse include in the discharge teaching? A. "Avoid sexual intercourse for 2 weeks." B. "Heavy bleeding is expected for 24 hours." C. "A temperature of 101o F (38.9 o C) is normal" D. "Birth control pills should not be taken for 30 days."

A. "Avoid sexual intercourse for 2 weeks." After an abortion teach the patient to avoid intercourse for 2 weeks. Contraception can be started the day of the procedure. Symptoms of possible complications include a fever and abnormal vaginal bleeding. These symptoms should be reported immediately.

After a vasectomy, what teaching should be included in the discharge teaching? A. "You will want to use an alternative form of contraception for 6 weeks." B. "You may lose some secondary sexual characteristics after this surgery." C. "You may have erectile dysfunction for several months after this surgery." D. "You will be uncomfortable, but you may safely have sexual intercourse today."

A. "You will want to use an alternative form of contraception for 6 weeks." As vasectomies are usually done for sterilization purposes, to safely have sexual intercourse, the patient will need to use an alternative form of contraception until semen examination reveals no sperm, usually at least 10 ejaculations or 6 weeks to evacuate sperm distal to the surgical site. Hormones are not affected, so there is no loss of secondary sexual characteristics or erectile function. Most men experience too much pain to have sexual intercourse on the day of their surgery, so this is not an appropriate comment by the nurse.

The nurse is caring for a 62-year-old man after a transurethral resection of the prostate (TURP). Which instructions should the nurse include in the teaching plan? A. Avoid straining during defecation. B. Restrict fluids to prevent incontinence. C. Sexual functioning will not be affected. D. Prostate exams are not needed after surgery.

A. Avoid straining during defecation. Activities that increase abdominal pressure, such as sitting or walking for prolonged periods and straining to have a bowel movement (Valsalva maneuver), should be avoided in the postoperative recovery period to prevent a postoperative hemorrhage. Instruct the patient to drink at least 2 L of fluid every day. Digital rectal examinations should be performed yearly. The prostate gland is not totally removed and may enlarge after a TURP. Sexual functioning may change after prostate surgery. Changes may include retrograde ejaculation, erectile dysfunction, and decreased orgasmic sensation.

The nurse is caring for a 25-year-old patient who has polycystic ovary syndrome (PCOS). When preparing to teach this patient, which classic manifestation should the nurse know is associated with the severity of symptoms, including infertility? A. Obesity B. Hirsutism C. Amenorrhea D. Irregular menstrual periods

A. Obesity Obesity has been associated with the severity of symptoms such as excess androgens, oligorrhea, amenorrhea, and infertility. This knowledge will affect the teaching the nurse does for this patient to prevent cardiovascular disease and abnormal insulin resistance. Hirsutism, amenorrhea, and irregular menstrual periods are not associated with the severity of the symptoms.

A 71-year-old patient with a diagnosis of benign prostatic hyperplasia (BPH) has been scheduled for a contact laser technique. What is the primary goal of this intervention? A. Resumption of normal urinary drainage B. Maintenance of normal sexual functioning C. Prevention of acute or chronic renal failure D. Prevention of fluid and electrolyte imbalances

A. Resumption of normal urinary drainage The most significant signs and symptoms of BPH relate to the disruption of normal urinary drainage and consequent urine retention, incontinence, and pain. A laser technique vaporizes prostate tissue and cauterizes blood vessels and is used as an effective alternative to a TURP to resolve these problems. Fluid imbalances, sexual functioning, and kidney disease may result from uncontrolled BPH, but the central focus remains urinary drainage.

A 33-year-old patient noticed a painless lump in his scrotum on self-examination of his testicles and a feeling of heaviness. The nurse should first teach him about what diagnostic test? A. Ultrasound B. Cremasteric reflex C. Doppler ultrasound D. Transillumination with a flashlight

A. Ultrasound When the scrotum has a painless lump, scrotal swelling, and a feeling of heaviness, testicular cancer is suspected, and an ultrasound of the testes is indicated. Blood tests will also be done. The cremasteric reflex and Doppler ultrasound are done to diagnose testicular torsion. Transillumination with a flashlight is done to diagnose a hydrocele.

A 58-year-old patient who has undergone a radical vulvectomy for vulvar carcinoma returns to the medical-surgical unit after the surgery. The priority nursing diagnosis for the patient at this time is a. risk for infection related to contact of the wound with urine and stool. b. self-care deficit: bathing/hygiene related to pain and difficulty moving. c. imbalanced nutrition: less than body requirements related to low-residue diet. d. risk for ineffective sexual pattern related to disfiguration caused by the surgery.

ANS: A Complex and meticulous wound care is needed to prevent infection and delayed wound healing. The other nursing diagnoses may also be appropriate for the patient but are not the highest priority immediately after surgery

The nurse will plan to teach the female patient with genital warts about the a. importance of regular Pap tests. b. increased risk for endometrial cancer. c. appropriate use of oral contraceptives. d. symptoms of pelvic inflammatory disease (PID).

ANS: A Genital warts are caused by the human papillomavirus (HPV) and increase the risk for cervical cancer. There is no indication that the patient needs teaching about PID, oral contraceptives, or endometrial cancer

A 22-year-old tells the nurse that she has not had a menstrual period for the last 2 months. Which action is most important for the nurse to take? a. Obtain a urine specimen for a pregnancy test. b. Ask about any recent stressful lifestyle changes. c. Measure the patient's current height and weight. d. Question the patient about prescribed medications.

ANS: A Pregnancy should always be considered a possible cause of amenorrhea in women of childbearing age. The other actions are also appropriate, but it is important to check for pregnancy in this patient because pregnancy will require rapid implementation of actions to promote normal fetal development such as changes in lifestyle, folic acid intake, etc.

A 54-year-old patient is on the surgical unit after a radical abdominal hysterectomy. Which finding is most important to report to the health care provider? a. Urine output of 125 mL in the first 8 hours after surgery b. Decreased bowel sounds in all four abdominal quadrants c. One-inch area of bloody drainage on the abdominal dressing d. Complaints of abdominal pain at the incision site with coughing

ANS: A The decreased urine output indicates possible low blood volume and further assessment is needed to assess for possible internal bleeding. Decreased bowel sounds, minor drainage on the dressing, and abdominal pain with coughing are expected after this surgery

A 32-year-old woman brought to the emergency department reports being sexually assaulted. The patient is confused about where she is and she has a large laceration above the right eye. Which action should the nurse take first? a. Assess the patient's neurologic status. b. Assist the patient to remove her clothing. c. Contact the sexual assault nurse examiner (SANE). d. Ask the patient to describe what occurred during the assault.

ANS: A The first priority is to treat urgent medical problems associated with the sexual assault. The patient's head injury may be associated with a head trauma such as a skull fracture or subdural hematoma. Therefore her neurologic status should be assessed first. The other nursing actions are also appropriate, but they are not as high in priority as assessment and treatment for acute physiologic injury

Which topic will the nurse include in the preoperative teaching for a patient admitted for an abdominal hysterectomy? a. Purpose of ambulation and leg exercises b. Adverse effects of systemic chemotherapy c. Decrease in vaginal sensation after surgery d. Symptoms caused by the drop in estrogen level

ANS: A Venous thromboembolism (VTE) is a potential complication after the surgery, and the nurse will instruct the patient about ways to prevent it. Vaginal sensation is decreased after a vaginal hysterectomy but not after abdominal hysterectomy. Leiomyomas are benign tumors, so chemotherapy and radiation will not be prescribed. Because the patient will still have her ovaries, the estrogen level will not decrease

A 31-year-old patient who has been diagnosed with human papillomavirus (HPV) infection gives a health history that includes smoking tobacco, taking oral contraceptives, and having been treated twice for vaginal candidiasis. Which topic will the nurse include in patient teaching? a. Use of water-soluble lubricants b. Risk factors for cervical cancer c. Antifungal cream administration d. Possible difficulties with conception

ANS: B Because HPV infection and smoking are both associated with increased cervical cancer risk, the nurse should emphasize the importance of avoiding smoking. An HPV infection does not decrease vaginal lubrication, decrease ability to conceive, or require the use of antifungal creams.

When caring for a 58-year-old patient with persistent menorrhagia, the nurse will plan to monitor the a. estrogen level. b. complete blood count (CBC). c. gonadotropin-releasing hormone (GNRH) level. d. serial human chorionic gonadotropin (hCG) results.

ANS: B Because anemia is a likely complication of menorrhagia, the nurse will need to check the CBC. Estrogen and GNRH levels are checked for patients with other problems, such as infertility. Serial hCG levels are monitored in patients who may be pregnant, which is not likely for this patient

A 25-year-old woman has an induced abortion with suction curettage at an ambulatory surgical center. Which instructions will the nurse include when discharging the patient? a. "Heavy vaginal bleeding is expected for about 2 weeks." b. "You should abstain from sexual intercourse for 2 weeks." c. "Contraceptives should be avoided until your reexamination." d. "Irregular menstrual periods are expected for the next few months."

ANS: B Because infection is a possible complication of this procedure, the patient is advised to avoid intercourse until the reexamination in 2 weeks. Patients may be started on contraceptives on the day of the procedure. The patient should call the doctor if heavy vaginal bleeding occurs. No change in the regularity of the menstrual periods is expected

A 32-year-old who was admitted to the emergency department with severe abdominal pain is diagnosed with an ectopic pregnancy. The patient begins to cry and asks the nurse to leave her alone to grieve. Which action should the nurse take next? a. Stay with the patient and encourage her to discuss her feelings. b. Explain the reason for taking vital signs every 15 to 30 minutes. c. Close the door to the patient's room and minimize disturbances. d. Provide teaching about options for termination of the pregnancy.

ANS: B Because the patient is at risk for rupture of the fallopian tube and hemorrhage, frequent monitoring of vital signs is needed. The patient has asked to be left alone, so staying with her and encouraging her to discuss her feelings are inappropriate actions. Minimizing contact with her and closing the door of the room is unsafe because of the risk for hemorrhage. Because the patient has requested time to grieve, it would be inappropriate to provide teaching about options for pregnancy termination

A 29-year-old patient who is trying to become pregnant asks the nurse how to determine when she is most likely to conceive. The nurse explains that a. ovulation is unpredictable unless there are regular menstrual periods. b. ovulation prediction kits provide accurate information about ovulation. c. she will need to bring a specimen of cervical mucus to the clinic for testing. d. she should take her body temperature daily and have intercourse when it drops.

ANS: B Ovulation prediction kits indicate when luteinizing hormone (LH) levels first rise. Ovulation occurs about 28 to 36 hours after the first rise of LH. This information can be used to determine the best time for intercourse. Body temperature rises at ovulation. Postcoital cervical smears are used in infertility testing, but they do not predict the best time for conceiving and are not obtained by the patient. Determination of the time of ovulation can be predicted by basal body temperature charts or ovulation prediction kits and is not dependent on regular menstrual periods

A 24-year-old patient with pelvic inflammatory disease (PID) is being treated with oral antibiotics as an outpatient. Which instruction will be included in patient teaching? a. Abdominal pain may persist for several weeks. b. Return for a follow-up appointment in 2 to 3 days. c. Instruct a male partner to use a condom during sexual intercourse for the next week. d. Nonsteroidal antiinflammatory drug (NSAID) use may prevent pelvic organ scarring

ANS: B The patient is instructed to return for follow-up in 48 to 72 hours. The patient should abstain from intercourse for 3 weeks. Abdominal pain should subside with effective antibiotic therapy. Corticosteroids may help prevent inflammation and scarring, but NSAIDs will not decrease scarring

A 63-year-old woman undergoes an anterior and posterior (A&P) colporrhaphy for repair of a cystocele and rectocele. Which nursing action will be included in the postoperative care plan? a. Encourage a high-fiber diet. b. Perform indwelling catheter care. c. Repack the vagina with gauze daily. d. Teach the patient to insert a pessary.

ANS: B The patient will have a retention catheter for several days after surgery to keep the bladder empty and decrease strain on the suture. A pessary will not be needed after the surgery. Vaginal wound packing is not usually used after an A&P repair. A low-residue diet will be ordered after posterior colporrhaphy

A 34-year-old woman who is discussing contraceptive options with the nurse says, "I want to have children, but not for a few years." Which response by the nurse is appropriate? a. "If you do not become pregnant within the next few years, you never will." b. "You may have more difficulty becoming pregnant after about age 35." c. "You have many years of fertility left, so there is no rush to have children." d. "You should plan to stop taking oral contraceptives several years before you want to become pregnant."

ANS: B The probability of successfully becoming pregnant decreases after age 35, although some patients may have no difficulty in becoming pregnant. Oral contraceptives do not need to be withdrawn for several years for a woman to become pregnant. Although the patient may be fertile for many years, it would be inaccurate to indicate that there is no concern about fertility as she becomes older. Although the risk for infertility increases after age 35, not all patients have difficulty in conceiving

Which assessment finding in a woman who recently started taking hormone therapy (HT) is most important for the nurse to report to the health care provider? a. Breast tenderness b. Left calf swelling c. Weight gain of 3 lb d. Intermittent spotting

ANS: B Unilateral calf swelling may indicate deep vein thrombosis caused by the changes in coagulation associated with HT and would indicate that the HT should be discontinued. Breast tenderness, weight gain, and intermittent spotting are common side effects of HT and do not indicate a need for a change in therapy

A 32-year-old woman is scheduled for an induced abortion using instillation of hypertonic saline solution. Which information will the nurse plan to discuss with the patient before the procedure? a. The patient will require a general anesthetic. b. The expulsion of the fetus may take 1 to 2 days. c. There is a possibility that the patient may deliver a live fetus. d. The procedure may be unsuccessful in terminating the pregnancy.

ANS: B Uterine contractions take 12 to 36 hours to begin after the hypertonic saline is instilled. Because the saline is feticidal, the nurse does not need to discuss any possibility of a live delivery or that the pregnancy termination will not be successful. General anesthesia is not needed for this procedure

Which information will the nurse include when teaching a patient who has developed a small vesicovaginal fistula 2 weeks into the postpartum period? a. Take stool softeners to prevent fecal contamination of the vagina. b. Limit oral fluid intake to minimize the quantity of urinary drainage. c. Change the perineal pad frequently to prevent perineal skin breakdown. d. Call the health care provider immediately if urine drains from the vagina.

ANS: C Because urine will leak from the bladder, the patient should plan to use perineal pads and change them frequently. A high fluid intake is recommended to decrease the risk for urinary tract infections. Drainage of urine from the vagina is expected with vesicovaginal fistulas. Fecal contamination is not a concern with vesicovaginal fistulas.

A 49-year-old woman is considering the use of combined estrogen-progesterone hormone replacement therapy (HT) during menopause. Which information will the nurse include during their discussion? a. Use of estrogen-containing vaginal creams provides most of the same benefits as oral HT. b. Increased incidence of colon cancer in women taking HT requires more frequent colonoscopy. c. HT decreases osteoporosis risk and increases the risk for cardiovascular disease and breast cancer. d. Use of HT for up to 10 years to prevent symptoms such as hot flashes is generally considered safe.

ANS: C Data from the Women's Health Initiative indicate an increased risk for cardiovascular disease and breast cancer in women taking combination HT but a decrease in hip fractures. Vaginal creams decrease symptoms related to vaginal atrophy and dryness, but they do not offer the other benefits of HT, such as decreased hot flashes. Most women who use HT are placed on short-term treatment and are not treated for up to 10 years. The incidence of colon cancer decreases in women taking HRT

A 27-year-old patient tells the nurse that she would like a prescription for oral contraceptives to control her premenstrual dysphoric disorder (PMD-D) symptoms. Which patient information is most important to communicate to the health care provider? a. Bilateral breast tenderness b. Frequent abdominal bloating c. History of migraine headaches d. Previous spontaneous abortion

ANS: C Oral contraceptives are contraindicated in patients with a history of migraine headaches. The other patient information would not prevent the patient from receiving oral contraceptives

A 18-year-old requests a prescription for birth control pills to control severe abdominal cramping and headaches during her menstrual periods. Which should the nurse take first? a. Determine whether the patient is sexually active. b. Teach about the side effects of oral contraceptives. c. Take a personal and family health history from the patient. d. Suggest nonsteroidal antiinflammatory drugs (NSAIDs) for relief.

ANS: C Oral contraceptives may be appropriate to control this patient's symptoms, but the patient's health history may indicate contraindications to oral contraceptive use. Because the patient is requesting contraceptives for management of dysmenorrhea, whether she is sexually active is irrelevant. Because the patient is asking for birth control pills, responding that she should try NSAIDs is nontherapeutic. The patient does not need teaching about oral contraceptive side effects at this time.

Which patient in the women's health clinic will the nurse expect to teach about an endometrial biopsy? a. The 55-year-old patient who has 3 to 4 alcoholic drinks each day b. The 35-year-old patient who has used oral contraceptives for 15 years c. The 25-year-old patient who has a family history of hereditary nonpolyposis colorectal cancer d. The 45-year-old patient who has had 6 full-term pregnancies and 2 spontaneous abortions

ANS: C Patients with a personal or familial history of hereditary nonpolyposis colorectal cancer are at increased risk for endometrial cancer. Alcohol addiction does not increase this risk. Multiple pregnancies and oral contraceptive use offer protection from endometrial cancer

Which action should the nurse take when a 35-year-old patient has a result of minor cellular changes on her Pap test? a. Teach the patient about colposcopy. b. Teach the patient about punch biopsy. c. Schedule another Pap test in 4 months. d. Administer the human papillomavirus (HPV) vaccine.

ANS: C Patients with minor changes on the Pap test can be followed with Pap tests every 4 to 6 months because these changes may revert to normal. Punch biopsy or colposcopy may be used if the Pap test shows more prominent changes. The HPV vaccine may reduce the risk for cervical cancer, but it is recommended only for ages 9 through 26.

The nurse is caring for a 20-year-old patient with pelvic inflammatory disease (PID) requiring hospitalization. Which nursing intervention will be included in the plan of care? a. Monitor liver function tests. b. Use cold packs PRN for pelvic pain. c. Elevate the head of the bed to at least 30 degrees. d. Teach the patient how to perform Kegel exercises.

ANS: C The head of the bed should be elevated to at least 30 degrees to promote drainage of the pelvic cavity and prevent abscess formation higher in the abdomen. Although a possible complication of PID is acute perihepatitis, liver function tests will remain normal. There is no indication for increased fluid intake. Application of heat is used to reduce pain. Kegel exercises are not helpful in PID

A 47-year-old woman asks whether she is going into menopause if she has not had a menstrual period for 3 months. The best response by the nurse is which of the following? a. "Have you thought about using hormone replacement therapy?" b. "Most women feel a little depressed about entering menopause." c. "What was your menstrual pattern before your periods stopped?" d. "Since you are in your mid-40s, it is likely that you are menopausal."

ANS: C The initial response by the nurse should be to assess the patient's baseline menstrual pattern. Although many women do enter menopause in the mid-40s, more information about this patient is needed before telling her that it is likely she is menopausal. Although hormone therapy (HT) may be prescribed, further assessment of the patient is needed before discussing therapies for menopause. Because the response to menopause is very individual, the nurse should not assume that the patient is experiencing any adverse emotional reactions.

A nursing diagnosis that is likely to be appropriate for a 67-year-old woman who has just been diagnosed with stage III ovarian cancer is a. sexual dysfunction related to loss of vaginal sensation. b. risk for infection related to impaired immune function. c. anxiety related to cancer diagnosis and need for treatment decisions. d. situational low self-esteem related to guilt about delaying medical care.

ANS: C The patient with stage III ovarian cancer is likely to be anxious about the poor prognosis and about the need to make decisions about the multiple treatments that may be used. Decreased vaginal sensation does not occur with ovarian cancer. The patient may develop immune dysfunction when she receives chemotherapy, but she is not currently at risk. It is unlikely that the patient has delayed seeking medical care because the symptoms of ovarian cancer are vague and occur late in the course of the cancer

The nurse has just received change-of-shift report about the following four patients. Which patient should be assessed first? a. A patient with a cervical radium implant in place who is crying in her room b. A patient who is complaining of 5/10 pain after an abdominal hysterectomy c. A patient with a possible ectopic pregnancy who is complaining of shoulder pain d. A patient in the fifteenth week of gestation who has uterine cramping and spotting

ANS: C The patient with the ectopic pregnancy has symptoms consistent with rupture and needs immediate assessment for signs of hemorrhage and possible transfer to surgery. The other patients should also be assessed as quickly as possible but do not have symptoms of life-threatening complications

A female patient tells the nurse that she has been having nightmares and acute anxiety around men since being sexually assaulted 3 months ago. The most appropriate nursing diagnosis for the patient is a. anxiety related to effects of being raped. b. sleep deprivation related to frightening dreams. c. rape-trauma syndrome related to rape experience. d. ineffective coping related to inability to resolve incident.

ANS: C The patient's symptoms are most consistent with the nursing diagnosis of rape-trauma syndrome. The nursing diagnoses of sleep deprivation, ineffective coping, and anxiety address some aspects of the patient's symptoms but do not address the problem as completely as the rape-trauma syndrome diagnosis.

The nurse in the infertility clinic is explaining in vitro fertilization (IVF) to a couple. The woman tells the nurse that they cannot afford IVF on her husband's salary. The man replies that if his wife worked outside the home, they would have enough money. Which nursing diagnosis is appropriate? a. Decisional conflict related to inadequate financial resources b. Ineffective sexuality patterns related to psychological stress c. Defensive coping related to anxiety about lack of conception d. Ineffective denial related to frustration about continued infertility

ANS: C The statements made by the couple are consistent with the diagnosis of defensive coping. No data indicate that ineffective sexuality and ineffective denial are problems. Although the couple is quarreling about finances, the data do not provide information indicating that the finances are inadequate

The nurse notes that a patient who has a large cystocele, admitted 10 hours ago, has not yet voided. Which action should the nurse take first? a. Insert a straight catheter per the PRN order. b. Encourage the patient to increase oral fluids. c. Notify the health care provider of the inability to void. d. Use an ultrasound scanner to check for urinary retention.

ANS: D Because urinary retention is common with a large cystocele, the nurse's first action should be to use an ultrasound bladder scanner to check for the presence of urine in the bladder. The other actions may be appropriate, depending on the findings with the bladder scanner

A 31-year-old patient has just been instructed in the treatment for a Chlamydia trachomatis vaginal infection. Which patient statement indicates that the nurse's teaching has been effective? a. "I can purchase an over-the-counter medication to treat this infection." b. "The symptoms are due to the overgrowth of normal vaginal bacteria." c. "The medication will need to be inserted once daily with an applicator." d. "Both my partner and I will need to take the medication for a full week."

ANS: D Chlamydia is a sexually transmitted bacterial infection that requires treatment of both partners with antibiotics for 7 days. The other statements are true for the treatment of Candida albicans infection

A 49-year-old woman tells the nurse that she is postmenopausal but has occasional spotting. Which initial response by the nurse is most appropriate? a. "A frequent cause of spotting is endometrial cancer." b. "How long has it been since your last menstrual period?" c. "Breakthrough bleeding is not unusual in women your age." d. "Are you using prescription hormone replacement therapy?"

ANS: D In postmenopausal women, a common cause of spotting is hormone therapy (HT). Because breakthrough bleeding may be a sign of problems such as cancer or infection, the nurse would not imply that this is normal. The length of time since the last menstrual period is not relevant to the patient's symptoms. Although endometrial cancer may cause spotting, this information is not appropriate as an initial response

A 19-year-old has been diagnosed with primary dysmenorrhea. How will the nurse suggest that the patient prevent discomfort? a. Avoid aerobic exercise during her menstrual period. b. Use cold packs on the abdomen and back for pain relief. c. Talk with her health care provider about beginning antidepressant therapy. d. Take nonsteroidal antiinflammatory drugs (NSAIDs) when her period starts.

ANS: D NSAIDs should be started as soon as the menstrual period begins and taken at regular intervals during the usual time frame in which pain occurs. Aerobic exercise may help reduce symptoms. Heat therapy, such as warm packs, is recommended for relief of pain. Antidepressant therapy is not a typical treatment for dysmenorrhea

A 28-year-old patient was recently diagnosed with polycystic ovary syndrome. It is most important for the nurse to teach the patient a. reasons for a total hysterectomy. b. how to decrease facial hair growth. c. ways to reduce the occurrence of acne. d. methods to maintain appropriate weight.

ANS: D Obesity exacerbates the problems associated with polycystic ovary syndrome, such as insulin resistance and type 2 diabetes. The nurse should also address the problems of acne and hirsutism, but these symptoms are lower priority because they do not have long-term health consequences. Although some patients do require total hysterectomy, this is usually performed only after other therapies have been unsuccessful

When caring for a patient who has a radium implant for treatment of cancer of the cervix, the nurse will a. assist the patient to ambulate every 2 to 3 hours. b. use gloves and gown when changing the patient's bed. c. flush the toilet several times right after the patient voids. d. encourage the patient to discuss needs or concerns by telephone.

ANS: D The nurse should spend minimal time in the patient's room to avoid exposure to radiation. The patient and nurse can have longer conversations by telephone between the patient room and nursing station. To prevent displacement of the implant, absolute bed rest is required. Wearing of gloves and gown when changing linens, and flushing the toilet several times are not necessary because the isotope is confined to the implant

What is the primary purpose of a three-way urinary catheter after a transurethral resection of the prostate (TURP)? A. Promote hemostasis and drainage of clots B. Relieve bladder spasms C. Reduce edema D. Increase bladder tone

Answer: A A large three-way indwelling catheter with a 30-mL balloon is inserted into the bladder after the procedure to provide hemostasis and to facilitate urinary drainage. The bladder is irrigated, either continuously or intermittently, usually for the first 24 hours to prevent obstruction from mucus and blood clots.

A patient scheduled for a prostatectomy for prostate cancer expresses the fear that he will have erectile dysfunction. In responding to the patient, you should keep in mind that A. erectile dysfunction can occur even with a nerve-sparing procedure. B. retrograde ejaculation affects sexual functioning more frequently than erectile dysfunction. C. the most common complication of this surgery is postoperative bowel incontinence. D. preoperative sexual functioning is the most important factor in determining postoperative erectile dysfunction.

Answer: A A major complication after a prostatectomy (even with nerve-sparing procedures) is erectile dysfunction

An elderly male patient is experiencing difficulty in initiating voiding and a feeling of incomplete bladder emptying. What causes these symptoms in benign prostatic hyperplasia (BPH)? A. Obstruction of the urethra B. Untreated chronic prostatitis C. Decreased bladder compliance D. Excessive secretion of testosterone

Answer: A BPH is a benign enlargement of the prostate gland. The enlargement of the prostate gradually compresses the urethra, eventually causing partial or complete obstruction. Compression of the urethra ultimately leads to the development of clinical symptoms.

A patient with benign prostatic hyperplasia is scheduled for TURP. After you assess the patient's knowledge of the procedure and its effects on reproductive function, you determine a need for further teaching when the patient says, A. "It is possible that I'll be sterile after this procedure." B. "I understand that some retrograde ejaculation may occur." C. "I will have a catheter for several days to keep my urinary system open." D. "It is unlikely that I would become impotent from this procedure."

Answer: A The patient will not be sterile; he may experience retrograde ejaculation and some erectile dysfunction. It is unlikely he will become impotent. He will need a catheter.

You have provided teaching to the male patient regarding a vasectomy procedure. Which statement by the patient indicates the teaching has been successful? A. "I will need to not eat or drink anything for 24 hours before the procedure." B. "The physician will use local anesthetic, and I will be awake." C. "I will be able to have unprotected intercourse in 3 weeks." D. "I acknowledge there is a risk of impotence."

Answer: B The procedure requires only 15 to 30 minutes and is usually performed with the patient under local anesthesia on an outpatient basis. Vasectomy is considered a permanent form of sterilization but does not affect the ability to achieve an erection. After vasectomy, the patient should not notice any difference in the look or feel of the ejaculate because its major component is seminal and prostatic fluid. The patient should use an alternative form of contraception until semen examination reveals no sperm. This usually requires at least 10 ejaculations or 6 weeks to evacuate sperm distal to the surgical site.

What is the most significant factor in the development of clinical symptoms associated with BPH? A. Size of the prostate B. Location of the enlargement C. Age of the patient D. Length of the urethra

Answer: B There is no direct relationship between the size of the prostate and degree of obstruction. The location of the enlargement significantly affects development of obstructive symptoms. For example, it is possible for mild hyperplasia to cause severe obstruction, and it is possible for extreme hyperplasia to cause few obstructive symptoms.

A 62-year-old man is seen at the health clinic because he is concerned about a gradual decrease in sexual performance. The nursing history does not identify any specific risk factors related to erectile dysfunction. What is your priority at this time? A. Consult with the health care provider about testing the patient for hypogonadism. B. Refer the patient to a qualified therapist to explore possible psychologic causes of decreased function. C. Explain normal age-related changes in sexual performance to the patient. D. Discuss the variety of aids and devices available to increase sexual performance.

Answer: C Normal physiologic age-related changes are associated with changes in erectile function and may be an underlying cause of ED for some men.

You expect which finding in a patient as a complication of prostatic hyperplasia? A. Dysuria B. Hematuria C. Urinary retention D. Urinary frequency

Answer: C Obstructive symptoms caused by prostate enlargement include a decrease in the caliber and force of the urinary stream, difficulty in initiating voiding, intermittency (stopping and starting stream several times while voiding), and dribbling at the end of urination. These symptoms result from urinary retention.

During the first 4 hours after TURP, the patient receives 1200 mL of bladder irrigation solution, and his urine output is 1000 mL. What is your priority intervention? A. Slowing the rate of bladder irrigation B. Continuing to observe the patient C. Checking catheter patency D. Encouraging oral fluids

Answer: C You should continuously monitor the inflow and outflow of the irrigant. If outflow is less than inflow, assess the catheter for kinks or clots. If the outflow is blocked and patency cannot be reestablished by manual irrigation, stop the CBI and notify the physician.

In assessing a patient for testicular cancer, you understand that the manifestations of this disease often include A. acute back spasms and testicular pain. B. rapid onset of scrotal swelling and fever. C. fertility problems and bilateral scrotal tenderness. D. painless mass and heaviness sensation in the scrotal area.

Answer: D Clinical manifestations of testicular cancer include a painless lump in the scrotum, scrotal swelling, and a feeling of heaviness. The scrotal mass usually is not tender and is very firm. Some patients complain of a dull ache or heavy sensation in the lower abdomen, perianal area, or scrotum.

A priority nursing diagnosis for the patient with bacterial prostatitis is A. activity intolerance related to fatigue. B. sexual dysfunction related to painful ejaculation. C. deficient fluid volume related to decreased fluid intake. D. impaired urinary elimination related to urethral compression.

Answer: D Acute urinary retention can develop in acute prostatitis, and it requires bladder drainage with suprapubic catheterization. The patient may experience fatigue, but this is not the priority. Sexual dysfunction can occur, but the pain occurs after ejaculation. Fluid volume should be increased.

Postoperatively, a patient who has had a laser prostatectomy has continuous bladder irrigation with a three-way urinary catheter with a 30-mL balloon. When he complains of bladder spasms with the catheter in place, you should A. deflate the catheter balloon to 10 mL to decrease bulk in the bladder. B. deflate the catheter balloon and then reinflate it to ensure that it is patent. C. encourage the patient to try to have a bowel movement to relieve colon pressure. D. explain that this feeling is normal and that he should not try to urinate around the catheter.

Answer: D Bladder spasms occur as a result of irritation of the bladder mucosa from the insertion of the resectoscope, presence of a catheter, or clots leading to obstruction of the catheter. Instruct the patient not to urinate around the catheter because this increases the likelihood of spasm.

The patient at the clinic complains about her abdominal bloating, depression, and irritability related to her premenstrual syndrome. What should the nurse first recommend (select all that apply)? A. Take diuretics. B. Exercise regularly. C. Take antidepressants. D. Take antianxiety agents. E. Increase pork, chicken, and milk intake.

B, E B. Exercise regularly. E. Increase pork, chicken, and milk intake. The nurse can recommend regular exercise to help manage stress, elevate the mood, and have a relaxing effect. Eating foods rich in vitamin B6 (pork, milk, and legumes) and tryptophan (dairy and poultry) will promote serotonin production and improve symptoms. Diuretics, antidepressants, and antianxiety agents are not prescribed unless symptoms persist or interfere with daily functioning.

A male patient complains of fever, dysuria, and cloudy urine. What additional information may indicate that these manifestations may be something other than a urinary tract infection (UTI)? A. E. coli bacteria in his urine B. A very tender prostate gland C. Complaints of chills and rectal pain D. Complaints of urgency and frequency

B. A very tender prostate gland A tender and swollen prostate is indicative of prostatis, which is a more serious male reproductive problem because an acute episode can result in chronic prostatis and lead to epididymitis or cystitis. E. coli in his urine, chills and rectal pain, and urgency and frequency are all present with a UTI and not specifically indicative of prostatis.

A 73-year-old male patient admitted for total knee replacement states during the health history interview that he has no problems with urinary elimination except that the "stream is less than it used to be." The nurse should give the patient anticipatory guidance that what condition may be developing? A. A tumor of the prostate B. Benign prostatic hyperplasia C. Bladder atony because of age D. Age-related altered innervation of the bladder

B. Benign prostatic hyperplasia Benign prostatic hyperplasia is an enlarged prostate gland because of an increased number of epithelial cells and stromal tissue. It occurs in about 50% of men over age 50 and 80% of men over age 80. Only about 16% of men develop prostate cancer. Bladder atony and age-related altered innervations of the bladder do not lead to a weakened stream.

A 23-year-old woman who is admitted with a possible ectopic tubal pregnancy reports sudden intense pelvic pain radiating to the left shoulder. Which action by the nurse should receive the highest priority? A. Observe the amount of vaginal bleeding every 15 minutes for 1 hour. B. Check the vital signs and immediately notify the health care provider. C. Administer the prescribed pain medication and reassess in 30 minutes. D. Assess the fetal heart tones and determine the presence of fetal movement.

B. Check the vital signs and immediately notify the health care provider. A ruptured ectopic pregnancy may produce pelvic or abdominal pain and vaginal bleeding. If the tube ruptures, the pain is intense and may be referred to the shoulder. External vaginal bleeding may not be an accurate indicator of actual blood loss. Vital signs should be monitored closely along with observation for signs of shock. A ruptured ectopic pregnancy is an emergency because of the risk of hemorrhage and hypovolemic shock. The patient may need a blood transfusion and IV fluid therapy. In addition, the patient will need emergency surgery. Fetal assessment is not indicated for an ectopic pregnancy.

The nurse is teaching health promotion to a variety of women in a community center. When asked when a female should begin having a Pap smear, how should the nurse respond? A. Every year beginning at age 30 B. Every 3 years beginning at age 21 C. Every 3 years beginning at age 18 if sexually active D. Every year beginning at the onset of menarche and continuing until menopause

B. Every 3 years beginning at age 21 A Pap test (Pap smear) should be done at least once every 3 years at the age of 21 regardless of when a woman becomes sexually active. Women 65 years or older may stop having Pap tests after having no abnormal Pap tests for the last 2 years.

A 45-year-old man reports having recent problems attaining an erection. Which medication will the nurse further explore as the possible etiology of this patient's sexual dysfunction? A. Furosemide (Lasix) B. Fluoxetine (Prozac) C. Clopidogrel (Plavix) D. Nitroglycerin (Nitrostat)

B. Fluoxetine (Prozac) Fluoxetine is a selective serotonin reuptake inhibitor used in the treatment of depression. A common adverse effect of this medication is sexual problems (impotence, delayed or absent orgasm, delayed or absent ejaculation, decreased sexual interest) in nearly 70% of men and women.

To prevent or decrease age-related changes that occur after menopause in a patient who chooses not to take hormone therapy, the most important self-care measure to teach is: A. maintaining usual sexual activity B. increasing the intake of dairy products C. performing regular aerobic weight bearing exercises D. taking vitamin E and B-complex vitamin supplements

C

Postoperative nursing care for the woman with a gynecologic fistula includes (select all that apply): A. ambulation B. bladder training C. warm sitz baths D. perineal hygiene

C,D

The nurse obtains a history from a 42-year-old patient diagnosed with premenstrual syndrome (PMS). It is most important for the nurse to follow up on which patient statement? A. "Exercise is relaxing and reduces anxiety and irritability." B. "Milk, beans, and soy nuts may decrease symptoms of PMS." C. "Evening primrose oil will reduce bleeding during my period." D. "A low-salt diet decreases weight gain and bloating before my period."

C. "Evening primrose oil will reduce bleeding during my period." Evening primrose oil may increase the risk of bleeding. Foods rich in vitamin B6 (e.g., pork, milk, egg yolk, legumes) promote serotonin production, which improves the symptoms of PMS. Aerobic exercise can have a relaxing effect and can reduce PMS clinical manifestations. Limiting salt intake and increasing calcium intake have been proposed to alleviate fluid retention, weight gain, bloating, breast swelling, and tenderness.

The nurse provides drug teaching for a 30-year-old woman who is prescribed clomiphene (Clomid). It is most important for the nurse to follow up on which patient statement? A. "Hormone production and release will be increased." B. "This drug is like estrogen and is used to treat infertility." C. "I should avoid intercourse while taking this medication." D. "This medication will stimulate my ovaries to produce eggs."

C. "I should avoid intercourse while taking this medication." Clomiphene is an oral medication administered for infertility. The medication is a selective estrogen-stimulation modulator that stimulates ovulation, making pregnancy after intercourse or artificial insemination more likely. The drug increases gonadotropin-releasing hormone production. In addition, the release of the follicle-stimulating hormone and luteinizing hormone is increased.

A pregnant female is experiencing amenorrhea, morning sickness, and breast tenderness. In the ninth week after her last menstrual period, she is rushed to the hospital with severe left shoulder pain, blood pressure of 90/60 mm Hg, and heart rate of 112 beats/minute. What is the best diagnostic test to determine the cause of her problem? A. Serum hemoglobin B. 12-lead electrocardiogram C. A transvaginal ultrasound D. Serial β-human chorionic gonadotropin levels

C. A transvaginal ultrasound Because the patient is known to be pregnant, a transvaginal ultrasound will be used to assess for ectopic pregnancy and tubal rupture. Serum hemoglobin and 12-lead ECG would not define a diagnosis related to the manifestations that she has. Serial β-human chorionic gonadotropin levels could be used if the patient was stable to determine if a spontaneous abortion is occurring, as the levels would decrease over time.

The nurse teaches a 30-year-old man with a family history of prostate cancer about dietary factors associated with prostate cancer. The nurse determines that teaching is successful if the patient selects which menu? A. Grilled steak, French fries, and vanilla shake B. Hamburger with cheese, pudding, and coffee C. Baked chicken, peas, apple slices, and skim milk D. Grilled cheese sandwich, onion rings, and hot tea

C. Baked chicken, peas, apple slices, and skim milk A diet high in red meat and high-fat dairy products along with a low intake of vegetables and fruits may increase the risk of prostate cancer.

To accurately monitor progression of a symptom of decreased urinary stream, the nurse should encourage the patient to have which primary screening measure done on a regular basis? A. Uroflowmetry B. Transrectal ultrasound C. Digital rectal examination (DRE) D. Prostate-specific antigen (PSA) monitoring

C. Digital rectal examination (DRE) Digital rectal examination is part of a regular physical examination and is a primary means of assessing symptoms of decreased urinary stream, which is often caused by benign prostatic hyperplasia in men over 50 years of age. The uroflowmetry helps determine the extent of urethral blockage and the type of treatment needed but is not done on a regular basis. Transrectal ultrasound is indicated with an abnormal DRE and elevated PSA to differentiate between BPH and prostate cancer. The PSA monitoring is done to rule out prostate cancer, although levels may be slightly elevated in patients with BPH.

A 58-year-old woman is one-day postoperative following an abdominal hysterectomy. Which intervention should the nurse perform in order to prevent deep vein thrombosis (DVT)? A. Place the patient in a high Fowler's position. B. Provide pillows to place under the patient's knees. C. Encourage the patient to change positions frequently. D. Teach the patient deep breathing and coughing exercises.

C. Encourage the patient to change positions frequently. The patient should be encouraged to change positions frequently and ambulate to prevent venous stasis. The high Fowler's position and pressure under the knees should be avoided in order to prevent DVT. Deep breathing and coughing are therapeutic exercises but do not directly address the risk of DVT.

What should the nurse emphasize when teaching a woman who has been diagnosed with pelvic inflammatory disease (PID)? A. The importance of contraception B. Benefits of hormone therapy (HT) C. Manifestations of further infection D. The importance of maintaining hygiene

C. Manifestations of further infection PID frequently progresses to serious infection of the reproductive structures. The diagnosis does not present a particular need for contraception or specific hygiene measures. HT is not used to treat PID.

The patient has had cardiovascular disease for some time and has now developed erectile dysfunction. He is frustrated because he cannot take erectogenic medications because he takes nitrates for his cardiac disease. What should the nurse do first to help this patient? A. Give the patient choices for penile implant surgery. B. Recommend counseling for the patient and his partner. C. Obtain a thorough sexual, health, and psychosocial history. D. Assess levels of testosterone, prolactin, LH, and thyroid hormones.

C. Obtain a thorough sexual, health, and psychosocial history. The nurse's first action to help this patient is to obtain a thorough sexual, health, and psychosocial history. Alternative treatments for the cardiac disease would then be explored if that had not already been done. Further examination or diagnostic testing would be based on the history and physical assessment, including hormone levels, counseling, or penile implant options.

What should the nurse include when teaching about early detection of ovarian cancer? A. Report any pelvic or vaginal bleeding soon. B. Use estrogen with progestin for menopause. C. Obtain annual bimanual pelvic examinations. D. Receive a preventive bilateral oophorectomy.

C. Obtain annual bimanual pelvic examinations. Because it is difficult for a patient to detect early clinical indicators of ovarian cancer, the best method of early detection is to have a yearly bimanual pelvic examination to palpate for an ovarian mass. Although pelvic or vaginal bleeding should be reported soon after it occurs, this rarely occurs with ovarian cancer and is not an early symptom. Oral contraceptives may be used or a preventive bilateral oophorectomy may be done to reduce the risk, but they would not be done to detect early ovarian cancer.

A patient is one day postoperative following a transurethral resection of the prostate (TURP). Which event is not an expected normal finding in the care of this patient? A. The patient requires two tablets of Tylenol #3 during the night. B. The patient complains of fatigue and claims to have minimal appetite. C. The patient has continuous bladder irrigation (CBI) infusing, but output has decreased. D. The patient has expressed anxiety about his planned discharge home the following day.

C. The patient has continuous bladder irrigation (CBI) infusing, but output has decreased. A decrease or cessation of output in a patient with CBI requires immediate intervention. The nurse should temporarily stop the CBI and attempt to resume output by repositioning the patient or irrigating the catheter. Complaints of pain, fatigue, and low appetite at this early postoperative stage are not unexpected. Discharge planning should be addressed, but this should not precede management of the patient's CBI.

A 60-year-old woman comes to the clinic reporting unexpected bleeding. What should the nurse tell the patient about diagnosing the cause of this bleeding? A. "It is probably only the end of menopause." B. "You will need a hysterectomy to treat this bleeding." C. "You will need a Pap smear to see if you have endometrial cancer." D. "You will need an endometrial biopsy to determine the cause of bleeding.

D. "You will need an endometrial biopsy to determine the cause of bleeding. With unexpected bleeding in a postmenopausal woman, an endometrial biopsy should be done to exclude or diagnose endometrial cancer. The abnormal bleeding should not be ignored just because she is postmenopausal. A hysterectomy with bilateral salpingo-oophorectomy with lymph node biopsies will be done to treat endometrial cancer if it is diagnosed. A Pap smear will not diagnose endometrial cancer unless it has spread to the cervix.

Because of the risks associated with hormone therapy (HT), a 50-year-old patient does not want to take HT and asks the nurse how she can handle her perimenopausal symptoms of hot flashes and sweating at night. What should the nurse first advise this patient? A. Increase warmth to avoid chills. B. Good nutrition to avoid osteoporosis C. Vitamin B complex and vaginal lubrication D. Decrease heat production and increase heat loss.

D. Decrease heat production and increase heat loss. To avoid hot flashes and sweating at night, decrease heat production with a cool environment, limit caffeine and alcohol, and practice relaxation techniques. Heat loss may be facilitated with increased circulation in the room, avoidance of heavy bedding, and wearing loose-fitting clothes. Warmth will facilitate hot flashes. Nutrition, vitamin B complex, and vaginal lubrication will help with other complications of perimenopause, but not hot flashes and sweating at night.

What should the nurse advise a woman to do who is recovering from surgical repair of a fistula? A. Douche daily to prevent postoperative infection. B. Remove and cleanse her pessary on a daily basis. C. Resume normal activity to prevent adhesion formation. D. Ensure that she does not place stress on the repaired area.

D. Ensure that she does not place stress on the repaired area. Following surgical repair of a fistula, the patient should avoid placing stress on the repaired region. Normal activity is not commonly resumed until significant healing has occurred. Douching is contraindicated, and pessaries are used to treat prolapses, not fistulas.

What should be included in nursing responsibilities when providing immediate care for a female patient who has experienced a sexual assault? A. Administering a pregnancy test B. Close monitoring of the patient's vital signs C. Ensuring the patient is left alone when possible D. Informing the patient about possible financial support

D. Informing the patient about possible financial support Many sexual assault survivors are unaware of the availability of financial compensation (a law in most states) and appreciate information about the application process. A pregnancy test is premature, and the patient should not be left alone. There is rarely a specific indication for close monitoring of vital signs unless the extent of physical injury indicates a need.

The nurse coordinates postoperative care for a 70-year-old man with osteoarthritis after prostate surgery. Which task is appropriate for the nurse to delegate to a licensed practical/vocational nurse (LPN/LVN)? A. Teach the patient how to perform Kegel exercises. B. Provide instructions to the patient on catheter care. C. Administer oxybutynin (Ditropan) for bladder spasms. D. Manually irrigate the urinary catheter to determine patency.

D. Manually irrigate the urinary catheter to determine patency. The nurse may delegate the following to an LPN/LVN: monitor catheter drainage for increased blood or clots, increase flow of irrigating solution to maintain light pink color in outflow, administer antispasmodics and analgesics as needed. A registered nurse may not delegate teaching, assessments, or clinical judgments to a LPN/LVN.

The woman, who has finished having children, has been suffering from endometriosis and is seeking a cure. The nurse should teach this patient about what treatment? A. Leuprolide (Lupron) B. Danazol (Danocrine) C. Nonsteroidal anti-inflammatory drugs D. Surgical removal of endometrial implants

D. Surgical removal of endometrial implants The only cure for endometriosis is the surgical removal of all endometrial implants that may include the uterus, fallopian tubes, and ovaries. Leuprolide is a gonadotropin-releasing hormone agonist that causes amenorrhea with menopausal side effects. Danazol is a synthetic androgen that inhibits the anterior pituitary. Nonsteroidal anti-inflammatory drugs relieve pain but do not affect the problem of endometriosis.

A 55-year-old woman diagnosed with endometrial cancer is receiving brachytherapy. The nurse is most concerned if what is observed? A. The patient develops foul-smelling vaginal discharge. B. The patient has 5 to 8 liquid stools over a period of 24 hours. C. The patient uses a bedpan instead of a bedside commode or toilet. D. The patient requests a nursing assistant to stay in the room for company.

D. The patient requests a nursing assistant to stay in the room for company. Brachytherapy is internal radiation applied directly to the tumor. Health care providers should limit close contact with the patient to less than 30 minutes per day. Internal radiation causes the destruction of cells and results in a foul-smelling vaginal discharge. Internal radiation may cause systemic reactions such as nausea, vomiting, diarrhea, and malaise. The patient receiving brachytherapy is placed in a lead-lined private room and on absolute bed rest.

Postoperative goals in caring for the patient who has undergone an abdominal hysterectomy include (select all that apply) a. monitoring urine output b. changing position frequently c. restricting all food for 24 hours d. observing perineal pad for bleeding e. encouraging leg exercises to promote circulation

a, b, e.

Nursing responsibilities related to the patient receiving brachytherapy for endometrial cancer include a. maintaining absolute bed rest b. keeping the patient in high Fowler's position c. allowing visitors if they stay 3 ft (1 m) from the bed d. limiting direct nurse-to-patient contact to 30 minutes per shift

a.

The first nursing intervention for the patient who has been sexually assaulted is to a. treat urgent medical problems b. contact support person for the patient c. provide supplies for the patient to cleanse self d. document bruises and lacerations of the perineum and the cervix

a.

What do you tell the patient with chronic bacterial prostatitis who is undergoing antibiotic therapy (select all that apply)? A. Hospitalization is required of all patients. B. Pain will lessen after treatment has ended. C. The course of treatment usually is 2 to 4 weeks. D. Long-term therapy may be indicated for immunocompromised patients. E. If the condition is unresolved and untreated, the patient is at risk for prostate cancer.

answer: B, D Patients with chronic bacterial prostatitis are usually given antibiotics for 4 to 12 weeks. However, antibiotics may be given for a lifetime if the patient is immunocompromised. Although patients with chronic bacterial prostatitis tend to experience a great amount of discomfort, the pain resolves as the infection is treated. If the patient with acute bacterial prostatitis has high fever or other signs of impending sepsis, hospitalization and intravenous antibiotics are prescribed.

In telling a patient with infertility what she and her partner can expect, the nurse explains that a. ovulatory studies can help determine tube patency b. a hysterosalpingogram is a common diagnostic study c. the cause will remain unexplained for 40% of couples d. if postcoital studies are normal, infection tests will be done

b.

A 32-year-old patient has oral contraceptives prescribed for endometriosis. The nurse will teach the patient to a. expect to experience side effects such as facial hair. b. take the medication every day for the next 9 months. c. take calcium supplements to prevent developing osteoporosis during therapy. d. use a second method of contraception to ensure that she will not become pregnant.

b. take the medication every day for the next 9 months. When oral contraceptives are prescribed to treat endometriosis, the patient should take the medications continuously for 9 months. Facial hair is a side effect of synthetic androgens. The patient does not need to use additional contraceptive methods. The hormones in oral contraceptives will protect against osteoporosis

The family of a patient newly diagnosed with hepatitis A asks the nurse what they can do to prevent becoming ill themselves. Which response by the nurse is most appropriate? "The hepatitis vaccine will provide immunity from this exposure and future exposures." "I am afraid there is nothing you can do since the patient was infectious before admission." "You will need to be tested first to make sure you don't have the virus before we can treat you." "An injection of immunoglobulin will need to be given to prevent or minimize the effects from this exposure."

"An injection of immunoglobulin will need to be given to prevent or minimize the effects from this exposure." Immunoglobulin provides temporary (1-2 months) passive immunity and is effective for preventing hepatitis A if given within 2 weeks after exposure. It may not prevent infection in all persons, but it will at least modify the illness to a subclinical infection. The hepatitis vaccine is only used for preexposure prophylaxis.

what type of urinary tract calculi are the most common and frequently obstruct the ureter? a. cystine b. uric acid c. calcium oxalate d. calcium phosphate

c. calcium oxalate

The nurse provides discharge instructions for a 64-year-old woman with ascites and peripheral edema related to cirrhosis. Which statement, if made by the patient, indicates teaching was effective? "It is safe to take acetaminophen up to four times a day for pain." "Lactulose (Cephulac) should be taken every day to prevent constipation." "Herbs and other spices should be used to season my foods instead of salt." "I will eat foods high in potassium while taking spironolactone (Aldactone)."

"Herbs and other spices should be used to season my foods instead of salt." A low-sodium diet is indicated for the patient with ascites and edema related to cirrhosis. Table salt is a well-known source of sodium and should be avoided. Alternatives to salt to season foods include the use of seasonings such as garlic, parsley, onion, lemon juice, and spices. Pain medications such as acetaminophen, aspirin, and ibuprofen should be avoided as these medications may be toxic to the liver. The patient should avoid potentially hepatotoxic over-the-counter drugs (e.g., acetaminophen) because the diseased liver is unable to metabolize these drugs. Spironolactone is a potassium-sparing diuretic. Lactulose results in the acidification of feces in bowel and trapping of ammonia, causing its elimination in feces.

The patient states that she "loses urine" every time she laughs or coughs. The nurse teaches the patient measures to regain urinary control. The nurse recognizes the need for further teaching when the patient states: A) "I will perform my Kegel exercises every day." B) "I joined weight watchers." C) "I drink two glasses of wine with dinner." D) "I have tried urinating every 3 hours."

"I drink two glasses of wine with dinner."

The patient with cirrhosis is being taught self-care. Which statement indicates the patient needs more teaching? "If I notice a fast heart rate or irregular beats, this is normal for cirrhosis." "I need to take good care of my belly and ankle skin where it is swollen." "A scrotal support may be more comfortable when I have scrotal edema." "I can use pillows to support my head to help me breathe when I am in bed."

"If I notice a fast heart rate or irregular beats, this is normal for cirrhosis." Correct If the patient with cirrhosis experiences a fast or irregular heart rate, it may be indicative of hypokalemia and should be reported to the health care provider, as this is not normal for cirrhosis. Edematous tissue is subject to breakdown and needs meticulous skin care. Pillows and a semi-Fowler's or Fowler's position will increase respiratory efficiency. A scrotal support may improve comfort if there is scrotal edema.

The nurse instructs a 50-year-old woman about cholestyramine to reduce pruritis caused by gallbladder disease. Which statement by the patient to the nurse indicates she understands the instructions? "This medication will help me digest fats and fat-soluble vitamins." "I will apply the medicated lotion sparingly to the areas where I itch." "The medication is a powder and needs to be mixed with milk or juice." "I should take this medication on an empty stomach at the same time each day."

"The medication is a powder and needs to be mixed with milk or juice." For treatment of pruritus, cholestyramine may provide relief. This is a resin that binds bile salts in the intestine, increasing their excretion in the feces. Cholestyramine is in powder form and should be mixed with milk or juice before oral administration.

The nurse is planning the care of a client diagnosed with Addison's disease. Which intervention should be included? 1. Administer steroid medications. 2. Place the client on fluid restriction. 3. Provide frequent stimulation. 4. Consult physical therapy for gait training.

1. Administer steroid medications. Clients diagnosed with Addison's dis- ease have adrenal gland hypofunction. The hormones normally produced by the gland must be replaced. Steroids and androgens are produced by the adrenal gland.

The client diagnosed with a pituitary tumor developed syndrome of inappropriate antidiuretic hormone (SIADH). Which interventions should the nurse implement? 1. Assess for dehydration and monitor blood glucose levels. 2. Assess for nausea and vomiting and weigh daily. 3. Monitor potassium levels and encourage fluid intake. 4. Administer vasopressin IV and conduct a fluid deprivation test.

2. Early signs and symptoms are nausea and vomiting.

Which signs/symptoms should make the nurse suspect the client is experiencing a thyroid storm? 1. Obstipation and hypoactive bowel sounds. 2. Hyperpyrexia and extreme tachycardia. 3. Hypotension and bradycardia. 4. Decreased respirations and hypoxia.

2. Hyperpyrexia (high fever) and heart rate above 130 beats per minute are signs of thyroid storm,

The client is admitted to rule out Cushing's syndrome. Which laboratory tests should the nurse anticipate being ordered? 1. Plasma drug levels of quinidine, digoxin, and hydralazine. 2. Plasma levels of ACTH and cortisol. 3. A 24-hour urine for metanephrine and catecholamine. 4. Spot urine for creatinine and white blood cells.

2. The adrenal gland secretes cortisol and the pituitary gland secretes adrenocorticotropic hormone (ACTH), a hormone used by the body to stimulate the production of cortisol.

The nurse is admitting a client to the neurological intensive care unit who is postoperative transsphenoidal hypophysectomy. Which data warrant immediate intervention? 1. The client is alert to name but is unable to tell the nurse the location. 2. The client has an output of 2,500 mL since surgery and an intake of 1,000 mL. 3. The client's vital signs are T 97.6 F, P 88, R 20, and BP 130/80. 4. The client has a 3-cm amount of dark-red drainage on the turban dressing.

2. The output is more than double the intake in a short time.

Which sign/symptom indicates to the nurse the client is experiencing hyperparathyroidism? 1. A negative Trousseau's sign. 2. A positive Chvostek's sign. 3. Nocturnal muscle cramps. 4. Tented skin turgor.

2. When a sharp tapping over the facial nerve elicits a spasm or twitching of the mouth, nose, or eyes, the client is hypocalcemic

A patient has been diagnosed with hypoparathyroidism. What manifestations should the nurse expect to observe (select all that apply)? a. Skeletal pain b. Dry, scaly skin c. Personality changes d. Abdominal cramping e. Cardiac dysrhythmias f. Muscle spasms and stiffness

28. b, c, d, e, f. In hypoparathyroidism the patient has inadequate circulating parathyroid hormone (PTH) that leads to hypocalcemia from the inability to maintain serum calcium levels. With hypocalcemia there is muscle stiffness and spasms, which can lead to cardiac dysrhythmias and abdominal cramps. There can also be personality and visual changes and dry, scaly skin.

The client diagnosed with Cushing's disease has undergone a unilateral adrenalectomy. Which discharge instructions should the nurse discuss with the client? 1. Instruct the client to take the glucocorticoid and mineralocorticoid medications as prescribed. 2. Teach the client regarding sexual functioning and androgen replacement therapy. 3. Explain the signs and symptoms of infection and when to call the health-care provider. 4. Demonstrate turn, cough, and deep-breathing exercises the client should perform every (2) hours.

3. Notifying the HCP if signs/symptoms of infection develop is an instruction given to all surgical clients on discharge.

The nurse is discussing the endocrine system with the client. Which endocrine gland secretes epinephrine and norepinephrine? 1. The pancreas. 2. The adrenal cortex. 3. The adrenal medulla. 4. The anterior pituitary gland.

3. The adrenal medulla secretes the catecholamines epinephrine and norepinephrine.

The client diagnosed with hypothyroidism is prescribed the thyroid hormone levothyroxine (Synthroid). Which assessment data indicate the medication has been effective? 1. The client has a three (3)-pound weight gain. 2. The client has a decreased pulse rate. 3. The client's temperature is WNL. 4. The client denies any diaphoresis.

3. The client with hypothyroidism frequently has a subnormal temperature, so a temperature WNL indicates the medication is effective.

The patient has a history of chronic diarrhea of three or more liquid stools for the past 4 weeks. What potential imbalance is of greatest concern? A. Hypokalemia B. Hyponatremia C. Metabolic alkalosis D. Thrombocytopenia

A Rationale Diarrhea can cause electrolyte imbalances, particularly hypokalemia. Because dehydration can result, the patient is at risk for hypernatremia. Metabolic acidosis can develop with severe diarrhea. Low platelet levels are not necessarily associated with diarrhea. Reference: 1008

The charge nurse of an intensive care unit is making assignments for the night shift. Which client should be assigned to the most experienced intensive care nurse? 1. The client diagnosed with respiratory failure who is on a ventilator and requires frequent sedation. 2. The client diagnosed with lung cancer and iatrogenic Cushing's disease with ABGs of pH 7.35, PaO2 88, PaCO2 44, and HCO3 22. 3. The client diagnosed with Addison's disease who is lethargic and has a BP of 80/45, P 124, and R 28. 4. The client diagnosed with hyperthyroidism who has undergone a thyroidectomy two (2) days ago and has a negative Trousseau's sign.

3. This client has a low blood pressure and tachycardia. This client may be experiencing an addisonian crisis

You are caring for a patient in the emergency department with complaints of acute abdominal pain, nausea, and vomiting. When you palpate the patient's left lower abdominal quadrant, the patient complains of pain in the right lower quadrant. You will document this as which diagnostic sign of appendicitis? A. Rovsing sign B. Referred pain C. Chvostek's sign D. Rebound tenderness

A Rationale In patients with suspected appendicitis, the Rovsing sign may be elicited by palpation of the left lower quadrant, causing pain to be felt in the right lower quadrant. Reference: 1020

The nurse is providing an in-service on thyroid disorders. One of the attendees asks the nurse, "Why don't the people in the United States get goiters as often?" Which statement by the nurse is the best response? 1. "It is because of the screening techniques used in the United States." 2. "It is a genetic predisposition rare in North Americans." 3. "The medications available in the United States decrease goiters." 4. "Iodized salt helps prevent the development of goiters in the United States."

4. Almost all of the iodine entering the body is retained in the thyroid gland. A deficiency in iodine will cause the thyroid gland to work hard and enlarge, which is called a goiter. Goiters are commonly seen in geographical regions having an iodine deficiency. Most table salt in the United States has iodine added.

The client is three (3) days postoperative unilateral adrenalectomy. Which discharge instructions should the nurse teach? 1. Discuss the need for lifelong steroid replacement. 2. Instruct the client on administration of vasopressin. 3. Teach the client to care for the suprapubic Foley catheter. 4. Tell the client to notify the HCP if the incision is inflamed.

4. Any inflammation of the incision indicates an infection

The nurse is admitting a client to rule out aldosteronism. Which assessment data support the client's diagnosis? 1. Temperature. 2. Pulse. 3. Respirations. 4. Blood pressure.

4. Blood pressure is affected by aldosteronism, with hypertension being the most prominent and universal sign of aldosteronism.

Which statement made by the client makes the nurse suspect the client is experiencing hyperthyroidism? 1. "I just don't seem to have any appetite anymore." 2. "I have a bowel movement about every 3 to 4 days." 3. "My skin is really becoming dry and coarse." 4. "I have noticed all my collars are getting tighter."

4. The thyroid gland (in the neck) en larges as a result of the increased need for thyroid hormone production; an enlarged gland is called a goiter.

8

8

A patient with a history of peptic ulcer disease has presented to the emergency department reporting severe abdominal pain and has a rigid, boardlike abdomen that prompts the health care team to suspect a perforated ulcer. What intervention should the nurse anticipate? a) Providing IV fluids and inserting a nasogastric (NG) tube b) Administering oral bicarbonate and testing the patient's gastric pH level c) Performing a fecal occult blood test and administering IV calcium gluconate d) Starting parenteral nutrition and placing the patient in a high-Fowler's position

A A perforated peptic ulcer requires IV replacement of fluid losses and continued gastric aspiration by NG tube. Nothing is given by mouth, and gastric pH testing is not a priority. Calcium gluconate is not a medication directly relevant to the patient's suspected diagnosis, and parenteral nutrition is not a priority in the short term.

The appropriate collaborative therapy for the patient with acute diarrhea caused by a viral infection is to A. increase fluid intake. B. administer an antibiotic. C. administer antimotility drugs. D. quarantine the patient to prevent spread of the virus.

A Rationale Acute diarrhea from infectious causes (e.g., a virus) is usually self-limiting. The major concerns are preventing transmission, fluid and electrolyte replacement, and resolution of the diarrhea. Antidiarrheal agents are contraindicated in the treatment of infectious diarrhea because they potentially prolong exposure to the infectious organism. Antibiotics are rarely used to treat acute diarrhea. To prevent transmission of diarrhea caused by a virus, hand washing should be performed before and after contact with the patient and when body fluids of any kind are handled. Flush vomitus and stool in the toilet, and wash contaminated clothing immediately with soap and hot water. Reference: 1008-1009

Which factor in a patient's history would make the patient at risk for constipation? A. Takes codeine daily for chronic back problem B. Runs a marathon yearly C. Takes an angiotensin-converting enzyme (ACE) inhibitor for hypertension D. Follows a vegan diet

A Rationale Common causes of constipation include insufficient dietary fiber, inadequate fluid intake, decreased physical activity, ignoring the defecation urge, and medications such as opioids. Constipation from opioids is a side effect that does not decrease with time. Training for and running a marathon would help regularity. An ACE inhibitor does not cause constipation. A vegan diet tends to have increased bulk, but it is not a factor as long as the patient has adequate fluids and fiber. Reference: 1012

A patient is suspected of having a large intestine obstruction. What is the best indication that an obstruction is present? A. Lack of flatus B. Nausea C. Temperature of 100.4° F (38° C) D. Thirst

A Rationale Inability to pass gas or constipation is a common manifestation of a large intestinal obstruction. Reference: 1032

You are conducting discharge teaching for a patient with metastatic lung cancer who was admitted with a bowel impaction. Which instruction would be most helpful to prevent further episodes of constipation? A. Maintain a high intake of fluid and fiber in the diet. B. Reduce intake of medications causing constipation. C. Eat several small meals per day to maintain bowel motility. D. Sit upright during meals to increase bowel motility by gravity.

A Rationale Increased fluid intake and a high-fiber diet reduce the incidence of constipation caused by immobility, medications, and other factors. Fluid and fiber provide bulk that in turn increases peristalsis and bowel motility. Reference: 1013

The elderly patient was informed that outpouches were found in the descending colon during the screening colonoscopy. The patient asks you what this finding means. What is the best explanation? A. Most people get these outpouchings as they age. B. These findings respond well to treatment with sulfa antibiotics. C. It is a precursor to colon cancer, and routine screening is essential. D. They contribute to malabsorption of cobalamin (vitamin B12) and fat.

A Rationale It is believed that 65% of people have the saccular dilations or outpouchings of the mucosa by the time they are 85 years old. It is believed to be from high intraluminal pressure on weakened areas of the bowel wall from inadequate dietary fiber. It is typically asymptomatic and not a concern unless inflamed or diverticulitis develops. Antibiotics are not required unless they are inflamed (diverticulitis). Polyps are a precursor to colon cancer; diverticula are not precancerous. Malabsorption of cobalamin (vitamin B12) and fat is seen with disease of the terminal ileum; the sigmoid colon mainly absorbs water. Reference: 1046

A 60-year-old man reports to the clinic nurse that he feels tired. The CBC count shows a hemoglobin level of 9 g/dL. What action should you advocate? A. Fecal occult blood test (FOBT) B. Consuming foods high in iron, such as red meat C. Assessing the patient's stress and sleep schedule D. Encouraging over-the-counter iron supplement

A Rationale Symptoms of colorectal cancer are nonspecific or absent until the disease is advanced. Anemia can be a symptom in an older adult. The cause of the anemia, including screening for colorectal cancer, needs to be done before advocating other treatment. Cancer assessment takes priority over lifestyle assessment. Reference: 1035-1036

What is a classic diagnostic finding in a patient with appendicitis? A. Elevated white blood cell (WBC) count B. Elevated level of lipase C. Left lower quadrant tenderness D. Positive Kernig's sign

A Rationale The WBC count is mildly to moderately elevated in about 90% of cases. The lipase level is elevated in patients with pancreatitis. Left lower quadrant tenderness is typically seen in diverticulitis. The classic location for appendicitis is McBurney's point in the right lower quadrant. Positive Kernig's sign indicates meningitis. Reference: 1020

The patient has fecal incontinence. You are working to promote bowel training. What is the best time to have the patient attempt to defecate every day? A. Thirty minutes after breakfast B. Before going to bed C. Before noon meal D. After performing exercise

A Rationale The gastrocolic reflex is strongest in most people right after breakfast. A good time for many persons to schedule elimination is within 30 minutes after breakfast. The other times are not as conducive; exercise often makes fecal incontinence worse. Reference: 1012

You explain to the patient undergoing ostomy surgery that the procedure that maintains the most normal functioning of the bowel is A. A sigmoid colostomy B. A transverse colostomy C. A descending colostomy D. An ascending colostomy

A Rationale The more distal the ostomy, the more the intestinal contents resemble feces that are eliminated from an intact colon and rectum. Output from a sigmoid colostomy resembles normally formed stool, and some patients are able to regulate emptying time so they do not need to wear a collection bag. Reference: 1039

What is important to teach a patient who is learning to manage a colostomy? A. Change the drainage bag when it is one-third full. B. Cut the skin protective wafer about 1 inch larger than the stoma. C. Poke a hole in the collection bag if it is tight and taunt. D. Maintain sterile procedure during the drainage bag change.

A Rationale The weight of drainage from the stoma pulls the wafer away from the skin so ostomy bags should be emptied when one-third full. The wafer is 1/8 to 1/16 larger than the stoma to prevent drainage on the skin that would cause irritation. Holes are not poked in the bag. A distended bag is full of flatus, which should be expelled by opening the bag. The gastrointestinal system is not sterile, and clean protocol is maintained. Reference: 1042

The nurse is conducting discharge teaching for a patient with metastatic lung cancer who was admitted with a bowel impaction. Which instructions would be most helpful to prevent further episodes of constipation? A. Maintain a high intake of fluid and fiber in the diet. B. Reduce intake of medications causing constipation. C. Eat several small meals per day to maintain bowel motility. D. Sit upright during meals to increase bowel motility by gravity.

A Increased fluid intake and a high-fiber diet reduce the incidence of constipation caused by immobility, medications, and other factors. Fluid and fiber provide bulk that in turn increases peristalsis and bowel motility. Analgesics taken for lung cancer probably cannot be reduced. Other medications may decrease constipation, but it is best to avoid laxatives. Eating several small meals per day and position do not facilitate bowel motility. Defecation is easiest when the person sits on the commode with the knees higher than the hips.

A stroke patient who primarily uses a wheelchair for mobility has diarrhea with fecal incontinence. What should the nurse assess first? A. Fecal impaction B. Perineal hygiene C. Dietary fiber intake D. Antidiarrheal agent use

A Patients with limited mobility are at risk for fecal impactions due to constipation that may lead to liquid stool leaking around the hardened impacted feces, so assessing for fecal impaction is the priority. Perineal hygiene can be assessed at the same time. Assessing the dietary fiber and fluid intake and antidiarrheal agent use will be assessed and considered next.

Following a transsphenoidal resection of a pituitary tumor, an important N assessment is a. monitoring hourly urine output. b. checking the dressings for serous drainage. c. palpating for dependent pitting edema. d. obtaining continuous pulse oximetry.

A R: After pituitary surgery, the pt is at risk for diabetes insipidus caused by cerebral edema and monitoring of urine output and urine specific gravity is essential. There will be no dressing when transsphenoidal approach is used. The pt is at risk for dehydration, not volume overload. The pt is not at high risk for problems with oxygenation, and continuous pulse oximetry is not needed.

RN observes a nursing assistant (NA) caring for a patient after a hypophysectomy. Which action by the NA requires that the RN intervene? a. The NA lowers the HOB to the flat position. b. The NA cautions the patient to avoid coughing. c. The NA cleans the patient's mouth with a swab. d. NA collects a urine specimen for specific gravity

A R: HOB should be elevated about 30 degrees to decrease pressure on the sella turcica and avoid headaches. The other actions by the NA are appropriate after this surgery. (Cognitive Level: Application Text : p. 1293 NProcess: Implementation NCLEX: Safe and Effective Care Enviro)

A patient with symptoms of DI is admitted to the hospital for evaluation and treatment of the condition. An appropriate nursing diagnosis for the patient is a. insomnia related to waking at night to void. b. risk for impaired skin integrity related to generalized edema. c. excess fluid volume related to intake greater than output. d. activity intolerance related to muscle cramps and weakness.

A R: Nocturia occurs as a result of the polyuria caused by diabetes insipidus. Edema will not be expected because dehydration is a concern with polyuria. The pt drinks large amnts of fluid to compensate for losses experienced from diuresis. The pt's fluid and electrolyte status remain normal as long as the patient's oral intake can keep up w fluid losses, muscle cramps and weakness arent concerns.

A patient with hyperthyroidism is treated with radioactive iodine (RAI) at a clinic. Before the patient is discharged, the nurse instructs the pt a. to monitor for symptoms of hypothyroidism, such as easy bruising and cold intolerance. b. to discontinue the antithyroid medications taken before the radioactive therapy. c. that symptoms of hyperthyroidism should be relieved in about a week. d. about radioactive precautions to take with urine, stool, and other body secretions.

A R: There is a high incidence of post-radiation hypothyroidism after RAI, and the pt should be monitored for symptoms of hypothyroidism. RAI has a delayed response, with maximum effect not seen for 2-3 months, and pt will continue to take antithyroid medications during this time. The therapeutic dose of radioactive iodine is low enough that no radiation safety precautions are needed.

A patient is hospitalized with possible SIADH. The patient is confused and reports a headache, muscle cramps, and twitching. The nurse would expect the initial laboratory results to include a a. serum sodium of 125 mEq/L (125 mmol/L). b. hematocrit of 52%. c. blood urea nitrogen (BUN) of 22 mg/dl (11.5 mmol/L). d. serum chloride of 110 mEq/L (110 mmol/L).

A R: When water is retained, the serum sodium level will drop below normal, causing the CMs reported by the patient. The hematocrit will decrease because of the dilution caused by water retention. The BUN is not helpful in diagnosis of SIADH and this BUN value is increased. The serum chloride level will usually decrease along with the sodium level. This chloride value is elevated.

A 63-year-old patient is newly diagnosed with type 2 diabetes. When developing an education plan, the nurse's first action should be to a. assess the patient's perception of what it means to have type 2 diabetes. b. demonstrate how to check glucose using capillary blood glucose monitoring. c. ask the patient's family to participate in the diabetes education program. d. discuss the need for the patient to actively participate in diabetes management.

A Rationale: Before planning education, the nurse should assess the patient's interest in and ability to self-manage the diabetes. After assessing the patient, the other nursing actions may be appropriate, but planning needs to be individualized to each patient. Cognitive Level: Application Text Reference: p.1264 Nursing Process: Planning NCLEX: Health Promotion and Maintenance

A patient with type 2 diabetes that is controlled with diet and metformin (Glucophage) also has severe rheumatoid arthritis (RA). During an acute exacerbation of the patient's arthritis, the health care provider prescribes prednisone (Deltasone) to control inflammation. The nurse will anticipate that the patient may a. require administration of insulin while taking prednisone. b. develop acute hypoglycemia during the RA exacerbation. c. have rashes caused by metformin-prednisone interactions. d. need a diet higher in calories while receiving prednisone.

A Rationale: Glucose levels increase when patients are taking corticosteroids, and insulin may be required to control blood glucose. Hypoglycemia is not a complication of RA exacerbation or prednisone use. Rashes are not an adverse effect caused by taking metformin and prednisone simultaneously. The patient is likely to have an increased appetite when taking prednisone, but it will be important to avoid weight gain for the patient with RA. Cognitive Level: Application Text Reference: pp. 1258, 1267 Nursing Process: Planning NCLEX: Physiological Integrity

After the home health nurse has taught a patient and family about how to use glargine and regular insulin safely, which action by the patient indicates that the teaching has been successful? a. The patient disposes of the open insulin vials after 4 weeks. b. The patient draws up the regular insulin in the syringe and then draws up the glargine. c. The patient stores extra vials of both types of insulin in the freezer until needed. d. The patient's family prefills the syringes weekly and stores them in the refrigerator.

A Rationale: Insulin can be stored at room temperature for 4 weeks. Glargine should not be mixed with other insulins or prefilled and stored. Freezing alters the insulin molecule and should not be done. Cognitive Level: Application Text Reference: p. 1261 Nursing Process: Evaluation NCLEX: Physiological Integrity

A patient using a split mixed-dose insulin regimen asks the nurse about the use of intensive insulin therapy to achieve tighter glucose control. The nurse should teach the patient that a. intensive insulin therapy requires three or more injections a day in addition to an injection of a basal long-acting insulin. b. intensive insulin therapy is indicated only for newly diagnosed type 1 diabetics who have never experienced ketoacidosis. c. studies have shown that intensive insulin therapy is most effective in preventing the macrovascular complications characteristic of type 2 diabetes. d. an insulin pump provides the best glucose control and requires about the same amount of attention as intensive insulin therapy.

A Rationale: Patients using intensive insulin therapy must check their glucose level four to six times daily and administer insulin accordingly. A previous episode of ketoacidosis is not a contraindication for intensive insulin therapy. Intensive insulin therapy is not confined to type 2 diabetics and would prevent microvascular changes as well as macrovascular changes. Intensive insulin therapy and an insulin pump are comparable in glucose control. Cognitive Level: Application Text Reference: p. 1263 Nursing Process: Implementation NCLEX: Physiological Integrity

When teaching a patient with chronic SIADH about long-term management of the disorder, the nurse determines that additional instruction is needed when the patient says, a. "I need to shop for foods that are low in sodium and avoid adding salt to foods." b. "I should weigh myself daily and report any sudden weight loss or gain." c. "I need to limit my fluid intake to no more than 1 quart of liquids a day." d. "I will eat foods high in potassium because the diuretics cause potassium loss."

A Rationale: Pts with SIADH are at risk for hyponatremia, and a sodium supplement may be prescribed. The other pt statements are correct and indicate successful teaching has occurred.

Intramuscular glucagon is administered to an unresponsive patient for treatment of hypoglycemia. Which action should the nurse take after the patient regains consciousness? a. Give the patient a snack of cheese and crackers. b. Have the patient drink a glass of orange juice or nonfat milk. c. Administer a continuous infusion of 5% dextrose for 24 hours. d. Assess the patient for symptoms of hyperglycemia.

A Rationale: Rebound hypoglycemia can occur after glucagon administration, but having a meal containing complex carbohydrates plus protein and fat will help prevent hypoglycemia. Orange juice and nonfat milk will elevate blood sugar rapidly, but the cheese and crackers will stabilize blood sugar. Administration of glucose intravenously might be used in patients who were unable to take in nutrition orally. The patient should be assessed for symptoms of hypoglycemia after glucagon administration. Cognitive Level: Application Text Reference: p. 1282 Nursing Process: Implementation NCLEX: Physiological Integrity

The nurse has been teaching the patient to administer a dose of 10 units of regular insulin and 28 units of NPH insulin. The statement by the patient that indicates a need for additional instruction is, a. "I need to rotate injection sites among my arms, legs, and abdomen each day." b. "I will buy the 0.5-ml syringes because the line markings will be easier to see." c. "I should draw up the regular insulin first after injecting air into the NPH bottle." d. "I do not need to aspirate the plunger to check for blood before I inject the insulin."

A Rationale: Rotating sites is no longer necessary because all insulin is now purified human insulin, and the risk for lipodystrophy is low. The other patient statements are accurate and indicate that no additional instruction is needed. Cognitive Level: Application Text Reference: p. 1262 Nursing Process: Evaluation NCLEX: Health Promotion and Maintenance

When teaching a patient with type 2 diabetes about taking glipizide (Glucotrol), the nurse determines that additional teaching about the medication is needed when the patient says, a. "Since I can take oral drugs rather than insulin, my diabetes is not serious and won't cause many complications." b. "If I overeat at a meal, I will still take just the usual dose of medication." c. "If I become ill, I may have to take insulin to control my blood sugar." d. "I should check with my doctor before taking any other medications because there are many that will affect glucose levels."

A Rationale: The patient should understand that type 2 diabetes places the patient at risk for many complications and that good glucose control is as important when taking oral agents as when using insulin. The other statements are accurate and indicate good understanding of the use of glipizide. Cognitive Level: Application Text Reference: p. 1275 Nursing Process: Evaluation NCLEX: Health Promotion and Maintenance

While hospitalized and recovering from an episode of diabetic ketoacidosis, the patient calls the nurse and reports feeling anxious, nervous, and sweaty. Based on the patient's report, the nurse should a. obtain a glucose reading using a finger stick. b. administer 1 mg glucagon subcutaneously. c. have the patient eat a candy bar. d. have the patient drink 4 ounces of orange juice.

A Rationale: The patient's clinical manifestations are consistent with hypoglycemia and the initial action should be to check the patient's glucose with a finger stick or order a stat blood glucose. If the glucose is low, the patient should ingest a rapid-acting carbohydrate, such as orange juice. Glucagon might be given if the patient's symptoms become worse or if the patient is unconscious. Candy bars contain fat, which would slow down the absorption of sugar and delay the response to treatment. Cognitive Level: Application Text Reference: p. 1282 Nursing Process: Implementation NCLEX: Physiological Integrity

Glyburide (Micronase, DiaBeta, Glynase) is prescribed for a patient whose type 2 diabetes has not been controlled with diet and exercise. When teaching the patient about glyburide, the nurse explains that a. glyburide stimulates insulin production and release from the pancreas. b. the patient should not take glyburide for 48 hours after receiving IV contrast media. c. glyburide should be taken even when the blood glucose level is low in the morning. d. glyburide decreases glucagon secretion.

A Rationale: The sulfonylureas stimulate the production and release of insulin from the pancreas. If the glucose level is low, the patient should contact the health care provider before taking the glyburide, since hypoglycemia can occur with this category of medication. Metformin should be held for 48 hours after administration of IV contract, but this is not necessary for glyburide. Cognitive Level: Application Text Reference: pp. 1265-1266 Nursing Process: Implementation NCLEX: Physiological Integrity

Amitriptyline (Elavil) is prescribed for a diabetic patient with peripheral neuropathy who has burning foot pain occurring mostly at night. Which information should the nurse include when teaching the patient about the new medication? a. Amitriptyline will help prevent the transmission of pain impulses to the brain. b. Amitriptyline will improve sleep and make you less aware of nighttime pain. c. Amitriptyline will decrease the depression caused by the pain. d. Amitriptyline will correct some of the blood vessel changes that cause pain.

A Rationale: Tricyclic antidepressants decrease the transmission of pain impulses to the spinal cord and brain. Tricyclics also improve sleep quality and are used for depression, but that is not the major purpose for their use in diabetic neuropathy. The blood vessel changes that contribute to neuropathy are not affected by tricyclics. Cognitive Level: Application Text Reference: p. 1285 Nursing Process: Implementation NCLEX: Physiological Integrity

When assessing the patient experiencing the onset of symptoms of type 1 diabetes, which question should the nurse ask? a. "Have you lost any weight lately?" b. "Do you crave fluids containing sugar?" c. "How long have you felt anorexic?" d. "Is your urine unusually dark-colored?"

A Rationale: Weight loss occurs because the body is no longer able to absorb glucose and starts to break down protein and fat for energy. The patient is thirsty but does not necessarily crave sugar- containing fluids. Increased appetite is a classic symptom of type 1 diabetes. With the classic symptom of polyuria, urine will be very dilute. Cognitive Level: Application Text Reference: pp. 1255, 1258 Nursing Process: Assessment NCLEX: Physiological Integrity

The nurse is preparing to insert a nasogastric (NG) tube into a 68-year-old female patient who is nauseated and vomiting. She has an abdominal mass and suspected small intestinal obstruction. The patient asks the nurse why this procedure is necessary. What response by the nurse is most appropriate? A. "The tube will help to drain the stomach contents and prevent further vomiting." B. "The tube will push past the area that is blocked and thus help to stop the vomiting." C. "The tube is just a standard procedure before many types of surgery to the abdomen." D. "The tube will let us measure your stomach contents so that we can plan what type of IV fluid replacement would be best."

A The NG tube is used to decompress the stomach by draining stomach contents and thereby prevent further vomiting. The NG tube will not push past the blocked area. Potential surgery is not currently indicated. The location of the obstruction will determine the type of fluid to use, not measure the amount of stomach contents

When providing discharge teaching for the patient after a laparoscopic cholecystectomy, what information should the nurse include? A lower-fat diet may be better tolerated for several weeks. Do not return to work or normal activities for 3 weeks. Bile-colored drainage will probably drain from the incision. Keep the bandages on and the puncture site dry until it heals.

A - A lower-fat diet may be better tolerated for several weeks. Correct Although the usual diet can be resumed, a low-fat diet is usually better tolerated for several weeks following surgery. Normal activities can be gradually resumed as the patient tolerates. Bile-colored drainage or pus, redness, swelling, severe pain, and fever may all indicate infection. The bandage may be removed the day after surgery, and the patient can shower.

The nurse is caring for a group of patients. Which patient is at highest risk for pancreatic cancer? A 38-year-old Hispanic female who is obese and has hyperinsulinemia A 23-year-old who has cystic fibrosis-related pancreatic enzyme insufficiency A 72-year-old African American male who has smoked cigarettes for 50 years A 19-year-old who has a 5-year history of uncontrolled type 1 diabetes mellitus

A 72-year-old African American male who has smoked cigarettes for 50 years Risk factors for pancreatic cancer include chronic pancreatitis, diabetes mellitus, age, cigarette smoking, family history of pancreatic cancer, high-fat diet, and exposure to chemicals such as benzidine. African Americans have a higher incidence of pancreatic cancer than whites. The most firmly established environmental risk factor is cigarette smoking. Smokers are two or three times more likely to develop pancreatic cancer as compared with nonsmokers. The risk is related to duration and number of cigarettes smoked.

The nurse is caring for a woman recently diagnosed with viral hepatitis A. Which individual should the nurse refer for an immunoglobin (IG) injection? A caregiver who lives in the same household with the patient A friend who delivers meals to the patient and family each week A relative with a history of hepatitis A who visits the patient daily A child living in the home who received the hepatitis A vaccine 3 months ago

A caregiver who lives in the same household with the patient IG is recommended for persons who do not have anti-HAV antibodies and are exposed as a result of close contact with persons who have HAV or foodborne exposure. Persons who have received a dose of HAV vaccine more than 1 month previously or who have a history of HAV infection do not require IG.

A patient is hospitalized with possible syndrome of inappropriate antidiuretic hormone (SIADH). The patient is confused and reports a headache, muscle cramps, and twitching. The nurse would expect the initial laboratory results to include A. An elevated hematocrit. B. A decreased serum sodium. C. An increased serum chloride. D. A low urine specific gravity

A decreased serum sodium. Feedback When water is retained, the serum sodium level will drop below normal, causing the clinical manifestations reported by the patient. The hematocrit will decrease because of the dilution caused by water retention. Urine will be more concentrated with a higher specific gravity. The serum chloride level will usually decrease along with the sodium level.

The client is admitted to the intensive care department diagnosed with myxedema coma. Which assessment data warrant immediate intervention by the nurse? 1. Serum blood glucose level of 74 mg/dL. 2. Pulse oximeter reading of 90%. 3. Telemetry reading showing sinus bradycardia. 4. The client is lethargic and sleeps all the time.

A pulse oximeter reading of less than 93% is significant.

As a precaution for vocal cord paralysis from damage to the recurrent laryngeal nerve during thyroidectomy surgery, what equipment should be in the room in case it is needed for this emergency situation? a. Tracheostomy tray b. Oxygen equipment c. IV calcium gluconate d. Paper and pencil for communication

A tracheostomy tray is in the room to use if vocal cord paralysis occurs from recurrent laryngeal nerve damage or for laryngeal stridor from tetany. The oxygen equipment may be useful but will not improve oxygenation with vocal cord paralysis without a tracheostomy. IV calcium salts will be used if hypocalcemia occurs from parathyroid damage. The paper and pencil for communication may be helpful, especially if a tracheostomy is performed, but will not aid in emergency oxygenation of the patient.

Which nursing action will be included in the postoperative plan of care for a patient who has had a transsphenoidal resection of a pituitary tumor? A. Monitor urine output every hour. B. Palpate extremities for dependent edema. C. Check hematocrit hourly for first 12 hours. D. Obtain continuous pulse oximetry for 24 hours.

A. Monitor urine output every hour

The nurse is assigned to the care of a 64-year-old patient diagnosed with type 2 diabetes. In formulating a teaching plan that encourages the patient to actively participate in management of the diabetes, what should be the nurse's initial intervention? A) Assess patient's perception of what it means to have diabetes. B) Ask the patient to write down current knowledge about diabetes. C) Set goals for the patient to actively participate in managing his diabetes. D) Assume responsibility for all of the patient's care to decrease stress level.

A) Assess patient's perception of what it means to have diabetes. In order for teaching to be effective, the first step is to assess the patient. Teaching can be individualized once the nurse is aware of what a diagnosis of diabetes means to the patient. After the initial assessment, current knowledge can be assessed, and goals should be set with the patient.

The nurse is assisting a patient with newly diagnosed type 2 diabetes to learn dietary planning as part of the initial management of diabetes. The nurse would encourage the patient to limit intake of which foods to help reduce the percent of fat in the diet? A) Cheese B) Broccoli C) Chicken D) Oranges

A) Cheese Cheese is a product derived from animal sources and is higher in fat and calories than vegetables, fruit, and poultry. Excess fat in the diet is limited to help avoid macrovascular changes.

The nurse has been teaching a patient with diabetes mellitus how to perform self-monitoring of blood glucose (SMBG). During evaluation of the patient's technique, the nurse identifies a need for additional teaching when the patient does what? A) Chooses a puncture site in the center of the finger pad. B) Washes hands with soap and water to cleanse the site to be used. C) Warms the finger before puncturing the finger to obtain a drop of blood. D) Tells the nurse that the result of 110 mg/dL indicates good control of diabetes.

A) Chooses a puncture site in the center of the finger pad. The patient should select a site on the sides of the fingertips, not on the center of the finger pad as this area contains many nerve endings and would be unnecessarily painful. Washing hands, warming the finger, and knowing the results that indicate good control all show understanding of the teaching.

A female patient reports that she is experiencing burning on urination, frequency, and urgency. The nurse notes that a clean-voided urine specimen is markedly cloudy. The probable cause of these symptoms and findings is: A) Cystitis. B) Hematuria. C) Pyelonephritis. D) Dysuria.

A) Cystitis.

After administering a dose of promethazine (Phenergan) to a patient with nausea and vomiting, the nurse explains that which of the following may be experienced as a common temporary adverse effect of the medication? A) Drowsiness B) Reduced hearing C) Sensation of falling D) Photosensitivity

A) Drowsiness (Although being given to this patient as an antiemetic, promethazine also has sedative and amnesic properties. For this reason, the patient is likely to experience drowsiness as an adverse effect of the medication.)

The nurse is evaluating a 45-year-old patient diagnosed with type 2 diabetes mellitus. Which symptom reported by the patient is considered one of the classic clinical manifestations of diabetes? A) Excessive thirst B) Gradual weight gain C) Overwhelming fatigue D) Recurrent blurred vision

A) Excessive thirst The classic symptoms of diabetes are polydipsia (excessive thirst), polyuria, (excessive urine output), and polyphagia (increased hunger). Weight gain, fatigue, and blurred vision may all occur with type 2 diabetes, but are NOT classic manifestations.

Elimination changes that result from inability of the bladder to empty properly may cause which of the following? (Select all that apply.) A) Incontinence B) Frequency C) Urgency D) Urinary retention E) Urinary tract infection

A) Incontinence B) Frequency C) Urgency D) Urinary retention E) Urinary tract infection ---- Any condition resulting in urinary retention increases the risk for urinary tract infection. As retention progresses, retention with overflow develops. Pressure in the bladder builds to a point at which the external urethral sphincter is unable to hold back urine. With retention the patient may void small amounts of urine 2 to 3 times an hour and have urgency. He or she may continually dribble urine. Urinary retention results from inability of the bladder to empty.

Laboratory results have been obtained for a 50-year-old patient with a 15-year history of type 2 diabetes. Which result reflects the expected pattern accompanying macrovascular disease as a complication of diabetes? A) Increased triglyceride levels B) Increased high-density lipoproteins (HDL) C) Decreased low-density lipoproteins (LDL) D) Decreased very-low-density lipoproteins (VLDL)

A) Increased triglyceride levels Macrovascular complications of diabetes include changes to large- and medium-sized blood vessels. They include cerebrovascular, cardiovascular, and peripheral vascular disease. Increased triglyceride levels are associated with these macrovascular changes. Increased HDL, decreased LDL, and decreased VLDL are positive in relation to atherosclerosis development.

A pt with a history of peptic ulcer disease has presented to the emergency department with complaints of severe abdominal pain and a rigid, boardlike abdomen, prompting the health care team to suspect a perforated ulcer. Which of the following actions should the nurse anticipate? A) Providing IV fluids and inserting a nasogastric tube B) Administering oral bicarbonate and testing the patient's gastric pH level C) Performing a fecal occult blood test and administering IV calcium gluconate D) Starting parenteral nutrition and placing the patient in a high-Fowler's position

A) Providing IV fluids and inserting a nasogastric tube A perforated peptic ulcer requires IV replacement of fluid losses and continued gastric aspiration by NG tube. Nothing is given by mouth and gastric pH testing is not a priority. Calcium gluconate is not a medication directly relevant to the patient's suspected diagnosis and parenteral nutrition is not a priority in the short term.

A patient is admitted with diabetes mellitus, malnutrition, and cellulitis. The patient's potassium level is 5.6 mEq/L. The nurse understands that what could be contributing factors for this laboratory result (select all that apply)? A) The level may be increased as a result of dehydration that accompanies hyperglycemia. B) The patient may be excreting extra sodium and retaining potassium because of malnutrition. C) The level is consistent with renal insufficiency that can develop with renal nephropathy. D) The level may be raised as a result of metabolic ketoacidosis caused by hyperglycemia. E) This level demonstrates adequate treatment of the cellulitis and effective serum glucose control.

A) The level may be increased as a result of dehydration that accompanies hyperglycemia. C) The level is consistent with renal insufficiency that can develop with renal nephropathy. E) This level demonstrates adequate treatment of the cellulitis and effective serum glucose control. The additional stress of cellulitis may lead to an increase in the patient's serum glucose levels. Dehydration may cause hemoconcentration, resulting in elevated serum readings. Kidneys may have difficulty excreting potassium if renal insufficiency exists. Finally, the nurse must consider the potential for metabolic ketoacidosis since potassium will leave the cell when hydrogen enters in an attempt to compensate for a low pH.

A patient with type 2 diabetes and cirrhosis asks the nurse if it would be okay to take silymarin (milk thistle) to help minimize liver damage. The nurse responds based on knowledge that: A. Milk thistle may affect liver enzymes and thus alter drug metabolism. B. Milk thistle is generally safe in recommended doses for up to 10 years. C. is unclear scientific evidence for the use of milk thistle in treating cirrhosis. D. Milk thistle may elevate the serum glucose levels and is thus contraindicated in diabetes.

A) There is good scientific evidence that there is no real benefit from using milk thistle to protect the liver cells from toxic damage in the treatment of cirrhosis. Milk thistle does affect liver enzymes and thus could alter drug metabolism. Therefore patients will need to be monitored for drug interactions. It is noted to be safe for up to 6 years, not 10 years, and it may lower, not elevate, blood glucose levels.

Select the results (in italics) that are normal in a urinalysis. A. pH 6 B. Specific gravity 1.015 C. Protein small D. Sugar negative E. Nitrate small F. Leukocyte esterase positive G. Bilirubin negative

A, B, D, G Rationale: The abnormal values are indicative of a urinary tract infection. As a result of protein, nitrates, and leukoesterase in the urine, the nurse can expect the laboratory to analyze microscopic sediment including evaluating the sample for the presence of crystals, casts, WBCs, and RBCs.

You perform a detailed assessment of the abdomen of a patient with a possible bowel obstruction, knowing that manifestations of an obstruction in the large intestine include (select all that apply) A. a largely distended abdomen. B. diarrhea that is loose or liquid. C. persistent, colicky abdominal pain. D. profuse vomiting that relieves abdominal pain.

A, C Rationale Persistent, colicky abdominal pain occurs with lower intestinal obstruction. Abdominal distention is markedly increased in lower intestinal obstructions. Onset of a large intestine obstruction is gradual, and vomiting is rare. Absolute constipation usually is associated with large intestine obstructions. Reference: 1033

When caring for a patient with a biliary obstruction, the nurse will anticipate administering which of the following vitamin supplements (select all that apply)? A. Vitamin A B. Vitamin D C. Vitamin E D. Vitamin K E. Vitamin B

A,B,C,D) Biliary obstruction prevents bile from entering the small intestine and thus prevents the absorption of fat-soluble vitamins. Vitamins A, D, E, and K are all fat-soluble and thus would need to be supplemented in a patient with biliary obstruction.

When caring for a patient with liver disease, the nurse recognizes the need to prevent bleeding resulting from altered clotting factors and rupture of varices. Which of the following nursing interventions would be appropriate to achieve this outcome (select all that apply)? A. Use smallest gauge possible when giving injections or drawing blood. B. Teach patient to avoid straining at stool, vigorous blowing of nose, and coughing. C. Advise patient to use soft-bristle toothbrush and avoid ingestion of irritating food. D. Apply gentle pressure for the shortest possible time period after performing venipuncture. E. Instruct patient to avoid aspirin and NSAIDs to prevent hemorrhage when varices are present.

A,B,C,E) Using the smallest gauge needle for injections will minimize the risk of bleeding into the tissues. Avoiding straining, nose blowing, and coughing will reduce the risk of hemorrhage at these sites. The use of a soft-bristle toothbrush and avoidance of irritating food will reduce injury to highly vascular mucous membranes. The nurse should apply gentle but prolonged pressure to venipuncture sites to minimize the risk of bleeding. Aspirin and NSAIDs should not be used in patients with liver disease because they interfere with platelet aggregation, thus increasing the risk for bleeding.

The nurse instructs a 28-year-old man with acromegaly resulting from an unresectable benign pituitary tumor about octreotide (Sandostatin). The nurse should intervene if the patient makes which statement? A. "I will come in to receive this medication IV every 2 to 4 weeks." B. "I will inject the medication in the subcutaneous layer of the skin." C. "The medication will decrease the growth hormone production to normal." D. "If radiation treatment is not effective, I may need to take the medication."

A. "I will come in to receive this medication IV every 2 to 4 weeks Drugs are most commonly used in patients who have had an inadequate response to or cannot be treated with surgery and/or radiation therapy. The most common drug used for acromegaly is octreotide (Sandostatin), a somatostatin analog that reduces growth hormone levels to within the normal range in many patients. Octreotide is given by subcutaneous injection three times a week. Two long-acting analogs, octreotide (Sandostatin LAR) and lanreotide SR (Somatuline Depot), are available as intramuscular (IM) injections given every 2 to 4 weeks.

Which statement by the nurse regarding continuous ambulatory peritoneal dialysis (CAPD) would be of highest priority when teaching a patient new to this procedure? A. "It is essential that you maintain aseptic technique to prevent peritonitis." B. "You will be allowed a more liberal protein diet once you complete CAPD." C. "It is important for you to maintain a daily written record of blood pressure and weight." D. "You will need to continue regular medical and nursing follow-up visits while performing CAPD."

A. "It is essential that you maintain aseptic technique to prevent peritonitis." Peritonitis is a potentially fatal complication of peritoneal dialysis, and thus it is imperative to teach the patient methods of preventing this from occurring. Although the other teaching statements are accurate, they do not have the potential for morbidity and mortality as does peritonitis, thus making that statement of highest priority.

what is hte effect of finasteride (proscar) in the treatment of BPH a. a reduction in the size of the prostate gland b. relaxation of the smooth muscle and the urethra c. increased bladder tone that promotes bladder emptying d. relaxation of the bladder detrusor muscle promoting urine flow

a. a reduction in the size of the prostate gland

A patient with a history of recurrent urinary tract infections has been scheduled for a cystoscopy. What teaching point should the nurse emphasize before the procedure? A. "You might have pink-tinged urine and burning after your cystoscopy." B. "You'll need to refrain from eating or drinking after midnight the day before the test." C. "You'll require a urinary catheter inserted before the cystoscopy, and it will be in place for a few days." D. "The morning of the test, the nurse will ask you to drink some water that contains a contrast solution."

A. "You might have pink-tinged urine and burning after your cystoscopy." Pink-tinged urine, burning, and frequency are common following a cystoscopy. The patient does not need to be NPO prior to the test, and contrast media is not needed. A cystoscopy does not always necessitate catheterization before or after the procedure.

The nurse performs a physical assessment on a 74-year-old woman with possible endocrine dysfunction. The patient's weight was 142 pounds 6 months ago compared to a current weight of 125 pounds. What percent weight change will the nurse document in the patient's health record? A. 12% weight loss B. 17% weight loss C. 25% weight loss D. 74% weight loss

A. 12% weight loss 142 pounds - 125 pounds = 17 pounds; (17/142) × 100 = 12%. Weight change (%) is calculated by dividing the current body weight change by the usual body weight and multiplying the result by 100. Weight change greater than 5% in 1 month, 7.5% in 3 months, or 10% in 6 months is considered severe.

The nurse is teaching clinic patients about risk factors for testicular cancer. Which individual is at highest risk for developing testicular cancer? A. A 30-year-old white male with a history of cryptorchidism B. A 48-year-old African American male with erectile dysfunction C. A 19-year-old Asian male who had surgery for testicular torsion D. A 28-year-old Hispanic male with infertility caused by a varicocele

A. A 30-year-old white male with a history of cryptorchidism The incidence of testicular cancer is four times higher in white males than in African American males. Testicular tumors are also more common in males who have had undescended testes (cryptorchidism) or a family history of testicular cancer or anomalies. Other predisposing factors include orchitis, human immunodeficiency virus infection, maternal exposure to DES, and testicular cancer in the contralateral testis.

Which nursing diagnosis is a priority in the care of a patient with renal calculi? A. Acute pain B. Risk for constipation C. Deficient fluid volume D. Risk for powerlessness

A. Acute Pain Urinary stones are associated with severe abdominal or flank pain. Deficient fluid volume is unlikely to result from urinary stones, whereas constipation is more likely to be an indirect consequence rather than a primary clinical manifestation of the problem. The presence of pain supersedes powerlessness as an immediate focus of nursing care.

What should the nurse expect to do to prepare a patient for an intravenous pyelogram (IVP)? A. Administer a cathartic or enema. B. Assess patient for allergies to penicillin. C. Keep the patient NPO for 4 hours preprocedure. D. Advise the patient that a metallic taste may occur during procedure.

A. Administer a cathartic or enema. Nursing responsibilities in caring for a patient undergoing an IVP include administration of a cathartic or enema to empty the colon of feces and gas. The nurse will also assess the patient for iodine sensitivity, keep the patient NPO for 8 hours preprocedure, and advise the patient that warmth, a flushed face, and a salty taste during injection of contrast material may occur.

A 78-year-old patient has Stage 3 CKD and is being taught about a low potassium diet. The nurse knows the patient understands the diet when the patient selects which foods to eat? A. Apple, green beans, and a roast beef sandwich B. Granola made with dried fruits, nuts, and seeds C. Watermelon and ice cream with chocolate sauce D. Bran cereal with ½ banana and milk and orange juice

A. Apple, green beans, and a roast beef sandwich When the patient selects an apple, green beans, and a roast beef sandwich, the patient demonstrates understanding of the low potassium diet. Granola, dried fruits, nuts and seeds, milk products, chocolate sauce, bran cereal, banana, and orange juice all have elevated levels of potassium, at or above 200 mg per 1/2 cup.

A patient scheduled for a prostatectomy for prostate cancer expresses the fear that he will have erectile dysfunction. In responding to this patient, the nurse should keep in mind that? A. Erectile dysfunction can occur even with a nerve-sparing procedure B. Retrograde ejaculation affects sexual function more frequently than erectile dysfunction C. The most common complication of this surgery is postoperative bowel incontinence D. Preoperative sexual function is the most important factor in determining postoperative erectile dysfunction

A. Erectile dysfunction can occur even with a nerve-sparing procedure

The nurse is caring for a 68-year-old woman after a parathyroidectomy related to hyperparathyroidism. The nurse should administer IV calcium gluconate if the patient exhibits which clinical manifestations? A. Facial muscle spasms or laryngospasms B. Decreased muscle tone or muscle weakness C. Tingling in the hands and around the mouth D. Shortened QT interval on the electrocardiogram

A. Facial muscle spasms or laryngospasms Nursing care for the patient following a parathyroidectomy includes monitoring for a sudden decrease in serum calcium levels causing tetany, a condition of neuromuscular hyperexcitability. If tetany is severe (e.g., muscular spasms or laryngospasms develop), IV calcium gluconate should be administered. Mild tetany, characterized by unpleasant tingling of the hands and around the mouth, may be present but should decrease over time without treatment. Decreased muscle tone, muscle weakness, and shortened QT interval are clinical manifestations of hyperparathyroidism.

What is the nurse's priority when changing the appliance of a patient with an ileal conduit? A. Keep the skin free of urine. B. Inspect the peristomal area. C. Cleanse and dry the area gently. D. Affix the appliance to the faceplate.

A. Keep the skin free of urine The nurse's priority is to keep the skin free of urine because the peristomal skin is at high risk for damage from the urine if it is alkaline. The peristomal area will be assessed; the area will be gently cleaned and dried, and the appliance will be affixed to the faceplate if one is being used, but these are not as much of a priority as keeping the skin free of urine to prevent skin damage.

Eight months after the delivery of her first child, a 31-year-old woman has sought care because of occasional incontinence that she experiences when sneezing or laughing. Which measure should the nurse first recommend in an attempt to resolve the woman's incontinence? A. Kegel exercises B. Use of adult incontinence pads C. Intermittent self-catheterization D. Dietary changes including fluid restriction

A. Kegel exercises Patients who experience stress incontinence frequently benefit from Kegel exercises (pelvic floor muscle exercises). The use of incontinence pads does not resolve the problem, and intermittent self-catheterization would be a premature recommendation. Dietary changes are not likely to influence the patient's urinary continence

The urinalysis of a male patient reveals a high microorganism count. What data should the nurse use to determine the area of the urinary tract that is infected (select all that apply)? A. Pain location B. Fever and chills C. Mental confusion D. Urinary hesitancy E. Urethral discharge F. Post-void dribbling

A. Pain location E. Urethral discharge Although all the manifestations are evident with urinary tract infections (UTIs), pain location is useful in differentiating among pyelonephritis, cystitis, and urethritis because flank pain is characteristic of pyelonephritis, but dysuria is characteristic of cystitis and urethritis. Urethral discharge is indicative of urethritis, not pyelonephritis or cystitis. Fever and chills and mental confusion are nonspecific indicators of UTIs. Urinary hesitancy and postvoid dribbling may occur with a UTI but may also occur with prostate enlargement in the male patient.

A patient is to receive methylprednisolone (Solu-Medrol) 100 mg. The label on the medication states: methylprednisolone 125 mg in 2 mL. How many milliliters will the nurse administer?

ANS: 1.6 A concentration of 125 mg in 2 mL will result in 100 mg in 1.6 mL.

Vasopressin (Pitressin) 0.2 units/min infusion is prescribed for a patient with acute arterial gastrointestinal (GI) bleeding. The vasopressin label states vasopressin 100 units/250 mL normal saline. How many mL/hr will the nurse infuse?

ANS: 30 There are 0.4 units/1 mL. An infusion of 30 mL/hr will result in the patient receiving 0.2 units/min as prescribed.

While caring for a comatose patient who is receiving continuous enteral nutrition through a soft nasogastric tube, the nurse notes the presence of new crackles in the patient's lungs. In which order will the nurse take the following actions? Put a comma and space between each answer choice (a, b, c, d, etc.) ____________________ a. Turn off the tube feeding. b. Obtain the patient's oxygen saturation. c. Check the tube feeding residual volume. d. Notify the patient's health care provider.

ANS: A, B, C, D The assessment data indicate that aspiration may have occurred, and the nurse's first action should be to turn off the tube feeding to avoid further aspiration. The next action should be to check the oxygen saturation because this may indicate the need for immediate respiratory suctioning or oxygen administration. The residual volume should be obtained because it provides data about possible causes of aspiration. Finally, the health care provider should be notified and informed of all the assessment data the nurse has just obtained.

In which order will the nurse take these steps to prepare NPH 20 units and regular insulin 2 units using the same syringe? (Put a comma and a space between each answer choice [A, B, C, D, E]). a. Rotate NPH vial. b. Withdraw regular insulin. c. Withdraw 20 units of NPH. d. Inject 20 units of air into NPH vial. e. Inject 2 units of air into regular insulin vial

ANS: A, D, E, B, C When mixing regular insulin with NPH, it is important to avoid contact between the regular insulin and the additives in the NPH that slow the onset, peak, and duration of activity in the longer-acting insulin.

In which order will the nurse take the following actions when caring for a patient who develops watery diarrhea and a fever after prolonged omeprazole (Prilosec) therapy? (Put a comma and a space between each answer choice [A, B, C, D].) a. Contact the health care provider. b. Assess blood pressure and heart rate. c. Give the PRN acetaminophen (Tylenol). d. Place the patient on contact precautions.

ANS: D, B, A, C Proton pump inhibitors including omeprazole (Prilosec) may increase the risk of Clostridium difficile-associated colitis. Because the patient's history and symptoms are consistent with C. difficile infection, the initial action should be initiation of infection control measures to protect other patients. Assessment of blood pressure and pulse is needed to determine whether the patient has symptoms of hypovolemia and/or shock. The health care provider should be notified so that actions such as obtaining stool specimens and antibiotic therapy can be started. Tylenol may be administered, but is the lowest priority of the actions.

Which observation by the nurse best indicates that bladder irrigation for urinary retention has been effective? a. Recording an output that is larger than the amount instilled b. Presence of blood clots or sediment in the drainage bag c. Reduction in discomfort from bladder distention d. Visualizing clear urinary catheter tubing

ANS: A Recording an output that is greater than what was irrigated into the bladder shows progress that the bladder is draining urine. The other observations do not objectively measure the increase in urine output.

Which menu choice by the patient with diverticulosis is best for preventing diverticulitis? a. Navy bean soup and vegetable salad b. Whole grain pasta with tomato sauce c. Baked potato with low-fat sour cream d. Roast beef sandwich on whole wheat bread

ANS: A A diet high in fiber and low in fats and red meat is recommended to prevent diverticulitis. Although all of the choices have some fiber, the bean soup and salad will be the highest in fiber and the lowest in fat

The nurse admitting a patient with acute diverticulitis explains that the initial plan of care is to a. administer IV fluids. b. give stool softeners and enemas. c. order a diet high in fiber and fluids. d. prepare the patient for colonoscopy.

ANS: A A patient with acute diverticulitis will be NPO and given parenteral fluids. A diet high in fiber and fluids will be implemented before discharge. Bulk-forming laxatives, rather than stool softeners, are usually given, and these will be implemented later in the hospitalization. The patient with acute diverticulitis will not have enemas or a colonoscopy because of the risk for perforation and peritonitis.

A 56-year-old woman is concerned about having a moderate amount of vaginal bleeding after 4 years of menopause. The nurse will anticipate teaching the patient about a. endometrial biopsy. b. endometrial ablation. c. uterine balloon therapy. d. dilation and curettage (D&C).

ANS: A A postmenopausal woman with vaginal bleeding should be evaluated for endometrial cancer, and endometrial biopsy is the primary test for endometrial cancer. D&C will be needed only if the biopsy does not provide sufficient information to make a diagnosis. Endometrial ablation and balloon therapy are used to treat menorrhagia, which is unlikely in this patient

A 50-year-old female patient calls the clinic to report a new onset of severe diarrhea. The nurse anticipates that the patient will need to a. collect a stool specimen. b. prepare for colonoscopy. c. schedule a barium enema. d. have blood cultures drawn.

ANS: A Acute diarrhea is usually caused by an infectious process, and stool specimens are obtained for culture and examined for parasites or white blood cells. There is no indication that the patient needs a colonoscopy, blood cultures, or a barium enema.

When admitting a patient with acute glomerulonephritis, it is most important that the nurse ask the patient about a. recent sore throat and fever. b. history of high blood pressure. c. frequency of bladder infections. d. family history of kidney stones.

ANS: A Acute glomerulonephritis frequently occurs after a streptococcal infection such as strep throat. It is not caused by hypertension, urinary tract infection (UTI), or kidney stones. DIF: Cognitive Level: Application REF: 1131-1132

Which prescribed medication should the nurse administer first to a 60-year-old patient admitted to the emergency department in thyroid storm? a. Propranolol (Inderal) b. Propylthiouracil (PTU) c. Methimazole (Tapazole) d. Iodine (Lugol's solution)

ANS: A Adrenergic blockers work rapidly to decrease the cardiovascular manifestations of thyroid storm. The other medications take days to weeks to have an impact on thyroid function.

A 34-year-old female patient with a new ileostomy asks how much drainage to expect. The nurse explains that after the bowel adjusts to the ileostomy, the usual drainage will be about _____ cups. a. 2 b. 3 c. 4 d. 5

ANS: A After the proximal small bowel adapts to reabsorb more fluid, the average amount of ileostomy drainage is about 500 mL daily. One cup is about 240 mL.

A patient complains of gas pains and abdominal distention two days after a small bowel resection. Which nursing action is best to take? a. Encourage the patient to ambulate. b. Instill a mineral oil retention enema. c. Administer the ordered IV morphine sulfate. d. Offer the ordered promethazine (Phenergan) suppository.

ANS: A Ambulation will improve peristalsis and help the patient eliminate flatus and reduce gas pain. A mineral oil retention enema is helpful for constipation with hard stool. A return-flow enema might be used to relieve persistent gas pains. Morphine will further reduce peristalsis. Promethazine (Phenergan) is used as an antiemetic rather than to decrease gas pains or distention

A patient will need vascular access for hemodialysis. Which statement by the nurse accurately describes an advantage of a fistula over a graft? a. A fistula is much less likely to clot. b. A fistula increases patient mobility. c. A fistula can accommodate larger needles. d. A fistula can be used sooner after surgery.

ANS: A Arteriovenous (AV) fistulas are much less likely to clot than grafts, although it takes longer for them to mature to the point where they can be used for dialysis. The choice of an AV fistula or a graft does not have an impact on needle size or patient mobility.

Which question during the assessment of a diabetic patient will help the nurse identify autonomic neuropathy? a. "Do you feel bloated after eating?" b. "Have you seen any skin changes?" c. "Do you need to increase your insulin dosage when you are stressed?" d. "Have you noticed any painful new ulcerations or sores on your feet?"

ANS: A Autonomic neuropathy can cause delayed gastric emptying, which results in a bloated feeling for the patient. The other questions are also appropriate to ask but would not help in identifying autonomic neuropathy

In order to assist an older diabetic patient to engage in moderate daily exercise, which action is most important for the nurse to take? a. Determine what type of activities the patient enjoys. b. Remind the patient that exercise will improve self-esteem. c. Teach the patient about the effects of exercise on glucose level. d. Give the patient a list of activities that are moderate in intensity.

ANS: A Because consistency with exercise is important, assessment for the types of exercise that the patient finds enjoyable is the most important action by the nurse in ensuring adherence to an exercise program. The other actions will also be implemented but are not the most important in improving compliance

The nurse is caring for a 63-year-old with a possible pituitary tumor who is scheduled for a computed tomography (CT) scan with contrast. Which information about the patient is most important to discuss with the health care provider before the test? a. History of renal insufficiency b. Complains of chronic headache c. Recent bilateral visual field loss d. Blood glucose level of 134 mg/dL

ANS: A Because contrast media may cause acute kidney injury in patients with poor renal function, the health care provider will need to prescribe therapies such as IV fluids to prevent this complication. The other findings are consistent with the patient's diagnosis of a pituitary tumor.

When performing discharge teaching for a patient after a vasectomy, the nurse instructs the patient that he a. should continue to use other methods of birth control for 6 weeks. b. should not have sexual intercourse until his 6-week follow-up visit. c. may have temporary erectile dysfunction (ED) because of swelling. d. will notice a decrease in the appearance and volume of his ejaculate.

ANS: A Because it takes about 6 weeks to evacuate sperm that are distal to the vasectomy site, the patient should use contraception for 6 weeks. ED that occurs after vasectomy is psychologic in origin and not related to postoperative swelling. The patient does not need to abstain from intercourse. The appearance and volume of the ejaculate are not changed because sperm are a minor component of the ejaculate.

Which nursing action will be included in the plan of care for a 27-year-old male patient with bowel irregularity and a new diagnosis of irritable bowel syndrome (IBS)? a. Encourage the patient to express concerns and ask questions about IBS. b. Suggest that the patient increase the intake of milk and other dairy products. c. Educate the patient about the use of alosetron (Lotronex) to reduce symptoms. d. Teach the patient to avoid using nonsteroidal antiinflammatory drugs (NSAIDs).

ANS: A Because psychologic and emotional factors can affect the symptoms for IBS, encouraging the patient to discuss emotions and ask questions is an important intervention. Alosetron has serious side effects, and is used only for female patients who have not responded to other therapies. Although yogurt may be beneficial, milk is avoided because lactose intolerance can contribute to symptoms in some patients. NSAIDs can be used by patients with IBS.

A patient is admitted to the emergency department with possible renal trauma after an automobile accident. Which prescribed intervention will the nurse implement first? a. Check blood pressure and heart rate. b. Administer morphine sulfate 4 mg IV. c. Transport to radiology for an intravenous pyelogram. d. Insert a urethral catheter and obtain a urine specimen.

ANS: A Because the kidney is very vascular, the initial action with renal trauma will be assessment for bleeding and shock. The other actions are also important once the patient's cardiovascular status has been determined and stabilized.

The health care provider prescribes the following therapies for a patient who has been admitted with dehydration and hypotension after 3 days of nausea and vomiting. Which order will the nurse implement first? a. Infuse normal saline at 250 mL/hr. b. Administer IV ondansetron (Zofran). c. Provide oral care with moistened swabs. d. Insert a 16-gauge nasogastric (NG) tube.

ANS: A Because the patient has severe dehydration, rehydration with IV fluids is the priority. The other orders should be accomplished as quickly as possible after the IV fluids are initiated.

A female patient being admitted with pneumonia has a history of neurogenic bladder as a result of a spinal cord injury. Which action will the nurse plan to take first? a. Ask about the usual urinary pattern and any measures used for bladder control. b. Assist the patient to the toilet at scheduled times to help ensure bladder emptying. c. Check the patient for urinary incontinence every 2 hours to maintain skin integrity. d. Use intermittent catheterization on a regular schedule to avoid the risk of infection.

ANS: A Before planning any interventions, the nurse should complete the assessment and determine the patient's normal bladder pattern and the usual measures used by the patient at home. All the other responses may be appropriate, but until the assessment is complete, an individualized plan for the patient cannot be developed

Which finding by the nurse when assessing a patient with Hashimoto's thyroiditis and a goiter will require the most immediate action? a. New-onset changes in the patient's voice b. Apical pulse rate at rest 112 beats/minute c. Elevation in the patient's T3 and T4 levels d. Bruit audible bilaterally over the thyroid gland

ANS: A Changes in the patient's voice indicate that the goiter is compressing the laryngeal nerve and may lead to airway compression. The other findings will also be reported but are expected with Hashimoto's thyroiditis and do not require immediate action.

The nurse is caring for a patient who has had an ileal conduit for several years. Which nursing action could be delegated to unlicensed assistive personnel (UAP)? a. Change the ostomy appliance. b. Choose the appropriate ostomy bag. c. Monitor the appearance of the stoma. d. Assess for possible urinary tract infection (UTI).

ANS: A Changing the ostomy appliance for a stable patient could be done by UAP. Assessments of the site, choosing the appropriate ostomy bag, and assessing for (UTI) symptoms require more education and scope of practice and should be done by the registered nurse (RN).

Which nursing action can the nurse delegate to unlicensed assistive personnel (UAP) who are working in the diabetic clinic? a. Measure the ankle-brachial index. b. Check for changes in skin pigmentation. c. Assess for unilateral or bilateral foot drop. d. Ask the patient about symptoms of depression.

ANS: A Checking systolic pressure at the ankle and brachial areas and calculating the ankle-brachial index is a procedure that can be done by UAP who have been trained in the procedure. The other assessments require more education and critical thinking and should be done by the registered nurse (RN).

Which statement by the patient indicates that the nurse's teaching about treating vaginal candidiasis has been effective? a. "I should clean carefully after each urination and bowel movement." b. "I can douche with warm water if the itching continues to bother me." c. "I will insert the antifungal cream right before I get up in the morning." d. "I will tell my husband that we cannot have intercourse for the next month."

ANS: A Cleaning of the perineal area will decrease itching caused by contact of the irritated tissues with urine and reduce the chance of further infection of irritated tissues by bacteria in the stool. Sexual intercourse should be avoided for 1 week. Douching will disrupt normal protective mechanisms in the vagina. The cream should be used at night so that it will remain in the vagina for longer periods of time

When assessing a patient who is a vegan, which finding may indicate the need for cobalamin supplementation? a. Paresthesias b. Ecchymoses c. Dry, scaly skin d. Gingival swelling

ANS: A Cobalamin (vitamin B12) cannot be obtained from foods of plant origin, so the patient will be most at risk for signs of cobalamin deficiency, such as anemia and peripheral neuropathy. The other symptoms listed are associated with other nutritional deficiencies but would not be associated with a vegan diet.

Which patient statement indicates that the nurse's teaching following a gastroduodenostomy has been effective? a. "Vitamin supplements may prevent anemia." b. "Persistent heartburn is common after surgery." c. "I will try to drink more liquids with my meals." d. "I will need to choose high carbohydrate foods."

ANS: A Cobalamin deficiency may occur after partial gastrectomy, and the patient may need to receive cobalamin via injections or nasal spray. Although peptic ulcer disease may recur, persistent heartburn is not expected after surgery and the patient should call the health care provider if this occurs. Ingestion of liquids with meals is avoided to prevent dumping syndrome. Foods that have moderate fat and low carbohydrate should be chosen to prevent dumping syndrome

Cobalamin injections have been prescribed for a patient with chronic atrophic gastritis. The nurse determines that teaching regarding the injections has been effective when the patient states, a. "The cobalamin injections will prevent me from becoming anemic." b. "These injections will increase the hydrochloric acid in my stomach." c. "These injections will decrease my risk for developing stomach cancer." d. "The cobalamin injections need to be taken until my inflamed stomach heals."

ANS: A Cobalamin supplementation prevents the development of pernicious anemia. The incidence of stomach cancer is higher in patients with chronic gastritis, but cobalamin does not reduce the risk for stomach cancer. Chronic gastritis may cause achlorhydria, but cobalamin does not correct this. The loss of intrinsic factor secretion with chronic gastritis is permanent, and the patient will need lifelong supplementation with cobalamin.

A 28-year-old patient with endometriosis asks why she is being treated with medroxyprogesterone (Depo-Provera), a medication that she thought was an oral contraceptive. The nurse explains that this therapy a. suppresses the menstrual cycle by mimicking pregnancy. b. will relieve symptoms such as vaginal atrophy and hot flashes. c. prevents a pregnancy that could worsen the menstrual bleeding. d. will lead to permanent suppression of abnormal endometrial tissues.

ANS: A Depo-Provera induces a pseudopregnancy, which suppresses ovulation and causes shrinkage of endometrial tissue. Menstrual bleeding does not occur during pregnancy. Vaginal atrophy and hot flashes are caused by synthetic androgens such as danazol or gonadotropin-releasing hormone agonists (GNRH) such as leuprolide. Although hormonal therapies will control endometriosis while the therapy is used, endometriosis will recur once the menstrual cycle is reestablished

A patient has just arrived on the postoperative unit after having a laparoscopic esophagectomy for treatment of esophageal cancer. Which nursing action should be included in the postoperative plan of care? a. Elevate the head of the bed to at least 30 degrees. b. Reposition the nasogastric (NG) tube if drainage stops or decreases. c. Notify the doctor immediately about bloody NG drainage. d. Start oral fluids when the patient has active bowel sounds.

ANS: A Elevation of the head of the bed decreases the risk for reflux and aspiration of gastric secretions. The NG tube should not be repositioned without consulting with the health care provider. Bloody NG drainage is expected for the first 8 to 12 hours. A swallowing study is needed before oral fluids are started.

The nurse will anticipate preparing a 71-year-old female patient who is vomiting "coffee-ground" emesis for a. endoscopy. b. angiography. c. barium studies. d. gastric analysis.

ANS: A Endoscopy is the primary tool for visualization and diagnosis of upper gastrointestinal (GI) bleeding. Angiography is used only when endoscopy cannot be done because it is more invasive and has more possible complications. Barium studies are helpful in determining the presence of gastric lesions, but not whether the lesions are actively bleeding. Gastric analysis testing may help with determining the cause of gastric irritation, but it is not used for acute GI bleeding. DIF: Cognitive Level: Apply (application) REF: 954 TOP: Nursing Process: Planning MSC: NCLEX: Physiological Integrity

A patient is hospitalized with vomiting of "coffee-ground" emesis. The nurse will anticipate preparing the patient for a. endoscopy. b. angiography. c. gastric analysis testing. d. barium contrast studies.

ANS: A Endoscopy is the primary tool for visualization and diagnosis of upper gastrointestinal (GI) bleeding. Angiography is used only when endoscopy cannot be done because it is more invasive and has more possible complications. Gastric analysis testing may help with determining the cause of gastric irritation, but it is not used for acute GI bleeding. Barium studies are helpful in determining the presence of gastric lesions, but not whether the lesions are actively bleeding.

A 36-year-old male patient in the outpatient clinic is diagnosed with acute hepatitis C (HCV) infection. Which action by the nurse is appropriate? a. Schedule the patient for HCV genotype testing. b. Administer the HCV vaccine and immune globulin. c. Teach the patient about ribavirin (Rebetol) treatment. d. Explain that the infection will resolve over a few months.

ANS: A Genotyping of HCV has an important role in managing treatment and is done before drug therapy is initiated. Because most patients with acute HCV infection convert to the chronic state, the nurse should not teach the patient that the HCV will resolve in a few months. Immune globulin or vaccine is not available for HCV. Ribavirin is used for chronic HCV infection

A patient has elevated blood urea nitrogen (BUN) and serum creatinine levels. Which bowel preparation order would the nurse question for this patient who is scheduled for a renal arteriogram? a. Fleet enema b. Tap-water enema c. Senna/docusate (Senokot-S) d. Bisacodyl (Dulcolax) tablets

ANS: A High-phosphate enemas, such as Fleet enemas, should be avoided in patients with elevated BUN and creatinine because phosphate cannot be excreted by patients with renal failure. The other medications for bowel evacuation are more appropriate.

A 27-year-old patient admitted with diabetic ketoacidosis (DKA) has a serum glucose level of 732 mg/dL and serum potassium level of 3.1 mEq/L. Which action prescribed by the health care provider should the nurse take first? a. Place the patient on a cardiac monitor. b. Administer IV potassium supplements. c. Obtain urine glucose and ketone levels. d. Start an insulin infusion at 0.1 units/kg/hr.

ANS: A Hypokalemia can lead to potentially fatal dysrhythmias such as ventricular tachycardia and ventricular fibrillation, which would be detected with electrocardiogram (ECG) monitoring. Because potassium must be infused over at least 1 hour, the nurse should initiate cardiac monitoring before infusion of potassium. Insulin should not be administered without cardiac monitoring because insulin infusion will further decrease potassium levels. Urine glucose and ketone levels are not urgently needed to manage the patient's care.

A 58-year-old woman has just returned to the nursing unit after an esophagogastroduodenoscopy (EGD). Which action by unlicensed assistive personnel (UAP) requires that the registered nurse (RN) intervene? a. Offering the patient a drink of water b. Positioning the patient on the right side c. Checking the vital signs every 30 minutes d. Swabbing the patient's mouth with cold water

ANS: A Immediately after EGD, the patient will have a decreased gag reflex and is at risk for aspiration. Assessment for return of the gag reflex should be done by the RN. The other actions by the UAP are appropriate

Which patient action indicates a good understanding of the nurse's teaching about the use of an insulin pump? a. The patient programs the pump for an insulin bolus after eating. b. The patient changes the location of the insertion site every week. c. The patient takes the pump off at bedtime and starts it again each morning. d. The patient plans for a diet that is less flexible when using the insulin pump.

ANS: A In addition to the basal rate of insulin infusion, the patient will adjust the pump to administer a bolus after each meal, with the dosage depending on the oral intake. The insertion site should be changed every 2 or 3 days. There is more flexibility in diet and exercise when an insulin pump is used. The pump will deliver a basal insulin rate 24 hours a day.

A 50-year-old patient who underwent a gastroduodenostomy (Billroth I) earlier today complains of increasing abdominal pain. The patient has no bowel sounds and 200 mL of bright red nasogastric (NG) drainage in the last hour. The highest priority action by the nurse is to a. contact the surgeon. b. irrigate the NG tube. c. monitor the NG drainage. d. administer the prescribed morphine.

ANS: A Increased pain and 200 mL of bright red NG drainage 12 hours after surgery indicate possible postoperative hemorrhage, and immediate actions such as blood transfusion and/or return to surgery are needed. Because the NG is draining, there is no indication that irrigation is needed. Continuing to monitor the NG drainage is not an adequate response. The patient may need morphine, but this is not the highest priority action

Twelve hours after undergoing a gastroduodenostomy (Billroth I), a patient complains of increasing abdominal pain. The patient has absent bowel sounds and 200 mL of bright red nasogastric (NG) drainage in the last hour. The most appropriate action by the nurse at this time is to a. notify the surgeon. b. irrigate the NG tube. c. administer the prescribed morphine. d. continue to monitor the NG drainage.

ANS: A Increased pain and 200 mL of bright red NG drainage 12 hours after surgery indicate possible postoperative hemorrhage, and immediate actions such as blood transfusion and/or return to surgery are needed. Because the NG is draining, there is no indication that irrigation is needed. The patient may need morphine, but this is not the highest priority action. Continuing to monitor the NG drainage is not an adequate response.

A 22-year-old patient is being seen in the clinic with increased secretion of the anterior pituitary hormones. The nurse would expect the laboratory results to show a. increased urinary cortisol. b. decreased serum thyroxine. c. elevated serum aldosterone levels. d. low urinary catecholamines excretion.

ANS: A Increased secretion of adrenocorticotropic hormone (ACTH) by the anterior pituitary gland will lead to an increase in serum and urinary cortisol levels. An increase, rather than a decrease, in thyroxine level would be expected with increased secretion of thyroid stimulating hormone (TSH) by the anterior pituitary. Aldosterone and catecholamine levels are not controlled by the anterior pituitary.

The nurse will be teaching self-management to patients after gastric bypass surgery. Which information will the nurse plan to include? a. Drink fluids between meals but not with meals. b. Choose high-fat foods for at least 30% of intake. c. Developing flabby skin can be prevented by exercise. d. Choose foods high in fiber to promote bowel function.

ANS: A Intake of fluids with meals tends to cause dumping syndrome and diarrhea. Food choices should be low in fat and fiber. Exercise does not prevent the development of flabby skin

A 50-year-old patient is diagnosed with uterine bleeding caused by a leiomyoma. Which information will the nurse include in the patient teaching plan? a. The symptoms may decrease after the patient undergoes menopause. b. The tumor size is likely to increase throughout the patient's lifetime. c. Aspirin or acetaminophen may be used to control mild to moderate pain. d. The patient will need frequent monitoring to detect any malignant changes.

ANS: A Leiomyomas appear to depend on ovarian hormones and will atrophy after menopause, leading to a decrease in symptoms. Aspirin use is discouraged because the antiplatelet effects may lead to heavier uterine bleeding. The size of the tumor will shrink after menopause. Leiomyomas are benign tumors that do not undergo malignant changes.

A patient who has just been started on continuous tube feedings of a full-strength commercial formula at 100 mL/hr using a closed system method has six diarrhea stools the first day. Which action should the nurse plan to take? a. Slow the infusion rate of the tube feeding. b. Check gastric residual volumes more frequently. c. Change the enteral feeding system and formula every 8 hours. d. Discontinue administration of water through the feeding tube.

ANS: A Loose stools indicate poor absorption of nutrients and indicate a need to slow the feeding rate or decrease the concentration of the feeding. Water should be given when patients receive enteral feedings to prevent dehydration. When a closed enteral feeding system is used, the tubing and formula are changed every 24 hours. High residual volumes do not contribute to diarrhea.

A 51-year-old male patient has a new diagnosis of Crohn's disease after having frequent diarrhea and a weight loss of 10 pounds (4.5 kg) over 2 months. The nurse will plan to teach about a. medication use. b. fluid restriction. c. enteral nutrition. d. activity restrictions.

ANS: A Medications are used to induce and maintain remission in patients with inflammatory bowel disease (IBD). Decreased activity level is indicated only if the patient has severe fatigue and weakness. Fluids are needed to prevent dehydration. There is no advantage to enteral feedings

A patient in the urology clinic is diagnosed with monilial urethritis. Which action will the nurse include in the plan of care? a. Teach the patient about the use of antifungal medications. b. Tell the patient to avoid tub baths until the symptoms resolve. c. Instruct the patient to refer recent sexual partners for treatment. d. Teach the patient to avoid nonsteroidal antiinflammatory drugs (NSAIDs).

ANS: A Monilial urethritis is caused by a fungus and antifungal medications such as nystatin (Mycostatin) or fluconazole (Diflucan) are usually used as treatment. Because monilial urethritis is not sexually transmitted, there is no need to refer sexual partners. Warm baths and NSAIDS may be used to treat symptoms.

A female patient is scheduled for an oral glucose tolerance test. Which information from the patient's health history is most important for the nurse to communicate to the health care provider? a. The patient uses oral contraceptives. b. The patient runs several days a week. c. The patient has been pregnant three times. d. The patient has a family history of diabetes.

ANS: A Oral contraceptive use may falsely elevate oral glucose tolerance test (OGTT) values. Exercise and a family history of diabetes both can affect blood glucose but will not lead to misleading information from the OGTT. History of previous pregnancies may provide informational about gestational glucose tolerance, but will not lead to misleading information from the OGTT

The nurse will plan to provide teaching for a 67-year-old patient who has been diagnosed with orchitis about a. pain management. b. emergency surgical repair. c. application of heat to the scrotum. d. aspiration of fluid from the scrotal sac.

ANS: A Orchitis is very painful, and effective pain management will be needed. Heat, aspiration, and surgery are not used to treat orchitis.

The nurse determines that additional instruction is needed for a 60-year-old patient with chronic syndrome of inappropriate antidiuretic hormone (SIADH) when the patient says which of the following? a. "I need to shop for foods low in sodium and avoid adding salt to food." b. "I should weigh myself daily and report any sudden weight loss or gain." c. "I need to limit my fluid intake to no more than 1 quart of liquids a day." d. "I will eat foods high in potassium because diuretics cause potassium loss."

ANS: A Patients with SIADH are at risk for hyponatremia, and a sodium supplement may be prescribed. The other patient statements are correct and indicate successful teaching has occurred.

A patient who has had several episodes of bloody diarrhea is admitted to the emergency department. Which action should the nurse anticipate taking? a. Obtain a stool specimen for culture. b. Administer antidiarrheal medications. c. Teach about adverse effects of nonsteroidal anti-inflammatory drugs (NSAIDs). d. Provide education about antibiotic therapy.

ANS: A Patients with bloody diarrhea should have a stool culture for E. coli O157:H7. NSAIDs may cause occult blood in the stools, but not diarrhea. Antidiarrheal medications usually are avoided for possible infectious diarrhea to avoid prolonging the infection. Antibiotic therapy in the treatment of infectious diarrhea is controversial because it may precipitate kidney complications.

The nurse is caring for a 68-year-old hospitalized patient with a decreased glomerular filtration rate who is scheduled for an intravenous pyelogram (IVP). Which action will be included in the plan of care? a. Monitor the urine output after the procedure. b. Assist with monitored anesthesia care (MAC). c. Give oral contrast solution before the procedure. d. Insert a large size urinary catheter before the IVP.

ANS: A Patients with impaired renal function are at risk for decreased renal function after IVP because the contrast medium used is nephrotoxic, so the nurse should monitor the patient's urine output. MAC sedation and retention catheterization are not required for the procedure. The contrast medium is given IV, not orally.

The health care provider orders intravenous (IV) ranitidine (Zantac) for a patient with gastrointestinal (GI) bleeding caused by peptic ulcer disease. When teaching the patient about the effect of the medication, which information will the nurse include? a. "Ranitidine decreases secretion of gastric acid." b. "Ranitidine neutralizes the acid in the stomach." c. "Ranitidine constricts the blood vessels in the stomach and decreases bleeding." d. "Ranitidine covers the ulcer with a protective material that promotes healing."

ANS: A Ranitidine is a histamine-2 (H2) receptor blocker, which decreases the secretion of gastric acid. The response beginning, "Ranitidine constricts the blood vessels" describes the effect of vasopressin. The response beginning "Ranitidine neutralizes the acid" describes the effect of antacids. And the response beginning "Ranitidine covers the ulcer" describes the action of sucralfate (Carafate).

A 32-year-old patient with diabetes is starting on intensive insulin therapy. Which type of insulin will the nurse discuss using for mealtime coverage? a. Lispro (Humalog) b. Glargine (Lantus) c. Detemir (Levemir) d. NPH (Humulin N)

ANS: A Rapid- or short-acting insulin is used for mealtime coverage for patients receiving intensive insulin therapy. NPH, glargine, or detemir will be used as the basal insulin.

The nurse is planning care for a 48-year-old woman with acute severe pancreatitis. The highest priority patient outcome is a. maintaining normal respiratory function. b. expressing satisfaction with pain control. c. developing no ongoing pancreatic disease. d. having adequate fluid and electrolyte balance.

ANS: A Respiratory failure can occur as a complication of acute pancreatitis, and maintenance of adequate respiratory function is the priority goal. The other outcomes would also be appropriate for the patient

Which nursing action will be most helpful in decreasing the risk for hospital-acquired infection (HAI) of the urinary tract in patients admitted to the hospital? a. Avoid unnecessary catheterizations. b. Encourage adequate oral fluid intake. c. Test urine with a dipstick daily for nitrites. d. Provide thorough perineal hygiene to patients.

ANS: A Since catheterization bypasses many of the protective mechanisms that prevent urinary tract infection (UTI), avoidance of catheterization is the most effective means of reducing HAI. The other actions will also be helpful, but are not as useful as decreasing urinary catheter use. DIF: Cognitive Level: Application REF: 1125-1127

When counseling a patient with a family history of stomach cancer about ways to decrease risk for developing stomach cancer, the nurse will teach the patient to avoid a. smoked foods such as bacon and ham. b. foods that cause abdominal distention. c. chronic use of H2 blocking medications. d. emotionally or physically stressful situations.

ANS: A Smoked foods such as bacon, ham, and smoked sausage increase the risk for stomach cancer. Use of H2 blockers, stressful situations, and abdominal distention are not associated with an increased incidence of stomach cancer.

Sodium polystyrene sulfonate (Kayexalate) is ordered for a patient with hyperkalemia. Before administering the medication, the nurse should assess the a. bowel sounds. b. blood glucose. c. blood urea nitrogen (BUN). d. level of consciousness (LOC).

ANS: A Sodium polystyrene sulfonate (Kayexalate) should not be given to a patient with a paralytic ileus (as indicated by absent bowel sounds) because bowel necrosis can occur. The BUN and creatinine, blood glucose, and LOC would not affect the nurse's decision to give the medication.

The nurse has administered 4 oz of orange juice to an alert patient whose blood glucose was 62 mg/dL. Fifteen minutes later, the blood glucose is 67 mg/dL. Which action should the nurse take next? a. Give the patient 4 to 6 oz more orange juice. b. Administer the PRN glucagon (Glucagon) 1 mg IM. c. Have the patient eat some peanut butter with crackers. d. Notify the health care provider about the hypoglycemia.

ANS: A The "rule of 15" indicates that administration of quickly acting carbohydrates should be done 2 to 3 times for a conscious patient whose glucose remains less than 70 mg/dL before notifying the health care provider. More complex carbohydrates and fats may be used once the glucose has stabilized. Glucagon should be used if the patient's level of consciousness decreases so that oral carbohydrates can no longer be given

A 48-year-old woman in the emergency department reports that she has been sexually assaulted. Which action by the nurse will be most important in maintaining the medicolegal chain of evidence? a. Labeling all specimens and other materials obtained from the patient. b. Assisting the patient in filling out the application for financial compensation. c. Discussing the availability of the "morning-after pill" for pregnancy prevention. d. Educating the patient about baseline sexually transmitted infection (STI) testing.

ANS: A The careful labeling of specimens and materials will assist in maintaining the chain of evidence. Assisting with paperwork, and discussing STIs and pregnancy prevention are interventions that might be appropriate after sexual assault, but they do not help maintain the legal chain of evidence

A 38-year-old patient who has type 1 diabetes plans to swim laps daily at 1:00 PM. The clinic nurse will plan to teach the patient to a. check glucose level before, during, and after swimming. b. delay eating the noon meal until after the swimming class. c. increase the morning dose of neutral protamine Hagedorn (NPH) insulin. d. time the morning insulin injection so that the peak occurs while swimming.

ANS: A The change in exercise will affect blood glucose, and the patient will need to monitor glucose carefully to determine the need for changes in diet and insulin administration. Because exercise tends to decrease blood glucose, patients are advised to eat before exercising. Increasing the morning NPH or timing the insulin to peak during exercise may lead to hypoglycemia, especially with the increased exercise.

A 55-year-old female patient with type 2 diabetes has a nursing diagnosis of imbalanced nutrition: more than body requirements. Which goal is most important for this patient? a. The patient will reach a glycosylated hemoglobin level of less than 7%. b. The patient will follow a diet and exercise plan that results in weight loss. c. The patient will choose a diet that distributes calories throughout the day. d. The patient will state the reasons for eliminating simple sugars in the diet.

ANS: A The complications of diabetes are related to elevated blood glucose, and the most important patient outcome is the reduction of glucose to near-normal levels. The other outcomes also are appropriate but are not as high in priority

What glomerular filtration rate (GFR) would the nurse estimate for a 30-year-old patient with a creatinine clearance result of 60 mL/min? a. 60 mL/min b. 90 mL/min c. 120 mL/min d. 180 mL/min

ANS: A The creatinine clearance approximates the GFR. The other responses are not accurate

A 19-year-old visits the health clinic for a routine checkup. Which question should the nurse ask to determine whether a Pap test is needed? a. "Have you had sexual intercourse?" b. "Do you use any illegal substances?" c. "Do you have cramping with your periods?" d. "At what age did your menstrual periods start?"

ANS: A The current American Cancer Society recommendation is that a Pap test be done every 3 years, starting 3 years after the first sexual intercourse and no later than age 21. The information about menstrual periods and substance abuse will not help determine whether the patient requires a Pap test

To palpate the liver during a head-to-toe physical assessment, the nurse a. places one hand on the patient's back and presses upward and inward with the other hand below the patient's right costal margin. b. places one hand on top of the other and uses the upper fingers to apply pressure and the bottom fingers to feel for the liver edge. c. presses slowly and firmly over the right costal margin with one hand and withdraws the fingers quickly after the liver edge is felt. d. places one hand under the patient's lower ribs and presses the left lower rib cage forward, palpating below the costal margin with the other hand.

ANS: A The liver is normally not palpable below the costal margin. The nurse needs to push inward below the right costal margin while lifting the patient's back slightly with the left hand. The other methods will not allow palpation of the liver

A 54-year-old patient is admitted with diabetic ketoacidosis. Which admission order should the nurse implement first? a. Infuse 1 liter of normal saline per hour. b. Give sodium bicarbonate 50 mEq IV push. c. Administer regular insulin 10 U by IV push. d. Start a regular insulin infusion at 0.1 units/kg/hr.

ANS: A The most urgent patient problem is the hypovolemia associated with diabetic ketoacidosis (DKA), and the priority is to infuse IV fluids. The other actions can be done after the infusion of normal saline is initiated

Which medication taken at home by a 47-year-old patient with decreased renal function will be of most concern to the nurse? a. ibuprofen (Motrin) b. warfarin (Coumadin) c. folic acid (vitamin B9) d. penicillin (Bicillin LA)

ANS: A The nonsteroidal antiinflammatory medications (NSAIDs) are nephrotoxic and should be avoided in patients with impaired renal function. The nurse also should ask about reasons the patient is taking the other medications, but the medication of most concern is the ibuprofen.

A patient who is diagnosed with nephrotic syndrome has 3+ ankle and leg edema and ascites. Which nursing diagnosis is a priority for the patient? a. Excess fluid volume related to low serum protein levels b. Activity intolerance related to increased weight and fatigue c. Disturbed body image related to peripheral edema and ascites d. Altered nutrition: less than required related to protein restriction

ANS: A The patient has massive edema, so the priority problem at this time is the excess fluid volume. The other nursing diagnoses also are appropriate, but the focus of nursing care should be resolution of the edema and ascites. DIF: Cognitive Level: Application REF: 1133-1135

A nurse anticipates urodynamic testing for a patient with which symptom? a. Involuntary urine leakage b. Severe flank pain c. Presence of blood in urine d. Dysuria

ANS: A Urodynamic testing evaluates the muscle function of the bladder and is used to look for the cause of urinary incontinence. Severe flank pain indicates renal calculi; CT scan or IVP would be a more efficient diagnostic test. Blood indicates trauma to the urethral or bladder mucosa. Pain on elimination may warrant cultures to check for infection.

After change-of-shift report, which patient should the nurse assess first? a. Patient with a urethral stricture who has not voided for 12 hours b. Patient who has cloudy urine after orthotopic bladder reconstruction c. Patient with polycystic kidney disease whose blood pressure is 186/98 mm Hg d. Patient who voided bright red urine immediately after returning from lithotripsy

ANS: A The patient information suggests acute urinary retention, a medical emergency. The nurse will need to assess the patient and consider whether to insert a retention catheter. The other patients will also be assessed, but their findings are consistent with their diagnoses and do not require immediate assessment or possible intervention.

A patient recovering from a gastrojejunostomy (Billroth II) for treatment of a duodenal ulcer develops dizziness, weakness, and palpitations about 20 minutes after eating. To avoid recurrence of these symptoms, the nurse teaches the patient to a. lie down for about 30 minutes after eating. b. choose foods that are high in carbohydrates. c. increase the amount of fluid intake with meals. d. drink sugared fluids or eat candy after each meal.

ANS: A The patient is experiencing symptoms of dumping syndrome, which may be reduced by lying down after eating. Increasing fluid intake and choosing high carbohydrate foods will increase the risk for dumping syndrome. Having a sweet drink or hard candy will correct the hypoglycemia that is associated with dumping syndrome but will not prevent dumping syndrome.

A patient is being scheduled for endoscopic retrograde cholangiopancreatography (ERCP) as soon as possible. Which actions from the agency policy for ERCP should the nurse take first? a. Place the patient on NPO status. b. Administer sedative medications. c. Ensure the consent form is signed. d. Teach the patient about the procedure.

ANS: A The patient will need to be NPO for 8 hours before the ERCP is done, so the nurse's initial action should be to place the patient on NPO status. The other actions can be done after the patient is NPO

The nurse is caring for a patient admitted with diabetes insipidus (DI). Which information is most important to report to the health care provider? a. The patient is confused and lethargic. b. The patient reports a recent head injury. c. The patient has a urine output of 400 mL/hr. d. The patient's urine specific gravity is 1.003.

ANS: A The patient's confusion and lethargy may indicate hypernatremia and should be addressed quickly. In addition, patients with DI compensate for fluid losses by drinking copious amounts of fluids, but a patient who is lethargic will be unable to drink enough fluids and will become hypovolemic. A high urine output, low urine specific gravity, and history of a recent head injury are consistent with diabetes insipidus, but they do not require immediate nursing action to avoid life-threatening complications.

A 23-year-old has been admitted with acute liver failure. Which assessment data are most important for the nurse to communicate to the health care provider? a. Asterixis and lethargy b. Jaundiced sclera and skin c. Elevated total bilirubin level d. Liver 3 cm below costal margin

ANS: A The patient's findings of asterixis and lethargy are consistent with grade 2 hepatic encephalopathy. Patients with acute liver failure can deteriorate rapidly from grade 1 or 2 to grade 3 or 4 hepatic encephalopathy and need early transfer to a transplant center. The other findings are typical of patients with hepatic failure and would be reported but would not indicate a need for an immediate change in the therapeutic plan.

A 28-year-old patient reports anxiety, headaches with dizziness, and abdominal bloating occurring before her menstrual periods. Which action is best for the nurse to take at this time? a. Ask the patient to keep track of her symptoms in a diary for 3 months. b. Suggest that the patient try aerobic exercise to decrease her symptoms. c. Teach the patient about appropriate lifestyle changes to reduce premenstrual syndrome (PMS) symptoms. d. Advise the patient to use nonsteroidal antiinflammatory drugs (NSAIDs) such as ibuprofen (Advil) to control symptoms.

ANS: A The patient's symptoms indicate possible PMS, but they also may be associated with other diagnoses. Having the patient keep a symptom diary for 2 or 3 months will help in confirming a diagnosis of PMS. The nurse should not implement interventions for PMS until a diagnosis is made

It is most important that the nurse ask a patient admitted with acute glomerulonephritis about a. history of kidney stones. b. recent sore throat and fever. c. history of high blood pressure. d. frequency of bladder infections.

ANS: B Acute glomerulonephritis frequently occurs after a streptococcal infection such as strep throat. It is not caused by kidney stones, hypertension, or urinary tract infection (UTI).

When caring for a patient with a left arm arteriovenous fistula, which action will the nurse include in the plan of care to maintain the patency of the fistula? a. Auscultate for a bruit at the fistula site. b. Assess the quality of the left radial pulse. c. Compare blood pressures in the left and right arms. d. Irrigate the fistula site with saline every 8 to 12 hours.

ANS: A The presence of a thrill and bruit indicates adequate blood flow through the fistula. Pulse rate and quality are not good indicators of fistula patency. Blood pressures should never be obtained on the arm with a fistula. Irrigation of the fistula might damage the fistula, and typically only dialysis staff would access the fistula.

A patient with a bleeding duodenal ulcer has a nasogastric (NG) tube in place, and the health care provider orders 30 mL of aluminum hydroxide/magnesium hydroxide (Maalox) to be instilled through the tube every hour. To evaluate the effectiveness of this treatment, the nurse a. periodically aspirates and tests gastric pH. b. monitors arterial blood gas values on a daily basis. c. checks each stool for the presence of occult blood. d. measures the amount of residual stomach contents hourly.

ANS: A The purpose for antacids is to increase gastric pH. Checking gastric pH is the most direct way of evaluating the effectiveness of the medication. Arterial blood gases may change slightly, but this does not directly reflect the effect of antacids on gastric pH. Because the patient has upper gastrointestinal (GI) bleeding, occult blood in the stools will appear even after the acute bleeding has stopped. The amount of residual stomach contents is not a reflection of resolution of bleeding or of gastric pH.

Which finding indicates to the nurse that lactulose (Cephulac) is effective for a 72-year-old man who has advanced cirrhosis? a. The patient is alert and oriented. b. The patient denies nausea or anorexia. c. The patient's bilirubin level decreases. d. The patient has at least one stool daily.

ANS: A The purpose of lactulose in the patient with cirrhosis is to lower ammonia levels and prevent encephalopathy. Although lactulose may be used to treat constipation, that is not the purpose for this patient. Lactulose will not decrease nausea and vomiting or lower bilirubin levels.

A patient receives aspart (NovoLog) insulin at 8:00 AM. Which time will it be most important for the nurse to monitor for symptoms of hypoglycemia? a. 10:00 AM b. 12:00 AM c. 2:00 PM d. 4:00 PM

ANS: A The rapid-acting insulins peak in 1 to 3 hours. The patient is not at a high risk for hypoglycemia at the other listed times, although hypoglycemia may occur

Which information will the nurse plan to include when teaching a community health group about testicular self-examination? a. Testicular self-examination should be done in a warm room. b. The only structure normally felt in the scrotal sac is the testis. c. Testicular self-examination should be done at least every week. d. Call the health care provider if one testis is larger than the other.

ANS: A The testes will hang lower in the scrotum when the temperature is warm (e.g., during a shower), and it will be easier to palpate. The epididymis is also normally palpable in the scrotum. One testis is normally larger. The patient should perform testicular self-examination monthly.

A patient who is receiving continuous enteral nutrition through a small-bore silicone feeding tube has a computed tomography (CT) scan ordered and will have to be placed in a flat position for the scan. Which action by the nurse is best? a. Shut the feeding off 30 to 60 minutes before the scan. b. Ask the health care provider to reschedule the CT scan. c. Connect the feeding tube to continuous suction during the scan. d. Send the patient to CT scan with oral suction in case of aspiration.

ANS: A The tube feeding should be shut off 30 to 60 minutes before any procedure requiring the patient to lie flat. Because the CT scan is ordered for diagnosis of patient problems, rescheduling is not usually an option. Prevention, rather than treatment, of aspiration is needed. Small-bore feeding tubes are soft and collapse easily with aspiration or suction, making nasogastric suction of gastric contents unreliable.

The nurse administering a-interferon and ribavirin (Rebetol) to a patient with chronic hepatitis C will plan to monitor for a. leukopenia. b. hypokalemia. c. polycythemia. d. hypoglycemia.

ANS: A Therapy with ribavirin and a-interferon may cause leukopenia. The other problems are not associated with this drug therapy

A female patient is awaiting surgery for acute peritonitis. Which action will the nurse include in the plan of care? a. Position patient with the knees flexed. b. Avoid use of opioids or sedative drugs. c. Offer frequent small sips of clear liquids. d. Assist patient to breathe deeply and cough.

ANS: A There is less peritoneal irritation with the knees flexed, which will help decrease pain. Opioids and sedatives are typically given to control pain and anxiety. Preoperative patients with peritonitis are given IV fluids for hydration. Deep breathing and coughing will increase the patient's discomfort

A patient who was admitted with myxedema coma and diagnosed with hypothyroidism is improving and expected to be discharged in 2 days. Which teaching strategy will be best for the nurse to use? a. Provide written reminders of self-care information. b. Offer multiple options for management of therapies. c. Ensure privacy for teaching by asking visitors to leave. d. Delay teaching until patient discharge date is confirmed.

ANS: A Written instructions will be helpful to the patient because initially the hypothyroid patient may be unable to remember to take medications and other aspects of self-care. Because the treatment regimen is somewhat complex, teaching should be initiated well before discharge. Family members or friends should be included in teaching because the hypothyroid patient is likely to forget some aspects of the treatment plan. A simpler regimen will be easier to understand until the patient is euthyroid.

A patient requests the nurse's assistance to the bedside commode and becomes frustrated when unable to void in front of the nurse. The nurse understands the patient's inability to void because a. Anxiety can make it difficult for abdominal and perineal muscles to relax enough to void. b. The patient does not recognize the physiological signals that indicate a need to void. c. The patient is lonely, and calling the nurse in under false pretenses is a way to get attention. d. The patient is not drinking enough fluids to produce adequate urine output.

ANS: A Attempting to void in the presence of another can cause anxiety and tension in the muscles that make voiding difficult. The nurse should give the patient privacy and adequate time if appropriate. No evidence suggests that an underlying physiological or psychological condition exists.

A nurse is caring for an 8-year-old patient who is embarrassed about urinating in his bed at night. Which intervention should the nurse suggest to reduce the frequency of this occurrence? a. "Drink your nightly glass of milk earlier in the evening." b. "Set your alarm clock to wake you every 2 hours, so you can get up to void." c. "Line your bedding with plastic sheets to protect your mattress." d. "Empty your bladder completely before going to bed."

ANS: A Nightly incontinence and nocturia are often resolved by limiting fluid intake 2 hours before bedtime. Setting the alarm clock to wake does not correct the physiological problem, nor does lining the bedding with plastic sheets. Emptying the bladder may help with early nighttime urination, but will not affect urine produced throughout the night from late-night fluid intake.

When reviewing laboratory results, the nurse should immediately notify the health care provider about which finding? a. Glomerular filtration rate of 20 mL/min b. Urine output of 80 mL/hr c. pH of 6.4 d. Protein level of 2 mg/100 mL

ANS: A Normal glomerular filtration rate should be around 125 mL/min; a severe decrease in renal perfusion could indicate a life-threatening problem such as shock or dehydration. Normal urine output is 1000 to 2000 mL/day; an output of 30 mL/hr or less for 2 or more hours would be cause for concern. The normal pH of urine is between 4.6 and 8.0. Protein up to 8 mg/100 mL is acceptable; however, values in excess of this could indicate renal disease.

A patient asks about treatment for urge urinary incontinence. The nurse's best response is to advise the patient to a. Perform pelvic floor exercises. b. Drink cranberry juice. c. Avoid voiding frequently. d. Wear an adult diaper.

ANS: A Poor muscle tone leads to an inability to control urine flow. The nurse should recommend pelvic muscle strengthening exercises such as Kegel exercises; this solution best addresses the patient's problem. Drinking cranberry juice is a preventative measure for urinary tract infection. The nurse should not encourage the patient to reduce voiding; residual urine in the bladder increases the risk of infection. Wearing an adult diaper could be considered if attempts to correct the root of the problem fail. DIF: Analyze REF: 1056 OBJ: Identify nursing diagnoses appropriate for patients with alterations in urinary elimination.

What signs and symptoms would the nurse expect to observe in a patient with excessive white blood cells present in the urine? a. Fever and chills b. Difficulty holding in urine c. Increased blood pressure d. Abnormal blood sugar

ANS: A The presence of white blood cells in urine indicates a urinary tract infection. Difficulty with urinary elimination indicates blockage or renal damage. Increased blood pressure is associated with renal disease or damage and some medications. Abnormal blood sugars would be seen in someone with ketones in the urine, as this finding indicates diabetes.

Upon palpation, the nurse notices that the bladder is firm and distended; the patient expresses an urge to urinate. The nurse should follow up by asking a. "When was the last time you voided?" b. "Do you lose urine when you cough or sneeze?" c. "Have you noticed any change in your urination patterns?" d. "Do you have a fever or chills?"

ANS: A To obtain an accurate assessment, the nurse should first determine the source of the discomfort. Urinary retention causes the bladder to be firm and distended. Further assessment to determine the pathology of the condition can be performed later. Questions concerning fever and chills, changing urination patterns, and losing urine during coughing or sneezing focus on specific pathological conditions.

The nurse anticipates urinary diversion from the kidneys to a site other than the bladder for which patient? a. A 12-year-old female with severe abdominal trauma b. A 24-year-old male with severe genital warts around the urethra c. A 50-year-old male with recent prostatectomy d. A 75-year-old female with end-stage renal disease

ANS: A Urinary diversion would be needed in a patient with abdominal trauma who might have injury to the urinary system. Genital warts are not needed for urinary diversion. Patients with a prostatectomy may require intermittent catheterization after the procedure. End-stage renal disease would not be affected by rerouting the flow of urine.

To reduce patient discomfort during closed catheter irrigation, the nurse should a. Use room temperature irrigation solution. b. Administer the solution as quickly as possible. c. Allow the solution to sit in the bladder for at least 1 hour. d. Raise the bag of irrigation solution at least 12 inches above the bladder.

ANS: A Using cold solutions, instilling solutions too quickly, and prolonging filling of the bladder can cause discomfort and cramping. To reduce this, ensure that the solution is at room temperature, lower the solution bag so it instills slowly, and drain the bladder fully after an ordered amount of time.

After a patient returns from a barium swallow, the nurse's priority is to a. Encourage the patient to increase fluids to flush out the barium. b. Monitor stools closely for bright red blood or mucus, which indicates trauma from the procedure. c. Inform the patient that his bowel movements are radioactive, and that he should be sure to flush the toilet three times. d. Thicken all patient drinks to prevent aspiration

ANS: A Encourage the patient to increase fluid intake to flush and remove excess barium from the body. Barium swallow is a noninvasive procedure for which no trauma would produce blood or mucus or increase aspiration risk. Barium is not a radioactive substance, so multiple flushes are not needed.

The client scheduled for intravenous urography informs the nurse of the following allergies. Which one should the nurse report to the physician immediately? A. Seafood B. Penicillin C. Bee stings D. Red food dye

ANS: A Clients with seafood allergies often have severe allergic reactions to the standard dyes used during intravenous urography.

The client scheduled to have an intravenous urogram is a diabetic and taking the antidiabetic agent metformin. What should the nurse tell this client? A. "Call your diabetes doctor and tell him or her that you are having an intravenous urogram performed using dye." B. "Do not take your metformin the morning of the test because you are not going to be eating anything and could become hypoglycemic." C. "You must start on an antibiotic before this test because your risk of infection is greater as a result of your diabetes." D. "You must take your metformin immediately before the test is performed because the IV fluid and the dye contain a significant amount of sugar."

ANS: A Metformin can cause a lactic acidosis and renal impairment as an interaction with the dye. This drug must be discontinued for 48 hours before the procedure and not started again after the procedure until urine output is well established.

What would be the response if a person's nephrons were not able to filter normally due to scarring of the proximal convoluted tubule leading to inhibition of reabsorption? A. Increased urine output, fluid volume deficit B. Decreased urine output, fluid volume deficit C. Increased urine output, fluid volume overload D. Decreased urine output, fluid volume overload

ANS: A The nephrons filter about 120 mL/min. Most of this filtrate is reabsorbed in the proximal convoluted tubule. If the tubule were not able to reabsorb the fluid that has been filtered, urine output would greatly increase, leading to rapid and severe dehydration.

The nurse properly obtains a 24-hour urine specimen collection by (Select all that apply.) a. Asking the patient to void and to discard the first sample. b. Keeping the urine collection container on ice. c. Withholding all patient medications for the day. d. Asking the patient to notify the staff before and after every void.

ANS: A, B When obtaining a 24-hour urine specimen, it is important to keep the urine in cool condition. The patient should be asked to void and to discard the urine before the procedure begins. Medications do not need to be held unless indicated by the provider. If properly educated about the collection procedure, the patient can maintain autonomy and perform the procedure alone, taking care to maintain the integrity of the solution.

Which nonhormonal therapies will the nurse suggest for a healthy perimenopausal woman who prefers not to use hormone therapy (HT) (select all that apply)? a. Reduce coffee intake. b. Exercise several times a week. c. Take black cohosh supplements. d. Have a glass of wine in the evening. e. Increase intake of dietary soy products.

ANS: A, B, C, E Reduction in caffeine intake, use of black cohosh, increasing dietary soy intake, and exercising three to four times weekly are recommended to reduce symptoms associated with menopause. Alcohol intake in the evening may increase the sleep problems associated with menopause

After an unimmunized individual is exposed to hepatitis B through a needle-stick injury, which actions will the nurse plan to take (select all that apply)? a. Administer hepatitis B vaccine. b. Test for antibodies to hepatitis B. c. Teach about a-interferon therapy. d. Give hepatitis B immune globulin. e. Teach about choices for oral antiviral therapy.

ANS: A, B, D The recommendations for hepatitis B exposure include both vaccination and immune globulin administration. In addition, baseline testing for hepatitis B antibodies will be needed. Interferon and oral antivirals are not used for hepatitis B prophylaxis

Which of the following are indications for irrigating a urinary catheter? (Select all that apply.) a. Sediment occluding within the tubing b. Blood clots in the bladder following surgery c. Rupture of the catheter balloon d. Bladder infection e. Presence of renal calculi

ANS: A, B, D Catheter irrigation is used to flush and remove blockage that may be impeding the catheter from properly draining the bladder. Irrigation is used to remove blood clots in the bladder following surgery. For patients with bladder infection, an antibiotic irrigation is often ordered. A ruptured catheter balloon will involve extensive follow-up and possible surgery to remove the particles. Renal calculi obstruct the ureters and therefore the flow of urine before it reaches the bladder.

Which diet choice by the patient with an acute exacerbation of inflammatory bowel disease (IBD) indicates a need for more teaching? a. Scrambled eggs b. White toast and jam c. Oatmeal with cream d. Pancakes with syrup

ANS: C During acute exacerbations of IBD, the patient should avoid high-fiber foods such as whole grains. High-fat foods also may cause diarrhea in some patients. The other choices are low residue and would be appropriate for this patient.

Which information will be included when the nurse is teaching self-management to a patient who is receiving peritoneal dialysis (select all that apply)? a. Avoid commercial salt substitutes. b. Drink 1500 to 2000 mL of fluids daily. c. Take phosphate-binders with each meal. d. Choose high-protein foods for most meals. e. Have several servings of dairy products daily.

ANS: A, C, D Patients who are receiving peritoneal dialysis should have a high-protein diet. Phosphate binders are taken with meals to help control serum phosphate and calcium levels. Commercial salt substitutes are high in potassium and should be avoided. Fluid intake is limited in patients requiring dialysis. Dairy products are high in phosphate and usually are limited.

Which information will the nurse include when teaching a patient how to avoid chronic constipation (select all that apply)? a. Many over-the-counter (OTC) medications can cause constipation. b. Stimulant and saline laxatives can be used regularly. c. Bulk-forming laxatives are an excellent source of fiber. d. Walking or cycling frequently will help bowel motility. e. A good time for a bowel movement may be after breakfast.

ANS: A, C, D, E Stimulant and saline laxatives should be used infrequently. Use of bulk-forming laxatives, regular early morning timing of defecation, regular exercise, and avoiding many OTC medications will help the patient avoid constipation

Which statements will the nurse include when teaching a patient who is scheduled for oral glucose tolerance testing in the outpatient clinic (select all that apply)? a. "You will need to avoid smoking before the test." b. "Exercise should be avoided until the testing is complete." c. "Several blood samples will be obtained during the testing." d. "You should follow a low-calorie diet the day before the test." e. "The test requires that you fast for at least 8 hours before testing."

ANS: A, C, E Smoking may affect the results of oral glucose tolerance tests. Blood samples are obtained at baseline and at 30, 60, and 120 minutes. Accuracy requires that the patient be fasting before the test. The patient should consume at least 1500 calories/day for 3 days before the test. The patient should be ambulatory and active for accurate test results.

During a busy day, the nurse admits all of the following patients to the medical-surgical unit. Which patients are most important to refer to the dietitian for a complete nutritional assessment (select all that apply)? a. A 24-year-old who has a history of weight gains and losses b. A 53-year-old who complains of intermittent nausea for the past 2 days c. A 66-year-old who is admitted for débridement of an infected surgical wound d. A 45-year-old admitted with chest pain and possible myocardial infarction (MI) e. A 32-year-old with rheumatoid arthritis who takes prednisone (Deltasone) daily

ANS: A, C, E Weight fluctuations, use of corticosteroids, and draining or infected wounds all suggest that the patient may be at risk for malnutrition. Patients with chest pain or MI are not usually poorly nourished. Although vomiting that lasts 5 days places a patient at risk, nausea that has persisted for 2 days does not always indicate poor nutritional status or risk for health problems caused by poor nutrition.

The nurse knows that urinary tract infection (UTI) is the most common health care-associated infection because a. Catheterization procedures are performed more frequently than indicated. b. Escherichia coli pathogens are transmitted during surgical or catheterization procedures. C.Perineal care is often neglected by nursing staff. D.Bedpans and urinals are not stored properly and transmit infection.

ANS: B E. coli is the leading pathogen causing UTIs; this pathogen enters during procedures. Sterile technique is imperative to prevent the spread of infection. Frequent catheterizations can place a patient at high risk for UTI; however, infection is caused by bacteria, not by the procedure itself. Perineal care is important, and buildup of bacteria can lead to infection, but this is not the greatest cause. Bedpans and urinals may become bacteria ridden and should be cleaned frequently. Bedpans and urinals are not inserted into the urinary tract, so they are unlikely to be the primary cause of UTI.

Which of the following is the primary function of the kidney? a. Metabolizing and excreting medications b. Maintaining fluid and electrolyte balance c. Storing and excreting urine d. Filtering blood cells and proteins

ANS: B The main purpose of the kidney is to maintain fluid and electrolyte balance by filtering waste products and regulating pressures. The kidneys filter the byproducts of medication metabolism. The bladder stores and excretes urine. The kidneys help to maintain red blood cell volume by producing erythropoietin.

If obstructed, which component of the urination system would cause peristaltic waves? a. Kidney b. Ureters c. Bladder d. Urethra

ANS: B Ureters drain urine from the kidneys into the bladder; if they become obstructed, peristaltic waves attempt to push the obstruction into the bladder. The kidney, bladder, and urethra do not produce peristaltic waves. Obstruction of both bladder and urethra typically does not occur.

The health care provider prescribes finasteride (Proscar) for a 67-year-old patient who has benign prostatic hyperplasia (BPH). When teaching the patient about the drug, the nurse informs him that a. he should change position from lying to standing slowly to avoid dizziness. b. his interest in sexual activity may decrease while he is taking the medication. c. improvement in the obstructive symptoms should occur within about 2 weeks. d. he will need to monitor his blood pressure frequently to assess for hypertension.

ANS: B A decrease in libido is a side effect of finasteride because of the androgen suppression that occurs with the drug. Although orthostatic hypotension may occur if the patient is also taking a medication for erectile dysfunction (ED), it should not occur with finasteride alone. Improvement in symptoms of obstruction takes about 6 months. The medication does not cause hypertension.

A 45-year-old male patient with suspected acromegaly is seen at the clinic. To assist in making the diagnosis, which question should the nurse ask? a. "Have you had a recent head injury?" b. "Do you have to wear larger shoes now?" c. "Is there a family history of acromegaly?" d. "Are you experiencing tremors or anxiety?"

ANS: B Acromegaly causes an enlargement of the hands and feet. Head injury and family history are not risk factors for acromegaly. Tremors and anxiety are not clinical manifestations of acromegaly.

The nurse recognizes that teaching a 44-year-old woman following a laparoscopic cholecystectomy has been effective when the patient states which of the following? a. "I can expect yellow-green drainage from the incision for a few days." b. "I can remove the bandages on my incisions tomorrow and take a shower." c. "I should plan to limit my activities and not return to work for 4 to 6 weeks." d. "I will always need to maintain a low-fat diet since I no longer have a gallbladder."

ANS: B After a laparoscopic cholecystectomy, the patient will have Band-Aids in place over the incisions. Patients are discharged the same (or next) day and have few restrictions on activities of daily living. Drainage from the incisions would be abnormal, and the patient should be instructed to call the health care provider if this occurs. A low-fat diet may be recommended for a few weeks after surgery but will not be a life-long requirement

The nurse is planning postoperative care for a patient who is being admitted to the surgical unit form the recovery room after transsphenoidal resection of a pituitary tumor. Which nursing action should be included? a. Palpate extremities for edema. b. Measure urine volume every hour. c. Check hematocrit every 2 hours for 8 hours. d. Monitor continuous pulse oximetry for 24 hours.

ANS: B After pituitary surgery, the patient is at risk for diabetes insipidus caused by cerebral edema. Monitoring of urine output and urine specific gravity is essential. Hemorrhage is not a common problem. There is no need to check the hematocrit hourly. The patient is at risk for dehydration, not volume overload. The patient is not at high risk for problems with oxygenation, and continuous pulse oximetry is not needed.

Parenteral nutrition (PN) containing amino acids and dextrose was ordered and hung 24 hours ago for a malnourished patient. The nurse observes that about 50 mL remain in the PN container. Which action is best for the nurse to take? a. Ask the health care provider to clarify the written PN order. b. Add a new container of PN using the current tubing and filter. c. Hang a new container of PN and change the IV tubing and filter. d. Infuse the remaining 50 mL and then hang a new container of PN.

ANS: B All PN solutions are changed at 24 hours. PN solutions containing dextrose and amino acids require a change in tubing and filter every 72 hours rather than daily. Infusion of the additional 50 mL will increase patient risk for infection. Changing the IV tubing and filter more frequently than required will unnecessarily increase costs. The nurse (not the health care provider) is responsible for knowing the indicated times for tubing and filter changes.

Before administration of captopril (Capoten) to a patient with stage 2 chronic kidney disease (CKD), the nurse will check the patient's a. glucose. b. potassium. c. creatinine. d. phosphate.

ANS: B Angiotensin-converting enzyme (ACE) inhibitors are frequently used in patients with CKD because they delay the progression of the CKD, but they cause potassium retention. Therefore careful monitoring of potassium levels is needed in patients who are at risk for hyperkalemia. The other laboratory values would also be monitored in patients with CKD but would not affect whether the captopril was given or not.

Which information given by a 70-year-old patient during a health history indicates to the nurse that the patient should be screened for hepatitis C? a. The patient had a blood transfusion in 2005. b. The patient used IV drugs about 20 years ago. c. The patient frequently eats in fast-food restaurants. d. The patient traveled to a country with poor sanitation.

ANS: B Any patient with a history of IV drug use should be tested for hepatitis C. Blood transfusions given after 1992 (when an antibody test for hepatitis C became available) do not pose a risk for hepatitis C. Hepatitis C is not spread by the oral-fecal route and therefore is not caused by contaminated food or by traveling in underdeveloped countries

The nurse is caring for a 73-year-old man who has cirrhosis. Which data obtained by the nurse during the assessment will be of most concern? a. The patient complains of right upper-quadrant pain with palpation. b. The patient's hands flap back and forth when the arms are extended. c. The patient has ascites and a 2-kg weight gain from the previous day. d. The patient's skin has multiple spider-shaped blood vessels on the abdomen.

ANS: B Asterixis indicates that the patient has hepatic encephalopathy, and hepatic coma may occur. The spider angiomas and right upper quadrant abdominal pain are not unusual for the patient with cirrhosis and do not require a change in treatment. The ascites and weight gain indicate the need for treatment but not as urgently as the changes in neurologic status

The nurse preparing for the annual physical exam of a 50-year-old man will plan to teach the patient about a. endoscopy. b. colonoscopy. c. computerized tomography screening. d. carcinoembryonic antigen (CEA) testing.

ANS: B At age 50, individuals with an average risk for colorectal cancer (CRC) should begin screening for CRC. Colonoscopy is the gold standard for CRC screening. The other diagnostic tests are not recommended as part of a routine annual physical exam at age 50.

A patient complains of leg cramps during hemodialysis. The nurse should first a. massage the patient's legs. b. reposition the patient supine. c. give acetaminophen (Tylenol). d. infuse a bolus of normal saline.

ANS: D Muscle cramps during dialysis are caused by rapid removal of sodium and water. Treatment includes infusion of normal saline. The other actions do not address the reason for the cramps.

A 34-year-old has a new diagnosis of type 2 diabetes. The nurse will discuss the need to schedule a dilated eye exam a. every 2 years. b. as soon as possible. c. when the patient is 39 years old. d. within the first year after diagnosis.

ANS: B Because many patients have some diabetic retinopathy when they are first diagnosed with type 2 diabetes, a dilated eye exam is recommended at the time of diagnosis and annually thereafter. Patients with type 1 diabetes should have dilated eye exams starting 5 years after they are diagnosed and then annually.

Which finding for a patient who has been taking orlistat (Xenical) is most important to report to the health care provider? a. The patient frequently has liquid stools. b. The patient is pale and has many bruises. c. The patient complains of bloating after meals. d. The patient is experiencing a weight loss plateau.

ANS: B Because orlistat blocks the absorption of fat-soluble vitamins, the patient may not be receiving an adequate amount of vitamin K, resulting in a decrease in clotting factors. Abdominal bloating and liquid stools are common side effects of orlistat and indicate that the nurse should remind the patient that fat in the diet may increase these side effects. Weight loss plateaus are normal during weight reduction

A patient in the emergency department has just been diagnosed with peritonitis caused by a ruptured diverticulum. Which prescribed intervention will the nurse implement first? a. Insert a urinary catheter to drainage. b. Infuse metronidazole (Flagyl) 500 mg IV. c. Send the patient for a computerized tomography scan. d. Place a nasogastric (NG) tube to intermittent low suction.

ANS: B Because peritonitis can be fatal if treatment is delayed, the initial action should be to start antibiotic therapy (after any ordered cultures are obtained). The other actions can be done after antibiotic therapy is initiated

A 36-year-old female patient is receiving treatment for chronic hepatitis C with pegylated interferon (PEG-Intron, Pegasys), ribavirin (Rebetol), and telaprevir (Incivek). Which finding is most important to communicate to the health care provider? a. Weight loss of 2 lb (1 kg) b. Positive urine pregnancy test c. Hemoglobin level of 10.4 g/dL d. Complaints of nausea and anorexia

ANS: B Because ribavirin is teratogenic, the medication will need to be discontinued immediately. Anemia, weight loss, and nausea are common adverse effects of the prescribed regimen and may require actions such as patient teaching, but they would not require immediate cessation of the therapy.

Which information in a patient's history indicates to the nurse that the patient is not an appropriate candidate for kidney transplantation? a. The patient has type 1 diabetes. b. The patient has metastatic lung cancer. c. The patient has a history of chronic hepatitis C infection. d. The patient is infected with the human immunodeficiency virus.

ANS: B Disseminated malignancies are a contraindication to transplantation. The conditions of the other patients are not contraindications for kidney transplant.

After change-of-shift report, which patient should the nurse assess first? a. 19-year-old with type 1 diabetes who has a hemoglobin A1C of 12% b. 23-year-old with type 1 diabetes who has a blood glucose of 40 mg/dL c. 40-year-old who is pregnant and whose oral glucose tolerance test is 202 mg/dL d. 50-year-old who uses exenatide (Byetta) and is complaining of acute abdominal pain

ANS: B Because the brain requires glucose to function, untreated hypoglycemia can cause unconsciousness, seizures, and death. The nurse will rapidly assess and treat the patient with low blood glucose. The other patients also have symptoms that require assessments and/or interventions, but they are not at immediate risk for life-threatening complications

The nurse will plan to teach a 34-year-old patient diagnosed with stage 0 cervical cancer about a. radiation. b. conization. c. chemotherapy. d. radical hysterectomy.

ANS: B Because the carcinoma is in situ, conization can be used for treatment. Radical hysterectomy, chemotherapy, or radiation will not be needed

A 49-year-old man has been admitted with hypotension and dehydration after 3 days of nausea and vomiting. Which order from the health care provider will the nurse implement first? a. Insert a nasogastric (NG) tube. b. Infuse normal saline at 250 mL/hr. c. Administer IV ondansetron (Zofran). d. Provide oral care with moistened swabs.

ANS: B Because the patient has severe dehydration, rehydration with IV fluids is the priority. The other orders should be accomplished as quickly as possible after the IV fluids are initiated

A 58-year-old male patient who weighs 242 lb (110 kg) undergoes a nephrectomy for massive kidney trauma due to a motor vehicle crash. Which postoperative assessment finding is most important to communicate to the surgeon? a. Blood pressure is 102/58. b. Urine output is 20 mL/hr for 2 hours. c. Incisional pain level is reported as 9/10. d. Crackles are heard at bilateral lung bases.

ANS: B Because the urine output should be at least 0.5 mL/kg/hr, a 40 mL output for 2 hours indicates that the patient may have decreased renal perfusion because of bleeding, inadequate fluid intake, or obstruction at the suture site. The blood pressure requires ongoing monitoring but does not indicate inadequate perfusion at this time. The patient should cough and deep breathe, but the crackles do not indicate a need for an immediate change in therapy. The incisional pain should be addressed, but this is not as potentially life threatening as decreased renal perfusion. In addition, the nurse can medicate the patient for pain.

A 35-year-old female patient with a possible pituitary adenoma is scheduled for a computed tomography (CT) scan with contrast media. Which patient information is most important for the nurse to communicate to the health care provider before the test? a. Bilateral poor peripheral vision b. Allergies to iodine and shellfish c. Recent weight loss of 20 pounds d. Complaint of ongoing headaches

ANS: B Because the usual contrast media is iodine-based, the health care provider will need to know about the allergy before the CT scan. The other findings are common with any mass in the brain such as a pituitary adenoma.

The nurse is preparing to teach a 43-year-old man who is newly diagnosed with type 2 diabetes about home management of the disease. Which action should the nurse take first? a. Ask the patient's family to participate in the diabetes education program. b. Assess the patient's perception of what it means to have diabetes mellitus. c. Demonstrate how to check glucose using capillary blood glucose monitoring. d. Discuss the need for the patient to actively participate in diabetes management.

ANS: B Before planning teaching, the nurse should assess the patient's interest in and ability to self-manage the diabetes. After assessing the patient, the other nursing actions may be appropriate, but planning needs to be individualized to each patient.

Which nursing action is appropriate when coaching obese adults enrolled in a behavior modification program? a. Having the adults write down the caloric intake of each meal b. Asking the adults about situations that tend to increase appetite c. Suggesting that the adults plan rewards, such as sugarless candy, for achieving their goals d. Encouraging the adults to eat small amounts frequently rather than having scheduled meals

ANS: B Behavior modification programs focus on how and when the person eats and de-emphasize aspects such as calorie counting. Nonfood rewards are recommended for achievement of weight-loss goals. Patients are often taught to restrict eating to designated meals when using behavior modification

Which information obtained by the nurse interviewing a 30-year-old male patient is most important to communicate to the health care provider? a. The patient has a history of constipation. b. The patient has noticed blood in the stools. c. The patient had an appendectomy at age 27. d. The patient smokes a pack/day of cigarettes.

ANS: B Blood in the stools is a possible clinical manifestation of colorectal cancer and requires further assessment by the health care provider. The other patient information will also be communicated to the health care provider, but does not indicate an urgent need for further testing or intervention

A 74-year-old patient preparing to undergo a colon resection for cancer of the colon asks about the elevated carcinoembryonic antigen (CEA) test result. The nurse explains that the test is used to a. identify any metastasis of the cancer. b. monitor the tumor status after surgery. c. confirm the diagnosis of a specific type of cancer. d. determine the need for postoperative chemotherapy.

ANS: B CEA is used to monitor for cancer recurrence after surgery. CEA levels do not help to determine whether there is metastasis of the cancer. Confirmation of the diagnosis is made on the basis of biopsy. Chemotherapy use is based on factors other than CEA

Which information will the nurse monitor in order to determine the effectiveness of prescribed calcium carbonate (Caltrate) for a patient with chronic kidney disease (CKD)? a. Blood pressure b. Phosphate level c. Neurologic status d. Creatinine clearance

ANS: B Calcium carbonate is prescribed to bind phosphorus and prevent mineral and bone disease in patients with CKD. The other data will not be helpful in evaluating the effectiveness of calcium carbonate.

The nurse is assessing a 41-year-old African American male patient diagnosed with a pituitary tumor causing panhypopituitarism. Assessment findings consistent with panhypopituitarism include a. high blood pressure. b. decreased facial hair. c. elevated blood glucose. d. tachycardia and cardiac palpitations.

ANS: B Changes in male secondary sex characteristics such as decreased facial hair, testicular atrophy, diminished spermatogenesis, loss of libido, impotence, and decreased muscle mass are associated with decreases in follicle stimulating hormone (FSH) and luteinizing hormone (LH). Fasting hypoglycemia and hypotension occur in panhypopituitarism as a result of decreases in adrenocorticotropic hormone (ACTH) and cortisol. Bradycardia is likely due to the decrease in thyroid stimulating hormone (TSH) and thyroid hormones associated with panhypopituitarism.

Which nursing action should the nurse who is caring for a patient who has had an ileal conduit for several years delegate to nursing assistive personnel (NAP)? a. Assess for symptoms of urinary tract infection (UTI). b. Change the ostomy appliance. c. Choose the appropriate ostomy bag. d. Monitor the appearance of the stoma.

ANS: B Changing the ostomy appliance for a stable patient could be done by NAP. Assessments of the site, choosing the appropriate ostomy bag, and assessing for (UTI) symptoms require more education and scope of practice and should be done by the RN. DIF: Cognitive Level: Application REF: 1157 | 1159-1160 | 1158

How will the nurse assess for flank tenderness in a 30-year-old female patient with suspected pyelonephritis? a. Palpate along both sides of the lumbar vertebral column. b. Strike a flat hand covering the costovertebral angle (CVA). c. Push fingers upward into the two lowest intercostal spaces. d. Percuss between the iliac crest and ribs along the midaxillary line.

ANS: B Checking for flank pain is best performed by percussion of the CVA and asking about pain. The other techniques would not assess for flank pain

A patient who has been NPO during treatment for nausea and vomiting caused by gastric irritation is to start oral intake. Which of these should the nurse offer to the patient? a. A glass of orange juice b. A dish of lemon gelatin c. A cup of coffee with cream d. A bowl of hot chicken broth

ANS: B Clear liquids are usually the first foods started after a patient has been nauseated. Acidic foods such as orange juice, very hot foods, and coffee are poorly tolerated when patients have been nauseated.

A patient who has had a transurethral resection with fulguration for bladder cancer 3 days previously calls the nurse at the urology clinic. Which information given by the patient is most important to report to the health care provider? a. The patient is using opioids for pain. b. The patient has noticed clots in the urine. c. The patient is very anxious about the cancer. d. The patient is voiding every 4 hours at night.

ANS: B Clots in the urine are not expected and require further follow-up. Voiding every 4 hours, use of opioids for pain, and anxiety are typical after this procedure. DIF: Cognitive Level: Application REF: 1145-1146

A female patient with chronic kidney disease (CKD) is receiving peritoneal dialysis with 2 L inflows. Which information should the nurse report immediately to the health care provider? a. The patient has an outflow volume of 1800 mL. b. The patient's peritoneal effluent appears cloudy. c. The patient has abdominal pain during the inflow phase. d. The patient's abdomen appears bloated after the inflow.

ANS: B Cloudy appearing peritoneal effluent is a sign of peritonitis and should be reported immediately so that treatment with antibiotics can be started. The other problems can be addressed through nursing interventions such as slowing the inflow and repositioning the patient.

The nurse determines that teaching regarding cobalamin injections has been effective when the patient with chronic atrophic gastritis states which of the following? a. "The cobalamin injections will prevent gastric inflammation." b. "The cobalamin injections will prevent me from becoming anemic." c. "These injections will increase the hydrochloric acid in my stomach." d. "These injections will decrease my risk for developing stomach cancer."

ANS: B Cobalamin supplementation prevents the development of pernicious anemia. Chronic gastritis may cause achlorhydria, but cobalamin does not correct this. The loss of intrinsic factor secretion with chronic gastritis is permanent, and the patient will need lifelong supplementation with cobalamin. The incidence of stomach cancer is higher in patients with chronic gastritis, but cobalamin does not reduce the risk for stomach cancer. DIF: Cognitive Level: Apply (application) REF: 941-942 TOP: Nursing Process: Evaluation MSC: NCLEX: Physiological Integrity

Which information about a 30-year-old patient who is scheduled for an oral glucose tolerance test should be reported to the health care provider before starting the test? a. The patient reports having occasional orthostatic dizziness. b. The patient takes oral corticosteroids for rheumatoid arthritis. c. The patient has had a 10-pound weight gain in the last month. d. The patient drank several glasses of water an hour previously.

ANS: B Corticosteroids can affect blood glucose results. The other information will be provided to the health care provider but will not affect the test results.

Which information will the nurse prioritize in planning preoperative teaching for a patient undergoing a Roux-en-Y gastric bypass? a. Educating the patient about the nasogastric (NG) tube b. Instructing the patient on coughing and breathing techniques c. Discussing necessary postoperative modifications in lifestyle d. Demonstrating passive range-of-motion exercises for the legs

ANS: B Coughing and deep breathing can prevent major postoperative complications such as carbon monoxide retention and hypoxemia. Information about passive range of motion, the NG tube, and postoperative modifications in lifestyle will also be discussed, but avoidance of respiratory complications is the priority goal after surgery

The nurse is assessing a 31-year-old female patient with abdominal pain. Th nurse,who notes that there is ecchymosis around the area of umbilicus, will document this finding as a. Cullen sign. b. Rovsing sign. c. McBurney sign. d. Grey-Turner's signt.

ANS: B Cullen sign is ecchymosis around the umbilicus. Rovsing sign occurs when palpation of the left lower quadrant causes pain in the right lower quadrant. Deep tenderness at McBurney's point (halfway between the umbilicus and the right iliac crest), known as McBurney's sign, is a sign of acute appendicitis

Which of these assessment findings in a patient with a hiatal hernia who returned from a laparoscopic Nissen fundoplication 4 hours ago is most important for the nurse to address immediately? a. The patient is experiencing intermittent waves of nausea. b. The patient has absent breath sounds throughout the left lung. c. The patient has decreased bowel sounds in all four quadrants. d. The patient complains of 6/10 (0 to 10 scale) abdominal pain.

ANS: B Decreased breath sounds on one side may indicate a pneumothorax, which requires rapid diagnosis and treatment. The abdominal pain and nausea also should be addressed but they are not as high priority as the patient's respiratory status. The patient's decreased bowel sounds are expected after surgery and require ongoing monitoring but no other action.

The nurse determines that demeclocycline (Declomycin) is effective for a patient with syndrome of inappropriate antidiuretic hormone (SIADH) based on finding that the patient's a. weight has increased. b. urinary output is increased. c. peripheral edema is decreased. d. urine specific gravity is increased.

ANS: B Demeclocycline blocks the action of antidiuretic hormone (ADH) on the renal tubules and increases urine output. An increase in weight or an increase in urine specific gravity indicates that the SIADH is not corrected. Peripheral edema does not occur with SIADH. A sudden weight gain without edema is a common clinical manifestation of this disorder.

A patient has arrived for a scheduled hemodialysis session. Which nursing action is most appropriate for the registered nurse (RN) to delegate to a dialysis technician? a. Teach the patient about fluid restrictions. b. Check blood pressure before starting dialysis. c. Assess for causes of an increase in predialysis weight. d. Determine the ultrafiltration rate for the hemodialysis.

ANS: B Dialysis technicians are educated in monitoring for blood pressure. Assessment, adjustment of the appropriate ultrafiltration rate, and patient teaching require the education and scope of practice of an RN.

A 58-year-old patient who has been recently diagnosed with benign prostatic hyperplasia (BPH) tells the nurse that he does not want to have a transurethral resection of the prostate (TURP) because it might affect his ability to maintain an erection during intercourse. Which action should the nurse take? a. Provide teaching about medications for erectile dysfunction (ED). b. Discuss that TURP does not commonly affect erectile function. c. Offer reassurance that sperm production is not affected by TURP. d. Discuss alternative methods of sexual expression besides intercourse.

ANS: B ED is not a concern with TURP, although retrograde ejaculation is likely and the nurse should discuss this with the patient. Erectile function is not usually affected by a TURP, so the patient will not need information about penile implants or reassurance that other forms of sexual expression may be used. Because the patient has not asked about fertility, reassurance about sperm production does not address his concerns.

Which action should the nurse take first in order to improve calorie and protein intake for a patient who eats only about 50% of each meal because of "feeling too tired to eat much." a. Teach the patient about the importance of good nutrition. b. Serve multiple small feedings of high-calorie, high-protein foods. c. Obtain an order for enteral feedings of liquid nutritional supplements. d. Consult with the health care provider about providing parenteral nutrition (PN).

ANS: B Eating small amounts of food frequently throughout the day is less fatiguing and will improve the patient's ability to take in more nutrients. Teaching the patient may be appropriate, but will not address the patient's inability to eat more because of fatigue. Tube feedings or PN may be needed if the patient is unable to take in enough nutrients orally, but increasing the oral intake should be attempted first.

Which assessment action will help the nurse determine if an obese patient has metabolic syndrome? a. Take the patient's apical pulse. b. Check the patient's blood pressure. c. Ask the patient about dietary intake. d. Dipstick the patient's urine for protein.

ANS: B Elevated blood pressure is one of the characteristics of metabolic syndrome. The other information also may be obtained by the nurse, but it will not assist with the diagnosis of metabolic syndrome

A 30-year-old patient seen in the emergency department for severe headache and acute confusion is found to have a serum sodium level of 118 mEq/L. The nurse will anticipate the need for which diagnostic test? a. Urinary 17-ketosteroids b. Antidiuretic hormone level c. Growth hormone stimulation test d. Adrenocorticotropic hormone level

ANS: B Elevated levels of antidiuretic hormone will cause water retention and decrease serum sodium levels. The other tests would not be helpful in determining possible causes of the patient's hyponatremia.

Which nursing action should be included in the postoperative plan of care for a patient after a laparoscopic esophagectomy? a. Notify the doctor about bloody nasogastric (NG) drainage. b. Elevate the head of the bed to at least 30 degrees. c. Reposition the NG tube if drainage stops. d. Start oral fluids when the patient has active bowel sounds.

ANS: B Elevation of the head of the bed decreases the risk for reflux and aspiration of gastric secretions. The NG tube should not be repositioned without consulting with the health care provider. Bloody NG drainage is expected for the first 8 to 12 hours. A swallowing study is needed before oral fluids are started

The cardiac telemetry unit charge nurse receives status reports from other nursing units about four patients who need cardiac monitoring. Which patient should be transferred to the cardiac unit first? a. Patient with Hashimoto's thyroiditis and a heart rate of 102 b. Patient with tetany who has a new order for IV calcium chloride c. Patient with Cushing syndrome and a blood glucose of 140 mg/dL d. Patient with Addison's disease who takes hydrocortisone twice daily

ANS: B Emergency treatment of tetany requires IV administration of calcium; ECG monitoring will be required because cardiac arrest may occur if high calcium levels result from too-rapid administration. The information about the other patients indicates that they are more stable than the patient with tetany.

After a total proctocolectomy and permanent ileostomy, the patient tells the nurse, "I cannot manage all these changes. I don't want to look at the stoma." What is the best action by the nurse? a. Reassure the patient that ileostomy care will become easier. b. Ask the patient about the concerns with stoma management. c. Develop a detailed written list of ostomy care tasks for the patient. d. Postpone any teaching until the patient adjusts to the ileostomy.

ANS: B Encouraging the patient to share concerns assists in helping the patient adjust to the body changes. Acknowledgment of the patient's feelings and concerns is important rather than offering false reassurance. Because the patient indicates that the feelings about the ostomy are the reason for the difficulty with the many changes, development of a detailed ostomy care plan will not improve the patient's ability to manage the ostomy. Although detailed ostomy teaching may be postponed, the nurse should offer teaching about some aspects of living with an ostomy.

Which action should the nurse take to evaluate treatment effectiveness for a patient who has hepatic encephalopathy? a. Request that the patient stand on one foot. b. Ask the patient to extend both arms forward. c. Request that the patient walk with eyes closed. d. Ask the patient to perform the Valsalva maneuver.

ANS: B Extending the arms allows the nurse to check for asterixis, a classic sign of hepatic encephalopathy. The other tests might also be done as part of the neurologic assessment but would not be diagnostic for hepatic encephalopathy.

A patient with a body mass index (BMI) of 17 kg/m2 and a low albumin level is being admitted by the nurse. Which assessment finding will the nurse expect to find? a. Restlessness b. Hypertension c. Pitting edema d. Food allergies

ANS: C Edema occurs when serum albumin levels and plasma oncotic pressure decrease. The blood pressure and level of consciousness are not directly affected by malnutrition. Food allergies are not an indicator of nutritional status.

A family member of a 28-year-old patient who has suffered massive abdominal trauma in an automobile accident asks the nurse why the patient is receiving famotidine (Pepcid). The nurse will explain that the medication will a. decrease nausea and vomiting. b. inhibit development of stress ulcers. c. lower the risk for H. pylori infection. d. prevent aspiration of gastric contents.

ANS: B Famotidine is administered to prevent the development of physiologic stress ulcers, which are associated with a major physiologic insult such as massive trauma. Famotidine does not decrease nausea or vomiting, prevent aspiration, or prevent H. pylori infection

The family member of a patient who has suffered massive abdominal trauma in an automobile accident asks the nurse why the patient is receiving famotidine (Pepcid). The nurse will explain that the medication will a. prevent aspiration of gastric contents. b. inhibit the development of stress ulcers. c. lower the chance for H. pylori infection. d. decrease the risk for nausea and vomiting.

ANS: B Famotidine is administered to prevent the development of physiologic stress ulcers, which are associated with a major physiologic insult such as massive trauma. Famotidine does not decrease nausea or vomiting, prevent aspiration, or prevent H. pylori infection.

A 54-year-old critically ill patient with sepsis is frequently incontinent of watery stools. What action by the nurse will prevent complications associated with ongoing incontinence? a. Apply incontinence briefs. b. Use a fecal management system c. Insert a rectal tube with a drainage bag. d. Assist the patient to a commode frequently.

ANS: B Fecal management systems are designed to contain loose stools and can be in place for as long as 4 weeks without causing damage to the rectum or anal sphincters. Although incontinence briefs may be helpful, unless they are changed frequently, they are likely to increase the risk for skin breakdown. Rectal tubes are avoided because of possible damage to the anal sphincter and ulceration of the rectal mucosa. A critically ill patient will not be able to tolerate getting up frequently to use the commode or bathroom.

A 24-year-old woman with Crohn's disease develops a fever and symptoms of a urinary tract infection (UTI) with tan, fecal-smelling urine. What information will the nurse add to a general teaching plan about UTIs in order to individualize the teaching for this patient? a. Bacteria in the perianal area can enter the urethra. b. Fistulas can form between the bowel and bladder. c. Drink adequate fluids to maintain normal hydration. d. Empty the bladder before and after sexual intercourse.

ANS: B Fistulas between the bowel and bladder occur in Crohn's disease and can lead to UTI. Teaching for UTI prevention in general includes good hygiene, adequate fluid intake, and voiding before and after intercourse

A patient with nephrotic syndrome develops flank pain. The nurse will anticipate teaching the patient about treatment with a. antibiotics. b. anticoagulants. c. corticosteroids. d. antihypertensives.

ANS: B Flank pain in a patient with nephrosis suggests a renal vein thrombosis, and anticoagulation is needed. Antibiotics are used to treat a patient with flank pain caused by pyelonephritis. Antihypertensives are used if the patient has high blood pressure. Corticosteroids may be used to treat nephrotic syndrome but will not resolve a thrombosis. DIF: Cognitive Level: Application REF: 1133-1134

A 22-year-old female patient seen in the clinic for a bladder infection describes the following symptoms. Which information is most important for the nurse to report to the health care provider? a. Urinary urgency b. Left-sided flank pain c. Intermittent hematuria d. Burning with urination

ANS: B Flank pain indicates that the patient may have developed pyelonephritis as a complication of the bladder infection. The other clinical manifestations are consistent with a lower urinary tract infection (UTI).

The nurse is titrating the IV fluid infusion rate immediately after a patient has had kidney transplantation. Which parameter will be most important for the nurse to consider? a. Heart rate b. Urine output c. Creatinine clearance d. Blood urea nitrogen (BUN) level

ANS: B Fluid volume is replaced based on urine output after transplant because the urine output can be as high as a liter an hour. The other data will be monitored but are not the most important determinants of fluid infusion rate.

An unresponsive patient with type 2 diabetes is brought to the emergency department and diagnosed with hyperosmolar hyperglycemic syndrome (HHS). The nurse will anticipate the need to a. give a bolus of 50% dextrose. b. insert a large-bore IV catheter. c. initiate oxygen by nasal cannula. d. administer glargine (Lantus) insulin.

ANS: B HHS is initially treated with large volumes of IV fluids to correct hypovolemia. Regular insulin is administered, not a long-acting insulin. There is no indication that the patient requires oxygen. Dextrose solutions will increase the patient's blood glucose and would be contraindicated

Which information will the nurse include in teaching a patient who had a proctocolectomy and ileostomy for ulcerative colitis? a. Restrict fluid intake to prevent constant liquid drainage from the stoma. b. Use care when eating high-fiber foods to avoid obstruction of the ileum. c. Irrigate the ileostomy daily to avoid having to wear a drainage appliance. d. Change the pouch every day to prevent leakage of contents onto the skin.

ANS: B High-fiber foods are introduced gradually and should be well chewed to avoid obstruction of the ileostomy. Patients with ileostomies lose the absorption of water in the colon and need to take in increased amounts of fluid. The pouch should be drained frequently but is changed every 5 to 7 days. The drainage from an ileostomy is liquid and continuous, so control by irrigation is not possible

A 62-year-old female patient has been hospitalized for 8 days with acute kidney injury (AKI) caused by dehydration. Which information will be most important for the nurse to report to the health care provider? a. The creatinine level is 3.0 mg/dL. b. Urine output over an 8-hour period is 2500 mL. c. The blood urea nitrogen (BUN) level is 67 mg/dL. d. The glomerular filtration rate is <30 mL/min/1.73m2.

ANS: B The high urine output indicates a need to increase fluid intake to prevent hypovolemia. The other information is typical of AKI and will not require a change in therapy.

After teaching a patient with interstitial cystitis about management of the condition, the nurse determines that further instruction is needed when the patient says, a. "I will have to stop having coffee and orange juice for breakfast." b. "I should start taking a high potency multiple vitamin every morning." c. "I will buy some calcium glycerophosphate (Prelief) at the pharmacy." d. "I should call the doctor about increased bladder pain or odorous urine."

ANS: B High-potency multiple vitamins may irritate the bladder and increase symptoms. The other patient statements indicate good understanding of the teaching.

When planning teaching for a 59-year-old male patient with benign nephrosclerosis the nurse should include instructions regarding a. preventing bleeding with anticoagulants. b. monitoring and recording blood pressure. c. obtaining and documenting daily weights. d. measuring daily intake and output volumes.

ANS: B Hypertension is the major symptom of nephrosclerosis. Measurements of intake and output and daily weights are not necessary unless the patient develops renal insufficiency. Anticoagulants are not used to treat nephrosclerosis.

Which nursing assessment of a 69-year-old patient is most important to make during initiation of thyroid replacement with levothyroxine (Synthroid)? a. Fluid balance b. Apical pulse rate c. Nutritional intake d. Orientation and alertness

ANS: B In older patients, initiation of levothyroxine therapy can increase myocardial oxygen demand and cause angina or dysrhythmias. The medication also is expected to improve mental status and fluid balance and will increase metabolic rate and nutritional needs, but these changes will not result in potentially life-threatening complications.

After receiving change-of-shift report about the following four patients, which patient should the nurse assess first? a. A 31-year-old female with Cushing syndrome and a blood glucose level of 244 mg/dL b. A 70-year-old female taking levothyroxine (Synthroid) who has an irregular pulse of 134 c. A 53-year-old male who has Addison's disease and is due for a scheduled dose of hydrocortisone (Solu-Cortef). d. A 22-year-old male admitted with syndrome of inappropriate antidiuretic hormone (SIADH) who has a serum sodium level of 130 mEq/L

ANS: B Initiation of thyroid replacement in older adults may cause angina and cardiac dysrhythmias. The patient's high pulse rate needs rapid investigation by the nurse to assess for and intervene with any cardiac problems. The other patients also require nursing assessment and/or actions but are not at risk for life-threatening complications.

A patient gives the nurse health information before a scheduled intravenous pyelogram (IVP). Which item has the most immediate implications for the patient's care? a. The patient has not had food or drink for 8 hours. b. The patient lists allergies to shellfish and penicillin. c. The patient complains of costovertebral angle (CVA) tenderness. d. The patient used a bisacodyl (Dulcolax) tablet the previous night.

ANS: B Iodine-based contrast dye is used during IVP and for many computed tomography (CT) scans. The nurse will need to notify the health care provider before the procedures so that the patient can receive medications such as antihistamines or corticosteroids before the procedures are started. The other information is also important to note and document but does not have immediate implications for the patient's care during the procedures

Which information will the nurse teach a 23-year-old patient with lactose intolerance? a. Ice cream is relatively low in lactose. b. Live-culture yogurt is usually tolerated. c. Heating milk will break down the lactose. d. Nonfat milk is a better choice than whole milk.

ANS: B Lactose-intolerant individuals can usually eat yogurt without experiencing discomfort. Ice cream, nonfat milk, and milk that has been heated are all high in lactose

The nurse in the dialysis clinic is reviewing the home medications of a patient with chronic kidney disease (CKD). Which medication reported by the patient indicates that patient teaching is required? a. Multivitamin with iron b. Magnesium hydroxide c. Acetaminophen (Tylenol) d. Calcium phosphate (PhosLo)

ANS: B Magnesium is excreted by the kidneys, and patients with CKD should not use over-the-counter products containing magnesium. The other medications are appropriate for a patient with CKD.

A 30-year-old man is being admitted to the hospital for elective knee surgery. Which assessment finding is most important to report to the health care provider? a. Tympany on percussion of the abdomen b. Liver edge 3 cm below the costal margin c. Bowel sounds of 20/minute in each quadrant d. Aortic pulsations visible in the epigastric area

ANS: B Normally the lower border of the liver is not palpable below the ribs, so this finding suggests hepatomegaly. The other findings are within normal range for the physical assessment

A patient who had radical neck surgery to remove a malignant tumor developed hypoparathyroidism. The nurse should plan to teach the patient about a. bisphosphonates to reduce bone demineralization. b. calcium supplements to normalize serum calcium levels. c. increasing fluid intake to decrease risk for nephrolithiasis. d. including whole grains in the diet to prevent constipation.

ANS: B Oral calcium supplements are used to maintain the serum calcium in normal range and prevent the complications of hypocalcemia. Whole grain foods decrease calcium absorption and will not be recommended. Bisphosphonates will lower serum calcium levels further by preventing calcium from being reabsorbed from bone. Kidney stones are not a complication of hypoparathyroidism and low calcium levels.

After change-of-shift report, which patient should the nurse assess first? a. 40-year-old male with celiac disease who has frequent frothy diarrhea b. 30-year-old female with a femoral hernia who has abdominal pain and vomiting c. 30-year-old male with ulcerative colitis who has severe perianal skin breakdown d. 40-year-old female with a colostomy bag that is pulling away from the adhesive wafer

ANS: B Pain and vomiting with a femoral hernia suggest possible strangulation, which will necessitate emergency surgery. The other patients have less urgent problems

Which action by a new registered nurse (RN) caring for a patient with a goiter and possible hyperthyroidism indicates that the charge nurse needs to do more teaching? a. The RN checks the blood pressure on both arms. b. The RN palpates the neck thoroughly to check thyroid size. c. The RN lowers the thermostat to decrease the temperature in the room. d. The RN orders nonmedicated eye drops to lubricate the patient's bulging eyes.

ANS: B Palpation can cause the release of thyroid hormones in a patient with an enlarged thyroid and should be avoided. The other actions by the new RN are appropriate when caring for a patient with an enlarged thyroid.

The nurse is taking a health history from a 29-year-old pregnant patient at the first prenatal visit. The patient reports no personal history of diabetes but has a parent who is diabetic. Which action will the nurse plan to take first? a. Teach the patient about administering regular insulin. b. Schedule the patient for a fasting blood glucose level. c. Discuss an oral glucose tolerance test for the twenty-fourth week of pregnancy. d. Provide teaching about an increased risk for fetal problems with gestational diabetes.

ANS: B Patients at high risk for gestational diabetes should be screened for diabetes on the initial prenatal visit. An oral glucose tolerance test may also be used to check for diabetes, but it would be done before the twenty-fourth week. The other actions may also be needed (depending on whether the patient develops gestational diabetes), but they are not the first actions that the nurse should take

After the home health nurse teaches a patient with a neurogenic bladder how to use intermittent catheterization for bladder emptying, which patient statement indicates that the teaching has been effective? a. "I will use a sterile catheter and gloves for each time I self-catheterize." b. "I will clean the catheter carefully before and after each catheterization." c. "I will need to buy seven new catheters weekly and use a new one every day." d. "I will need to take prophylactic antibiotics to prevent any urinary tract infections."

ANS: B Patients who are at home can use a clean technique for intermittent self-catheterization and change the catheter every 7 days. There is no need to use a new catheter every day, to use sterile catheters, or to take prophylactic antibiotics. DIF: Cognitive Level: Application REF: 1154

A new 19-year-old male patient has familial adenomatous polyposis (FAP). Which action will the nurse in the gastrointestinal clinic include in the plan of care? a. Obtain blood samples for DNA analysis. b. Schedule the patient for yearly colonoscopy. c. Provide preoperative teaching about total colectomy. d. Discuss lifestyle modifications to decrease cancer risk.

ANS: B Patients with FAP should have annual colonoscopy starting at age 16 and usually have total colectomy by age 25 to avoid developing colorectal cancer. DNA analysis is used to make the diagnosis, but is not needed now for this patient. Lifestyle modifications will not decrease cancer risk for this patient.

Which action will the nurse include in the plan of care for a patient who has been diagnosed with chronic hepatitis B? a. Advise limiting alcohol intake to 1 drink daily. b. Schedule for liver cancer screening every 6 months. c. Initiate administration of the hepatitis C vaccine series. d. Monitor anti-hepatitis B surface antigen (anti-HBs) levels annually.

ANS: B Patients with chronic hepatitis are at higher risk for development of liver cancer, and should be screened for liver cancer every 6 to 12 months. Patients with chronic hepatitis are advised to completely avoid alcohol. There is no hepatitis C vaccine. Because anti-HBs is present whenever there has been a past hepatitis B infection or vaccination, there is no need to regularly monitor for this antibody.

When a patient with acute kidney injury (AKI) has an arterial blood pH of 7.30, the nurse will expect an assessment finding of a. persistent skin tenting b. rapid, deep respirations. c. bounding peripheral pulses. d. hot, flushed face and neck.

ANS: B Patients with metabolic acidosis caused by AKI may have Kussmaul respirations as the lungs try to regulate carbon dioxide. Bounding pulses and vasodilation are not associated with metabolic acidosis. Because the patient is likely to have fluid retention, poor skin turgor would not be a finding in AKI.

Which prescribed intervention for a 61-year-old female patient with chronic short bowel syndrome will the nurse question? a. Ferrous sulfate (Feosol) 325 mg daily b. Senna (Senokot) 1 tablet every day c. Psyllium (Metamucil) 2.1 grams 3 times daily d. Diphenoxylate with atropine (Lomotil) prn loose stools

ANS: B Patients with short bowel syndrome have diarrhea because of decreased nutrient and fluid absorption and would not need stimulant laxatives. Iron supplements are used to prevent iron-deficiency anemia, bulk-forming laxatives help make stools less watery, and opioid antidiarrheal drugs are helpful in slowing intestinal transit time

A 34-year old patient with chronic hepatitis C infection has several medications prescribed. Which medication requires further discussion with the health care provider before administration? a. Ribavirin (Rebetol, Copegus) 600 mg PO bid b. Pegylated a-interferon (PEG-Intron, Pegasys) SQ daily c. Diphenhydramine (Benadryl) 25 mg PO every 4 hours PRN itching d. Dimenhydrinate (Dramamine) 50 mg PO every 6 hours PRN nausea

ANS: B Pegylated a-interferon is administered weekly. The other medications are appropriate for a patient with chronic hepatitis C infection

A 57-year-old patient is incontinent of urine following a radical retropubic prostatectomy. The nurse will plan to teach the patient a. to restrict oral fluid intake. b. pelvic floor muscle exercises. c. to perform intermittent self-catheterization. d. the use of belladonna and opium suppositories.

ANS: B Pelvic floor muscle training (Kegel) exercises are recommended to strengthen the pelvic floor muscles and improve urinary control. Belladonna and opium suppositories are used to reduce bladder spasms after surgery. Intermittent self-catheterization may be taught before surgery if the patient has urinary retention, but it will not be useful in reducing incontinence after surgery. The patient should have a daily oral intake of 2 to 3 L.

The nurse performing a focused examination to determine possible causes of infertility will assess for a. hydrocele. b. varicocele. c. epididymitis. d. paraphimosis.

ANS: B Persistent varicoceles are commonly associated with infertility. Hydrocele, epididymitis, and paraphimosis are not risk factors for infertility.

The nurse caring for a patient after cystoscopy plans that the patient a. learns to request narcotics for pain. b. understands to expect blood-tinged urine. c. restricts activity to bed rest for a 4 to 6 hours. d. remains NPO for 8 hours to prevent vomiting.

ANS: B Pink-tinged urine and urinary frequency are expected after cystoscopy. Burning on urination is common, but pain that requires opioids for relief is not expected. A good fluid intake is encouraged after this procedure. Bed rest is not required following cystoscopy

Which action should the nurse take after a 36-year-old patient treated with intramuscular glucagon for hypoglycemia regains consciousness? a. Assess the patient for symptoms of hyperglycemia. b. Give the patient a snack of peanut butter and crackers. c. Have the patient drink a glass of orange juice or nonfat milk. d. Administer a continuous infusion of 5% dextrose for 24 hours.

ANS: B Rebound hypoglycemia can occur after glucagon administration, but having a meal containing complex carbohydrates plus protein and fat will help prevent hypoglycemia. Orange juice and nonfat milk will elevate blood glucose rapidly, but the cheese and crackers will stabilize blood glucose. Administration of IV glucose might be used in patients who were unable to take in nutrition orally. The patient should be assessed for symptoms of hypoglycemia after glucagon administration.

A patient returns to the clinic with recurrent dysuria after being treated with trimethoprim and sulfamethoxazole (Bactrim) for 3 days. Which action will the nurse plan to take? a. Remind the patient about the need to drink 1000 mL of fluids daily. b. Obtain a midstream urine specimen for culture and sensitivity testing. c. Teach the patient to take the prescribed Bactrim for at least 3 more days. d. Suggest that the patient use acetaminophen (Tylenol) to treat the symptoms.

ANS: B Since uncomplicated urinary tract infections (UTIs) are usually successfully treated with 3 days of antibiotic therapy, this patient will need a urine culture and sensitivity to determine appropriate antibiotic therapy. Tylenol would not be as effective as other over-the-counter (OTC) medications such as phenazopyridine (Pyridium) in treating dysuria. The fluid intake should be increased to at least 1800 mL/day. Since the UTI has persisted after treatment with Bactrim, the patient is likely to need a different antibiotic. DIF: Cognitive Level: Application REF: 1123-1125

Which information will the nurse include when teaching a 50-year-old male patient about somatropin (Genotropin)? a. The medication will be needed for 3 to 6 months. b. Inject the medication subcutaneously every day. c. Blood glucose levels may decrease when taking the medication. d. Stop taking the medication if swelling of the hands or feet occurs.

ANS: B Somatropin is injected subcutaneously on a daily basis, preferably in the evening. The patient will need to continue on somatropin for life. If swelling or other common adverse effects occur, the health care provider should be notified. Growth hormone will increase blood glucose levels.

A serum potassium level of 3.2 mEq/L (3.2 mmol/L) is reported for a patient with cirrhosis who has scheduled doses of spironolactone (Aldactone) and furosemide (Lasix). due. Which action should the nurse take? a. Administer both drugs. b. Administer the spironolactone. c. Withhold the spironolactone and administer the furosemide. d. Withhold both drugs until discussed with the health care provider.

ANS: B Spironolactone is a potassium-sparing diuretic and will help increase the patient's potassium level. The nurse does not need to talk with the doctor before giving the spironolactone, although the health care provider should be notified about the low potassium value. The furosemide will further decrease the patient's potassium level and should be held until the nurse talks with the health care provider.

Which patient statement indicates that the nurse's teaching about sulfasalazine (Azulfidine) for ulcerative colitis has been effective? a. "The medication will be tapered if I need surgery." b. "I will need to use a sunscreen when I am outdoors." c. "I will need to avoid contact with people who are sick." d. "The medication will prevent infections that cause the diarrhea."

ANS: B Sulfasalazine may cause photosensitivity in some patients. It is not used to treat infections. Sulfasalazine does not reduce immune function. Unlike corticosteroids, tapering of sulfasalazine is not needed.

Which assessment information will be most important for the nurse to report to the health care provider about a patient with acute cholecystitis? a. The patient's urine is bright yellow. b. The patient's stools are tan colored. c. The patient has increased pain after eating. d. The patient complains of chronic heartburn.

ANS: B Tan or grey stools indicate biliary obstruction, which requires rapid intervention to resolve. The other data are not unusual for a patient with this diagnosis, although the nurse would also report the other assessment information to the health care provider

A patient who has bladder cancer had a cystectomy with creation of an Indiana pouch. Which topic will be included in patient teaching? a. Application of ostomy appliances b. Catheterization technique and schedule c. Analgesic use before emptying the pouch d. Use of barrier products for skin protection

ANS: B The Indiana pouch enables the patient to self-catheterize every 4 to 6 hours. There is no need for an ostomy device or barrier products. Catheterization of the pouch is not painful. DIF: Cognitive Level: Application REF: 1155-1156

Which information about dietary management should the nurse include when teaching a patient with peptic ulcer disease (PUD)? a. "You will need to remain on a bland diet." b. "Avoid foods that cause pain after you eat them." c. "High-protein foods are least likely to cause you pain." d. "You should avoid eating any raw fruits and vegetables."

ANS: B The best information is that each individual should choose foods that are not associated with postprandial discomfort. Raw fruits and vegetables may irritate the gastric mucosa, but chewing well seems to decrease this problem and some patients may tolerate these foods well. High-protein foods help neutralize acid, but they also stimulate hydrochloric (HCl) acid secretion and may increase discomfort for some patients. Bland diets may be recommended during an acute exacerbation of PUD, but there is little scientific evidence to support their use. DIF: Cognitive Level: Apply (application) REF: 947 TOP: Nursing Process: Implementation MSC: NCLEX: Physiological Integrity

The nurse will plan to monitor a patient diagnosed with a pheochromocytoma for a. flushing. b. headache. c. bradycardia. d. hypoglycemia.

ANS: B The classic clinical manifestations of pheochromocytoma are hypertension, tachycardia, severe headache, diaphoresis, and abdominal or chest pain. Elevated blood glucose may also occur because of sympathetic nervous system stimulation. Bradycardia and flushing would not be expected.

A 48-year-old patient with stage 2 chronic kidney disease (CKD) is scheduled for an intravenous pyelogram (IVP). Which order for the patient will the nurse question? a. NPO for 6 hours before procedure b. Ibuprofen (Advil) 400 mg PO PRN for pain c. Dulcolax suppository 4 hours before procedure d. Normal saline 500 mL IV infused before procedure

ANS: B The contrast dye used in IVPs is potentially nephrotoxic, and concurrent use of other nephrotoxic medications such as the nonsteroidal anti-inflammatory drugs (NSAIDs) should be avoided. The suppository and NPO status are necessary to ensure adequate visualization during the IVP. IV fluids are used to ensure adequate hydration, which helps reduce the risk for contrast-induced renal failure.

A healthy 28-year-old who has been vaccinated against human papillomavirus (HPV) has a normal Pap test. Which information will the nurse include in patient teaching when calling the patient with the results of the Pap test? a. You can wait until age 30 before having another Pap test. b. Pap testing is recommended every 3 years for women your age. c. No further Pap testing is needed until you decide to become pregnant. d. Yearly Pap testing is suggested for women with multiple sexual partners.

ANS: B The current national guidelines suggest Pap testing every 3 years for patients between ages 21 to 65. Although HPV immunization does protect against cervical cancer, the recommendations are unchanged for individuals who have received the HPV vaccination

The nurse is assessing a patient 4 hours after a kidney transplant. Which information is most important to communicate to the health care provider? a. The urine output is 900 to 1100 mL/hr. b. The patient's central venous pressure (CVP) is decreased. c. The patient has a level 7 (0 to 10 point scale) incisional pain. d. The blood urea nitrogen (BUN) and creatinine levels are elevated.

ANS: B The decrease in CVP suggests hypovolemia, which must be rapidly corrected to prevent renal hypoperfusion and acute tubular necrosis. The other information is not unusual in a patient after a transplant

Which topic is most important to include in patient teaching for a 41-year-old patient diagnosed with early alcoholic cirrhosis? a. Maintaining good nutrition b. Avoiding alcohol ingestion c. Taking lactulose (Cephulac) d. Using vitamin B supplements

ANS: B The disease progression can be stopped or reversed by alcohol abstinence. The other interventions may be used when cirrhosis becomes more severe to decrease symptoms or complications, but the priority for this patient is to stop the progression of the disease

Which medications will the nurse teach the patient about whose peptic ulcer disease is associated with Helicobacter pylori? a. Sucralfate (Carafate), nystatin (Mycostatin), and bismuth (Pepto-Bismol) b. Amoxicillin (Amoxil), clarithromycin (Biaxin), and omeprazole (Prilosec) c. Famotidine (Pepcid), magnesium hydroxide (Mylanta), and pantoprazole (Protonix) d. Metoclopramide (Reglan), bethanechol (Urecholine), and promethazine (Phenergan)

ANS: B The drugs used in triple drug therapy include a proton pump inhibitor such as omeprazole and the antibiotics amoxicillin and clarithromycin. The other combinations listed are not included in the protocol for H. pylori infection

A patient with peptic ulcer disease associated with the presence of Helicobacter pylori is treated with triple drug therapy. The nurse will plan to teach the patient about a. sucralfate (Carafate), nystatin (Mycostatin), and bismuth (Pepto-Bismol). b. amoxicillin (Amoxil), clarithromycin (Biaxin), and omeprazole (Prilosec). c. famotidine (Pepcid), magnesium hydroxide (Mylanta), and pantoprazole (Protonix). d. metoclopramide (Reglan), bethanechol (Urecholine), and promethazine (Phenergan).

ANS: B The drugs used in triple drug therapy include a proton pump inhibitor such as omeprazole and the antibiotics amoxicillin and clarithromycin. The other combinations listed are not included in the protocol for H. pylori infection.

The nurse in the clinic notes elevated prostate specific antigen (PSA) levels in the laboratory results of these patients. Which patient's PSA result is most important to report to the health care provider? a. A 38-year-old who is being treated for acute prostatitis b. A 48-year-old whose father died of metastatic prostate cancer c. A 52-year-old who goes on long bicycle rides every weekend d. A 75-year-old who uses saw palmetto to treat benign prostatic hyperplasia (BPH)

ANS: B The family history of prostate cancer and elevation of PSA indicate that further evaluation of the patient for prostate cancer is needed. Elevations in PSA for the other patients are not unusual.

When preparing to teach an 82-year-old Hispanic patient who lives with an adult daughter about ways to improve nutrition, which action should the nurse take first? a. Ask the daughter about the patient's food preferences. b. Determine who shops for groceries and prepares the meals. c. Question the patient about how many meals per day are eaten. d. Assure the patient that culturally appropriate foods will be included.

ANS: B The family member who shops for groceries and cooks will be in control of the patient's diet, so the nurse will need to ensure that this family member is involved in any teaching or discussion about the patient's nutritional needs. The other information also will be assessed and used but will not be useful in meeting the patient's nutritional needs unless nutritionally appropriate foods are purchased and prepared.

A 62- year-old man reports chronic constipation. To promote bowel evacuation, the nurse will suggest that the patient attempt defecation a. in the mid-afternoon. b. after eating breakfast. c. right after getting up in the morning. d. immediately before the first daily meal.

ANS: B The gastrocolic reflex is most active after the first daily meal. Arising in the morning, the anticipation of eating, and physical exercise do not stimulate these reflexes.

The client diagnosed with Cushing's disease has developed 1 peripheral edema. Theclient has received intravenous fluids at 100 mL/hr via IV pump for the past 79 hours. The client received IVPB medication in 50 mL of fluid every 6 hours for 15 doses. How many mL of fluid did the client receive? ________

The client has received 8,650 mL of intravenous fluid.

Which goal has the highest priority in the plan of care for a 26-year-old homeless patient admitted with viral hepatitis who has severe anorexia and fatigue? a. Increase activity level. b. Maintain adequate nutrition. c. Establish a stable environment. d. Identify sources of hepatitis exposure.

ANS: B The highest priority outcome is to maintain nutrition because adequate nutrition is needed for hepatocyte regeneration. Finding a home for the patient and identifying the source of the infection would be appropriate activities, but they do not have as high a priority as ensuring adequate nutrition. Although the patient's activity level will be gradually increased, rest is indicated during the acute phase of hepatitis

A 25-year-old male patient has been admitted with a severe crushing injury after an industrial accident. Which laboratory result will be most important to report to the health care provider? a. Serum creatinine level 2.1 mg/dL b. Serum potassium level 6.5 mEq/L c. White blood cell count 11,500/µL d. Blood urea nitrogen (BUN) 56 mg/dL

ANS: B The hyperkalemia associated with crushing injuries may cause cardiac arrest and should be treated immediately. The nurse also will report the other laboratory values, but abnormalities in these are not immediately life threatening.

A 27-year-old man who has testicular cancer is being admitted for a unilateral orchiectomy. The patient does not talk to his wife and speaks to the nurse only to answer the admission questions. Which action is best for the nurse to take? a. Teach the patient and the wife that impotence is unlikely after unilateral orchiectomy. b. Ask the patient if he has any questions or concerns about the diagnosis and treatment. c. Document the patient's lack of communication on the chart and continue preoperative care. d. Inform the patient's wife that concerns about sexual function are common with this diagnosis.

ANS: B The initial action by the nurse should be assessment for any anxiety or questions about the surgery or postoperative care. The nurse should address the patient, not the spouse, when discussing the diagnosis and any possible concerns. Without further assessment of patient concerns, the nurse should not offer teaching about complications after orchiectomy. Documentation of the patient's lack of interaction is not an adequate nursing action in this situation.

A 25-year-old male patient calls the clinic complaining of diarrhea for 24 hours. Which action should the nurse take first? a. Inform the patient that laboratory testing of blood and stools will be necessary. b. Ask the patient to describe the character of the stools and any associated symptoms. c. Suggest that the patient drink clear liquid fluids with electrolytes, such as Gatorade or Pedialyte. d. Advise the patient to use over-the-counter loperamide (Imodium) to slow gastrointestinal (GI) motility.

ANS: B The initial response by the nurse should be further assessment of the patient. The other responses may be appropriate, depending on what is learned in the assessment

The nurse will determine that teaching a 67-year-old man to irrigate his new colostomy has been effective if the patient a. inserts the irrigation tubing 4 to 6 inches into the stoma. b. hangs the irrigating container 18 inches above the stoma. c. stops the irrigation and removes the irrigating cone if cramping occurs. d. fills the irrigating container with 1000 to 2000 mL of lukewarm tap water.

ANS: B The irrigating container should be hung 18 to 24 inches above the stoma. If cramping occurs, the irrigation should be temporarily stopped and the cone left in place. Five hundred to 1000 mL of water should be used for irrigation. An irrigation cone, rather than tubing, should be inserted into the stoma; 4 to 6 inches would be too far for safe insertion.

The nurse completing a physical assessment for a newly admitted male patient is unable to feel either kidney on palpation. Which action should the nurse take next? a. Obtain a urine specimen to check for hematuria. b. Document the information on the assessment form. c. Ask the patient about any history of recent sore throat. d. Ask the health care provider about scheduling a renal ultrasound.

ANS: B The kidneys are protected by the abdominal organs, ribs, and muscles of the back, and may not be palpable under normal circumstances, so no action except to document the assessment information is needed. Asking about a recent sore throat, checking for hematuria, or obtaining a renal ultrasound may be appropriate when assessing for renal problems for some patients, but there is nothing in the question stem to indicate that they are appropriate for this patient

Which assessment finding for a patient who has just been admitted with acute pyelonephritis is most important for the nurse to report to the health care provider? a. Complaint of flank pain b. Blood pressure 90/48 mm Hg c. Cloudy and foul-smelling urine d. Temperature 100.1° F (57.8° C)

ANS: B The low blood pressure indicates that urosepsis and septic shock may be occurring and should be immediately reported. The other findings are typical of pyelonephritis.

A 37-year-old patient has just arrived in the postanesthesia recovery unit (PACU) after a thyroidectomy. Which information is most important to communicate to the surgeon? a. The patient reports 7/10 incisional pain. b. The patient has increasing neck swelling. c. The patient is sleepy and difficult to arouse. d. The patient's cardiac rate is 112 beats/minute.

ANS: B The neck swelling may lead to respiratory difficulty, and rapid intervention is needed to prevent airway obstruction. The incisional pain should be treated but is not unusual after surgery. A heart rate of 112 is not unusual in a patient who has been hyperthyroid and has just arrived in the PACU from surgery. Sleepiness in the immediate postoperative period is expected.

Which information about an 80-year-old man at the senior center is of most concern to the nurse? a. Decreased appetite b. Unintended weight loss c. Difficulty chewing food d. Complaints of indigestion

ANS: B Unintentional weight loss is not a normal finding and may indicate a problem such as cancer or depression. Poor appetite, difficulty in chewing, and complaints of indigestion are common in older patients. These will need to be addressed but are not of as much concern as the weight loss

A 38-year-old male patient is admitted to the hospital in Addisonian crisis. Which patient statement supports a nursing diagnosis of ineffective self-health management related to lack of knowledge about management of Addison's disease? a. "I frequently eat at restaurants, and my food has a lot of added salt." b. "I had the stomach flu earlier this week, so I couldn't take the hydrocortisone." c. "I always double my dose of hydrocortisone on the days that I go for a long run." d. "I take twice as much hydrocortisone in the morning dose as I do in the afternoon."

ANS: B The need for hydrocortisone replacement is increased with stressors such as illness, and the patient needs to be taught to call the health care provider because medication and IV fluids and electrolytes may need to be given. The other patient statements indicate appropriate management of the Addison's disease.

Which laboratory value reported to the nurse by the unlicensed assistive personnel (UAP) indicates the most urgent need for the nurse's assessment of the patient? a. Bedtime glucose of 140 mg/dL b. Noon blood glucose of 52 mg/dL c. Fasting blood glucose of 130 mg/dL d. 2-hr postprandial glucose of 220 mg/dL

ANS: B The nurse should assess the patient with a blood glucose level of 52 mg/dL for symptoms of hypoglycemia and give the patient a carbohydrate-containing beverage such as orange juice. The other values are within an acceptable range or not immediately dangerous for a diabetic patient

A 74-year-old who is progressing to stage 5 chronic kidney disease asks the nurse, "Do you think I should go on dialysis? Which initial response by the nurse is best? a. "It depends on which type of dialysis you are considering." b. "Tell me more about what you are thinking regarding dialysis." c. "You are the only one who can make the decision about dialysis." d. "Many people your age use dialysis and have a good quality of life."

ANS: B The nurse should initially clarify the patient's concerns and questions about dialysis. The patient is the one responsible for the decision and many people using dialysis do have good quality of life, but these responses block further assessment of the patient's concerns. Referring to which type of dialysis the patient might use only indirectly responds to the patient's question.

A 71-year-old male patient tells the nurse that growing old causes constipation so he has been using a suppository for constipation every morning. Which action should the nurse take first? a. Encourage the patient to increase oral fluid intake. b. Assess the patient about risk factors for constipation. c. Suggest that the patient increase intake of high-fiber foods. d. Teach the patient that a daily bowel movement is unnecessary.

ANS: B The nurse's initial action should be further assessment of the patient for risk factors for constipation and for his usual bowel pattern. The other actions may be appropriate but will be based on the assessment.

A 58-year-old male patient who is diagnosed with nephrotic syndrome has ascites and 4+ leg edema. Which nursing diagnosis is a priority for the patient? a. Activity intolerance related to rapidly increased weight b. Excess fluid volume related to low serum protein levels c. Disturbed body image related to peripheral edema and ascites d. Altered nutrition: less than required related to protein restriction

ANS: B The patient has massive edema, so the priority problem at this time is the excess fluid volume. The other nursing diagnoses are also appropriate, but the focus of nursing care should be resolution of the edema and ascites.

The nurse identifies a need for additional teaching when the patient who is self-monitoring blood glucose a. washes the puncture site using warm water and soap. b. chooses a puncture site in the center of the finger pad. c. hangs the arm down for a minute before puncturing the site. d. says the result of 120 mg indicates good blood sugar control.

ANS: B The patient is taught to choose a puncture site at the side of the finger pad because there are fewer nerve endings along the side of the finger pad. The other patient actions indicate that teaching has been effective

Which nursing action will be included in the plan of care for a 55-year-old patient with Graves' disease who has exophthalmos? a. Place cold packs on the eyes to relieve pain and swelling. b. Elevate the head of the patient's bed to reduce periorbital fluid. c. Apply alternating eye patches to protect the corneas from irritation. d. Teach the patient to blink every few seconds to lubricate the corneas.

ANS: B The patient should sit upright as much as possible to promote fluid drainage from the periorbital area. With exophthalmos, the patient is unable to close the eyes completely to blink. Lubrication of the eyes, rather than eye patches, will protect the eyes from developing corneal scarring. The swelling of the eye is not caused by excessive blood flow to the eye, so cold packs will not be helpful.

The nurse will plan to teach the patient scheduled for photovaporization of the prostate (PVP) a. that urine will appear bloody for several days. b. how to care for an indwelling urinary catheter. c. that symptom improvement takes 2 to 3 weeks. d. about complications associated with urethral stenting.

ANS: B The patient will have an indwelling catheter for 24 to 48 hours and will need teaching about catheter care. There is minimal bleeding with this procedure. Symptom improvement is almost immediate after PVP. Stent placement is not included in the procedure

Which intervention will be included in the plan of care for a male patient with acute kidney injury (AKI) who has a temporary vascular access catheter in the left femoral vein? a. Start continuous pulse oximetry. b. Restrict physical activity to bed rest. c. Restrict the patient's oral protein intake. d. Discontinue the urethral retention catheter.

ANS: B The patient with a femoral vein catheter must be on bed rest to prevent trauma to the vein. Protein intake is likely to be increased when the patient is receiving dialysis. The retention catheter is likely to remain in place because accurate measurement of output will be needed. There is no indication that the patient needs continuous pulse oximetry.

A 56-year-old female patient is admitted to the hospital with new onset nephrotic syndrome. Which assessment data will the nurse expect? a. Poor skin turgor b. Recent weight gain c. Elevated urine ketones d. Decreased blood pressure

ANS: B The patient with a nephrotic syndrome will have weight gain associated with edema. Hypertension is a clinical manifestation of nephrotic syndrome. Skin turgor is normal because of the edema. Urine protein is high.

A 63-year-old patient with primary hyperparathyroidism has a serum phosphorus level of 1.7 mg/dL (0.55 mmol/L) and calcium of 14 mg/dL (3.5 mmol/L). Which nursing action should be included in the plan of care? a. Restrict the patient to bed rest. b. Encourage 4000 mL of fluids daily. c. Institute routine seizure precautions. d. Assess for positive Chvostek's sign.

ANS: B The patient with hypercalcemia is at risk for kidney stones, which may be prevented by a high fluid intake. Seizure precautions and monitoring for Chvostek's or Trousseau's sign are appropriate for hypocalcemic patients. The patient should engage in weight-bearing exercise to decrease calcium loss from bone.

A 66-year-old patient has a body mass index (BMI) of 31 kg/m2, a normal C-reactive protein level, and low transferrin and albumin levels. The nurse will plan patient teaching to increase the patient's intake of foods that are high in a. iron. b. protein. c. calories. d. carbohydrate.

ANS: B The patient's C-reactive protein and transferrin levels indicate low protein stores. The BMI is in the obese range, so increasing caloric intake is not indicated. The data do not indicate a need for increased carbohydrate or iron intake.

Which laboratory test result will the nurse monitor when evaluating the effects of therapy for a 62-year-old female patient who has acute pancreatitis? a. Calcium b. Bilirubin c. Amylase d. Potassium

ANS: C Amylase is elevated in acute pancreatitis. Although changes in the other values may occur, they would not be useful in evaluating whether the prescribed therapies have been effective

A 79-year-old man has been admitted with benign prostatic hyperplasia. What is most appropriate to include in the nursing plan of care? a. Limit fluid intake to no more than 1000 mL/day. b. Leave a light on in the bathroom during the night. c. Ask the patient to use a urinal so that urine can be measured. d. Pad the patient's bed to accommodate overflow incontinence.

ANS: B The patient's age and diagnosis indicate a likelihood of nocturia, so leaving the light on in the bathroom is appropriate. Fluids should be encouraged because dehydration is more common in older patients. The information in the question does not indicate that measurement of the patient's output is necessary or that the patient has overflow incontinence.

A 34-year-old male patient seen at the primary care clinic complains of feeling continued fullness after voiding and a split, spraying urine stream. The nurse will ask about a history of a. recent kidney trauma. b. gonococcal urethritis. c. recurrent bladder infection. d. benign prostatic hyperplasia.

ANS: B The patient's clinical manifestations are consistent with urethral strictures, a possible complication of gonococcal urethritis. These symptoms are not consistent with benign prostatic hyperplasia, kidney trauma, or bladder infection.

A patient who had a subtotal thyroidectomy earlier today develops laryngeal stridor and a cramp in the right hand upon returning to the surgical nursing unit. Which collaborative action will the nurse anticipate next? a. Suction the patient's airway. b. Administer IV calcium gluconate. c. Plan for emergency tracheostomy. d. Prepare for endotracheal intubation.

ANS: B The patient's clinical manifestations of stridor and cramping are consistent with tetany caused by hypocalcemia resulting from damage to the parathyroid glands during surgery. Endotracheal intubation or tracheostomy may be needed if the calcium does not resolve the stridor. Suctioning will not correct the stridor.

During routine hemodialysis, the 68-year-old patient complains of nausea and dizziness. Which action should the nurse take first? a. Slow down the rate of dialysis. b. Check patient's blood pressure (BP). c. Review the hematocrit (Hct) level. d. Give prescribed PRN antiemetic drugs.

ANS: B The patient's complaints of nausea and dizziness suggest hypotension, so the initial action should be to check the BP. The other actions may also be appropriate based on the blood pressure obtained.

A 22-year-old who is hospitalized with anorexia nervosa is 5 ft 5 in (163 cm) tall and weighs 90 pounds (41 kg). Laboratory tests reveal hypokalemia and iron-deficiency anemia. Which nursing diagnosis has the highest priority for the patient? a. Risk for activity intolerance related to anemia and weakness b. Risk for electrolyte imbalance related to poor eating patterns c. Ineffective health maintenance related to obsession with body image d. Imbalanced nutrition: less than body requirements related to refusal to eat

ANS: B The patient's hypokalemia may lead to life-threatening cardiac dysrhythmias. The other diagnoses also are appropriate for this patient but are not associated with immediate risk for fatal complications.

The nurse is administering IV fluid boluses and nasogastric irrigation to a patient with acute gastrointestinal (GI) bleeding. Which assessment finding is most important for the nurse to communicate to the health care provider? a. The bowel sounds are hyperactive in all four quadrants. b. The patient's lungs have crackles audible to the midchest. c. The nasogastric (NG) suction is returning coffee-ground material. d. The patient's blood pressure (BP) has increased to 142/84 mm Hg.

ANS: B The patient's lung sounds indicate that pulmonary edema may be developing as a result of the rapid infusion of IV fluid and that the fluid infusion rate should be slowed. The return of coffee-ground material in an NG tube is expected for a patient with upper GI bleeding. The BP is slightly elevated but would not be an indication to contact the health care provider immediately. Hyperactive bowel sounds are common when a patient has GI bleeding

A 52-year-old man tells the nurse that he decided to seek treatment for erectile dysfunction (ED) because his wife "is losing patience with the situation." The most appropriate nursing diagnosis for the patient is a. situational low self-esteem related to effects of ED. b. ineffective role performance related to effects of ED. c. anxiety related to inability to have sexual intercourse. d. ineffective sexuality patterns related to infrequent intercourse.

ANS: B The patient's statement indicates that the relationship with his wife is his primary concern. Although anxiety, low self-esteem, and ineffective sexuality patterns may also be concerns, the patient information suggests that addressing the role performance problem will lead to the best outcome for this patient.

A 44-year-old man admitted with a peptic ulcer has a nasogastric (NG) tube in place. When the patient develops sudden, severe upper abdominal pain, diaphoresis, and a firm abdomen, which action should the nurse take? a. Irrigate the NG tube. b. Check the vital signs. c. Give the ordered antacid. d. Elevate the foot of the bed.

ANS: B The patient's symptoms suggest acute perforation, and the nurse should assess for signs of hypovolemic shock. Irrigation of the NG tube, administration of antacids, or both would be contraindicated because any material in the stomach will increase the spillage into the peritoneal cavity. Elevating the foot of the bed may increase abdominal pressure and discomfort, as well as making it more difficult for the patient to breathe

A patient with a peptic ulcer who has a nasogastric (NG) tube develops sudden, severe upper abdominal pain, diaphoresis, and a very firm abdomen. Which action should the nurse take next? a. Irrigate the NG tube. b. Obtain the vital signs. c. Listen for bowel sounds. d. Give the ordered antacid.

ANS: B The patient's symptoms suggest acute perforation, and the nurse should assess for signs of hypovolemic shock. Irrigation of the NG tube, administration of antacids, or both would be contraindicated because any material in the stomach will increase the spillage into the peritoneal cavity. The nurse should assess the bowel sounds, but this is not the first action that should be taken.

A 55-year-old patient admitted with an abrupt onset of jaundice and nausea has abnormal liver function studies but serologic testing is negative for viral causes of hepatitis. Which question by the nurse is most appropriate? a. "Is there any history of IV drug use?" b. "Do you use any over-the-counter drugs?" c. "Are you taking corticosteroids for any reason?" d. "Have you recently traveled to a foreign country?"

ANS: B The patient's symptoms, lack of antibodies for hepatitis, and the abrupt onset of symptoms suggest toxic hepatitis, which can be caused by commonly used over-the-counter drugs such as acetaminophen (Tylenol). Travel to a foreign country and a history of IV drug use are risk factors for viral hepatitis. Corticosteroid use does not cause the symptoms listed

A patient who had surgery for creation of an ileal conduit 3 days ago will not look at the stoma and requests that only the ostomy nurse specialist does the stoma care. The nurse identifies a nursing diagnosis of a. anxiety related to effects of procedure on lifestyle. b. disturbed body image related to change in function. c. readiness for enhanced coping related to need for information. d. self-care deficit, toileting, related to denial of altered body function

ANS: B The patient's unwillingness to look at the stoma or participate in care indicates that disturbed body image is the best diagnosis. No data suggest that the impact on lifestyle is a concern for the patient. The patient does not appear to be ready for enhanced coping. The patient's insistence that only the ostomy nurse care for the stoma indicates that denial is not present.

Two days after surgery for an ileal conduit, the patient will not look at the stoma or participate in care. The patient insists that no one but the ostomy nurse specialist care for the stoma. The nurse identifies a nursing diagnosis of a. anxiety related to effects of procedure on lifestyle. b. disturbed body image related to change in body function. c. readiness for enhanced coping related to need for information. d. self-care deficit, toileting, related to denial of altered body function.

ANS: B The patient's unwillingness to look at the stoma or participate in care indicates that disturbed body image is the best diagnosis. No data suggest that the impact on lifestyle is a concern for the patient. The patient does not appear to be ready for enhanced coping. The patient's insistence that only the ostomy nurse care for the stoma indicates that denial is not present. DIF: Cognitive Level: Application REF: 1157 | 1159-1160 | 1158-1159

The nurse assessing the urinary system of a 45-year-old female would use auscultation to a. determine kidney position. b. identify renal artery bruits. c. check for ureteral peristalsis. d. assess for bladder distention.

ANS: B The presence of a bruit may indicate problems such as renal artery tortuosity or abdominal aortic aneurysm. Auscultation would not be helpful in assessing for the other listed urinary tract information

Administration of hepatitis B vaccine to a healthy 18-year-old patient has been effective when a specimen of the patient's blood reveals a. HBsAg. b. anti-HBs. c. anti-HBc IgG. d. anti-HBc IgM.

ANS: B The presence of surface antibody to HBV (anti-HBs) is a marker of a positive response to the vaccine. The other laboratory values indicate current infection with HBV

The nurse is planning care for a patient with severe heart failure who has developed elevated blood urea nitrogen (BUN) and creatinine levels. The primary collaborative treatment goal in the plan will be a. augmenting fluid volume. b. maintaining cardiac output. c. diluting nephrotoxic substances. d. preventing systemic hypertension.

ANS: B The primary goal of treatment for acute kidney injury (AKI) is to eliminate the cause and provide supportive care while the kidneys recover. Because this patient's heart failure is causing AKI, the care will be directed toward treatment of the heart failure. For renal failure caused by hypertension, hypovolemia, or nephrotoxins, the other responses would be correct.

To detect possible complications in a patient with severe cirrhosis who has bleeding esophageal varices, it is most important for the nurse to monitor a. bilirubin levels. b. ammonia levels. c. potassium levels. d. prothrombin time.

ANS: B The protein in the blood in the gastrointestinal (GI) tract will be absorbed and may result in an increase in the ammonia level because the liver cannot metabolize protein very well. The prothrombin time, bilirubin, and potassium levels should also be monitored, but they will not be affected by the bleeding episode

The nurse is caring for a 36-year-old patient with pancreatic cancer. Which nursing action is the highest priority? a. Offer psychologic support for depression. b. Offer high-calorie, high-protein dietary choices. c. Administer prescribed opioids to relieve pain as needed. d. Teach about the need to avoid scratching any pruritic areas.

ANS: C Effective pain management will be necessary in order for the patient to improve nutrition, be receptive to teaching, or manage anxiety or depression

A 68-year-old patient with a bleeding duodenal ulcer has a nasogastric (NG) tube in place, and the health care provider orders 30 mL of aluminum hydroxide/magnesium hydroxide (Maalox) to be instilled through the tube every hour. To evaluate the effectiveness of this treatment, the nurse a. monitors arterial blood gas values daily. b. periodically aspirates and tests gastric pH. c. checks each stool for the presence of occult blood. d. measures the volume of residual stomach contents.

ANS: B The purpose for antacids is to increase gastric pH. Checking gastric pH is the most direct way of evaluating the effectiveness of the medication. Arterial blood gases may change slightly, but this does not directly reflect the effect of antacids on gastric pH. Because the patient has upper gastrointestinal (GI) bleeding, occult blood in the stools will appear even after the acute bleeding has stopped. The amount of residual stomach contents is not a reflection of resolution of bleeding or of gastric pH

How many grams of protein will the nurse recommend to meet the minimum daily requirement for a patient who weighs 145 pounds (66 kg)? a. 36 b. 53 c. 75 d. 98

ANS: B The recommended daily protein intake is 0.8 to 1 g/kg of body weight, which for this patient is 66 kg × 0.8 g = 52.8 or 53 g/day.

When using a soft, silicone nasogastric tube for enteral feedings, the nurse will need to a. avoid giving medications through the feeding tube. b. flush the tubing after checking for residual volumes. c. administer continuous feedings using an infusion pump. d. replace the tube every 3 to 5 days to avoid mucosal damage.

ANS: B The soft silicone feeding tubes are small in diameter and can easily become clogged unless they are flushed after the nurse checks the residual volume. Either intermittent or continuous feedings can be given. The tubes are less likely to cause mucosal damage than the stiffer polyvinyl chloride tubes used for nasogastric suction and do not need to be replaced at certain intervals. Medications can be given through these tubes, but flushing after medication administration is important to avoid clogging

A nurse will teach a patient who is scheduled to complete a 24-hour urine collection for 17-ketosteroids to a. insert and maintain a retention catheter. b. keep the specimen refrigerated or on ice. c. drink at least 3 L of fluid during the 24 hours. d. void and save that specimen to start the collection.

ANS: B The specimen must be kept on ice or refrigerated until the collection is finished. Voided or catheterized specimens are acceptable for the test. The initial voided specimen is discarded. There is no fluid intake requirement for the 24-hour collection.

Which information will the nurse include when teaching a 50-year-old patient who has type 2 diabetes about glyburide (Micronase, DiaBeta, Glynase)? a. Glyburide decreases glucagon secretion from the pancreas. b. Glyburide stimulates insulin production and release from the pancreas. c. Glyburide should be taken even if the morning blood glucose level is low. d. Glyburide should not be used for 48 hours after receiving IV contrast media.

ANS: B The sulfonylureas stimulate the production and release of insulin from the pancreas. If the glucose level is low, the patient should contact the health care provider before taking the glyburide, because hypoglycemia can occur with this class of medication. Metformin should be held for 48 hours after administration of IV contrast media, but this is not necessary for glyburide. Glucagon secretion is not affected by glyburide

During the physical examination of a 36-year-old female, the nurse finds that the patient's thyroid gland cannot be palpated. The most appropriate action by the nurse is to a. palpate the patient's neck more deeply. b. document that the thyroid was nonpalpable. c. notify the health care provider immediately. d. teach the patient about thyroid hormone testing.

ANS: B The thyroid is frequently nonpalpable. The nurse should simply document the finding. There is no need to notify the health care provider immediately about a normal finding. There is no indication for thyroid-stimulating hormone (TSH) testing unless there is evidence of thyroid dysfunction. Deep palpation of the neck is not appropriate.

Which action will the nurse anticipate taking for an otherwise healthy 50-year-old who has just been diagnosed with Stage 1 renal cell carcinoma? a. Prepare patient for a renal biopsy. b. Provide preoperative teaching about nephrectomy. c. Teach the patient about chemotherapy medications. d. Schedule for a follow-up appointment in 3 months.

ANS: B The treatment of choice in patients with localized renal tumors who have no co-morbid conditions is partial or total nephrectomy. A renal biopsy will not be needed in a patient who has already been diagnosed with renal cancer. Chemotherapy is used for metastatic renal cancer. Because renal cell cancer frequently metastasizes, treatment will be started as soon as possible after the diagnosis.

Which adult will the nurse plan to teach about risks associated with obesity? a. Man who has a BMI of 18 kg/m2 b. Man with a 42 in waist and 44 in hips c. Woman who has a body mass index (BMI) of 24 kg/m2 d. Woman with a waist circumference of 34 inches (86 cm)

ANS: B The waist-to-hip ratio for this patient is 0.95, which exceeds the recommended level of <0.80. A patient with a BMI of 18 kg/m2 is considered underweight. A BMI of 24 kg/m2 is normal. Health risks associated with obesity increase in women with a waist circumference larger than 35 in (89 cm) and men with a waist circumference larger than 40 in (102 cm).

A patient returning from surgery for a perineal radical prostatectomy will have a nursing diagnosis of risk for infection related to a. urinary incontinence. b. prolonged urinary stasis. c. possible fecal wound contamination. d. placement of a suprapubic bladder catheter.

ANS: C An annual digital rectal exam (DRE) and PSA are usually recommended starting at age 50 for men who have an average risk for prostate cancer. Urinalysis and uroflowmetry studies are done if patients have symptoms of urinary tract infection or changes in the urinary stream. TRUS may be ordered if the DRE or PSA is abnormal.

A female patient with a suspected urinary tract infection (UTI) is to provide a clean-catch urine specimen for culture and sensitivity testing. To obtain the specimen, the nurse will a. have the patient empty the bladder completely, then obtain the next urine specimen that the patient is able to void. b. teach the patient to clean the urethral area, void a small amount into the toilet, and then void into a sterile specimen cup. c. insert a short sterile "mini" catheter attached to a collecting container into the urethra and bladder to obtain the specimen. d. clean the area around the meatus with a povidone-iodine (Betadine) swab, and then have the patient void into a sterile container.

ANS: B This answer describes the technique for obtaining a clean-catch specimen. The answer beginning, "insert a short, small, 'mini' catheter attached to a collecting container" describes a technique that would result in a sterile specimen, but a health care provider's order for a catheterized specimen would be required. Using Betadine before obtaining the specimen is not necessary, and might result in suppressing the growth of some bacteria. The technique described in the answer beginning "have the patient empty the bladder completely" would not result in a sterile specimen

After the nurse teaches a patient about the recommended amounts of foods from animal and plant sources, which menu selections indicate that the initial instructions about diet have been understood? a. 3 oz of lean beef, 2 oz of low-fat cheese, and a tomato slice b. 3 oz of roasted pork, a cup of corn, and a cup of carrot sticks c. Cup of tossed salad and nonfat dressing topped with a chicken breast d. Half cup of tuna mixed with nonfat mayonnaise and a half cup of celery

ANS: B This selection is most consistent with the recommendation of the American Institute for Cancer Research that one third of the diet should be from animal sources and two thirds from plant source foods. The other choices all have higher ratios of animal origin foods to plant source foods than would be recommended

After bariatric surgery, a patient who is being discharged tells the nurse, "I prefer to be independent. I am not interested in any support groups." Which response by the nurse is best? a. "I hope you change your mind so that I can suggest a group for you." b. "Tell me what types of resources you think you might use after this surgery." c. "Support groups have been found to lead to more successful weight loss after surgery." d. "Because there are many lifestyle changes after surgery, we recommend support groups."

ANS: B This statement allows the nurse to assess the individual patient's potential needs and preferences. The other statements offer the patient more information about the benefits of support groups, but fail to acknowledge the patient's preferences

An active 28-year-old male with type 1 diabetes is being seen in the endocrine clinic. Which finding may indicate the need for a change in therapy? a. Hemoglobin A1C level 6.2% b. Blood pressure 146/88 mmHg c. Heart rate at rest 58 beats/minute d. High density lipoprotein (HDL) level 65 mg/dL

ANS: B To decrease the incidence of macrovascular and microvascular problems in patients with diabetes, the goal blood pressure is usually 130/80. An A1C less than 6.5%, a low resting heart rate (consistent with regular aerobic exercise in a young adult), and an HDL level of 65 mg/dL all indicate that the patient's diabetes and risk factors for vascular disease are well controlled.

A diabetic patient who has reported burning foot pain at night receives a new prescription. Which information should the nurse teach the patient about amitriptyline (Elavil)? a. Amitriptyline decreases the depression caused by your foot pain. b. Amitriptyline helps prevent transmission of pain impulses to the brain. c. Amitriptyline corrects some of the blood vessel changes that cause pain. d. Amitriptyline improves sleep and makes you less aware of nighttime pain.

ANS: B Tricyclic antidepressants decrease the transmission of pain impulses to the spinal cord and brain. Tricyclic antidepressants also improve sleep quality and are used for depression, but that is not the major purpose for their use in diabetic neuropathy. The blood vessel changes that contribute to neuropathy are not affected by tricyclic antidepressants

When caring for a patient with continuous bladder irrigation after having transurethral resection of the prostate, which action could the nurse delegate to unlicensed assistive personnel (UAP)? a. Teach the patient how to perform Kegel exercises. b. Report any complaints of pain or spasms to the nurse. c. Monitor for increases in bleeding or presence of clots. d. Increase the flow rate of the irrigation if clots are noted.

ANS: B UAP education and role includes reporting patient concerns to supervising nurses. Patient teaching, assessments for complications, and actions such as bladder irrigation require more education and should be done by licensed nursing staff.

A 40-year-old male patient has been newly diagnosed with type 2 diabetes mellitus. Which information about the patient will be most useful to the nurse who is helping the patient develop strategies for successful adaptation to this disease? a. Ideal weight b. Value system c. Activity level d. Visual changes

ANS: B When dealing with a patient with a chronic condition such as diabetes, identification of the patient's values and beliefs can assist the health care team in choosing strategies for successful lifestyle change. The other information also will be useful, but is not as important in developing an individualized plan for the necessary lifestyle changes.

A 56-year-old patient who is disoriented and reports a headache and muscle cramps is hospitalized with possible syndrome of inappropriate antidiuretic hormone (SIADH). The nurse would expect the initial laboratory results to include a(n) a. elevated hematocrit. b. decreased serum sodium. c. low urine specific gravity. d. increased serum chloride.

ANS: B When water is retained, the serum sodium level will drop below normal, causing the clinical manifestations reported by the patient. The hematocrit will decrease because of the dilution caused by water retention. Urine will be more concentrated with a higher specific gravity. The serum chloride level will usually decrease along with the sodium level.

When caring for a patient with urinary retention, the nurse would anticipate an order for a. Limited fluid intake. b. A urinary catheter. c. Diuretic medication. d. A renal angiogram.

ANS: B A urinary catheter would relieve urinary retention. Reducing fluids would reduce the amount of urine produced but would not alleviate the urine retention. Diuretic medication would increase urine production and may worsen the discomfort caused by urine retention. A renal angiogram is an inappropriate diagnostic test for urinary retention.

Many individuals have difficulty voiding in a bedpan or urinal while lying in bed because they a. Are embarrassed that they will urinate on the bedding. b. Would feel more comfortable assuming a normal voiding position. c. Feel they are losing their independence by asking the nursing staff to help. d. Are worried about acquiring a urinary tract infection.

ANS: B Assuming a normal voiding position helps patients relax and be able to void; lying in bed is not the typical position in which people void. Men usually are most comfortable when standing; women are more comfortable when sitting and squatting. Embarrassment at using the bedpan and worrying about a urinary tract infection are not related to the lying-in-bed position. Fear of loss of independence is not related to use of the bedpan or urinal.

When caring for a hospitalized patient with a urinary catheter, which nursing action best prevents the patient from acquiring an infection? a. Inserting the catheter using strict clean technique b. Performing hand hygiene before and after providing perineal care c. Fully inflating the catheter's balloon according to the manufacturer's recommendation d. Disconnecting and replacing the catheter drainage bag once per shift

ANS: B Hand hygiene helps prevent infection in patients with a urinary catheter. A catheter should be inserted in the hospital setting using sterile technique. Inflating the balloon fully prevents dislodgement and trauma, not infection. Disconnecting the drainage bag from the catheter creates a break in the system and an open portal of entry and increases risk of infection. DIF: Apply REF: 1048 OBJ: Discuss nursing measures

Which nursing diagnosis related to alternations in urinary function in an older adult should be a nurse's first priority? a. Self-care deficit related to decreased mobility b. Risk of infection c. Anxiety related to urinary frequency d. Impaired self-esteem related to lack of independence

ANS: B Older adults often experience poor muscle tone, which leads to an inability of the bladder to fully empty. Residual urine greatly increases the risk of infection. Following Maslow's hierarchy of needs, physical health risks should be addressed before emotional/cognitive risks such as anxiety and self-esteem. Decreased mobility can lead to self-care deficit; the nurse's priority concern for this diagnosis would be infection, because the elderly person must rely on others for basic hygiene.

A patient is having difficulty voiding in a bedpan but states that she feels her bladder is full. To stimulation micturition, which nursing intervention should the nurse try first? a. Exiting the room and informing the patient that the nurse will return in 30 minutes to check on the patient's progress b. Utilizing the power of suggestion by turning on the faucet and letting the water run c. Obtaining an order for a Foley catheter d. Administering diuretic medication

ANS: B To stimulate micturition, the nurse should attempt noninvasive procedures first. Running warm water or stroking the inner aspect of the upper thigh promotes sensory perception that leads to urination. A patient should not be left alone on a bedpan for 30 minutes because this could cause skin breakdown. Catheterization places the patient at increased risk of infection and should not be the first intervention attempted. Diuretics are useful if the patient is not producing urine, but they do not stimulate micturition.

The nurse would question an order to insert a urinary catheter on which patient? a. A 26-year-old patient with a recent spinal cord injury at T2 b. A 30-year-old patient requiring drug screening for employment c. A 40-year-old patient undergoing bladder repair surgery d. An 86-year-old patient requiring monitoring of urinary output for renal failure

ANS: B Urinary catheterization places the patient at increased risk for infection and should be performed only when necessary. Urine can be obtained via clean-catch technique for a drug screening or urinalysis. Spinal cord injury, surgery, and renal failure wi h critical t intake and output monitoring are all appropriate reasons for catheterization.

While receiving a shift report on a patient, the nurse is informed that the patient has urinary incontinence. Upon assessment, the nurse would expect to find a. An indwelling Foley catheter. b. Reddened irritated skin on the buttocks. c. Tiny blood clots in the patient's urine. d. Foul-smelling discharge indicative of a UTI.

ANS: B Urinary incontinence is uncontrolled urinary elimination; if the urine has prolonged contact with the skin, skin breakdown can occur. An indwelling Foley catheter is a solution for urine retention. Blood clots and foul-smelling discharge are often signs of infection.

The client is taking a medication for an endocrine problem that inhibits aldosterone secretion and release. To what complications of this therapy should the nurse be alert? A. Dehydration, hypokalemia B. Dehydration, hyperkalemia C. Overhydration, hyponatremia D. Overhydration, hypernatremia

ANS: B Aldosterone is a mineralocorticoid that increases the reabsorption of water and sodium in the kidney at the same time that it promotes excretion of potassium. Any drug or condition that disrupts aldosterone secretion or release increases the client's risk for excessive water loss and potassium reabsorption.

Which condition would trigger the release of antidiuretic hormone (ADH)? A. Plasma osmolarity decreased secondary to overhydration. B. Plasma osmolarity increased secondary to dehydration. C. Plasma volume decreased secondary to hemorrhage. D. Plasma volume increased with edema formation.

ANS: B Antidiuretic hormone is triggered by a rising ECF osmolarity, especially hypernatremia.

Which assessment maneuvers should the nurse perform first when assessing the renal system at the same time as the abdomen? A. Abdominal percussion B. Abdominal auscultation C. Abdominal palpation D. Renal palpation

ANS: B Auscultation precedes percussion and palpation because the nurse needs to auscultate for abdominal bruits before palpation or percussion of the abdominal and renal components of a physical assessment.

The female client's urinalysis shows all the following characteristics. Which should the nurse document as abnormal? A. pH 5.6 B. Ketone bodies present C. Specific gravity is 1.030 D. Two white blood cells per high-power field

ANS: B Ketone bodies are byproducts of incomplete metabolism of fatty acids. Normally, there are no ketones in urine. Ketone bodies are produced when fat sources are used instead of glucose to provide cellular energy.

Which of the following conditions are associated with oversecretion of rennin? A. Alzheimer's disease B. Hypertension C. Diabetes mellitus D. Diabetes insipidus

ANS: B Renin is secreted when special cells in the DCT, called the macula densa, sense changes in blood volume and pressure. When the macula densa cells sense that blood volume is low, blood pressure is low, or blood sodium levels are low, renin is secreted. Renin then converts angiotensinogen into angiotensin I. This leads to a series of reactions that cause the secretion of the hormone aldosterone. This hormone increases kidney reabsorption of sodium and water, increasing blood pressure, blood volume, and blood sodium levels. Inappropriate or excessive renin secretion is a major cause of persistent hypertension.

The client's urine specific gravity is 1.018. What is the nurse's best action? A. Ask the client for a 24-hour recall of liquid intake. B. Document the finding as the only action. C. Obtain a specimen for culture. D. Notify the physician.

ANS: B This specific gravity is within the normal range for urine.

Which of the following muscle actions results in voluntary urination? A. Detrusor contraction, external sphincter contraction B. Detrusor contraction, external sphincter relaxation C. Detrusor relaxation, external sphincter contraction D. Detrusor relaxation, external sphincter relaxation

ANS: B Voiding becomes a voluntary act as a result of learned responses controlled by the cerebral cortex that cause contraction of the bladder detrusor muscle and simultaneous relaxation of the external urethral sphincter muscle.

Which nursing actions can the nurse working in a women's health clinic delegate to unlicensed assistive personnel (UAP) (select all that apply)? a. Call a patient with the results of an endometrial biopsy. b. Assist the health care provider with performing a Pap test. c. Draw blood for CA-125 levels for a patient with ovarian cancer. d. Screen a patient for use of medications that may cause amenorrhea. e. Teach the parent of a 10-year-old about the human papilloma virus (HPV) vaccine (Gardasil).

ANS: B, C Assisting with a Pap test and drawing blood (if trained) are skills that require minimal critical thinking and judgment and can be safely delegated to UAP. Patient teaching, calling a patient who may have questions about results of diagnostic testing, and risk-factor screening all require more education and critical thinking and should be done by the registered nurse (RN).

The nurse understands that peritoneal dialysis and hemodialysis use which processes to clean the patient's blood? (Select all that apply.) a. Gravity b. Osmosis c. Diffusion d. Filtration

ANS: B, C Osmosis and diffusion are the two processes used to clean the patient's blood in both types of dialysis. In peritoneal dialysis, osmosis and dialysis occur across the semi-permeable peritoneal membrane. In hemodialysis, osmosis and dialysis occur through the filter membrane on the artificial kidney. In peritoneal dialysis, the dialysate flows by gravity out of the abdomen.Gravity has no effect on cleansing of the blood. Filtration is the process that occurs in the glomerulus as blood flows through the kidney.

Which of the following symptoms are most closely associated with uremic syndrome? (Select all that apply.) a. Fever b. Nausea and vomiting c. Headache d. Altered mental status e. Dysuria

ANS: B, C, D Uremic syndrome is associated with end-stage renal disease. Signs and symptoms include headache, altered mental status, coma, seizures, nausea, vomiting, and pericarditis.

To monitor for complications in a patient with type 2 diabetes, which tests will the nurse in the diabetic clinic schedule at least annually (select all that apply)? a. Chest x-ray b. Blood pressure c. Serum creatinine d. Urine for microalbuminuria e. Complete blood count (CBC) f. Monofilament testing of the foot

ANS: B, C, D, F Blood pressure, serum creatinine, urine testing for microalbuminuria, and monofilament testing of the foot are recommended at least annually to screen for possible microvascular and macrovascular complications of diabetes. Chest x-ray and CBC might be ordered if the diabetic patient presents with symptoms of respiratory or infectious problems but are not routinely included in screening

A patient has been diagnosed with urinary tract calculi that are high in uric acid. Which foods will the nurse teach the patient to avoid (select all that apply)? a. Milk b. Liver c. Spinach d. Chicken e. Cabbage f. Chocolate

ANS: B, D Meats contain purines, which are metabolized to uric acid. The other foods might be restricted in patients who have calcium or oxalate stones.

Which nursing actions are acceptable when collecting a urine specimen? (Select all that apply.) a. Growing urine cultures for up to 12 hours b. Labeling all specimens with date, time, and initials c. Wearing gown, gloves, and mask for all specimen handling d. Allowing the patient adequate time and privacy to void e. Squeezing urine from diapers into a urine specimen cup f. Transporting specimens to the laboratory in a timely fashion g. Placing a plastic bag over the child's urethra to catch urine

ANS: B, D, F, G All specimens should be labeled appropriately and processed in a timely fashion. Allow patients time and privacy to void. Children may have difficulty voiding; attaching a plastic bag gives the child more time and freedom to void. Urine cultures can take up to 48 hours to develop. Gown, gloves, and mask are not necessary for specimen handling unless otherwise indicated. Urine should not be squeezed from diapers.

A nurse notifies the provider immediately if a patient with an indwelling catheter a. Complains of discomfort upon insertion of the catheter. b. Places the drainage bag higher than the waist while ambulating. c. Has not collected any urine in the drainage bag for 2 hours. d. Is incontinent of stool and contaminates the external portion of the catheter.

ANS: C If the patient has not produced urine in 2 hours, the physician needs to be notified immediately because this could indicate renal failure. Discomfort upon catheter insertion is unpleasant but unavoidable. The nurse is responsible for maintaining the integrity of the catheter by ensuring that the drainage bag is below the patient's bladder. Stool left on the catheter can cause infection and should be removed as soon as it is noticed. The nurse should ensure that frequent perineal care is being provided.

The nurse would expect the urine of a patient with uncontrolled diabetes mellitus to be a. Cloudy. b. Discolored. c. Sweet smelling. d. Painful.

ANS: C Incomplete fat metabolism and buildup of ketones give urine a sweet or fruity odor. Cloudy urine may indicate infection or renal failure. Discolored urine may result from various medications. Painful urination indicates an alteration in urinary elimination.

A patient is experiencing oliguria. Which action should the nurse perform first? a. Increase the patient's intravenous fluid rate. b. Encourage the patient to drink caffeinated beverages. c. Assess for bladder distention. d. Request an order for diuretics.

ANS: C The nurse first should gather all assessment data to determine the potential cause of oliguria. It could be that the patient does not have adequate intake, or it could be that the bladder sphincter is not functioning and the patient is retaining water. Increasing fluids is effective if the patient does not have adequate intake, or if dehydration occurs. Caffeine can work as a diuretic but is not helpful if an underlying pathology is present. An order for diuretics can be obtained if the patient was retaining water, but this should not be

The nurse is providing preoperative teaching for a 61-year-old man scheduled for an abdominal-perineal resection. Which information will the nurse include? a. Another surgery in 8 to 12 weeks will be used to create an ileal-anal reservoir. b. The patient will begin sitting in a chair at the bedside on the first postoperative day. c. The patient will drink polyethylene glycol lavage solution (GoLYTELY) preoperatively. d. IV antibiotics will be started at least 24 hours before surgery to reduce the bowel bacteria.

ANS: C A bowel-cleansing agent is used to empty the bowel before surgery to reduce the risk for infection. A permanent colostomy is created with this surgery. Sitting is contraindicated after an abdominal-perineal resection. Oral antibiotics (rather than IV antibiotics) are given to reduce colonic and rectal bacteria

A 26-year-old woman is being evaluated for vomiting and abdominal pain. Which question from the nurse will be most useful in determining the cause of the patient's symptoms? a. "What type of foods do you eat?" b. "Is it possible that you are pregnant?" c. "Can you tell me more about the pain?" d. "What is your usual elimination pattern?"

ANS: C A complete description of the pain provides clues about the cause of the problem. Although the nurse should ask whether the patient is pregnant to determine whether the patient might have an ectopic pregnancy and before any radiology studies are done, this information is not the most useful in determining the cause of the pain. The usual diet and elimination patterns are less helpful in determining the reason for the patient's symptoms

Which laboratory value should the nurse review to determine whether a patient's hypothyroidism is caused by a problem with the anterior pituitary gland or with the thyroid gland? a. Thyroxine (T4) level b. Triiodothyronine (T3) level c. Thyroid-stimulating hormone (TSH) level d. Thyrotropin-releasing hormone (TRH) level

ANS: C A low TSH level indicates that the patient's hypothyroidism is caused by decreased anterior pituitary secretion of TSH. Low T3 and T4 levels are not diagnostic of the primary cause of the hypothyroidism. TRH levels indicate the function of the hypothalamus.

When the nurse is assessing the mouth of a patient who uses smokeless tobacco for signs of oral cancer, which finding will be of most concern? a. Bleeding during tooth brushing b. Painful blisters at the border of the lips c. Red, velvety patches on the buccal mucosa d. White, curdlike plaques on the posterior tongue

ANS: C A red, velvety patch suggests erythroplasia, which has a high incidence (greater than 50%) of progression to squamous cell carcinoma. The other lesions are suggestive of acute processes (gingivitis, oral candidiasis, and herpes simplex).

A male patient in the clinic provides a urine sample that is red-orange in color. Which action should the nurse take first? a. Notify the patient's health care provider. b. Teach correct midstream urine collection. c. Ask the patient about current medications. d. Question the patient about urinary tract infection (UTI) risk factors.

ANS: C A red-orange color in the urine is normal with some over-the-counter (OTC) medications such as phenazopyridine (Pyridium). The color would not be expected with urinary tract infection, is not a sign that poor technique was used in obtaining the specimen, and does not need to be communicated to the health care provider until further assessment is done

A 54-year-old man has just arrived in the recovery area after an upper endoscopy. Which information collected by the nurse is most important to communicate to the health care provider? a. The patient is very drowsy. b. The patient reports a sore throat. c. The oral temperature is 101.6° F. d. The apical pulse is 104 beats/minute.

ANS: C A temperature elevation may indicate that a perforation has occurred. The other assessment data are normal immediately after the procedure.

After assisting with a needle biopsy of the liver at a patient's bedside, the nurse should a. put pressure on the biopsy site using a sandbag. b. elevate the head of the bed to facilitate breathing. c. place the patient on the right side with the bed flat. d. check the patient's postbiopsy coagulation studies.

ANS: C After a biopsy, the patient lies on the right side with the bed flat to splint the biopsy site. Coagulation studies are checked before the biopsy. A sandbag does not exert adequate pressure to splint the site

The nurse is caring for a patient following an adrenalectomy. The highest priority in the immediate postoperative period is to a. protect the patient's skin. b. monitor for signs of infection. c. balance fluids and electrolytes. d. prevent emotional disturbances.

ANS: C After adrenalectomy, the patient is at risk for circulatory instability caused by fluctuating hormone levels, and the focus of care is to assess and maintain fluid and electrolyte status through the use of IV fluids and corticosteroids. The other goals are also important for the patient but are not as immediately life threatening as the circulatory collapse that can occur with fluid and electrolyte disturbances.

A patient has just arrived on the unit after a thyroidectomy. Which action should the nurse take first? a. Observe the dressing for bleeding. b. Check the blood pressure and pulse. c. Assess the patient's respiratory effort. d. Support the patient's head with pillows.

ANS: C Airway obstruction is a possible complication after thyroidectomy because of swelling or bleeding at the site or tetany. The priority nursing action is to assess the airway. The other actions are also part of the standard nursing care postthyroidectomy but are not as high of a priority.

Following rectal surgery, a patient voids about 50 mL of urine every 30 to 60 minutes for the first 4 hours. Which nursing action is most appropriate? a. Monitor the patient's intake and output over night. b. Have the patient drink small amounts of fluid frequently. c. Use an ultrasound scanner to check the postvoiding residual volume. d. Reassure the patient that this is normal after rectal surgery because of anesthesia.

ANS: C An ultrasound scanner can be used to check for residual urine after the patient voids. Because the patient's history and clinical manifestations are consistent with overflow incontinence, it is not appropriate to have the patient drink small amounts. Although overflow incontinence is not unusual after surgery, the nurse should intervene to correct the physiologic problem, not just reassure the patient. The patient may develop reflux into the renal pelvis and discomfort from a full bladder if the nurse waits to address the problem for several hours.

After a 22-year-old female patient with a pituitary adenoma has had a hypophysectomy, the nurse will teach about the need for a. sodium restriction to prevent fluid retention. b. insulin to maintain normal blood glucose levels. c. oral corticosteroids to replace endogenous cortisol. d. chemotherapy to prevent malignant tumor recurrence.

ANS: C Antidiuretic hormone (ADH), cortisol, and thyroid hormone replacement will be needed for life after hypophysectomy. Without the effects of adrenocorticotropic hormone (ACTH) and cortisol, the blood glucose and serum sodium will be low unless cortisol is replaced. An adenoma is a benign tumor, and chemotherapy will not be needed.

Which assessment finding may indicate that a patient is experiencing adverse effects to a corticosteroid prescribed after kidney transplantation? a. Postural hypotension b. Recurrent tachycardia c. Knee and hip joint pain d. Increased serum creatinine

ANS: C Aseptic necrosis of the weight-bearing joints can occur when patients take corticosteroids over a prolonged period. Increased creatinine level, orthostatic dizziness, and tachycardia are not caused by corticosteroid use.

Which action will the nurse include in the plan of care for a 42-year-old patient who is being admitted with Clostridium difficile? a. Educate the patient about proper food storage. b. Order a diet with no dairy products for the patient. c. Place the patient in a private room on contact isolation. d. Teach the patient about why antibiotics will not be used.

ANS: C Because C. difficile is highly contagious, the patient should be placed in a private room and contact precautions should be used. There is no need to restrict dairy products for this type of diarrhea. Metronidazole (Flagyl) is frequently used to treat C. difficile. Improper food handling and storage do not cause C. difficile.

A 68-year-old male patient with a stroke is unconscious and unresponsive to stimuli. After learning that the patient has a history of gastroesophageal reflux disease (GERD), the nurse will plan to do frequent assessments of the patient's a. apical pulse. b. bowel sounds. c. breath sounds. d. abdominal girth.

ANS: C Because GERD may cause aspiration, the unconscious patient is at risk for developing aspiration pneumonia. Bowel sounds, abdominal girth, and apical pulse will not be affected by the patient's stroke or GERD and do not require more frequent monitoring than the routine. DIF: Cognitive Level: Apply (application) REF: 932 TOP: Nursing Process: Assessment MSC: NCLEX: Physiological Integrity

A 28-year-old male patient is diagnosed with polycystic kidney disease. Which information is most appropriate for the nurse to include in teaching at this time? a. Complications of renal transplantation b. Methods for treating severe chronic pain c. Discussion of options for genetic counseling d. Differences between hemodialysis and peritoneal dialysis

ANS: C Because a 28-year-old patient may be considering having children, the nurse should include information about genetic counseling when teaching the patient. The well-managed patient will not need to choose between hemodialysis and peritoneal dialysis or know about the effects of transplantation for many years. There is no indication that the patient has chronic pain.

A 60-year-old patient is taking spironolactone (Aldactone), a drug that blocks the action of aldosterone on the kidney, for hypertension. The nurse will monitor for a. increased serum sodium. b. decreased urinary output. c. elevated serum potassium. d. evidence of fluid overload.

ANS: C Because aldosterone increases the excretion of potassium, a medication that blocks aldosterone will tend to cause hyperkalemia. Aldosterone also promotes the reabsorption of sodium and water in the renal tubules, so spironolactone will tend to cause increased urine output, a decreased or normal serum sodium level, and signs of dehydration.

Nursing staff on a hospital unit are reviewing rates of hospital-acquired infections (HAI) of the urinary tract. Which nursing action will be most helpful in decreasing the risk for HAI in patients admitted to the hospital? a. Encouraging adequate oral fluid intake b. Testing urine with a dipstick daily for nitrites c. Avoiding unnecessary urinary catheterizations d. Providing frequent perineal hygiene to patients

ANS: C Because catheterization bypasses many of the protective mechanisms that prevent urinary tract infection (UTI), avoidance of catheterization is the most effective means of reducing HAI. The other actions will also be helpful, but are not as useful as decreasing urinary catheter use.

Which statement to the nurse from a patient with jaundice indicates a need for teaching? a. "I used cough syrup several times a day last week." b. "I take a baby aspirin every day to prevent strokes." c. "I use acetaminophen (Tylenol) every 4 hours for back pain." d. "I need to take an antacid for indigestion several times a week"

ANS: C Chronic use of high doses of acetaminophen can be hepatotoxic and may have caused the patient's jaundice. The other patient statements require further assessment by the nurse, but do not indicate a need for patient education

A 53-year-old male patient with deep partial-thickness burns from a chemical spill in the workplace experiences severe pain followed by nausea during dressing changes. Which action will be most useful in decreasing the patient's nausea? a. Keep the patient NPO for 2 hours before and after dressing changes. b. Avoid performing dressing changes close to the patient's mealtimes. c. Administer the prescribed morphine sulfate before dressing changes. d. Give the ordered prochlorperazine (Compazine) before dressing changes.

ANS: C Because the patient's nausea is associated with severe pain, it is likely that it is precipitated by stress and pain. The best treatment will be to provide adequate pain medication before dressing changes. The nurse should avoid doing painful procedures close to mealtimes, but nausea/vomiting that occur at other times also should be addressed. Keeping the patient NPO does not address the reason for the nausea and vomiting and will have an adverse effect on the patient's nutrition. Administration of antiemetics is not the best choice for a patient with nausea caused by pain

A 49-year-old female patient with cirrhosis and esophageal varices has a new prescription for propranolol (Inderal). Which finding is the best indicator that the medication has been effective? a. The patient reports no chest pain. b. Blood pressure is 140/90 mm Hg. c. Stools test negative for occult blood. d. The apical pulse rate is 68 beats/minute.

ANS: C Because the purpose of b-blocker therapy for patients with esophageal varices is to decrease the risk for bleeding from esophageal varices, the best indicator of the effectiveness for propranolol is the lack of blood in the stools. Although propranolol is used to treat hypertension, angina, and tachycardia, the purpose for use in this patient is to decrease the risk for bleeding from esophageal varices

A patient undergoes a nephrectomy after having massive trauma to the kidney. Which assessment finding obtained postoperatively is most important to communicate to the surgeon? a. Blood pressure is 102/58. b. Incisional pain level is 8/10. c. Urine output is 20 mL/hr for 2 hours. d. Crackles are heard at both lung bases.

ANS: C Because the urine output should be at least 0.5 mL/kg/hr, a 40 mL output for 2 hours indicates that the patient may have decreased renal perfusion because of bleeding, inadequate fluid intake, or obstruction at the suture site. The blood pressure requires ongoing monitoring but does not indicate inadequate perfusion at this time. The patient should cough and deep breathe, but the crackles do not indicate a need for an immediate change in therapy. The incisional pain should be addressed, but this is not as potentially life threatening as decreased renal perfusion. In addition, the nurse can medicate the patient for pain. DIF: Cognitive Level: Application REF: 1154-1155

A 46-year-old female patient returns to the clinic with recurrent dysuria after being treated with trimethoprim and sulfamethoxazole (Bactrim) for 3 days. Which action will the nurse plan to take? a. Teach the patient to take the prescribed Bactrim for 3 more days. b. Remind the patient about the need to drink 1000 mL of fluids daily. c. Obtain a midstream urine specimen for culture and sensitivity testing. d. Suggest that the patient use acetaminophen (Tylenol) to treat the symptoms.

ANS: C Because uncomplicated urinary tract infections (UTIs) are usually successfully treated with 3 days of antibiotic therapy, this patient will need a urine culture and sensitivity to determine appropriate antibiotic therapy. Acetaminophen would not be as effective as other over-the-counter (OTC) medications such as phenazopyridine (Pyridium) in treating dysuria. The fluid intake should be increased to at least 1800 mL/day. Because the UTI has persisted after treatment with Bactrim, the patient is likely to need a different antibiotic.

After reviewing the electronic medical record shown in the accompanying figure for a patient who had transurethral resection of the prostate the previous day, which information requires the most rapid action by the nurse? a. Elevated temperature b. Respiratory rate and lung sounds c. Bladder spasms and decreased urine output d. No prescription for antihypertensive drugs

ANS: C Bladder spasms and lack of urine output indicate that the nurse needs to assess the continuous bladder irrigation for kinks and may need to manually irrigate the patient's catheter. The other information will also require actions, such as having the patient take deep breaths and cough and discussing the need for antihypertensive medication prescriptions with the health care provider, but the nurse's first action should be to address the problem with the urinary drainage system.

A 62-year-old patient has had a hemorrhoidectomy at an outpatient surgical center. Which instructions will the nurse include in discharge teaching? a. Maintain a low-residue diet until the surgical area is healed. b. Use ice packs on the perianal area to relieve pain and swelling. c. Take prescribed pain medications before a bowel movement is expected. d. Delay having a bowel movement for several days until healing has occurred.

ANS: C Bowel movements may be very painful, and patients may avoid defecation unless pain medication is taken before the bowel movement. A high-residue diet will increase stool bulk and prevent constipation. Delay of bowel movements is likely to lead to constipation. Warm sitz baths rather than ice packs are used to relieve pain and keep the surgical area clean

The nurse implements discharge teaching for a patient following a gastroduodenostomy for treatment of a peptic ulcer. Which patient statement indicates that the teaching has been effective? a. "Persistent heartburn is expected after surgery." b. "I will try to drink liquids along with my meals." c. "Vitamin supplements may be needed to prevent problems with anemia." d. "I will need to choose foods that are low in fat and high in carbohydrate."

ANS: C Cobalamin deficiency may occur after partial gastrectomy, and the patient may need to receive cobalamin via injections or nasal spray. Foods that have moderate fat and low carbohydrate should be chosen to prevent dumping syndrome. Ingestion of liquids with meals is avoided to prevent dumping syndrome. Although peptic ulcer disease may recur, persistent heartburn is not expected after surgery and the patient should call the health care provider if this occurs.

To evaluate an obese patient for adverse effects of lorcaserin (Belviq), which action will the nurse take? a. Take the apical pulse rate. b. Check sclera for jaundice. c. Ask about bowel movements. d. Assess for agitation or restlessness.

ANS: C Constipation is a common side effect of lorcaserin. The other assessments would be appropriate for other weight-loss medications

A 29-year-old patient in the outpatient clinic will be scheduled for blood cortisol testing. Which instruction will the nurse provide? a. "Avoid adding any salt to your foods for 24 hours before the test." b. "You will need to lie down for 30 minutes before the blood is drawn." c. "Come to the laboratory to have the blood drawn early in the morning." d. "Do not have anything to eat or drink before the blood test is obtained."

ANS: C Cortisol levels are usually drawn in the morning, when levels are highest. The other instructions would be given to patients who were having other endocrine testing.

Which information about a patient with Goodpasture syndrome requires the most rapid action by the nurse? a. Blood urea nitrogen level is 70 mg/dL. b. Urine output over the last 2 hours is 30 mL. c. Audible crackles bilaterally over the posterior chest to the midscapular level. d. Elevated level of antiglomerular basement membrane (anti-GBM) antibodies.

ANS: C Crackles heard to a high level indicate a need for rapid actions such as assessment of oxygen saturation, reporting the findings to the health care provider, initiating oxygen therapy, and dialysis. The other findings will also be reported, but are typical of Goodpasture syndrome and do not require immediate nursing action.

The nurse taking a focused health history for a patient with possible testicular cancer will ask the patient about a history of a. testicular torsion. b. testicular trauma. c. undescended testicles. d. sexually transmitted infection (STI).

ANS: C Cryptorchidism is a risk factor for testicular cancer if it is not corrected before puberty. STI, testicular torsion, and testicular trauma are risk factors for other testicular conditions but not for testicular cancer.

A patient born in 1955 had hepatitis A infection 1 year ago. According to Centers for Disease Control and Prevention (CDC) guidelines, which action should the nurse include in care when the patient is seen for a routine annual physical exam? a. Start the hepatitis B immunization series. b. Teach the patient about hepatitis A immune globulin. c. Ask whether the patient has been screened for hepatitis C. d. Test for anti-hepatitis-A virus immune globulin M (anti-HAV-IgM).

ANS: C Current CDC guidelines indicate that all patients who were born between 1945 and 1965 should be screened for hepatitis C because many individuals who are positive have not been diagnosed. Although routine hepatitis B immunization is recommended for infants, children, and adolescents, vaccination for hepatitis B is recommended only for adults at risk for blood-borne infections. Because the patient has already had hepatitis A, immunization and anti-HAV IgM levels will not be needed.

A patient returned from a laparoscopic Nissen fundoplication for hiatal hernia 4 hours ago. Which assessment finding is most important for the nurse to address immediately? a. The patient is experiencing intermittent waves of nausea. b. The patient complains of 7/10 (0 to 10 scale) abdominal pain. c. The patient has absent breath sounds in the left anterior chest. d. The patient has hypoactive bowel sounds in all four quadrants.

ANS: C Decreased breath sounds on one side may indicate a pneumothorax, which requires rapid diagnosis and treatment. The nausea and abdominal pain should also be addressed but they are not as high priority as the patient's respiratory status. The patient's decreased bowel sounds are expected after surgery and require ongoing monitoring but no other action. DIF: Cognitive Level: Apply (application) REF: 935 OBJ: Special Questions: Prioritization TOP: Nursing Process: Assessment MSC: NCLEX: Physiological Integrity

Which information will the nurse teach a patient who has chronic prostatitis? a. Ibuprofen (Motrin) should provide good pain control. b. Prescribed antibiotics should be taken for 7 to 10 days. c. Intercourse or masturbation will help relieve symptoms. d. Cold packs used every 4 hours will decrease inflammation.

ANS: C Ejaculation helps drain the prostate and relieve pain. Warm baths are recommended to reduce pain. Nonsteroidal antiinflammatory drugs (NSAIDs) are frequently prescribed but usually do not offer adequate pain relief. Antibiotics for chronic prostatitis are taken for 4 to 12 weeks.

The nurse will plan to teach the patient with newly diagnosed achalasia that a. a liquid or blenderized diet will be necessary. b. drinking fluids with meals should be avoided. c. endoscopic procedures may be used for treatment. d. lying down and resting after meals is recommended.

ANS: C Endoscopic and laparoscopic procedures are the most effective therapy for improving symptoms caused by achalasia. Patients are advised to drink fluid with meals. Keeping the head elevated after eating will improve esophageal emptying. A semisoft diet is recommended to improve esophageal emptying.

After the nurse has completed teaching a patient with newly diagnosed eosinophilic esophagitis about the management of the disease, which patient action indicates that the teaching has been effective? a. Patient orders nonfat milk for each meal. b. Patient uses the prescribed corticosteroid inhaler. c. Patient schedules an appointment for allergy testing. d. Patient takes ibuprofen (Advil) to control throat pain.

ANS: C Eosinophilic esophagitis is frequently associated with environmental allergens, so allergy testing is used to determine possible triggers. Corticosteroid therapy may be prescribed, but the medication will be swallowed, not inhaled. Milk is a frequent trigger for attacks. NSAIDs are not used for eosinophilic esophagitis

Which response by the nurse best explains the purpose of ranitidine (Zantac) for a patient admitted with bleeding esophageal varices? a. The medication will reduce the risk for aspiration. b. The medication will inhibit development of gastric ulcers. c. The medication will prevent irritation of the enlarged veins. d. The medication will decrease nausea and improve the appetite.

ANS: C Esophageal varices are dilated submucosal veins. The therapeutic action of H2-receptor blockers in patients with esophageal varices is to prevent irritation and bleeding from the varices caused by reflux of acid gastric contents. Although ranitidine does decrease the risk for peptic ulcers, reduce nausea, and help prevent aspiration pneumonia, these are not the primary purposes for H2-receptor blockade in this patient

While interviewing a 30-year-old man, the nurse learns that the patient has a family history of familial adenomatous polyposis (FAP). The nurse will plan to assess the patient's knowledge about a. preventing noninfectious hepatitis. b. treating inflammatory bowel disease. c. risk for developing colorectal cancer. d. using antacids and proton pump inhibitors.

ANS: C Familial adenomatous polyposis is a genetic condition that greatly increases the risk for colorectal cancer. Noninfectious hepatitis, use of medications that treat increased gastric pH, and inflammatory bowel disease are not related to FAP.

All of the following nursing actions are included in the plan of care for a patient who is malnourished. Which action is appropriate for the nurse to delegate to nursing assistive personnel (NAP)? a. Assist the patient to choose high nutrition items from the menu. b. Monitor the patient for skin breakdown over the bony prominences. c. Offer the patient the prescribed nutritional supplement between meals. d. Assess the patient's strength while ambulating the patient in the room.

ANS: C Feeding the patient and assisting with oral intake are included in NAP education and scope of practice. Assessing the patient and assisting the patient in choosing high nutrition foods require LPN/LVN- or RN-level education and scope of practice.

The nurse will anticipate teaching a patient with nephrotic syndrome who develops flank pain about treatment with a. antibiotics. b. antifungals. c. anticoagulants. d. antihypertensives.

ANS: C Flank pain in a patient with nephrotic syndrome suggests a renal vein thrombosis, and anticoagulation is needed. Antibiotics are used to treat a patient with flank pain caused by pyelonephritis. Fungal pyelonephritis is uncommon and is treated with antifungals. Antihypertensives are used if the patient has high blood pressure.

Which information noted by the nurse when caring for a patient with a bladder infection is most important to report to the health care provider? a. Dysuria b. Hematuria c. Left-sided flank pain d. Temperature 100.1° F

ANS: C Flank pain indicates that the patient may have developed pyelonephritis as a complication of the bladder infection. The other clinical manifestations are consistent with a lower urinary tract infection (UTI). DIF: Cognitive Level: Application REF: 1132-1133

Which statement by a nurse to a patient newly diagnosed with type 2 diabetes is correct? a. Insulin is not used to control blood glucose in patients with type 2 diabetes. b. Complications of type 2 diabetes are less serious than those of type 1 diabetes. c. Changes in diet and exercise may control blood glucose levels in type 2 diabetes. d. Type 2 diabetes is usually diagnosed when the patient is admitted with a hyperglycemic coma.

ANS: C For some patients with type 2 diabetes, changes in lifestyle are sufficient to achieve blood glucose control. Insulin is frequently used for type 2 diabetes, complications are equally severe as for type 1 diabetes, and type 2 diabetes is usually diagnosed with routine laboratory testing or after a patient develops complications such as frequent yeast infections.

What may occur with failure of the sodium-potassium pump during severe protein depletion? a. Ascites b. Anemia c. Hyperkalemia d. Hypoalbuminemia

Correct answer: c Rationale: The sodium-potassium pump uses 20% to 50% of all calories ingested. When energy sources are decreased, the pump fails to function, sodium and water are left in the cell, and potassium remains in extracellular fluids. Hyperkalemia, as well as hyponatremia, can occur.

When a patient who takes metformin (Glucophage) to manage type 2 diabetes develops an allergic rash from an unknown cause, the health care provider prescribes prednisone (Deltasone). The nurse will anticipate that the patient may a. need a diet higher in calories while receiving prednisone. b. develop acute hypoglycemia while taking the prednisone. c. require administration of insulin while taking prednisone. d. have rashes caused by metformin-prednisone interactions.

ANS: C Glucose levels increase when patients are taking corticosteroids, and insulin may be required to control blood glucose. Hypoglycemia is not a side effect of prednisone. Rashes are not an adverse effect caused by taking metformin and prednisone simultaneously. The patient may have an increased appetite when taking prednisone, but will not need a diet that is higher in calories

The nurse determines that further instruction is needed for a patient with interstitial cystitis when the patient says which of the following? a. "I should stop having coffee and orange juice for breakfast." b. "I will buy calcium glycerophosphate (Prelief) at the pharmacy." c. "I will start taking high potency multiple vitamins every morning." d. "I should call the doctor about increased bladder pain or odorous urine."

ANS: C High-potency multiple vitamins may irritate the bladder and increase symptoms. The other patient statements indicate good understanding of the teaching.

Before administration of calcium carbonate (Caltrate) to a patient with chronic kidney disease (CKD), the nurse should check laboratory results for a. potassium level. b. total cholesterol. c. serum phosphate. d. serum creatinine.

ANS: C If serum phosphate is elevated, the calcium and phosphate can cause soft tissue calcification. The calcium carbonate should not be given until the phosphate level is lowered. Total cholesterol, creatinine, and potassium values do not affect whether calcium carbonate should be administered.

The health care provider suspects the Somogyi effect in a 50-year-old patient whose 6:00 AM blood glucose is 230 mg/dL. Which action will the nurse teach the patient to take? a. Avoid snacking at bedtime. b. Increase the rapid-acting insulin dose. c. Check the blood glucose during the night d. Administer a larger dose of long-acting insulin.

ANS: C If the Somogyi effect is causing the patient's increased morning glucose level, the patient will experience hypoglycemia between 2:00 and 4:00 AM. The dose of insulin will be reduced, rather than increased. A bedtime snack is used to prevent hypoglycemic episodes during the night

A patient passing bloody urine is scheduled for a cystoscopy with cystogram. Which description of the procedure by the nurse is accurate? a. "Your doctor will place a catheter into an artery in your groin and inject a dye that will visualize the blood supply to the kidneys." b. "Your doctor will insert a lighted tube into the bladder, and little catheters will be inserted through the tube into your kidney." c. "Your doctor will insert a lighted tube into the bladder through your urethra, inspect the bladder, and instill a dye that will outline your bladder on x-ray." d. "Your doctor will inject a radioactive solution into a vein in your arm and the distribution of the isotope in your kidneys and bladder will be checked."

ANS: C In a cystoscope and cystogram procedure, a cystoscope is inserted into the bladder for direct visualization, and then contrast solution is injected through the scope so that x-rays can be taken. The response beginning, "Your doctor will place a catheter" describes a renal arteriogram procedure. The response beginning, "Your doctor will inject a radioactive solution" describes a nuclear scan. The response beginning, "Your doctor will insert a lighted tube into the bladder, and little catheters will be inserted" describes a retrograde pyelogram

A 26-year-old female with type 1 diabetes develops a sore throat and runny nose after caring for her sick toddler. The patient calls the clinic for advice about her symptoms and a blood glucose level of 210 mg/dL despite taking her usual glargine (Lantus) and lispro (Humalog) insulin. The nurse advises the patient to a. use only the lispro insulin until the symptoms are resolved. b. limit intake of calories until the glucose is less than 120 mg/dL. c. monitor blood glucose every 4 hours and notify the clinic if it continues to rise. d. decrease intake of carbohydrates until glycosylated hemoglobin is less than 7%.

ANS: C Infection and other stressors increase blood glucose levels and the patient will need to test blood glucose frequently, treat elevations appropriately with lispro insulin, and call the health care provider if glucose levels continue to be elevated. Discontinuing the glargine will contribute to hyperglycemia and may lead to diabetic ketoacidosis (DKA). Decreasing carbohydrate or caloric intake is not appropriate because the patient will need more calories when ill. Glycosylated hemoglobin testing is not used to evaluate short-term alterations in blood glucose.

An expected nursing diagnosis for a 30-year-old patient admitted to the hospital with symptoms of diabetes insipidus is a. excess fluid volume related to intake greater than output. b. impaired gas exchange related to fluid retention in lungs. c. sleep pattern disturbance related to frequent waking to void. d. risk for impaired skin integrity related to generalized edema.

ANS: C Nocturia occurs as a result of the polyuria caused by diabetes insipidus. Edema, excess fluid volume, and fluid retention are not expected.

When a patient with type 2 diabetes is admitted for a cholecystectomy, which nursing action can the nurse delegate to a licensed practical/vocational nurse (LPN/LVN)? a. Communicate the blood glucose level and insulin dose to the circulating nurse in surgery. b. Discuss the reason for the use of insulin therapy during the immediate postoperative period. c. Administer the prescribed lispro (Humalog) insulin before transporting the patient to surgery. d. Plan strategies to minimize the risk for hypoglycemia or hyperglycemia during the postoperative period.

ANS: C LPN/LVN education and scope of practice includes administration of insulin. Communication about patient status with other departments, planning, and patient teaching are skills that require RN education and scope of practice.

A 80-year-old who is hospitalized with peptic ulcer disease develops new-onset auditory hallucinations. Which prescribed medication will the nurse discuss with the health care provider before administration? a. Sucralfate (Carafate) b. Omeprazole (Prilosec) c. Metoclopramide (Reglan) d. Aluminum hydroxide (Amphojel)

ANS: C Metoclopramide can cause central nervous system (CNS) side effects ranging from anxiety to hallucinations. Hallucinations are not a side effect of proton-pump inhibitors, mucosal protectants, or antacids

After 6 hours of parenteral nutrition (PN) infusion, the nurse checks a patient's capillary blood glucose level and finds it to be 120 mg/dL. The most appropriate action by the nurse is to a. obtain a venous blood glucose specimen. b. slow the infusion rate of the PN infusion. c. recheck the capillary blood glucose in 4 hours. d. notify the health care provider of the glucose level.

ANS: C Mild hyperglycemia is expected during the first few days after PN is started and requires ongoing monitoring. Because the glucose elevation is small and expected, notification of the health care provider is not necessary. There is no need to obtain a venous specimen for comparison. Slowing the rate of the infusion is beyond the nurse's scope of practice and will decrease the patient's nutritional intake.

A patient admitted to the hospital with pneumonia has a history of functional urinary incontinence. Which nursing action will be included in the plan of care? a. Demonstrate the use of the Credé maneuver. b. Teach exercises to strengthen the pelvic floor. c. Place a bedside commode close to the patient's bed. d. Use an ultrasound scanner to check postvoiding residuals.

ANS: C Modifications in the environment make it easier to avoid functional incontinence. Checking for residual urine and performing the Credé maneuver are interventions for overflow incontinence. Kegel exercises are useful for stress incontinence.

The nurse determines a need for additional instruction when the patient with newly diagnosed type 1 diabetes says which of the following? a. "I can have an occasional alcoholic drink if I include it in my meal plan." b. "I will need a bedtime snack because I take an evening dose of NPH insulin." c. "I can choose any foods, as long as I use enough insulin to cover the calories." d. "I will eat something at meal times to prevent hypoglycemia, even if I am not hungry."

ANS: C Most patients with type 1 diabetes need to plan diet choices very carefully. Patients who are using intensified insulin therapy have considerable flexibility in diet choices but still should restrict dietary intake of items such as fat, protein, and alcohol. The other patient statements are correct and indicate good understanding of the diet instruction

Which statement by the nurse is most likely to help a morbidly obese 22-year-old man in losing weight on a 1000-calorie diet? a. "It will be necessary to change lifestyle habits permanently to maintain weight loss." b. "You will decrease your risk for future health problems such as diabetes by losing weight now." c. "You are likely to notice changes in how you feel with just a few weeks of diet and exercise." d. "Most of the weight that you lose during the first weeks of dieting is water weight rather than fat."

ANS: C Motivation is a key factor in successful weight loss and a short-term outcome provides a higher motivation. A 22-year-old patient is unlikely to be motivated by future health problems. Telling a patient that the initial weight loss is water will be discouraging, although this may be correct. Changing lifestyle habits is necessary, but this process occurs over time and discussing this is not likely to motivate the patient

When a patient's urine dipstick test indicates a small amount of protein, the nurse's next action should be to a. send a urine specimen to the laboratory to test for ketones. b. obtain a clean-catch urine for culture and sensitivity testing. c. inquire about which medications the patient is currently taking. d. ask the patient about any family history of chronic renal failure.

ANS: C Normally the urinalysis will show zero to trace amounts of protein, but some medications may give false-positive readings. The other actions by the nurse may be appropriate, but checking for medications that may affect the dipstick accuracy should be done first

A 64-year-old male patient who has had progressive chronic kidney disease (CKD) for several years has just begun regular hemodialysis. Which information about diet will the nurse include in patient teaching? a. Increased calories are needed because glucose is lost during hemodialysis. b. Unlimited fluids are allowed because retained fluid is removed during dialysis. c. More protein is allowed because urea and creatinine are removed by dialysis. d. Dietary potassium is not restricted because the level is normalized by dialysis.

ANS: C Once the patient is started on dialysis and nitrogenous wastes are removed, more protein in the diet is encouraged. Fluids are still restricted to avoid excessive weight gain and complications such as shortness of breath. Glucose is not lost during hemodialysis. Sodium and potassium intake continues to be restricted to avoid the complications associated with high levels of these electrolytes.

A licensed practical/vocational nurse (LPN/LVN) is caring for a patient with stage 2 chronic kidney disease. Which observation by the RN requires an intervention? a. The LPN/LVN administers the erythropoietin subcutaneously. b. The LPN/LVN assists the patient to ambulate out in the hallway. c. The LPN/LVN administers the iron supplement and phosphate binder with lunch. d. The LPN/LVN carries a tray containing low-protein foods into the patient's room.

ANS: C Oral phosphate binders should not be given at the same time as iron because they prevent the iron from being absorbed. The phosphate binder should be given with a meal and the iron given at a different time. The other actions by the LPN/LVN are appropriate for a patient with renal insufficiency.

The nurse will teach a patient with chronic pancreatitis to take the prescribed pancrelipase (Viokase) a. at bedtime. b. in the morning. c. with each meal. d. for abdominal pain.

ANS: C Pancreatic enzymes are used to help with digestion of nutrients and should be taken with every meal

Which action by the unlicensed assistive personnel (UAP) who are assisting with the care of patients with male reproductive problems indicates that the nurse should provide more teaching? a. The UAP apply a cold pack to the scrotum for a patient with mumps orchitis. b. The UAP help a patient who has had a prostatectomy to put on antiembolism hose. c. The UAP leave the foreskin pulled back after cleaning the glans of a patient who has a retention catheter. d. The UAP encourage a high oral fluid intake for patient who had transurethral resection of the prostate yesterday

ANS: C Paraphimosis can be caused by failing to replace the foreskin back over the glans after cleaning. The other actions by UAP are appropriate.

Which additional information will the nurse need to consider when reviewing the laboratory results for a patient's total calcium level? a. The blood glucose is elevated. b. The phosphate level is normal. c. The serum albumin level is low. d. The magnesium level is normal.

ANS: C Part of the total calcium is bound to albumin so hypoalbuminemia can lead to misinterpretation of calcium levels. The other laboratory values will not affect total calcium interpretation.

Which action by a 70-year-old patient who is using peritoneal dialysis (PD) indicates that the nurse should provide more teaching about PD? a. The patient leaves the catheter exit site without a dressing. b. The patient plans 30 to 60 minutes for a dialysate exchange. c. The patient cleans the catheter while taking a bath each day. d. The patient slows the inflow rate when experiencing abdominal pain.

ANS: C Patients are encouraged to take showers rather than baths to avoid infections at the catheter insertion side. The other patient actions indicate good understanding of peritoneal dialysis.

A 26-year-old patient with diabetes rides a bicycle to and from work every day. Which site should the nurse teach the patient to administer the morning insulin? a. thigh. b. buttock. c. abdomen. d. upper arm.

ANS: C Patients should be taught not to administer insulin into a site that will be exercised because exercise will increase the rate of absorption. The thigh, buttock, and arm are all exercised by riding a bicycle.

Which information will the nurse include when teaching the patient with a urinary tract infection (UTI) about the use of phenazopyridine (Pyridium)? a. Take the medication for at least 7 days. b. Use sunscreen while taking the Pyridium. c. The urine may turn a reddish-orange color. d. Use the Pyridium before sexual intercourse.

ANS: C Patients should be taught that Pyridium will color the urine deep orange. Urinary analgesics should only be needed for a few days until the prescribed antibiotics decrease the bacterial count. Taking Pyridium before intercourse will not be helpful in reducing the risk for UTI. Pyridium does not cause photosensitivity.

The nurse will teach a patient to plan to minimize physical and emotional stress while the patient is undergoing a. a water deprivation test. b. testing for serum T3 and T4 levels. c. a 24-hour urine test for free cortisol. d. a radioactive iodine (I-131) uptake test.

ANS: C Physical and emotional stress can affect the results of the free cortisol test. The other tests are not impacted by stress.

Which menu choice by the patient who is receiving hemodialysis indicates that the nurse's teaching has been successful? a. Split-pea soup, English muffin, and nonfat milk b. Oatmeal with cream, half a banana, and herbal tea c. Poached eggs, whole-wheat toast, and apple juice d. Cheese sandwich, tomato soup, and cranberry juice

ANS: C Poached eggs would provide high-quality protein, and apple juice is low in potassium. Cheese is high in salt and phosphate, and tomato soup would be high in potassium. Split-pea soup is high in potassium, and dairy products are high in phosphate. Bananas are high in potassium, and the cream would be high in phosphate.

A 37-year-old patient is being admitted with a diagnosis of Cushing syndrome. Which findings will the nurse expect during the assessment? a. Chronically low blood pressure b. Bronzed appearance of the skin c. Purplish streaks on the abdomen d. Decreased axillary and pubic hair

ANS: C Purplish-red striae on the abdomen are a common clinical manifestation of Cushing syndrome. Hypotension and bronzed-appearing skin are manifestations of Addison's disease. Decreased axillary and pubic hair occur with androgen deficiency.

Four hours after a bowel resection, a 74-year-old male patient with a nasogastric tube to suction complains of nausea and abdominal distention. The first action by the nurse should be to a. auscultate for hypotonic bowel sounds. b. notify the patient's health care provider. c. reposition the tube and check for placement. d. remove the tube and replace it with a new one.

ANS: C Repositioning the tube will frequently facilitate drainage. Because this is a common occurrence, it is not appropriate to notify the health care provider unless other interventions do not resolve the problem. Information about the presence or absence of bowel sounds will not be helpful in improving drainage. Removing the tube and replacing it are unnecessarily traumatic to the patient, so that would only be done if the tube was completely occluded

Which finding by the nurse for a patient admitted with glomerulonephritis indicates that treatment has been effective? a. The patient denies pain with voiding. b. The urine dipstick is negative for nitrites. c. Peripheral and periorbital edema is resolved. d. The antistreptolysin-O (ASO) titer is decreased.

ANS: C Since edema is a common clinical manifestation of glomerulonephritis, resolution of the edema indicates that the prescribed therapies have been effective. Antibodies to streptococcus will persist after a streptococcal infection. Nitrites will be negative and the patient will not experience dysuria since the patient does not have a urinary tract infection. DIF: Cognitive Level: Application REF: 1131-1133

After the nurse has finished teaching a patient who has a new prescription for exenatide (Byetta), which patient statement indicates that the teaching has been effective? a. "I may feel hungrier than usual when I take this medicine." b. "I will not need to worry about hypoglycemia with the Byetta." c. "I should take my daily aspirin at least an hour before the Byetta." d. "I will take the pill at the same time I eat breakfast in the morning."

ANS: C Since exenatide slows gastric emptying, oral medications should be taken at least an hour before the exenatide to avoid slowing absorption. Exenatide is injected and increases feelings of satiety. Hypoglycemia can occur with this medication

Which information obtained by the nurse in the endocrine clinic about a patient who has been taking prednisone (Deltasone) 40 mg daily for 3 weeks is most important to report to the health care provider? a. Patient's blood pressure is 148/94 mm Hg. b. Patient has bilateral 2+ pitting ankle edema. c. Patient stopped taking the medication 2 days ago. d. Patient has not been taking the prescribed vitamin D.

ANS: C Sudden cessation of corticosteroids after taking the medication for a week or more can lead to adrenal insufficiency, with problems such as severe hypotension and hypoglycemia. The patient will need immediate evaluation by the health care provider to prevent and/or treat adrenal insufficiency. The other information will also be reported, but does not require rapid treatment.

A 63-year-old male patient had a cystectomy with an ileal conduit yesterday. Which new assessment data is most important for the nurse to communicate to the physician? a. Cloudy appearing urine b. Hypotonic bowel sounds c. Heart rate 102 beats/minute d. Continuous stoma drainage

ANS: C Tachycardia may indicate infection, hemorrhage, or hypovolemia, which are all serious complications of this surgery. The urine from an ileal conduit normally contains mucus and is cloudy. Hypotonic bowel sounds are expected after bowel surgery. Continuous drainage of urine from the stoma is normal.

Which assessment finding for a patient who has had a cystectomy with an ileal conduit the previous day is most important for the nurse to communicate to the physician? a. Cloudy appearing urine b. Hypotonic bowel sounds c. Heart rate 102 beats/minute d. Continuous drainage from stoma

ANS: C Tachycardia may indicate infection, hemorrhage, or hypovolemia, which are all serious complications of this surgery. The urine from an ileal conduit normally contains mucus and is cloudy. Hypotonic bowel sounds are expected after bowel surgery. Continuous drainage of urine from the stoma is normal. DIF: Cognitive Level: Application REF: 1157 | 1159-1160 | 1158-1159

A 44-year-old patient is admitted with tetany. Which laboratory value should the nurse monitor? a. Total protein b. Blood glucose c. Ionized calcium d. Serum phosphate

ANS: C Tetany is associated with hypocalcemia. The other values would not be useful for this patient.

A 68-year-old male patient who has bladder cancer had a cystectomy with creation of an Indiana pouch. Which topic will be included in patient teaching? a. Application of ostomy appliances b. Barrier products for skin protection c. Catheterization technique and schedule d. Analgesic use before emptying the pouch

ANS: C The Indiana pouch enables the patient to self-catheterize every 4 to 6 hours. There is no need for an ostomy device or barrier products. Catheterization of the pouch is not painful.

A 26-year-old patient with a history of polycystic kidney disease is admitted to the surgical unit after having knee surgery. Which of the routine postoperative orders is most important for the nurse to discuss with the health care provider? a. Infuse 5% dextrose in normal saline at 75 mL/hr. b. Order regular diet after patient is awake and alert. c. Give ketorolac (Toradol) 10 mg PO PRN for pain. d. Obtain blood urea nitrogen (BUN), creatinine, and electrolytes in 2 hours.

ANS: C The NSAIDs should be avoided in patients with decreased renal function because nephrotoxicity is a potential adverse effect. The other orders do not need any clarification or change. DIF: Cognitive Level: Application REF: 1142-1143

A patient who has acute glomerulonephritis is hospitalized with hyperkalemia. Which information will the nurse monitor to evaluate the effectiveness of the prescribed calcium gluconate IV? a. Urine volume b. Calcium level c. Cardiac rhythm d. Neurologic status

ANS: C The calcium gluconate helps prevent dysrhythmias that might be caused by the hyperkalemia. The nurse will monitor the other data as well, but these will not be helpful in determining the effectiveness of the calcium gluconate.

A few weeks after an 82-year-old with a new diagnosis of type 2 diabetes has been placed on metformin (Glucophage) therapy and taught about appropriate diet and exercise, the home health nurse makes a visit. Which finding by the nurse is most important to discuss with the health care provider? a. Hemoglobin A1C level is 7.9%. b. Last eye exam was 18 months ago. c. Glomerular filtration rate is decreased. d. Patient has questions about the prescribed diet.

ANS: C The decrease in renal function may indicate a need to adjust the dose of metformin or change to a different medication. In older patients, the goal for A1C may be higher in order to avoid complications associated with hypoglycemia. The nurse will plan on scheduling the patient for an eye exam and addressing the questions about diet, but the biggest concern is the patient's decreased renal function

Which assessment finding of a 42-year-old patient who had a bilateral adrenalectomy requires the most rapid action by the nurse? a. The blood glucose is 176 mg/dL. b. The lungs have bibasilar crackles. c. The blood pressure (BP) is 88/50 mm Hg. d. The patient reports 5/10 incisional pain.

ANS: C The decreased BP indicates possible adrenal insufficiency. The nurse should immediately notify the health care provider so that corticosteroid medications can be administered. The nurse should also address the elevated glucose, incisional pain, and crackles with appropriate collaborative or nursing actions, but prevention and treatment of acute adrenal insufficiency is the priority after adrenalectomy.

A patient is receiving tube feedings through a percutaneous endoscopic gastrostomy (PEG). Which action will the nurse include in the plan of care? a. Keep the patient positioned on the left side. b. Obtain a daily x-ray to verify tube placement. c. Check the gastric residual volume every 4 to 6 hours. d. Avoid giving bolus tube feedings through the PEG tube.

ANS: C The gastric residual volume is assessed every 4 to 6 hours to decrease the risk for aspiration. The patient does not need to be positioned on the left side. An x-ray is obtained immediately after placement of the PEG tube to check position, but daily x-rays are not needed. Bolus feedings can be administered through a PEG tube.

After a ureterolithotomy, a female patient has a left ureteral catheter and a urethral catheter in place. Which action will the nurse include in the plan of care? a. Provide teaching about home care for both catheters. b. Apply continuous steady tension to the ureteral catheter. c. Call the health care provider if the ureteral catheter output drops suddenly. d. Clamp the ureteral catheter off when output from the urethral catheter stops.

ANS: C The health care provider should be notified if the ureteral catheter output decreases because obstruction of this catheter may result in an increase in pressure in the renal pelvis. Tension on the ureteral catheter should be avoided in order to prevent catheter displacement. To avoid pressure in the renal pelvis, the catheter is not clamped. Because the patient is not usually discharged with a ureteral catheter in place, patient teaching about both catheters is not needed.

Which information from a patient's urinalysis requires that the nurse notify the health care provider? a. pH 6.2 b. Trace protein c. WBC 20 to 26/hpf d. Specific gravity 1.021

ANS: C The increased number of white blood cells (WBCs) indicates the presence of urinary tract infection or inflammation. The other findings are normal

Which assessment of a 62-year-old patient who has just had an intravenous pyelogram (IVP) requires immediate action by the nurse? a. The heart rate is 58 beats/minute. b. The patient complains of a dry mouth. c. The respiratory rate is 38 breaths/minute. d. The urine output is 400 mL after 2 hours.

ANS: C The increased respiratory rate indicates that the patient may be experiencing an allergic reaction to the contrast medium used during the procedure. The nurse should immediately assess the patient's oxygen saturation and breath sounds. The other data are not unusual findings following an IVP.

A patient with acute kidney injury (AKI) has longer QRS intervals on the electrocardiogram (ECG) than were noted on the previous shift. Which action should the nurse take first? a. Notify the patient's health care provider. b. Document the QRS interval measurement. c. Check the medical record for most recent potassium level. d. Check the chart for the patient's current creatinine level.

ANS: C The increasing QRS interval is suggestive of hyperkalemia, so the nurse should check the most recent potassium and then notify the patient's health care provider. The BUN and creatinine will be elevated in a patient with AKI, but they would not directly affect the electrocardiogram (ECG). Documentation of the QRS interval is also appropriate, but interventions to decrease the potassium level are needed to prevent life-threatening dysrhythmias.

A 76-year-old patient with obstipation has a fecal impaction and is incontinent of liquid stool. Which action should the nurse take first? a. Administer bulk-forming laxatives. b. Assist the patient to sit on the toilet. c. Manually remove the impacted stool. d. Increase the patient's oral fluid intake.

ANS: C The initial action with a fecal impaction is manual disimpaction. The other actions will be used to prevent future constipation and impactions

A 19-year-old female is brought to the emergency department with a knife handle protruding from the abdomen. During the initial assessment of the patient, the nurse should a. remove the knife and assess the wound. b. determine the presence of Rovsing sign. c. check for circulation and tissue perfusion. d. insert a urinary catheter and assess for hematuria.

ANS: C The initial assessment is focused on determining whether the patient has hypovolemic shock. The knife should not be removed until the patient is in surgery, where bleeding can be controlled. Rovsing sign is assessed in the patient with suspected appendicitis. A patient with a knife in place will be taken to surgery and assessed for bladder trauma there

After successfully losing 1 lb weekly for several months, a patient at the clinic has not lost any weight for the last month. The nurse should first a. review the diet and exercise guidelines with the patient. b. instruct the patient to weigh and record weights weekly. c. ask the patient whether there have been any changes in exercise or diet patterns. d. discuss the possibility that the patient has reached a temporary weight loss plateau.

ANS: C The initial nursing action should be assessment of any reason for the change in weight loss. The other actions may be needed, but further assessment is required before any interventions are planned or implemented

A 32-year-old patient with a history of polycystic kidney disease is admitted to the surgical unit after having shoulder surgery. Which of the routine postoperative orders is most important for the nurse to discuss with the health care provider? a. Infuse 5% dextrose in normal saline at 75 mL/hr. b. Order regular diet after patient is awake and alert. c. Give ketorolac (Toradol) 10 mg PO PRN for pain. d. Draw blood urea nitrogen (BUN) and creatinine in 2 hours.

ANS: C The nonsteroidal antiinflammatory drugs (NSAIDs) should be avoided in patients with decreased renal function because nephrotoxicity is a potential adverse effect. The other orders do not need any clarification or change.

Which action should the nurse in the emergency department take first for a new patient who is vomiting blood? a. Insert a large-gauge IV catheter. b. Draw blood for coagulation studies. c. Check blood pressure (BP), heart rate, and respirations. d. Place the patient in the supine position.

ANS: C The nurse's first action should be to determine the patient's hemodynamic status by assessing vital signs. Drawing blood for coagulation studies and inserting an IV catheter are also appropriate. However, the vital signs may indicate the need for more urgent actions. Because aspiration is a concern for this patient, the nurse will need to assess the patient's vital signs and neurologic status before placing the patient in a supine position

At his first postoperative checkup appointment after a gastrojejunostomy (Billroth II), a patient reports that dizziness, weakness, and palpitations occur about 20 minutes after each meal. The nurse will teach the patient to a. increase the amount of fluid with meals. b. eat foods that are higher in carbohydrates. c. lie down for about 30 minutes after eating. d. drink sugared fluids or eat candy after meals.

ANS: C The patient is experiencing symptoms of dumping syndrome, which may be reduced by lying down after eating. Increasing fluid intake and choosing high carbohydrate foods will increase the risk for dumping syndrome. Having a sweet drink or hard candy will correct the hypoglycemia that is associated with dumping syndrome but will not prevent dumping syndrome. DIF: Cognitive Level: Apply (application) REF: 949-950 TOP: Nursing Process: Implementation MSC: NCLEX: Physiological Integrity

To prepare a 56-year-old male patient with ascites for paracentesis, the nurse a. places the patient on NPO status. b. assists the patient to lie flat in bed. c. asks the patient to empty the bladder. d. positions the patient on the right side.

ANS: C The patient should empty the bladder to decrease the risk of bladder perforation during the procedure. The patient would be positioned in Fowler's position and would not be able to lie flat without compromising breathing. Because no sedation is required for paracentesis, the patient does not need to be NPO.

When preparing a female patient with bladder cancer for intravesical chemotherapy, the nurse will teach about a. premedicating to prevent nausea. b. obtaining wigs and scarves to wear. c. emptying the bladder before the medication. d. maintaining oral care during the treatments.

ANS: C The patient will be asked to empty the bladder before instillation of the chemotherapy. Systemic side effects are not usually experienced with intravesical chemotherapy.

A patient is admitted to the hospital with new onset nephrotic syndrome. Which assessment data will the nurse expect to find related to this illness? a. Poor skin turgor b. High urine ketones c. Recent weight gain d. Low blood pressure

ANS: C The patient with a nephrotic syndrome will have weight gain associated with edema. Hypertension is a clinical manifestation of nephrotic syndrome. Skin turgor is normal because of the edema. Urine protein is high. DIF: Cognitive Level: Comprehension REF: 1132-1134

When taking the blood pressure (BP) on the right arm of a patient with severe acute pancreatitis, the nurse notices carpal spasms of the patient's right hand. Which action should the nurse take next? a. Ask the patient about any arm pain. b. Retake the patient's blood pressure. c. Check the calcium level in the chart. d. Notify the health care provider immediately.

ANS: C The patient with acute pancreatitis is at risk for hypocalcemia, and the assessment data indicate a positive Trousseau's sign. The health care provider should be notified after the nurse checks the patient's calcium level. There is no indication that the patient needs to have the BP rechecked or that there is any arm pain

Which statement by a 62-year-old patient with stage 5 chronic kidney disease (CKD) indicates that the nurse's teaching about management of CKD has been effective? a. "I need to get most of my protein from low-fat dairy products." b. "I will increase my intake of fruits and vegetables to 5 per day." c. "I will measure my urinary output each day to help calculate the amount I can drink." d. "I need to take erythropoietin to boost my immune system and help prevent infection."

ANS: C The patient with end-stage kidney disease is taught to measure urine output as a means of determining an appropriate oral fluid intake. Erythropoietin is given to increase the red blood cell count and will not offer any benefit for immune function. Dairy products are restricted because of the high phosphate level. Many fruits and vegetables are high in potassium and should be restricted in the patient with CKD.

Which care activity for a patient with a paralytic ileus is appropriate for the registered nurse (RN) to delegate to unlicensed assistive personnel (UAP)? a. Auscultation for bowel sounds b. Nasogastric (NG) tube irrigation c. Applying petroleum jelly to the lips d. Assessment of the nares for irritation

ANS: C UAP education and scope of practice include patient hygiene such as oral care. The other actions require education and scope of practice appropriate to the RN.

A patient who was admitted with diabetic ketoacidosis secondary to a urinary tract infection has been weaned off an insulin drip 30 minutes ago. The patient reports feeling lightheaded and sweaty. Which action should the nurse take first? a. Infuse dextrose 50% by slow IV push. b. Administer 1 mg glucagon subcutaneously. c. Obtain a glucose reading using a finger stick. d. Have the patient drink 4 ounces of orange juice.

ANS: C The patient's clinical manifestations are consistent with hypoglycemia and the initial action should be to check the patient's glucose with a finger stick or order a stat blood glucose. If the glucose is low, the patient should ingest a rapid-acting carbohydrate, such as orange juice. Glucagon or dextrose 50% might be given if the patient's symptoms become worse or if the patient is unconscious

When assessing a 30-year-old man who complains of a feeling of incomplete bladder emptying and a split, spraying urine stream, the nurse asks about a history of a. bladder infection. b. recent kidney trauma. c. gonococcal urethritis. d. benign prostatic hyperplasia.

ANS: C The patient's clinical manifestations are consistent with urethral strictures, a possible complication of gonococcal urethritis. These symptoms are not consistent with benign prostatic hyperplasia, kidney trauma, or bladder infection. DIF: Cognitive Level: Application REF: 1141

After the insertion of an arteriovenous graft (AVG) in the right forearm, a 54-year-old patient complains of pain and coldness of the right fingers. Which action should the nurse take? a. Teach the patient about normal AVG function. b. Remind the patient to take a daily low-dose aspirin tablet. c. Report the patient's symptoms to the health care provider. d. Elevate the patient's arm on pillows to above the heart level.

ANS: C The patient's complaints suggest the development of distal ischemia (steal syndrome) and may require revision of the AVG. Elevation of the arm above the heart will further decrease perfusion. Pain and coolness are not normal after AVG insertion. Aspirin therapy is not used to maintain grafts.

Which patient should the nurse assess first after receiving change-of-shift report? a. A patient with nausea who has a dose of metoclopramide (Reglan) due b. A patient who is crying after receiving a diagnosis of esophageal cancer c. A patient with esophageal varices who has a blood pressure of 92/58 mm Hg d. A patient admitted yesterday with gastrointestinal (GI) bleeding who has melena

ANS: C The patient's history and blood pressure indicate possible hemodynamic instability caused by GI bleeding. The data about the other patients do not indicate acutely life-threatening complications

After receiving change-of-shift report, which patient should the nurse assess first? a. A patient who was admitted yesterday with gastrointestinal (GI) bleeding and has melena b. A patient who is crying after receiving a diagnosis of esophageal cancer c. A patient with esophageal varices who has a blood pressure of 96/54 mm Hg d. A patient with nausea who has a dose of metoclopramide (Reglan) scheduled

ANS: C The patient's history and blood pressure indicate possible hemodynamic instability caused by GI bleeding. The data about the other patients do not indicate acutely life-threatening complications.

A 48-year-old male patient screened for diabetes at a clinic has a fasting plasma glucose level of 120 mg/dL (6.7 mmol/L). The nurse will plan to teach the patient about a. self-monitoring of blood glucose. b. using low doses of regular insulin. c. lifestyle changes to lower blood glucose. d. effects of oral hypoglycemic medications.

ANS: C The patient's impaired fasting glucose indicates prediabetes, and the patient should be counseled about lifestyle changes to prevent the development of type 2 diabetes. The patient with prediabetes does not require insulin or oral hypoglycemics for glucose control and does not need to self-monitor blood glucose.

A patient with acute gastrointestinal (GI) bleeding is receiving normal saline IV at a rate of 500 mL/hr. Which assessment finding obtained by the nurse is most important to communicate immediately to the health care provider? a. The patient's blood pressure (BP) has increased to 142/94 mm Hg. b. The nasogastric (NG) suction is returning coffee-ground material. c. The patient's lungs have crackles audible to the midline. d. The bowel sounds are very hyperactive in all four quadrants.

ANS: C The patient's lung sounds indicate that pulmonary edema may be developing as a result of the rapid infusion of IV fluid and that the fluid infusion rate should be slowed. The return of coffee-ground material in an NG tube is expected for a patient with upper GI bleeding. The BP is slightly elevated but would not be an indication to contact the health care provider immediately. Hyperactive bowel sounds are common when a patient has GI bleeding.

A patient develops carpopedal spasms and tingling of the lips following a parathyroidectomy. Which action should the nurse take first? a. Administer the ordered muscle relaxant. b. Give the ordered oral calcium supplement. c. Have the patient rebreathe from a paper bag. d. Start the PRN oxygen at 2 L/min per cannula.

ANS: C The patient's symptoms suggest mild hypocalcemia. The symptoms of hypocalcemia will be temporarily reduced by having the patient breathe into a paper bag, which will raise the PaCO2 and create a more acidic pH. The muscle relaxant will have no impact on the ionized calcium level. Although severe hypocalcemia can cause laryngeal stridor, there is no indication that this patient is experiencing laryngeal stridor or needs oxygen. Calcium supplements will be given to normalize calcium levels quickly, but oral supplements will take time to be absorbed.

A 72-year-old male patient with dehydration caused by an exacerbation of ulcerative colitis is receiving 5% dextrose in normal saline at 125 mL/hour. Which assessment finding by the nurse is most important to report to the health care provider? a. Patient has not voided for the last 4 hours. b. Skin is dry with poor turgor on all extremities. c. Crackles are heard halfway up the posterior chest. d. Patient has had 5 loose stools over the last 6 hours.

ANS: C The presence of crackles in an older patient receiving IV fluids at a high rate suggests volume overload and a need to reduce the rate of the IV infusion. The other data will also be reported, but are consistent with the patient's age and diagnosis and do not require a change in the prescribed treatment

A 38-year-old patient with cirrhosis has ascites and 4+ edema of the feet and legs. Which nursing action will be included in the plan of care? a. Restrict daily dietary protein intake. b. Reposition the patient every 4 hours. c. Place the patient on a pressure-relieving mattress. d. Perform passive range of motion daily.

ANS: C The pressure-relieving mattress will decrease the risk for skin breakdown for this patient. Adequate dietary protein intake is necessary in patients with ascites to improve oncotic pressure. Repositioning the patient every 4 hours will not be adequate to maintain skin integrity. Passive range of motion will not take the pressure off areas such as the sacrum that are vulnerable to breakdown

What information will the nurse include for an overweight 35-year-old woman who is starting a weight-loss plan? a. Weigh yourself at the same time every morning and evening. b. Stick to a 600- to 800-calorie diet for the most rapid weight loss. c. Low carbohydrate diets lead to rapid weight loss but are difficult to maintain. d. Weighing all foods on a scale is necessary to choose appropriate portion sizes.

ANS: C The restrictive nature of fad diets makes the weight loss achieved by the patient more difficult to maintain. Portion size can be estimated in other ways besides weighing. Severely calorie-restricted diets are not necessary for patients in the overweight category of obesity and need to be closely supervised. Patients should weigh weekly rather than daily

Which finding is most important for the nurse to communicate to the health care provider about a patient who received a liver transplant 1 week ago? a. Dry palpebral and oral mucosa b. Crackles at bilateral lung bases c. Temperature 100.8° F (38.2° C) d. No bowel movement for 4 days

ANS: C The risk of infection is high in the first few months after liver transplant and fever is frequently the only sign of infection. The other patient data indicate the need for further assessment or nursing actions and might be communicated to the health care provider, but they do not indicate a need for urgent action

A 62-year-old patient with hyperthyroidism is to be treated with radioactive iodine (RAI). The nurse instructs the patient a. about radioactive precautions to take with all body secretions. b. that symptoms of hyperthyroidism should be relieved in about a week. c. that symptoms of hypothyroidism may occur as the RAI therapy takes effect. d. to discontinue the antithyroid medications taken before the radioactive therapy.

ANS: C There is a high incidence of postradiation hypothyroidism after RAI, and the patient should be monitored for symptoms of hypothyroidism. RAI has a delayed response, with the maximum effect not seen for 2 to 3 months, and the patient will continue to take antithyroid medications during this time. The therapeutic dose of radioactive iodine is low enough that no radiation safety precautions are needed.

A 42-year-old female patient is scheduled for transsphenoidal hypophysectomy to treat a pituitary adenoma. During preoperative teaching, the nurse instructs the patient about the need to a. cough and deep breathe every 2 hours postoperatively. b. remain on bed rest for the first 48 hours after the surgery. c. avoid brushing teeth for at least 10 days after the surgery. d. be positioned flat with sandbags at the head postoperatively.

ANS: C To avoid disruption of the suture line, the patient should avoid brushing the teeth for 10 days after surgery. It is not necessary to remain on bed rest after this surgery. Coughing is discouraged because it may cause leakage of cerebrospinal fluid (CSF) from the suture line. The head of the bed should be elevated 30 degrees to reduce pressure on the sella turcica and decrease the risk for headaches.

Which information is most important for the nurse to report to the health care provider before a patient with type 2 diabetes is prepared for a coronary angiogram? a. The patient's most recent HbA1C was 6.5%. b. The patient's admission blood glucose is 128 mg/dL. c. The patient took the prescribed metformin (Glucophage) today. d. The patient took the prescribed captopril (Capoten) this morning.

ANS: C To avoid lactic acidosis, metformin should be discontinued a day or 2 before the coronary arteriogram and should not be used for 48 hours after IV contrast media are administered. The other patient data will also be reported but do not indicate any need to reschedule the procedure

The nurse notes that the peripheral parenteral nutrition (PN) bag has only 20 mL left and a new PN bag has not yet arrived from the pharmacy. Which intervention is the priority? a. Monitor the patient's capillary blood glucose until a new PN bag is hung b. Flush the peripheral line with saline and wait until the new PN bag is available c. Infuse 5% dextrose in water until the new PN bag is delivered from the pharmacy d. Decrease the rate of the current PN infusion to 10 mL/hr until the new bag arrives

ANS: C To prevent hypoglycemia, the nurse should infuse a 5% dextrose solution until the next PN bag can be started. Decreasing the rate of the ordered PN infusion is beyond the nurse's scope of practice. Flushing the line and then waiting for the next bag may lead to hypoglycemia. Monitoring the capillary blood glucose is appropriate but is not the priority.

A 57-year-old man with Escherichia coli O157:H7 food poisoning is admitted to the hospital with bloody diarrhea and dehydration. Which order will the nurse question? a. Infuse lactated Ringer's solution at 250 mL/hr. b. Monitor blood urea nitrogen and creatinine daily. c. Administer loperamide (Imodium) after each stool. d. Provide a clear liquid diet and progress diet as tolerated.

ANS: C Use of antidiarrheal agents is avoided with this type of food poisoning. The other orders are appropriate

A 22-year-old patient with Escherichia coli O157:H7 food poisoning is admitted to the hospital with bloody diarrhea and dehydration. All of the following orders are received. Which order will the nurse question? a. Infuse lactated Ringer's solution at 250 mL/hr. b. Monitor blood urea nitrogen and creatinine daily. c. Administer loperamide (Imodium) after each stool. d. Provide a clear liquid diet and progress diet as tolerated.

ANS: C Use of antidiarrheal agents is avoided with this type of food poisoning. The other orders are appropriate.

The nurse is caring for a 54-year-old female patient on the first postoperative day after a Roux-en-Y gastric bypass procedure. Which assessment finding should be reported immediately to the surgeon? a. Bilateral crackles audible at both lung bases b. Redness, irritation, and skin breakdown in skinfolds c. Emesis of bile-colored fluid past the nasogastric (NG) tube d. Use of patient-controlled analgesia (PCA) several times an hour for pain

ANS: C Vomiting with an NG tube in place indicates that the NG tube needs to be repositioned by the surgeon to avoid putting stress on the gastric sutures. The nurse should implement actions to decrease skin irritation and have the patient cough and deep breathe, but these do not indicate a need for rapid notification of the surgeon. Frequent PCA use after bariatric surgery is expected

The nurse is assessing a 22-year-old patient experiencing the onset of symptoms of type 1 diabetes. Which question is most appropriate for the nurse to ask? a. "Are you anorexic?" b. "Is your urine dark colored?" c. "Have you lost weight lately?" d. "Do you crave sugary drinks?"

ANS: C Weight loss occurs because the body is no longer able to absorb glucose and starts to break down protein and fat for energy. The patient is thirsty but does not necessarily crave sugar-containing fluids. Increased appetite is a classic symptom of type 1 diabetes. With the classic symptom of polyuria, urine will be very dilute

A patient with diabetes who has bacterial pneumonia is being treated with IV gentamicin (Garamycin) 60 mg IV BID. The nurse will monitor for adverse effects of the medication by evaluating the patient's a. blood glucose. b. urine osmolality. c. serum creatinine. d. serum potassium.

ANS: C When a patient at risk for chronic kidney disease (CKD) receives a potentially nephrotoxic medication, it is important to monitor renal function with BUN and creatinine levels. The other laboratory values would not be useful in assessing for the adverse effects of the gentamicin.

A 22-year-old female patient with an exacerbation of ulcerative colitis is having 15 to 20 stools daily and has excoriated perianal skin. Which patient behavior indicates that teaching regarding maintenance of skin integrity has been effective? a. The patient uses incontinence briefs to contain loose stools. b. The patient asks for antidiarrheal medication after each stool. c. The patient uses witch hazel compresses to decrease irritation. d. The patient cleans the perianal area with soap after each stool.

ANS: C Witch hazel compresses are suggested to reduce anal irritation and discomfort. Incontinence briefs may trap diarrhea and increase the incidence of skin breakdown. Antidiarrheal medications are not given 15 to 20 times a day. The perianal area should be washed with plain water after each stool.

A 82-year-old patient in a long-term care facility has several medications prescribed. After the patient is newly diagnosed with hypothyroidism, the nurse will need to consult with the health care provider before administering a. docusate (Colace). b. ibuprofen (Motrin). c. diazepam (Valium). d. cefoxitin (Mefoxin).

ANS: C Worsening of mental status and myxedema coma can be precipitated by the use of sedatives, especially in older adults. The nurse should discuss the use of diazepam with the health care provider before administration. The other medications may be given safely to the patient.

The nurse would anticipate inserting a Coudé catheter for which patient? a. An 8-year-old male undergoing anesthesia for a tonsillectomy b. A 24-year-old female who is going into labor c. A 56-year-old male admitted for bladder irrigation d. An 86-year-old female admitted for a urinary tract infection.

ANS: C A Coudé catheter has a curved tip that is used for patients with enlarged prostates. This would be indicated for a middle-aged male who needs bladder irrigation. Coudé catheters are not indicated for children or women.

The nurse suspects that a urinary tract infection has progressed to cystitis when the patient complains of which symptom? a. Dysuria b. Flank pain c. Frequency d. Fever and chills

ANS: C Cystitis is inflammation of the bladder; associated symptoms include hematuria and urgency/frequency. Dysuria is a commo n symptom of a lower urinary tract infection. Flank pain, fever, and chills are all signs of pyelonephritis.

Which statement by the patient about an upcoming computed tomography (CT) scan indicates a need for further teaching? a. "I'm allergic to shrimp, so I should monitor myself for an allergic reaction." b. "I will complete my bowel prep program the night before the scan." c. "I will be anesthetized so that I lie perfectly still during the procedure." d. "I will ask the technician to play music to ease my anxiety."

ANS: C Patients are not put under anesthesia for a CT scan; instead the nurse should educate patients about the need to lie perfectly still and about possible methods of overcoming feelings of claustrophobia. The other options are correct. Patients need to be assessed for an allergy to shellfish if receiving contrast for the CT. Bowel cleansing is often performed before CT. Listening to music will help the patient relax and remain still during the examination.

The nurse knows that which indwelling catheter procedure places the patient at greatest risk for acquiring a urinary tract infection? a. Emptying the drainage bag every 8 hours or when half full b. Kinking the catheter tubing to obtain a urine specimen c. Placing the drainage bag on the side rail of the patient's bed d. Failing to secure the catheter tubing to the patient's thigh

ANS: C Placing the drainage bag on the side rail of the bed could allow the bag to be raised above the level of the bladder and urine to flow back into the bladder. The urine in the drainage bag is a medium for bacteria; allowing it to reenter the bladder can cause infection. The drainage bag should be emptied and output recorded every 8 hours or when needed. Urine specimens are obtained by temporarily kinking the tubing; a prolonged kink could lead to bladder distention. Failure to secure the catheter to the patient's thigh places the patient at risk for tissue injury from catheter dislodgment.

A patient has fallen several times in the past week when attempting to get to the bathroom. The patient informs the nurse that he gets up 3 or 4 times a night to urinate. Which recommendation by the nurse is most appropriate in correcting this urinary problem? a. Clear the path to the bathroom of all obstacles before bed. b. Leave the bathroom light on to illuminate a pathway. c. Limit fluid and caffeine intake before bed. d. Practice Kegel exercises to strengthen bladder muscles.

ANS: C Reducing fluids, especially caffeine and alcohol, before bedtime can reduce nocturia. Clearing a path to the restroom or illuminating the path, or shortening the distance to the restroom, may reduce falls but will not correct the urination problem. Kegel exercises are useful if a patient is experiencing incontinence.

A nurse is providing education to a patient being treated for a urinary tract infection. Which of the following statements by the patient indicates an understanding? a. "Since I'm taking medication, I do not need to worry about proper hygiene." b. "I should drink 15 to 20 glasses of fluid a day to help flush the bacteria out." c. "My medication may discolor my urine; this should resolve once the medication is stopped." d. "I should not have sexual intercourse until the infection has resolved."

ANS: C Some anti-infective medications turn urine colors; this is normal and will dissipate as the medication leaves the system. Even if the patient is on medication, hygiene is important to prevent spread or reinfection. Fluid intake should be increased to help flush out bacteria; however, 15 to 20 glasses is too much. Sexual intercourse is allowed with a urinary tract infection, as long as good hygiene and safe practices are used.

The nurse knows that indwelling catheters are placed before a cesarean because a. The patient may void uncontrollably during the procedure. b. A full bladder can cause the mother's heart rate to drop. c. Spinal anesthetics can temporarily disable urethral sphincters. d. The patient will not interrupt the procedure by asking to go to the bathroom.

ANS: C Spinal anesthetics may cause urinary retention due to the inability to sense or carry out the need to void. The patient is more likely to retain urine, rather than experience uncontrollable voiding. With spinal anesthesia, the patient will not be able to ambulate during the procedure. A full bladder has no impact on the pulse rate of the mother.

Which assessment question should the nurse ask if stress incontinence is suspected? a. "Does your bladder feel distended?" b. "Do you empty your bladder completely when you void?" c. "Do you experience urine leakage when you cough or sneeze?" d. "Do your symptoms increase with consumption of alcohol or caffeine?"

ANS: C Stress incontinence can be related to intra-abdominal pressure causing urine leakage, as would happen during coughing or sneezing. Asking the patient about the fullness of his bladder would rule out retention and overflow. An inability to void completely can refer to urge incontinence. Physiological causes and medications can effect elimination, but this is not related to stress incontinence.

To obtain a clean-voided urine specimen for a female patient, the nurse should teach the patient to a. Cleanse the urethral meatus from the area of most contamination to least. b. Initiate the first part of the urine stream directly into the collection cup. c. Hold the labia apart while voiding into the specimen cup. d. Drink fluids 5 minutes before collecting the urine specimen.

ANS: C The patient should hold the labia apart to reduce bacterial levels in the specimen. The urethral meatus should be cleansed from the area of least contamination to greatest contamination (or front-to-back). The initial steam flushes out microorganisms in the urethra and prevents bacterial transmission in the specimen. Drink fluids 30 to 60 minutes before giving a specimen.

Two hours after a closed percutaneous kidney biopsy, the client reports a dramatic increase in pain. What is the nurse's best first action? A. Reposition the client on the operative side. B. Administer prescribed opioid analgesic. C. Assess pulse rate and blood pressure. D. Check the Foley catheter for kinks.

ANS: C An increase in the intensity of pain after a percutaneous kidney biopsy is a symptom of internal hemorrhage.

With a renal threshold for glucose of 220 mg/dL, what is the expected response when a client has a blood glucose level of 400 mg/dL? A. 400 mg/dL of excreted glucose in the urine B. 220 mg/dL of excreted glucose in the urine C. 180 mg/dL of glucose is excreted in the urine D. No excreted glucose in the urine

ANS: C Blood glucose is freely filtered at the glomerulus. Therefore, if a client has a blood sugar level of 400 mg/dl, the filtrate in the proximal convoluted tubule will have a glucose concentration of 400 mg/dL. With a renal threshold of 220 mg/dl, a total of 220 mg/dL of the 400 mg/dL will be reabsorbed back into the systemic circulation, and the final urine will have a glucose concentration of 180 mg/dL.

Which of the following should a patient be taught after a hemorrhoidectomy? A. take mineral oil prior to bedtime B eat a low fiber diet to rest the colon C. administer oil retention enema to empty the colon D. use prescribed pain medication before a bowel movement

D

The client is going home after urography. Which instruction or precaution should the nurse teach this client? A. "Avoid direct contact with the urine for 24 hours until the radioisotope clears." B. "You are likely to experience some dribbling of urine for several weeks after this procedure." C. "Be sure to drink at least 3 L of fluids today to help eliminate the dye faster." D. "Your skin may become slightly yellow-tinged from the dye used in this procedure."

ANS: C Dyes used in urography are potentially nephrotoxic.

What is the result of stimulation of erythropoietin production in the kidney tissue? A. Increased blood flow to the kidney B. Inhibition of vitamin D and loss of bone density C. Increased bone marrow production of red blood cells D. Inhibition of the active transport of sodium, leading to hyponatremi

ANS: C Erythropoietin is produced in the kidney and released in response to decreased oxygen tension in the renal blood supply. Erythropoietin stimulates red blood cell (RBC) production in the bone marrow.

Confirmed by palpation and x-ray study, the client's right kidney is lower than the left kidney. What is the nurse's interpretation of this finding? A. The client has a problem involving the right kidney. B. The client has a problem involving the left kidney. C. The client has both kidneys in the normal position. D. The client is at increased risk for kidney impairment.

ANS: C Normally, the right kidney is positioned somewhat lower than the left kidney. This anatomic difference in otherwise symmetric organs is caused by liver displacement. The significance of this difference is that the right kidney is easier to palpate in an adult than is the left kidney.

After obtaining the information shown in the accompanying figure regarding a patient with Addison's disease, which prescribed action will the nurse take first a. Give 4 oz of fruit juice orally. b. Recheck the blood glucose level. c. Infuse 5% dextrose and 0.9% saline. d. Administer oxygen therapy as needed.

ANS: C The patient's poor skin turgor, hypotension, and hyponatremia indicate an Addisonian crisis. Immediate correction of the hypovolemia and hyponatremia is needed. The other actions may also be needed but are not the initial action for the patient

The client has an elevated blood urea nitrogen (BUN) level and an increased ratio of blood urea nitrogen to creatinine. What is the nurse's interpretation of these laboratory results? A. The client probably has a urinary tract infection. B. The client may be overhydrated. C. The kidney may be hypoperfused. D. The kidney may be damaged.

ANS: C When dehydration or renal hypoperfusion exist, the BUN level rises more rapidly than the serum creatinine level, causing the ratio to be increased, even when no renal dysfunction is present.

When establishing a diagnosis of altered urinary elimination, the nurse should first a. Establish normal voiding patterns for the patient. b. Encourage the patient to flush kidneys by drinking excessive fluids. c. Monitor patients' voiding attempts by assisting them with every attempt. d. Discuss causes and solutions to problems related to micturition.

ANS: D The nurse should assess first to determine cause, then should discuss and create goals with the patient, so nurse and patient can work in tandem to normalize voiding. The nurse should incorporate the patient's input into creating a plan of care for the patient. Drinking excessive fluid will not help and may worsen alterations in urinary elimination. The nurse does not need to monitor every void attempt by the patient; instead the nurse should provide patient education. The nurse asks the patient about normal voiding patterns, but establishing voiding patterns is a later intervention.

The nurse teaches a 64-year-old woman to prevent the recurrence of renal calculi by a. using a filter to strain all urine. b. avoiding dietary sources of calcium. c. choosing diuretic fluids such as coffee. d. drinking 2000 to 3000 mL of fluid a day.

ANS: D A fluid intake of 2000 to 3000 mL daily is recommended to help flush out minerals before stones can form. Avoidance of calcium is not usually recommended for patients with renal calculi. Coffee tends to increase stone recurrence. There is no need for a patient to strain all urine routinely after a stone has passed, and this will not prevent stones.

To prevent the recurrence of renal calculi, the nurse teaches the patient to: a. use a filter to strain all urine. b. avoid dietary sources of calcium. c. drink diuretic fluids such as coffee. d. have 2000 to 3000 mL of fluid a day.

ANS: D A fluid intake of 2000 to 3000 mL daily is recommended to help flush out minerals before stones can form. Avoidance of calcium is not usually recommended for patients with renal calculi. Coffee tends to increase stone recurrence. There is no need for a patient to strain all urine routinely after a stone has passed, and this will not prevent stones.

prevent the recurrence of renal calculi, the nurse teaches the patient to a. use a filter to strain all urine. b. avoid dietary sources of calcium. c. drink diuretic fluids such as coffee. d. have 2000 to 3000 mL of fluid a day.

ANS: D A fluid intake of 2000 to 3000 mL daily is recommended to help flush out minerals before stones can form. Avoidance of calcium is not usually recommended for patients with renal calculi. Coffee tends to increase stone recurrence. There is no need for a patient to strain all urine routinely after a stone has passed, and this will not prevent stones.

A patient who recently has been experiencing frequent heartburn is seen in the clinic. The nurse will anticipate teaching the patient about a. barium swallow. b. radionuclide tests. c. endoscopy procedures. d. proton pump inhibitors.

ANS: D Because diagnostic testing for heartburn that is probably caused by gastroesophageal reflux disease (GERD) is expensive and uncomfortable, proton pump inhibitors are frequently used for a short period as the first step in the diagnosis of GERD. The other tests may be used but are not usually the first step in diagnosis.

A patient is told to take replacement pancreatic enzymes to prevent malabsorption of fat. How should the enzymes be administered? A. Mixed in fruit juice B. Mixed in chocolate milk C. Enema D. IV infusion

a Rationale Pancreatic enzymes are mixed in fruit juice or applesauce. They should not be mixed with a protein. They are administered by mouth. Reference: 1093-1094

A 64-year-old woman who has chronic constipation asks the nurse about the use of psyllium (Metamucil). Which information will the nurse include in the response? a. Absorption of fat-soluble vitamins may be reduced by fiber-containing laxatives. b. Dietary sources of fiber should be eliminated to prevent excessive gas formation. c. Use of this type of laxative to prevent constipation does not cause adverse effects. d. Large amounts of fluid should be taken to prevent impaction or bowel obstruction.

ANS: D A high fluid intake is needed when patients are using bulk-forming laxatives to avoid worsening constipation. Although bulk-forming laxatives are generally safe, the nurse should emphasize the possibility of constipation or obstipation if inadequate fluid intake occurs. Although increased gas formation is likely to occur with increased dietary fiber, the patient should gradually increase dietary fiber and eventually may not need the psyllium. Fat-soluble vitamin absorption is blocked by stool softeners and lubricants, not by bulk-forming laxatives

Which assessment finding is of most concern for a 46-year-old woman with acute pancreatitis? a. Absent bowel sounds b. Abdominal tenderness c. Left upper quadrant pain d. Palpable abdominal mass

ANS: D A palpable abdominal mass may indicate the presence of a pancreatic abscess, which will require rapid surgical drainage to prevent sepsis. Absent bowel sounds, abdominal tenderness, and left upper quadrant pain are common in acute pancreatitis and do not require rapid action to prevent further complications

Which nursing action is essential for a patient immediately after a renal biopsy? a. Check blood glucose to assess for hyperglycemia or hypoglycemia. b. Insert a urinary catheter and test urine for gross or microscopic hematuria. c. Monitor the blood urea nitrogen (BUN) and creatinine to assess renal function. d. Apply a pressure dressing and keep the patient on the affected side for 30 minutes.

ANS: D A pressure dressing is applied and the patient is kept on the affected side for 30 to 60 minutes to put pressure on the biopsy side and decrease the risk for bleeding. The blood glucose and BUN/creatinine will not be affected by the biopsy. Although monitoring for hematuria is needed, there is no need for catheterization

Which information is most important for the nurse to communicate rapidly to the health care provider about a patient admitted with possible syndrome of inappropriate antidiuretic hormone (SIADH)? a. The patient has a recent weight gain of 9 lb. b. The patient complains of dyspnea with activity. c. The patient has a urine specific gravity of 1.025. d. The patient has a serum sodium level of 118 mEq/L.

ANS: D A serum sodium of less than 120 mEq/L increases the risk for complications such as seizures and needs rapid correction. The other data are not unusual for a patient with SIADH and do not indicate the need for rapid action

A 73-year-old patient with diverticulosis has a large bowel obstruction. The nurse will monitor for a. referred back pain. b. metabolic alkalosis. c. projectile vomiting. d. abdominal distention.

ANS: D Abdominal distention is seen in lower intestinal obstruction. Referred back pain is not a common clinical manifestation of intestinal obstruction. Metabolic alkalosis is common in high intestinal obstruction because of the loss of HCl acid from vomiting. Projectile vomiting is associated with higher intestinal obstruction.

A 23-year-old patient is admitted with diabetes insipidus. Which action will be most appropriate for the registered nurse (RN) to delegate to an experienced licensed practical/vocational nurse (LPN/LVN)? a. Titrate the infusion of 5% dextrose in water. b. Teach the patient how to use desmopressin (DDAVP) nasal spray. c. Assess the patient's hydration status every 8 hours. d. Administer subcutaneous DDAVP.

ANS: D Administration of medications is included in LPN/LVN education and scope of practice. Assessments, patient teaching, and titrating fluid infusions are more complex skills and should be done by the RN.

The nurse will ask a 64-year-old patient being admitted with acute pancreatitis specifically about a history of a. diabetes mellitus. b. high-protein diet. c. cigarette smoking. d. alcohol consumption.

ANS: D Alcohol use is one of the most common risk factors for pancreatitis in the United States. Cigarette smoking, diabetes, and high-protein diets are not risk factors

Which breakfast choice indicates a patient's good understanding of information about a diet for celiac disease? a. Oatmeal with nonfat milk b. Whole wheat toast with butter c. Bagel with low-fat cream cheese d. Corn tortilla with scrambled eggs

ANS: D Avoidance of gluten-containing foods is the only treatment for celiac disease. Corn does not contain gluten, while oatmeal and wheat do

The nurse will anticipate teaching a patient experiencing frequent heartburn about a. a barium swallow. b. radionuclide tests. c. endoscopy procedures. d. proton pump inhibitors.

ANS: D Because diagnostic testing for heartburn that is probably caused by gastroesophageal reflux disease (GERD) is expensive and uncomfortable, proton pump inhibitors are frequently used for a short period as the first step in the diagnosis of GERD. The other tests may be used but are not usually the first step in diagnosis

Which assessment finding is most important to report to the health care provider regarding a patient who has had left-sided extracorporeal shock wave lithotripsy? a. Blood in urine b. Left flank bruising c. Left flank discomfort d. Decreased urine output

ANS: D Because lithotripsy breaks the stone into small sand, which could cause obstruction, it is important to report a drop in urine output. Left flank pain, bruising, and hematuria are common after lithotripsy.

When the nurse is caring for a patient who has had left-sided extracorporeal shock wave lithotripsy, which assessment finding is most important to report to the health care provider? a. Blood in urine b. Left flank pain c. Left flank bruising d. Drop in urine output

ANS: D Because lithotripsy breaks the stone into small sand, which could cause obstruction, it is important to report a drop in urine output. Left flank pain, bruising, and hematuria are common after lithotripsy. DIF: Cognitive Level: Application REF: 1138-1139

A patient with protein calorie malnutrition who has had abdominal surgery is receiving parenteral nutrition (PN). Which assessment information obtained by the nurse is the best indicator that the patient is receiving adequate nutrition? a. Blood glucose is 110 mg/dL. b. Serum albumin level is 3.5 mg/dL. c. Fluid intake and output are balanced. d. Surgical incision is healing normally.

ANS: D Because poor wound healing is a possible complication of malnutrition for this patient, normal healing of the incision is an indicator of the effectiveness of the PN in providing adequate nutrition. Blood glucose is monitored to prevent the complications of hyperglycemia and hypoglycemia, but it does not indicate that the patient's nutrition is adequate. The intake and output will be monitored but do not indicate that the PN is effective. The albumin level is in the low-normal range but does not reflect adequate caloric intake, which is also important for the patient.

All of the following orders are received for a patient who has vomited 1500 mL of bright red blood. Which order will the nurse implement first? a. Insert a nasogastric (NG) tube and connect to suction. b. Administer intravenous (IV) famotidine (Pepcid) 40 mg. c. Draw blood for typing and crossmatching. d. Infuse 1000 mL of lactated Ringer's solution.

ANS: D Because the patient has vomited a large amount of blood, correction of hypovolemia and prevention of hypovolemic shock are the priorities. The other actions also are important to implement quickly but are not the highest priorities.

A patient who is receiving chemotherapy develops a Candida albicans oral infection. The nurse will anticipate the need for a. hydrogen peroxide rinses. b. the use of antiviral agents. c. referral to a dentist for professional tooth cleaning. d. administration of nystatin (Mycostatin) oral tablets.

ANS: D Candida albicans is treated with an antifungal such as nystatin. Oral saltwater rinses may be used but will not cure the infection. Antiviral agents are used for viral infections such as herpes simplex. Referral to a dentist is indicated for gingivitis but not for Candida infection.

A hospitalized diabetic patient received 38 U of NPH insulin at 7:00 AM. At 1:00 PM, the patient has been away from the nursing unit for 2 hours, missing the lunch delivery while awaiting a chest x-ray. To prevent hypoglycemia, the best action by the nurse is to a. save the lunch tray for the patient's later return to the unit. b. ask that diagnostic testing area staff to start a 5% dextrose IV. c. send a glass of milk or orange juice to the patient in the diagnostic testing area. d. request that if testing is further delayed, the patient be returned to the unit to eat.

ANS: D Consistency for mealtimes assists with regulation of blood glucose, so the best option is for the patient to have lunch at the usual time. Waiting to eat until after the procedure is likely to cause hypoglycemia. Administration of an IV solution is unnecessarily invasive for the patient. A glass of milk or juice will keep the patient from becoming hypoglycemic but will cause a rapid rise in blood glucose because of the rapid absorption of the simple carbohydrate in these items

A 24-year-old female contracts hepatitis from contaminated food. During the acute (icteric) phase of the patient's illness, the nurse would expect serologic testing to reveal a. antibody to hepatitis D (anti-HDV). b. hepatitis B surface antigen (HBsAg). c. anti-hepatitis A virus immunoglobulin G (anti-HAV IgG). d. anti-hepatitis A virus immunoglobulin M (anti-HAV IgM).

ANS: D Hepatitis A is transmitted through the oral-fecal route, and antibody to HAV IgM appears during the acute phase of hepatitis A. The patient would not have antigen for hepatitis B or antibody for hepatitis D. Anti-HAV IgG would indicate past infection and lifelong immunity.

Which finding by the nurse will be most helpful in determining whether a 67-year-old patient with benign prostatic hyperplasia has an upper urinary tract infection (UTI)? a. Bladder distention b. Foul-smelling urine c. Suprapubic discomfort d. Costovertebral tenderness

ANS: D Costovertebral tenderness is characteristic of pyelonephritis. Bladder distention, foul-smelling urine, and suprapubic discomfort are characteristic of lower UTI and are likely to be present if the patient also has an upper UTI.

A 72-year-old who has benign prostatic hyperplasia is admitted to the hospital with chills, fever, and vomiting. Which finding by the nurse will be most helpful in determining whether the patient has an upper urinary tract infection (UTI)? a. Suprapubic pain b. Bladder distention c. Foul-smelling urine d. Costovertebral tenderness

ANS: D Costovertebral tenderness is characteristic of pyelonephritis. The other symptoms are characteristic of lower UTI and are likely to be present if the patient also has an upper UTI. DIF: Cognitive Level: Application REF: 1128

The nurse will plan to teach a patient with Crohn's disease who has megaloblastic anemia about the need for a. oral ferrous sulfate tablets. b. regular blood transfusions. c. iron dextran (Imferon) infusions. d. cobalamin (B12) spray or injections.

ANS: D Crohn's disease frequently affects the ileum, where absorption of cobalamin occurs. Cobalamin must be administered regularly by nasal spray or IM to correct the anemia. Iron deficiency does not cause megaloblastic anemia. The patient may need occasional transfusions but not regularly scheduled transfusions

Which statement by a 50-year-old female patient indicates to the nurse that further assessment of thyroid function may be necessary? a. "I notice my breasts are tender lately." b. "I am so thirsty that I drink all day long." c. "I get up several times at night to urinate." d. "I feel a lump in my throat when I swallow."

ANS: D Difficulty in swallowing can occur with a goiter. Nocturia is associated with diseases such as diabetes mellitus, diabetes insipidus, or chronic kidney disease. Breast tenderness would occur with excessive gonadal hormone levels. Thirst is a sign of disease such as diabetes.

To assess whether there is any improvement in a patient's dysuria, which question will the nurse ask? a. "Do you have to urinate at night?" b. "Do you have blood in your urine?" c. "Do you have to urinate frequently?" d. "Do you have pain when you urinate?"

ANS: D Dysuria is painful urination. The alternate responses are used to assess other urinary tract symptoms: hematuria, nocturia, and frequency

When a 72-year-old patient is diagnosed with achalasia, the nurse will teach the patient that a. lying down after meals is recommended. b. a liquid or blenderized diet will be necessary. c. drinking fluids with meals should be avoided. d. treatment may include endoscopic procedures.

ANS: D Endoscopic and laparoscopic procedures are the most effective therapy for improving symptoms caused by achalasia. Keeping the head elevated after eating will improve esophageal emptying. A semisoft diet is recommended to improve esophageal emptying. Patients are advised to drink fluid with meals. DIF: Cognitive Level: Apply (application) REF: 940 TOP: Nursing Process: Planning MSC: NCLEX: Physiological Integrity

A 62-year-old asks the nurse for a perineal pad, stating that laughing or coughing causes leakage of urine. Which intervention is most appropriate to include in the care plan? a. Assist the patient to the bathroom q3hr. b. Place a commode at the patient's bedside. c. Demonstrate how to perform the Credé maneuver. d. Teach the patient how to perform Kegel exercises.

ANS: D Exercises to strengthen the pelvic floor muscles will help reduce stress incontinence. The Credé maneuver is used to help empty the bladder for patients with overflow incontinence. Placing the commode close to the bedside and assisting the patient to the bathroom are helpful for functional incontinence. DIF: Cognitive Level: Application REF: 1148

A 32-year-old patient who is employed as a hairdresser and has a 15 pack-year history of cigarette smoking is scheduled for an annual physical examination. The nurse will plan to teach the patient about the increased risk for a. renal failure. b. kidney stones. c. pyelonephritis. d. bladder cancer.

ANS: D Exposure to the chemicals involved with working as a hairdresser and in smoking both increase the risk of bladder cancer, and the nurse should assess whether the patient understands this risk. The patient is not at increased risk for renal failure, pyelonephritis, or kidney stones

A 42-year-old woman is admitted to the outpatient testing area for an ultrasound of the gallbladder. Which information obtained by the nurse indicates that the ultrasound may need to be rescheduled? a. The patient took a laxative the previous evening. b. The patient had a high-fat meal the previous evening. c. The patient has a permanent gastrostomy tube in place. d. The patient ate a low-fat bagel 4 hours ago for breakfast.

ANS: D Food intake can cause the gallbladder to contract and result in a suboptimal study. The patient should be NPO for 8 to 12 hours before the test. A high-fat meal the previous evening, laxative use, or a gastrostomy tube will not affect the results of the study.

After the nurse teaches a patient with gastroesophageal reflux disease (GERD) about recommended dietary modifications, which diet choice for a snack 2 hours before bedtime indicates that the teaching has been effective? a. Chocolate pudding b. Glass of low-fat milk c. Peanut butter sandwich d. Cherry gelatin and fruit

ANS: D Gelatin and fruit are low fat and will not decrease lower esophageal sphincter (LES) pressure. Foods like chocolate are avoided because they lower LES pressure. Milk products increase gastric acid secretion. High-fat foods such as peanut butter decrease both gastric emptying and LES pressure.

A 18-year-old male patient with a small stature is scheduled for a growth hormone stimulation test. In preparation for the test, the nurse will obtain a. ice in a basin. b. glargine insulin. c. a cardiac monitor. d. 50% dextrose solution.

ANS: D Hypoglycemia is induced during the growth hormone stimulation test, and the nurse should be ready to administer 50% dextrose immediately. Regular insulin is used to induce hypoglycemia (glargine is never given IV). The patient does not require cardiac monitoring during the test. Although blood samples for some tests must be kept on ice, this is not true for the growth hormone stimulation test.

The nurse receives the following information about a 51-year-old woman who is scheduled for a colonoscopy. Which information should be communicated to the health care provider before sending the patient for the procedure? a. The patient has a permanent pacemaker to prevent bradycardia. b. The patient is worried about discomfort during the examination. c. The patient has had an allergic reaction to shellfish and iodine in the past. d. The patient refused to drink the ordered polyethylene glycol (GoLYTELY).

ANS: D If the patient has had inadequate bowel preparation, the colon cannot be visualized and the procedure should be rescheduled. Because contrast solution is not used during colonoscopy, the iodine allergy is not pertinent. A pacemaker is a contraindication to magnetic resonance imaging (MRI), but not to colonoscopy. The nurse should instruct the patient about the sedation used during the examination to decrease the patient's anxiety about discomfort

The nurse will anticipate that a 61-year-old patient who has an enlarged prostate detected by digital rectal examination (DRE) and an elevated prostate specific antigen (PSA) level will need teaching about a. cystourethroscopy. b. uroflowmetry studies. c. magnetic resonance imaging (MRI). d. transrectal ultrasonography (TRUS).

ANS: D In a patient with an abnormal DRE and elevated PSA, transrectal ultrasound is used to visualize the prostate for biopsy. Uroflowmetry studies help determine the extent of urine blockage and treatment, but there is no indication that this is a problem for this patient. Cystoscopy may be used before prostatectomy but will not be done until after the TRUS and biopsy. MRI is used to determine whether prostatic cancer has metastasized but would not be ordered at this stage of the diagnostic process.

A 68-year-old female patient admitted to the hospital with dehydration is confused and incontinent of urine. Which nursing action will be best to include in the plan of care? a. Restrict fluids between meals and after the evening meal. b. Apply absorbent incontinent pads liberally over the bed linens. c. Insert an indwelling catheter until the symptoms have resolved. d. Assist the patient to the bathroom every 2 hours during the day.

ANS: D In older or confused patients, incontinence may be avoided by using scheduled toileting times. Indwelling catheters increase the risk for urinary tract infection (UTI). Incontinent pads increase the risk for skin breakdown. Restricting fluids is not appropriate in a patient with dehydration.

Which action by a patient indicates that the home health nurse's teaching about glargine and regular insulin has been successful? a. The patient administers the glargine 30 minutes before each meal. b. The patient's family prefills the syringes with the mix of insulins weekly. c. The patient draws up the regular insulin and then the glargine in the same syringe. d. The patient disposes of the open vials of glargine and regular insulin after 4 weeks.

ANS: D Insulin can be stored at room temperature for 4 weeks. Glargine should not be mixed with other insulins or prefilled and stored. Short-acting regular insulin is administered before meals, while glargine is given once daily.

When working in the urology/nephrology clinic, which patient could the nurse delegate to an experienced licensed practical/vocational nurse (LPN/LVN)? a. Patient who is scheduled for a renal biopsy after a recent kidney transplant b. Patient who will need monitoring for several hours after a renal arteriogram c. Patient who requires teaching about possible post-cystoscopy complications d. Patient who will have catheterization to check for residual urine after voiding

ANS: D LPN/LVN education includes common procedures such as catheterization of stable patients. The other patients require more complex assessments and/or patient teaching that are included in registered nurse (RN) education and scope of practice.

A 62-year-old man patient who requires daily use of a nonsteroidal antiinflammatory drug (NSAID) for the management of severe rheumatoid arthritis has recently developed melena. The nurse will anticipate teaching the patient about a. substitution of acetaminophen (Tylenol) for the NSAID. b. use of enteric-coated NSAIDs to reduce gastric irritation. c. reasons for using corticosteroids to treat the rheumatoid arthritis. d. misoprostol (Cytotec) to protect the gastrointestinal (GI) mucosa.

ANS: D Misoprostol, a prostaglandin analog, reduces acid secretion and the incidence of upper GI bleeding associated with NSAID use. Enteric coating of NSAIDs does not reduce the risk for GI bleeding. Corticosteroids increase the risk for ulcer development, and will not be substituted for NSAIDs for this patient. Acetaminophen will not be effective in treating the patient's rheumatoid arthritis

Which assessment finding would the nurse need to report most quickly to the health care provider regarding a patient with acute pancreatitis? a. Nausea and vomiting b. Hypotonic bowel sounds c. Abdominal tenderness and guarding d. Muscle twitching and finger numbness

ANS: D Muscle twitching and finger numbness indicate hypocalcemia, which may lead to tetany unless calcium gluconate is administered. Although the other findings should also be reported to the health care provider, they do not indicate complications that require rapid action

A 67-year-old male patient with acute pancreatitis has a nasogastric (NG) tube to suction and is NPO. Which information obtained by the nurse indicates that these therapies have been effective? a. Bowel sounds are present. b. Grey Turner sign resolves. c. Electrolyte levels are normal. d. Abdominal pain is decreased.

ANS: D NG suction and NPO status will decrease the release of pancreatic enzymes into the pancreas and decrease pain. Although bowel sounds may be hypotonic with acute pancreatitis, the presence of bowel sounds does not indicate that treatment with NG suction and NPO status has been effective. Electrolyte levels may be abnormal with NG suction and must be replaced by appropriate IV infusion. Although Grey Turner sign will eventually resolve, it would not be appropriate to wait for this to occur to determine whether treatment was effective

Which finding by the nurse when assessing a patient with a large pituitary adenoma is most important to report to the health care provider? a. Changes in visual field b. Milk leaking from breasts c. Blood glucose 150 mg/dL d. Nausea and projectile vomiting

ANS: D Nausea and projectile vomiting may indicate increased intracranial pressure, which will require rapid actions for diagnosis and treatment. Changes in the visual field, elevated blood glucose, and galactorrhea are common with pituitary adenoma, but these do not require rapid action to prevent life-threatening complications.

Which question from the nurse would help determine if a patient's abdominal pain might indicate irritable bowel syndrome? a. "Have you been passing a lot of gas?" b. "What foods affect your bowel patterns?" c. "Do you have any abdominal distention?" d. "How long have you had abdominal pain?"

ANS: D One criterion for the diagnosis of irritable bowel syndrome (IBS) is the presence of abdominal discomfort or pain for at least 3 months. Abdominal distention, flatulence, and food intolerance are also associated with IBS, but are not diagnostic criteria.

Which assessment data reported by a 28-year-old male patient is consistent with a lower urinary tract infection (UTI)? a. Poor urine output b. Bilateral flank pain c. Nausea and vomiting d. Burning on urination

ANS: D Pain with urination is a common symptom of a lower UTI. Urine output does not decrease, but frequency may be experienced. Flank pain and nausea are associated with an upper UTI.

When assessing the patient who has a lower urinary tract infection (UTI), the nurse will initially ask about a. nausea. b. flank pain. c. poor urine output. d. pain with urination.

ANS: D Pain with urination is a common symptom of a lower UTI. Urine output does not decrease, but frequency may be experienced. Flank pain and nausea are associated with an upper UTI. DIF: Cognitive Level: Application REF: 1123-1124

A 61-year-old female patient admitted with pneumonia has a total serum calcium level of 13.3 mg/dL (3.3 mmol/L). The nurse will anticipate the need to teach the patient about testing for _____ levels. a. calcitonin b. catecholamine c. thyroid hormone d. parathyroid hormone

ANS: D Parathyroid hormone is the major controller of blood calcium levels. Although calcitonin secretion is a countermechanism to parathyroid hormone, it does not play a major role in calcium balance. Catecholamine and thyroid hormone levels do not affect serum calcium level.

To prevent pregnancy in a patient who has been sexually assaulted, the nurse in the emergency department will plan to teach the patient about the use of a. mifepristone (RU-486). b. dilation and evacuation. c. methotrexate with misoprostol. d. levonorgestrel (Plan-B One-Step).

ANS: D Plan B One-Step reduces the risk of pregnancy when taken within 72 hours of intercourse. The other methods are used for therapeutic abortion, but not for pregnancy prevention after unprotected intercourse

A 51-year-old woman had an incisional cholecystectomy 6 hours ago. The nurse will place the highest priority on assisting the patient to a. choose low-fat foods from the menu. b. perform leg exercises hourly while awake. c. ambulate the evening of the operative day. d. turn, cough, and deep breathe every 2 hours.

ANS: D Postoperative nursing care after a cholecystectomy focuses on prevention of respiratory complications because the surgical incision is high in the abdomen and impairs coughing and deep breathing. The other nursing actions are also important to implement but are not as high a priority as ensuring adequate ventilation

The nurse is interviewing a new patient with diabetes who receives rosiglitazone (Avandia) through a restricted access medication program. What is most important for the nurse to report immediately to the health care provider? a. The patient's blood pressure is 154/92. b. The patient has a history of emphysema. c. The patient's blood glucose is 86 mg/dL. d. The patient has chest pressure when walking.

ANS: D Rosiglitazone can cause myocardial ischemia. The nurse should immediately notify the health care provider and expect orders to discontinue the medication. There is no urgent need to discuss the other data with the health care provider

Which action will be included in the care for a patient who has recently been diagnosed with asymptomatic nonalcoholic fatty liver disease (NAFLD)? a. Teach symptoms of variceal bleeding. b. Draw blood for hepatitis serology testing. c. Discuss the need to increase caloric intake. d. Review the patient's current medication list.

ANS: D Some medications can increase the risk for NAFLD, and they should be eliminated. NAFLD is not associated with hepatitis, weight loss is usually indicated, and variceal bleeding would not be a concern in a patient with asymptomatic NAFLD

The health care provider prescribes antacids and sucralfate (Carafate) for treatment of a patient's peptic ulcer. The nurse will teach the patient to take a. sucralfate at bedtime and antacids before each meal. b. sucralfate and antacids together 30 minutes before meals. c. antacids 30 minutes before each dose of sucralfate is taken. d. antacids after meals and sucralfate 30 minutes before meals.

ANS: D Sucralfate is most effective when the pH is low and should not be given with or soon after antacids. Antacids are most effective when taken after eating. Administration of sucralfate 30 minutes before eating and antacids just after eating will ensure that both drugs can be most effective. The other regimens will decrease the effectiveness of the medications.

Which finding indicates to the nurse that a patient's transjugular intrahepatic portosystemic shunt (TIPS) placed 3 months ago has been effective? a. Increased serum albumin level b. Decreased indirect bilirubin level c. Improved alertness and orientation d. Fewer episodes of bleeding varices

ANS: D TIPS is used to lower pressure in the portal venous system and decrease the risk of bleeding from esophageal varices. Indirect bilirubin level and serum albumin levels are not affected by shunting procedures. TIPS will increase the risk for hepatic encephalopathy

To determine the severity of the symptoms for a 68-year-old patient with benign prostatic hyperplasia (BPH) the nurse will ask the patient about a. blood in the urine. b. lower back or hip pain. c. erectile dysfunction (ED). d. force of the urinary stream.

ANS: D The American Urological Association (AUA) Symptom Index for a patient with BPH asks questions about the force and frequency of urination, nocturia, etc. Blood in the urine, ED, and back or hip pain are not typical symptoms of BPH.

Which finding indicates a need to contact the health care provider before the nurse administers metformin (Glucophage)? a. The patient's blood glucose level is 174 mg/dL. b. The patient has gained 2 lb (0.9 kg) since yesterday. c. The patient is scheduled for a chest x-ray in an hour. d. The patient's blood urea nitrogen (BUN) level is 52 mg/dL.

ANS: D The BUN indicates possible renal failure, and metformin should not be used in patients with renal failure. The other findings are not contraindications to the use of metformin.

The nurse observes nursing assistive personnel (NAP) taking the following actions when caring for a patient with a retention catheter. Which action requires that the nurse intervene? a. Taping the catheter to the skin on the patient's upper inner thigh b. Cleaning around the patient's urinary meatus with soap and water c. Using an alcohol-based hand cleaner before performing catheter care d. Disconnecting the catheter from the drainage tube to obtain a specimen

ANS: D The catheter should not be disconnected from the drainage tube because this increases the risk for urinary tract infection (UTI). The other actions are appropriate and do not require any intervention. DIF: Cognitive Level: Application REF: 1152-1154

A patient with type 2 diabetes is scheduled for a follow-up visit in the clinic several months from now. Which test will the nurse schedule to evaluate the effectiveness of treatment for the patient? a. Urine dipstick for glucose b. Oral glucose tolerance test c. Fasting blood glucose level d. Glycosylated hemoglobin level

ANS: D The glycosylated hemoglobin (A1C or HbA1C) test shows the overall control of glucose over 90 to 120 days. A fasting blood level indicates only the glucose level at one time. Urine glucose testing is not an accurate reflection of blood glucose level and does not reflect the glucose over a prolonged time. Oral glucose tolerance testing is done to diagnose diabetes, but is not used for monitoring glucose control once diabetes has been diagnosed

Which action will the nurse include in the plan of care for a patient who has had a ureterolithotomy and has a left ureteral catheter and a urethral catheter in place? a. Provide education about home care for both catheters. b. Apply continuous steady tension to the ureteral catheter. c. Clamp the ureteral catheter unless output from the urethral catheter stops. d. Call the health care provider if the ureteral catheter output drops suddenly.

ANS: D The health care provider should be notified if the ureteral catheter output decreases since obstruction of this catheter may result in an increase in pressure in the renal pelvis. Tension on the ureteral catheter should be avoided in order to prevent catheter displacement. To avoid pressure in the renal pelvis, the catheter is not clamped. Since the patient is not usually discharged with a ureteral catheter in place, patient teaching about both catheters is not needed. DIF: Cognitive Level: Application REF: 1153-1154

After vertical banded gastroplasty, a 42-year-old male patient returns to the surgical nursing unit with a nasogastric tube to low, intermittent suction and a patient-controlled analgesia (PCA) machine for pain control. Which nursing action should be included in the postoperative plan of care? a. Offer sips of fruit juices at frequent intervals. b. Irrigate the nasogastric (NG) tube frequently. c. Remind the patient that PCA use may slow the return of bowel function. d. Support the surgical incision during patient coughing and turning in bed.

ANS: D The incision should be protected from strain to decrease the risk for wound dehiscence. The patient should be encouraged to use the PCA because pain control will improve the cough effort and patient mobility. NG irrigation may damage the suture line or overfill the stomach pouch. Sugar-free clear liquids are offered during the immediate postoperative time to decrease the risk for dumping syndrome

Which data will the nurse monitor in relation to the 4+ pitting edema assessed in a patient with cirrhosis? a. Hemoglobin b. Temperature c. Activity level d. Albumin level

ANS: D The low oncotic pressure caused by hypoalbuminemia is a major pathophysiologic factor in the development of edema. The other parameters should also be monitored, but they are not directly associated with the patient's current symptoms

The nurse is performing an admission assessment on a 20-year-old college student who is being admitted for electrolyte disorders of unknown etiology. Which assessment finding is most important to report to the health care provider? a. The patient's knuckles are macerated. b. The patient uses laxatives on a daily basis. c. The patient has a history of weight fluctuations. d. The patient's serum potassium level is 2.9 mEq/L.

ANS: D The low serum potassium level may cause life-threatening cardiac dysrhythmias and potassium supplementation is needed rapidly. The other information also will be reported because it suggests that bulimia may be the etiology of the patient's electrolyte disturbances, but it does not suggest imminent life-threatening complications.

A 44-year-old female patient with Cushing syndrome is admitted for adrenalectomy. Which intervention by the nurse will be most helpful for a nursing diagnosis of disturbed body image related to changes in appearance? a. Reassure the patient that the physical changes are very common in patients with Cushing syndrome. b. Discuss the use of diet and exercise in controlling the weight gain associated with Cushing syndrome. c. Teach the patient that the metabolic impact of Cushing syndrome is of more importance than appearance. d. Remind the patient that most of the physical changes caused by Cushing syndrome will resolve after surgery.

ANS: D The most reassuring communication to the patient is that the physical and emotional changes caused by the Cushing syndrome will resolve after hormone levels return to normal postoperatively. Reassurance that the physical changes are expected or that there are more serious physiologic problems associated with Cushing syndrome are not therapeutic responses. The patient's physiological changes are caused by the high hormone levels, not by the patient's diet or exercise choices.

A patient with bladder cancer is scheduled for intravesical chemotherapy. In preparation for the treatment the nurse will teach the patient about a. premedicating to prevent nausea. b. where to obtain wigs and scarves. c. the importance of oral care during treatment. d. the need to empty the bladder before treatment.

ANS: D The patient will be asked to empty the bladder before instillation of the chemotherapy. Systemic side effects are not experienced with intravesical chemotherapy. DIF: Cognitive Level: Application REF: 1146

Which assessment should the nurse perform first for a patient who just vomited bright red blood? a. Measuring the quantity of emesis b. Palpating the abdomen for distention c. Auscultating the chest for breath sounds d. Taking the blood pressure (BP) and pulse

ANS: D The nurse is concerned about blood loss and possible hypovolemic shock in a patient with acute gastrointestinal (GI) bleeding. BP and pulse are the best indicators of these complications. The other information is important to obtain, but BP and pulse rate are the best indicators for assessing intravascular volume. DIF: Cognitive Level: Apply (application) REF: 925 | 956 OBJ: Special Questions: Prioritization TOP: Nursing Process: Assessment MSC: NCLEX: Physiological Integrity

A patient with a recent 20-pound unintended weight loss is diagnosed with stomach cancer. Which nursing action will be included in the plan of care? a. Refer the patient for hospice services. b. Infuse IV fluids through a central line. c. Teach the patient about antiemetic therapy. d. Offer supplemental feedings between meals.

ANS: D The patient data indicate a poor nutritional state and improvement in nutrition will be helpful in improving response to therapies such as surgery, chemotherapy, or radiation. Nausea and vomiting are not common clinical manifestations of stomach cancer. There is no indication that the patient requires hospice or IV fluid infusions.

The nurse receives change-of-shift report about the following four patients. Which patient will the nurse assess first? a. A patient who has malnutrition associated with 4+ generalized pitting edema b. A patient whose parenteral nutrition has 10 mL of solution left in the infusion bag c. A patient whose gastrostomy tube is plugged after crushed medications were given through the tube d. A patient who is receiving continuous enteral feedings and has new-onset crackles throughout the lungs

ANS: D The patient data suggest aspiration has occurred and rapid assessment and intervention are needed. The other patients also should be assessed as quickly as possible, but the data about them do not suggest any immediately life-threatening complications.

The nurse has been teaching a patient with type 2 diabetes about managing blood glucose levels and taking glipizide (Glucotrol). Which patient statement indicates a need for additional teaching? a. "If I overeat at a meal, I will still take the usual dose of medication." b. "Other medications besides the Glucotrol may affect my blood sugar." c. "When I am ill, I may have to take insulin to control my blood sugar." d. "My diabetes won't cause complications because I don't need insulin."

ANS: D The patient should understand that type 2 diabetes places the patient at risk for many complications and that good glucose control is as important when taking oral agents as when using insulin. The other statements are accurate and indicate good understanding of the use of glipizide

After receiving change-of-shift report, which patient should the nurse assess first? a. Patient who is scheduled for the drain phase of a peritoneal dialysis exchange b. Patient with stage 4 chronic kidney disease who has an elevated phosphate level c. Patient with stage 5 chronic kidney disease who has a potassium level of 3.4 mEq/L d. Patient who has just returned from having hemodialysis and has a heart rate of 124/min

ANS: D The patient who is tachycardic after hemodialysis may be bleeding or excessively hypovolemic and should be assessed immediately for these complications. The other patients also need assessments or interventions but are not at risk for life-threatening complications.

When caring for a patient with a history of a total gastrectomy, the nurse will monitor for a. constipation. b. dehydration. c. elevated total serum cholesterol. d. cobalamin (vitamin B12) deficiency.

ANS: D The patient with a total gastrectomy does not secrete intrinsic factor, which is needed for cobalamin (vitamin B12) absorption. Because the stomach absorbs only small amounts of water and nutrients, the patient is not at higher risk for dehydration, elevated cholesterol, or constipation.

A nurse is considering which patient to admit to the same room as a patient who had a liver transplant 3 weeks ago and is now hospitalized with acute rejection. Which patient would be the best choice? a. Patient who is receiving chemotherapy for liver cancer b. Patient who is receiving treatment for acute hepatitis C c. Patient who has a wound infection after cholecystectomy d. Patient who requires pain management for chronic pancreatitis

ANS: D The patient with chronic pancreatitis does not present an infection risk to the immunosuppressed patient who had a liver transplant. The other patients either are at risk for infection or currently have an infection, which will place the immunosuppressed patient at risk for infection.

Which patient should the nurse assess first after receiving change-of-shift report? a. 60-year-old patient whose new ileostomy has drained 800 mL over the previous 8 hours b. 50-year-old patient with familial adenomatous polyposis who has occult blood in the stool c. 40-year-old patient with ulcerative colitis who has had six liquid stools in the previous 4 hours d. 30-year-old patient who has abdominal distention and an apical heart rate of 136 beats/minute

ANS: D The patient's abdominal distention and tachycardia suggest hypovolemic shock caused by problems such as peritonitis or intestinal obstruction, which will require rapid intervention. The other patients should also be assessed as quickly as possible, but the data do not indicate any life-threatening complications associated with their diagnoses

A 27-year-old female patient is admitted to the hospital for evaluation of right lower quadrant abdominal pain with nausea and vomiting. Which action should the nurse take? a. Encourage the patient to sip clear liquids. b. Assess the abdomen for rebound tenderness. c. Assist the patient to cough and deep breathe. d. Apply an ice pack to the right lower quadrant.

ANS: D The patient's clinical manifestations are consistent with appendicitis, and application of an ice pack will decrease inflammation at the area. Checking for rebound tenderness frequently is unnecessary and uncomfortable for the patient. The patient should be NPO in case immediate surgery is needed. The patient will need to know how to cough and deep breathe postoperatively, but coughing will increase pain at this time

Which statement by a patient who had a cystoscopy the previous day should be reported immediately to the health care provider? a. "My urine looks pink." b. "My IV site is bruised." c. "My sleep was restless." d. "My temperature is 101."

ANS: D The patient's elevated temperature may indicate a bladder infection, a possible complication of cystoscopy. The health care provider should be notified so that antibiotic therapy can be started. Pink-tinged urine is expected after a cystoscopy. The insomnia and bruising should be discussed further with the patient but do not indicate a need to notify the health care provider.

The nurse in the women's health clinic has four patients who are waiting to be seen. Which patient should the nurse see first? a. 22-year-old with persistent red-brown vaginal drainage 3 days after having balloon thermotherapy b. 42-year-old with secondary amenorrhea who says that her last menstrual cycle was 3 months ago c. 35-year-old with heavy spotting after having a progestin-containing IUD (Mirena) inserted a month ago d. 19-year-old with menorrhagia who has been using superabsorbent tampons and has fever with weakness

ANS: D The patient's history and clinical manifestations suggest possible toxic shock syndrome, which will require rapid intervention. The symptoms for the other patients are consistent with their diagnoses and do not indicate life-threatening complications

After several days of antibiotic therapy, an older hospitalized patient develops watery diarrhea. Which action should the nurse take first? a. Notify the health care provider. b. Obtain a stool specimen for analysis. c. Teach the patient about handwashing. d. Place the patient on contact precautions.

ANS: D The patient's history and new onset diarrhea suggest a C. difficile infection, which requires implementation of contact precautions to prevent spread of the infection to other patients. The other actions are also appropriate but can be accomplished after contact precautions are implemented

A patient who has a wound infection after major surgery has only been taking in about 50% to 75% of the ordered meals and states, "Nothing on the menu really appeals to me." Which action by the nurse will be most effective in improving the patient's oral intake? a. Make a referral to the dietician. b. Order at least six small meals daily. c. Teach the patient about high-calorie, high-protein foods. d. Have family members bring in favorite foods from home.

ANS: D The patient's statement that the hospital foods are unappealing indicates that favorite home-cooked foods might improve intake. The other interventions also may help improve the patient's intake, but the most effective action will be to offer the patient more appealing foods.

A 45-year-old patient is admitted to the emergency department with severe abdominal pain and rebound tenderness. Vital signs include temperature 102° F (38.3° C), pulse 120, respirations 32, and blood pressure (BP) 82/54. Which prescribed intervention should the nurse implement first? a. Administer IV ketorolac (Toradol) 15 mg. b. Draw blood for a complete blood count (CBC). c. Obtain a computed tomography (CT) scan of the abdomen. d. Infuse 1 liter of lactated Ringer's solution over 30 minutes.

ANS: D The priority for this patient is to treat the patient's hypovolemic shock with fluid infusion. The other actions should be implemented after starting the fluid infusion

The nurse explaining esomeprazole (Nexium) to a patient with recurring heartburn describes that the medication a. reduces gastroesophageal reflux by increasing the rate of gastric emptying. b. neutralizes stomach acid and provides relief of symptoms in a few minutes. c. coats and protects the lining of the stomach and esophagus from gastric acid. d. treats gastroesophageal reflux disease by decreasing stomach acid production.

ANS: D The proton pump inhibitors decrease the rate of gastric acid secretion. Promotility drugs such as metoclopramide (Reglan) increase the rate of gastric emptying. Cryoprotective medications such as sucralfate (Carafate) protect the stomach. Antacids neutralize stomach acid and work rapidly

Which information about continuous bladder irrigation will the nurse teach to a patient who is being admitted for a transurethral resection of the prostate (TURP)? a. Bladder irrigation decreases the risk of postoperative bleeding. b. Hydration and urine output are maintained by bladder irrigation. c. Antibiotics are infused continuously through the bladder irrigation. d. Bladder irrigation prevents obstruction of the catheter after surgery.

ANS: D The purpose of bladder irrigation is to remove clots from the bladder and to prevent obstruction of the catheter by clots. The irrigation does not decrease bleeding or improve hydration. Antibiotics are given by the IV route, not through the bladder irrigation.

A 47-year-old female patient is transferred from the recovery room to a surgical unit after a transverse colostomy. The nurse observes the stoma to be deep pink with edema and a small amount of sanguineous drainage. The nurse should a. place ice packs around the stoma. b. notify the surgeon about the stoma. c. monitor the stoma every 30 minutes. d. document stoma assessment findings.

ANS: D The stoma appearance indicates good circulation to the stoma. There is no indication that surgical intervention is needed or that frequent stoma monitoring is required. Swelling of the stoma is normal for 2 to 3 weeks after surgery, and an ice pack is not needed

After change-of-shift report, which patient should the nurse assess first? a. 42-year-old who has acute gastritis and ongoing epigastric pain b. 70-year-old with a hiatal hernia who experiences frequent heartburn c. 53-year-old who has dumping syndrome after a recent partial gastrectomy d. 60-year-old with nausea and vomiting who has dry oral mucosa and lethargy

ANS: D This older patient is at high risk for problems such as aspiration, dehydration, and fluid and electrolyte disturbances. The other patients will also need to be assessed, but the information about them indicates symptoms that are typical for their diagnoses and are not life threatening. DIF: Cognitive Level: Apply (application) REF: 928 OBJ: Special Questions: Prioritization

A patient who is nauseated and vomiting up blood-streaked fluid is admitted to the hospital with acute gastritis. To determine possible risk factors for gastritis, the nurse will ask the patient about a. the amount of fat in the diet. b. history of recent weight gain or loss. c. any family history of gastric problems. d. use of nonsteroidal anti-inflammatory drugs (NSAIDs).

ANS: D Use of an NSAID is associated with damage to the gastric mucosa, which can result in acute gastritis. Family history, recent weight gain or loss, and fatty foods are not risk factors for acute gastritis.

A 50-year-old man vomiting blood-streaked fluid is admitted to the hospital with acute gastritis. To determine possible risk factors for gastritis, the nurse will ask the patient about a. the amount of saturated fat in the diet. b. any family history of gastric or colon cancer. c. a history of a large recent weight gain or loss. d. use of nonsteroidal antiinflammatory drugs (NSAIDs).

ANS: D Use of an NSAID is associated with damage to the gastric mucosa, which can result in acute gastritis. Family history, recent weight gain or loss, and fatty foods are not risk factors for acute gastritis. DIF: Cognitive Level: Understand (comprehension) REF: 941 TOP: Nursing Process: Assessment MSC: NCLEX: Physiological Integrity

The nurse determines that instruction regarding prevention of future urinary tract infections (UTIs) has been effective for a 22-year-old female patient with cystitis when the patient states which of the following? a. "I can use vaginal antiseptic sprays to reduce bacteria." b. "I will drink a quart of water or other fluids every day." c. "I will wash with soap and water before sexual intercourse." d. "I will empty my bladder every 3 to 4 hours during the day."

ANS: D Voiding every 3 to 4 hours is recommended to prevent UTIs. Use of vaginal sprays is discouraged. The bladder should be emptied before and after intercourse, but cleaning with soap and water is not necessary. A quart of fluids is insufficient to provide adequate urine output to decrease risk for UTI.

A 28-year-old male patient with type 1 diabetes reports how he manages his exercise and glucose control. Which behavior indicates that the nurse should implement additional teaching? a. The patient always carries hard candies when engaging in exercise. b. The patient goes for a vigorous walk when his glucose is 200 mg/dL. c. The patient has a peanut butter sandwich before going for a bicycle ride. d. The patient increases daily exercise when ketones are present in the urine.

ANS: D When the patient is ketotic, exercise may result in an increase in blood glucose level. Type 1 diabetic patients should be taught to avoid exercise when ketosis is present. The other statements are correct

A frail 72-yr-old woman with stage 3 chronic kidney disease is cared for at home by her family. The patient has a history of taking many over-the-counter medications. Which over-the-counter medications should the nurse teach the patient to avoid? Aspirin Acetaminophen Diphenhydramine Aluminum hydroxide

Aluminum hydroxide

A 86-year-old patient asks the nurse what lifestyle changes will reduce the chance of a urinary tract infection. Which response is accurate? a. Urinary tract infections are unavoidable in the elderly because of a weakened immune system. b. Decreasing fluid intake will decrease the amount of urine with bacteria produced. c. Making sure to cleanse the perineal area from back to front after voiding will reduce the chance of infection. d. Increasing consumption of acidic foods such as cranberry juice will reduce the chance of infection.

ANS: D Cranberry juice and other acidic foods decrease adherence of bacteria to the bladder wall. Urinary tract infections are avoidable in the elderly population with proper knowledge and hygiene. Perineal skin should be cleansed from front to back to avoid spreading fecal matter to the urethra. Increasing fluids will help to flush bacteria, thus preventing them from residing in the bladder for prolonged periods of time.

The nurse would anticipate an order for which diagnostic test for a patient who has severe flank pain and calcium phosphate crystals revealed on urinalysis? a. Renal ultrasound b. Bladder scan c. KUB x-ray d. Intravenous pyelogram

ANS: D Flank pain and calcium phosphate crystals are associated with renal calculi. An intravenous pyelogram allows the provider to observe pathological problems such as obstruction of the ureter. A renal ultrasound is performed to identify gross structures. A bladder scan measures the amount of urine in the bladder. A KUB x-ray shows size, shape, symmetry, and location of the kidneys.

A nurse is caring for a patient who just underwent intravenous pyelography that revealed a renal calculus obstructing the left ureter. What is the nurse's first priority in caring for this patient? a. Turn the patient on the right side to alleviate pressure on the left kidney. b. Encourage the patient to increase fluid intake to flush the obstruction. c. Administer narcotic medications to alleviate pain. d. Monitor the patient for fever, rash, and difficulty breathing.

ANS: D Intravenous pyelography is performed by administering iodine-based dye to view functionality of the urinary system. Many individuals are allergic to shellfish; therefore, the first nursing priority is to assess the patient for an allergic reaction that could be life threatening. The nurse should then encourage the patient to drink fluids to flush dye resulting from the procedure. Narcotics can be administered but are not the first priority. Turning the patient on the side will not affect patient safety.

A nurse observes that the client's left flank region is larger than the right flank region. What is the nurse's best action? A. Ask the client if he or she participates in contact sports and has been recently injured. B. Document the finding as the only action on the appropriate flowsheet. C. Apply a heating pad to the left flank after inspecting the site for signs of infection. D. Anticipate further diagnostic testing after sharing informing the physician of this finding

ANS: D Asymmetry of the flank or a unilateral protrusion may indicate an enlargement of a kidney. The enlargement may be benign or may be associated with a hydronephrosis or mass on the kidney.

Which change in renal or urinary functioning as a result of the normal aging process increases the older client's risk for infection? A. Decreased glomerular filtration B. Decreased filtrate reabsorption C. Weakened sphincter muscles D. Urinary retention

ANS: D Incomplete bladder emptying for whatever reason increases the client's risk for urinary tract infections as a result of urine stasis providing an excellent culture medium that promotes the growth of microorganisms.

The client reports the regular use of all the following medications. Which one alerts the nurse to the possibility of renal impairment when used consistently? A. Antacids B. Penicillin C. Antihistamine nasal sprays D. Nonsteroidal anti-inflammatory drug

ANS: D NSAIDs inhibit prostaglandin production and decrease blood flow to the nephrons. They can cause an interstitial nephritis and renal impairment.

Six weeks after a vasectomy, the patient has a sperm-free semen analysis result. What should the couple conclude? A. The patient is sterile. B. One additional semen free test is required. C. Alternative contraception is still required. D. Intercourse should be avoided for 2 more weeks.

Answer: A After vasectomy, the patient should not notice any difference in the look or feel of the ejaculate because its major component is seminal and prostatic fluid. The patient should use an alternative form of contraception until semen examination reveals no sperm. This usually requires at least 10 ejaculations or 6 weeks to evacuate sperm distal to the surgical site.

Where is the urethra opening located in cases of hypospadias? A. On the ventral surface of the penis B. On the dorsal surface of the penis C. At the upper portion of the testicular vault D. Within a bladder extrophy

Answer: A Hypospadias is a urologic abnormality in which the urethral opening is located on the ventral surface of the penis anywhere from the corona to the perineum. Hormonal influences in utero, environmental factors, and genetic factors are possible causes.

Which instructions should you include in the discharge instructions for a patient with epididymitis? A. Refrain from sexual intercourse. B. Continue to participate in liberal exercise. C. Take frequent sitz baths. D. Eat a low-salt diet.

Answer: A Patients with epididymitis should be encouraged to refrain from sexual intercourse during the acute phase. Conservative treatment consists of bed rest with elevation of the scrotum, use of ice packs, and analgesics. Ambulation places the scrotum in a dependent position and increases pain. Salt does not affect epididymitis recovery.

During the bath of a male patient, the nursing assistive personnel reports to you that the patient's foreskin cannot be retracted over the head of the penis. You recognize this condition as A. phimosis. B. prostatitis. C. priapism. D. epispadias.

Answer: A Phimosis is tightness or constriction of the foreskin around the head of the penis, which makes retraction difficult. Paraphimosis is tightness of the foreskin resulting in the inability to pull it forward from a retracted position and preventing normal return over the glans. Prostatitis is inflammation of the prostate, and epispadias occurs when the urethral opening is on the dorsal surface of the penis.

What is an erection lasting longer than 6 hours called? A. Priapism B. Peyronie's disease C. Hydrocele D. Hypospadias

Answer: A Priapism is a painful erection lasting longer than 6 hours. It is caused by an obstruction of the venous outflow in the penis. The condition may constitute a medical emergency. Causes of priapism include thrombosis of the corpus cavernosal veins, leukemia, sickle cell anemia, diabetes mellitus, degenerative lesions of the spine, neoplasms of the brain or spinal cord, vasoactive medications injected into the corpus cavernosa, and medications (e.g., sildenafil, cocaine, trazodone).

After you perform teaching regarding medication therapy for erectile dysfunction (ED), you know the teaching was successful when the patient states, A. "I will take the drug once each day about an hour before sexual activity." B. "I will take the drug two times each day but not more than three." C. "I can take the drug in the morning, and I will be able to have an erection anytime throughout the day." D. "There are no precautions to take while I am on this medication."

Answer: A Medication is taken orally about 1 hour before sexual activity, but not more than once each day. These drugs have been found to be generally safe and effective for the treatment of most types of ED.

Which ethnic group has the highest incidence of prostate cancer? A. African Americans B. Asians C. Whites D. Hispanics

Answer: A The incidence of prostate cancer worldwide is higher among African Americans than in any other ethnic group. The reasons for the higher rate are unknown.

You notice that the patient's urinary drainage 4 hours after TURP is redder than 1 hour ago. What is your priority intervention? A. To increase the rate of bladder irrigation B. To manually irrigate the urinary catheter C. To notify the physician D. To obtain vital signs

Answer: A With CBI, irrigating solution is continuously infused and drained from the bladder. The rate of infusion is based on the color of drainage. Ideally, urine drainage should be light pink without clots. Continuously monitor the inflow and outflow of the irrigant.

A patient asks you, "How can I decrease my risk of prostate cancer?" You teach the patient to avoid which foods (select all that apply)? A. Red meat B. High-fat dairy products C. Fruits D. Vegetables E. Chicken

Answer: A, B Dietary factors may be associated with prostate cancer. A diet high in red meat and high-fat dairy products, along with a low intake of vegetables and fruits, may increase the risk of prostate cancer.

Which factors place a patient at high risk for prostate cancer (select all that apply)? A. Age older than 65 years B. Asian or Native American ethnicity C. Personal history of BPH D. Brother diagnosed and treated for prostate cancer E. History of undescended testicle and testicular cancer

Answer: A,D Age, ethnicity, and family history are known risk factors for prostate cancer. The incidence of prostate cancer rises markedly after age 50, and more than 66% of men diagnosed are older than 65 years. The incidence of prostate cancer worldwide is higher among African Americans than in any other ethnic group. A family history of prostate cancer, especially first-degree relatives (fathers, brothers), is associated with an increased risk.

Which childhood disease is often associated with orchitis? A. Measles B. Mumps C. Strep throat D. Chickenpox

Answer: B Orchitis refers to an acute inflammation of the testis. In orchitis, the testis is painful, tender, and swollen. It usually occurs after an episode of bacterial or viral infection, such as mumps. The other disorders are not associated with mumps.

Which diagnostic evaluation is used for a patient with a suspected testicular cancer? A. Prostate-specific antigen (PSA) B. α-Fetoprotein (AFP) C. Complete blood cell count D. Urine and semen analyses

Answer: B Palpation of the scrotal contents is the first step in diagnosing testicular cancer. A cancerous mass is firm and does not transilluminate. Ultrasound of the testes is indicated when testicular cancer (e.g., palpable mass) is suspected or when persistent or painful testicular swelling is present. If testicular cancer is suspected, blood is obtained to determine the serum levels of AFP, lactate dehydrogenase (LDH), and human chorionic gonadotropin (hCG).

For the patient with inoperable prostate cancer, you expect the physician to order which type of hormone? A. Gonadotropin-releasing hormone B. Androgen deprivation C. Luteinizing hormone D. Estrogen

Answer: B Prostate cancer growth largely depends on androgens, and androgen deprivation is a primary therapeutic approach for some men with prostatic cancer. Hormone therapy, also known as androgen-deprivation therapy (ADT), focuses on reducing the levels of circulating androgens to diminish tumor growth.

Which fact in the patient's history could be related to the presence of testicular cancer? A. Epispadias B. Cryptorchidism C. Hernia repair D. Uncircumcised penis

Answer: B The incidence of testicular cancer is four times higher among white men (especially those of Scandinavian descent) than African American males. It occurs more commonly in the right testicle than the left. Testicular tumors are also more common in men who have had undescended testes (cryptorchidism) or have a family history of testicular cancer or anomalies. Epispadias and lack of circumcision are not related to testicular cancer. Hernia repair is related to future hernias, but not testicular cancer.

What is the most common cause of male infertility? A. A hydrocele B. A varicocele C. Alcohol use D. Epididymitis.

Answer: B The most common cause of male infertility is a varicocele, not a hydrocele. Other factors that influence the testes include infection such as epididymitis and alcohol use, but they are not the most common causes.

You should encourage a patient with bacterial prostatitis to A. remain on bed rest until antibiotic therapy has been in place for 72 hours. B. significantly increase fluid intake of noncaffeinated beverages. C. empty the bladder every hour with a straight catheter. D. weigh daily before the first voiding of the morning.

Answer: B Because the prostate can serve as a source of bacteria, fluid intake should be kept at a high level for all patients experiencing prostatitis. Nursing interventions are aimed at encouraging the patient to drink plenty of fluids. This is especially important for those with acute bacterial prostatitis because of the increased fluid needs associated with fever and infection. Bed rest is not indicated, and activity should be encouraged. The bladder should be emptied, although not with a catheter, and it is not necessary to do so every hour. Weight gain should be reported, but the patient does not need to weigh before voiding.

You are teaching the patient with BPH about interventions that can assist in alleviating symptoms. Which behavior in the patient indicates successful teaching? A. The patient increases use of decongestants. B. The patient decreases intake of caffeinated beverages and artificial sweeteners. C. The patient increases activities such as walking. D. The patient voids every 30 minutes.

Answer: B In some patients who have symptoms that appear and then disappear, a conservative treatment approach has value. Dietary changes (decreasing intake of caffeine and artificial sweeteners, limiting spicy or acidic foods), avoiding medications such as decongestants and anticholinergics, and restricting evening fluid intake may result in improvement of symptoms. A timed voiding schedule may reduce or eliminate symptoms, negating the need for further intervention, but 30 minutes is too frequent.

Which best indicates that treatment for cancer of the prostate is effective? A. Increase in urinary stream B. Decrease of PSA to 2 ng/mL C. Decreased blood in the urine D. White blood cell (WBC) count of 10,000/μL

Answer: B The PSA value is used to detect prostate cancer and to monitor the success of treatment. When treatment has been successful in removing prostate cancer, PSA levels should decrease and reach normal levels (less than 4 ng/mL). The regular measurement of PSA levels after treatment is important to evaluate the effectiveness of treatment and possible recurrence of prostate cancer.

A complication of prostatitis is A. prostatic cancer. B. benign prostatic hyperplasia. C. acute urinary retention. D. hydrocele.

Answer: C Acute urinary retention can develop in acute prostatitis, and it requires bladder drainage with suprapubic catheterization. Passage of a catheter through an inflamed urethra is contraindicated in acute prostatitis.

When providing patient teaching about medication therapy for BPH with 5α-reductase inhibitors such as finasteride (Proscar), what information should you include? A. Ninety percent of patients show improvement with the drug. B. The drug can be taken periodically as symptoms occur. C. Women who are pregnant should not handle the drug. D. Effects are seen in 1 week.

Answer: C Although more than 50% of men who are treated with the drug show symptom improvement, it takes about 6 months to be effective. The drug must be taken on a continuous basis to maintain therapeutic results. Women who may be or are pregnant should not handle tablets.

To decrease the patient's discomfort about care involving his reproductive organs, you should A. relate his sexual concerns to his sexual partner. B. arrange to have male nurses care for the patient. C. maintain a nonjudgmental attitude toward his sexual practices. D. use technical terminology when discussing reproductive function.

Answer: C Conducting routine health assessments on men places you in a unique position. It provides an opportunity to ask the patient questions pertaining to general health and to sexual health and function. Given the opportunity, men are less hesitant to answer these questions when they know that someone cares and can provide them with answers. You must remain nonjudgmental about sexual practices.

The patient with epididymitis asks you when he can resume his exercise routine. You should respond with which piece of information? A. When there is no evidence of urethral discharge B. After 1 week of antibiotic therapy C. When scrotal pain has subsided D. As soon as he feels able

Answer: C Most tenderness subsides within 1 week, although swelling may last for weeks or months. One week of antibiotic therapy may not relieve pain and swelling. "When the patient feels able" is a vague response, and the patient should remain on rest until scrotal swelling has subsided. There may be no urethral discharge in epididymitis.

What is the most common cause of erectile dysfunction? A. Aging B. Benign prostatic hyperplasia C. Vascular disease D. Psychological distress

Answer: C Common causes of ED include diabetes, vascular disease, side effects from medications, results of surgery (e.g., prostatectomy), trauma, chronic illness, decreased gonadal hormone secretion, stress, difficulty in a relationship, or depression. The most common cause of ED is vascular disease.

When working in the emergency department you recognize the need for immediate diagnosis and treatment of the patient with A. orchitis. B. phimosis. C. epididymitis. D. testicular torsion.

Answer: D Torsion constitutes a surgical emergency. If the blood supply to the affected testicle is not restored within 4 to 6 hours, testicular ischemia will occur, leading to necrosis and possible removal.

What should you explain to the patient who has had a vasectomy? A. The procedure blocks the production of sperm B. Erectile dysfunction is temporary and will return with sexual activity. C. The ejaculate will be about half the volume it was before the procedure. D. An alternative form of contraception will be necessary for 6 to 8 weeks.

Answer: D After a vasectomy, the patient should use an alternative form of contraception until semen examination reveals no sperm. This usually requires at least 10 ejaculations or 6 weeks to evacuate sperm distal to the surgical site.

You recognize the need to begin testosterone therapy when testosterone levels drop below A. 750 ng/dL. B. 500 ng/dL. C. 400 ng/dL. D. 250 ng/dL.

Answer: D Normal testosterone levels can range from 280 to 1100 ng/dL. Replacement therapy may be considered when levels are below 250 ng/dL.

A 45-year-old man asks you if it is advisable to have his prostate-specific antigen (PSA) level tested, because his father and brother have prostate cancer. What is your response? A. "You should wait until you are age 50." B. "You should have a transurethral resection of the prostate as a preventive measure." C. "You should have a voiding cystourethrogram yearly." D. "You should have annual PSA levels assessed and a digital examination of the prostate."

Answer: D The American Cancer Society recommends an annual digital rectal examination (DRE) and a blood test for PSA beginning at age 50 for men who are at average risk for prostate cancer. During DRE, an abnormal prostate may feel hard, nodular, and asymmetric.

When providing teaching regarding potential complications after perineal resection of the prostate, what should you include? A. Deep vein thrombosis B. Pulmonary embolism C. Colonic constipation D. Urinary incontinence

Answer: D The two major complications after a radical prostatectomy are erectile dysfunction and urinary incontinence.

Which information will the nurse include when teaching a patient who has been newly diagnosed with Graves' disease? A. Exercise is contraindicated to avoid increasing metabolic rate. B. Restriction of iodine intake is needed to reduce thyroid activity. C. Surgery will eventually be required to remove the thyroid gland. D. Antithyroid medications may take several weeks to have an effect.

Antithyroid medications may take several weeks to have an effect. Feedback Medications used to block the synthesis of thyroid hormones may take several weeks before an effect is seen. Large doses of iodine are used to inhibit the synthesis of thyroid hormones. Exercise using large muscle groups is encouraged to decrease the irritability and hyperactivity associated with high levels of thyroid hormones. Radioactive iodine is the most common treatment for Graves' disease, although surgery may be used

A 72-year-old patient is diagnosed with hypothyroidism and levothyroxine (Synthroid) is prescribed. Which assessment is most important for the nurse to make during initiation of thyroid replacement? A. Apical pulse rate B. Nutritional intake C. Intake and output D. Orientation and alertness

Apical pulse rate Feedback In older patients, initiation of levothyroxine therapy can increase myocardial oxygen demand and cause angina or dysrhythmias. The medication also is expected to improve mental status and fluid balance and will increase metabolic rate and nutritional needs, but these changes will not result in potentially life-threatening complications.

Which action should the nurse take first when caring for a patient who has just arrived on the unit after a thyroidectomy? A. Check the dressing for bleeding. B. Assess respiratory rate and effort. C. Take the blood pressure and pulse. D. Support the patient's head with pillows.

Assess respiratory rate and effort. ANS: B Airway obstruction is a possible complication after thyroidectomy because of swelling or bleeding at the site or tetany, and the priority nursing action is to assess the airway. The other actions also are part of the standard nursing care postthyroidectomy but are not as high in priority.

A 52-yr-old man with stage 2 chronic kidney disease is scheduled for an outpatient diagnostic procedure using contrast media. Which priority action should the nurse perform? Assess skin turgor to determine hydration status. Insert a urinary catheter for the expected diuresis. Evaluate the patient's lower extremities for edema. Check the patient's urine for the presence of ketones.

Assess skin turgor to determine hydration status.

The patient is incontinent, and a condom catheter is placed. The nurse should take which action? A) Secure the condom with adhesive tape B) Change the condom every 48 hours C) Assess the patient for skin irritation D) Use sterile technique for placement

Assess the patient for skin irritation

During preoperative teaching for a patient scheduled for transsphenoidal hypophysectomy for treatment of a pituitary adenoma, the nurse instructs the patient about the need to A. Cough and deep breathe every 2 hours postoperatively. B. Remain on bed rest for the first 48 hours after the surgery. C. Be positioned flat with sandbags at the head postoperatively. D. Avoid brushing the teeth for at least 10 days after the surgery.

Avoid brushing the teeth for at least 10 days after the surgery. Feedback To avoid disruption of the suture line, the patient should avoid brushing the teeth for 10 days after surgery. It is not necessary to remain on bed rest after this surgery. Coughing is discouraged because it may cause leakage of cerebrospinal fluid (CSF) from the suture line. The head of the bed should be elevated 30 degrees to reduce pressure on the sella turcica and

The patient with Crohn's disease has an ileostomy, with the terminal ileum removed. Absorption of what nutrient is a key concern? A. Carbohydrate B. Cobalamin C. Gluten D. Lactose

B Rationale Patients who had the terminal ileum removed have reduced absorption of cobalamin (vitamin B12). Instrinsic factor is secreted in the stomach but absorbed in the small intestine. Fat absorption is decreased because bile salts are absorbed in the terminal ileum, but not carbohydrates. Lack of absorption of gluten occurs in celiac disease. Lactose intolerance occurs because the enzyme lactase is deficient. Reference: 1045

The nurse cares for a postoperative patient who has just vomited yellow green liquid and reports nausea. Which action would be an appropriate nursing intervention? a) Offer the patient a herbal supplement such as ginseng. b) Apply a cool washcloth to the forehead and provide mouth care. c) Take the patient for a walk in the hallway to promote peristalsis. d) Discontinue any medications that may cause nausea or vomiting.

B Cleansing the face and hands with a cool washcloth and providing mouth care are appropriate comfort interventions for nausea and vomiting. Ginseng is not used to treat postoperative nausea and vomiting. Unnecessary activity should be avoided. The patient should rest in a quiet environment. Medications may be temporarily withheld until the acute phase is over, but the *medications should not be discontinued without consultation with the health care provider.*

You are planning care for a 68-year-old patient with an abdominal mass and suspected bowel obstruction. Which factor in the patient's history increases the patient's risk for colorectal cancer? A. Osteoarthritis B. History of rectal polyps C. History of lactose intolerance D. Use of herbs as dietary supplements

B Rationale A history of rectal polyps places this patient at risk for colorectal cancer. This tissue can degenerate over time and become malignant. The other factors identified do not pose an additional risk to the patient. Reference: 1035

A woman with irritable bowel syndrome is prescribed alosetron (Lotronex). The patient should be taught to discontinue the drug if which symptom occurs? A. Diarrhea B. Constipation C. Urinary burning D. Renal calculi

B Rationale A potential complication of alosetron is severe constipation with ischemic colitis. The drug should be discontinued if constipation occurs. Diarrhea is the reason the drug is prescribed. The drug does not affect the urinary system. Reference: 1018

The patient is diagnosed with infectious diarrhea caused by E. coli. Which treatment do you anticipate? A. Antidiarrheal agent B. Pedialyte C. Antibiotic D. Stool transplantation

B Rationale Acute diarrhea from infectious causes is usually self-limiting. The major concerns are preventing transmission, fluid and electrolyte replacement, and resolution of the diarrhea. Oral solutions such as Gatorade or Pedialyte may be sufficient. Antidiarrheal agents are contraindicated in the treatment of infectious diarrhea because they potentially prolong exposure to the infectious organisms. Antibiotics are rarely used to treat acute diarrhea. Exceptions are certain infections (C. difficile) or patients who are immunosuppressed. Stool transplantation is used as a last resort for recurrent C. difficile infections. Reference: 1008

Which is correct information about the treatment of Crohn's disease? A. Surgery is the preferred treatment. B. Aminosalicylates are frequently used first. C. Corticosteroids are given for long-term therapy. D. High-fiber foods are encouraged to add bulk to diarrheal stool.

B Rationale Aminosalicylates (5-ASAs) are used first because they are less toxic, although there is a movement to using biologic and targeted therapy as first-line therapy. Drugs with 5-ASA suppress the proinflammatory cytokines and inflammatory mediators. Because there is a high rate of recurrence after surgical treatment of Crohn's disease, medication is the preferred treatment, although up to 75% of patients eventually must have surgery. Corticosteroids are given for the shortest time possible due to the complications of long-term therapy. Patients are put on a low-residue diet. Reference: 1025

The patient is brought in with a piece of wood impaled in his lower abdomen after being rescued from a collapsing building. What action should you take? A. Assess for positive rebound pressure. B. Stabilize the wood until examined by a physician. C. Remove the wood and apply direct pressure. D. Note the presence of bowel sounds.

B Rationale An impaled object should never be removed until skilled care is available. Removal may cause further injury and bleeding. Reference: 1020

The patient presents with abdominal pain. Which assessment finding should you follow-up? A. 130/80 mm Hg, 100 beats/minute, 16 breaths/minute B. Fetal posture C. Vomit × 1 green liquid D. Negative rebound tenderness

B Rationale Fetal posture is common with peritoneal irritation. It lessens the tension on the inflamed perineal area. The elevated pulse and blood pressure may be caused by pain and anxiety. Signs of shock include elevated pulse and respirations with normal or decreased blood pressure. Green emesis is likely a sign of bile, and one episode is not of major concern. Negative rebound tenderness is a normal finding. Reference: 1015

The patient with fecal incontinence has not responded to the bowel-retraining program and a medication will be used. Which is most appropriate? A. Oil retention enema B. Bisacodyl (Dulcolax) C. Diphenoxylate and atropine (Lomotil) D. Loperamide (Imodium)

B Rationale If nonpharmacologic treatments are ineffective in reestablishing bowel regularity, Dulcolax or a small phosphate enema can be administered 15 to 30 minutes before the usual evacuation time to stimulate the anorectal reflex. After a regular pattern is reestablished, the mediation is discontinued. An oil retention enema is used for impaction or constipation. Lomotil and Imodium are used for diarrhea and are not part of an incontinent program. Reference: 1012

The patient with ulcerative colitis has a total proctocolectomy with permanent ileostomy. Which instruction is most important to teach the patient? A. Contact the primary provider if there is more than 1500 mL of drainage per day. B. Monitor skin integrity. C. Irrigate to gain control of stool elimination. D. Assess for signs of cobalamin (vitamin B12) deficiency.

B Rationale If the stoma retracts, stool can contact the skin, causing loss of the epidermal layer. An ileostomy can initially drain 1500 to 2000 mL per 24 hours. It is not possible to obtain continence with an ileostomy. Cobalamin deficiency is a concern if the terminal ileus is removed because that is where this vitamin is absorbed. It is not a problem with large intestine removal. Reference: 1028

What is the main treatment for a patient with acute diverticulitis? A. Colon resection and ostomy B. Nasogastric tube and intravenous (IV) fluids C. Long-term course of oral corticosteroids D. Mechanical soft diet

B Rationale In acute diverticulitis, the goal of treatment is to allow the colon to rest and inflammation to subside. Bowel rest can be accomplished with the use of a nasogastric tube and IV fluids. Surgery is reserved for patients with complications, such as an abscess or obstruction that does not respond to medical management. The colon is reanastomosed after it has healed. Corticosteroids are not routinely used with diverticulitis; antibiotics are. During an acute attack, bowel rest is required. A mechanical soft diet is used for patients who lack the ability to adequately chew. Reference: 1047

The immunosuppressant azathioprine (Imuran) is given to maintain remission after corticosteroid induction therapy for an exacerbation of ulcerative colitis. What monitoring is required? A. Carcinogenic embryonic antigen (CEA) B. Complete blood cell count (CBC) C. Prostate-specific antigen (PSA) D. Potassium

B Rationale Regular CBC monitoring is required because the drug can suppress the bone marrow and lead to inflammation of the pancreas or gallbladder. CEA is used to monitor for recurrence of colorectal cancer. PSA is used to monitor for prostate cancer. It is not necessary to specifically monitor for potassium with this drug. Reference: 1026

How does the drug sulfasalazine (Azulfidine) work in the treatment of IBD? A. Destroys bacteria B. Suppresses inflammatory mediators C. Slows gastric motility D. Promotes electrolyte exchange across intestinal membrane

B Rationale Sulfasalazine contains sulfapyridine and 5-aminosalicylic acid (5-ASA). Although the exact action is unknown, it works by suppressing inflammatory mediators. IBD is an autoimmune inflammatory disease; no specific infectious agent has been identified, although antimicrobials (Flagyl, Cipro) occasionally are used. The last two options are not related to this drug. Reference: 1025

Compared with a colostomy, which complication is a patient with an ileostomy at an increased risk for? A. Constipation B. Obstruction C. Flatus D. Polyps

B Rationale The ileostomy patient is susceptible to obstruction because the lumen is less than an inch in diameter and may narrow further at the point where the bowel passes through the fascia-muscle layer of the abdomen. Ileostomies have loose drainage because fluid is not absorbed in the large colon. Flatus can occur in patients with either an ileostomy or a colostomy. Polyps are most often found in the distal colon and rectum, which has been removed if the patient has an ileostomy. Reference: 1045

You would increase the comfort of the patient with appendicitis by A. having the patient lie prone. B. flexing the patient's right knee. C. sitting the patient upright in a chair. D. turning the patient onto his or her left side.

B Rationale The patient with appendicitis usually prefers to lie still, often with the right leg flexed to decrease pain. Reference: 1020

What is a key distinction between ulcerative colitis and Crohn's disease? A. Presence of intermittent constipation B. Pattern of inflammation C. Age of onset D. Prescribed diet

B Rationale The pattern of inflammation differs between Crohn's disease and ulcerative colitis. Crohn's disease involves all layers of the bowel wall anywhere in the gastrointestinal tract. Ulcerative colitis usually starts in the rectum and moves progressively toward the cecum, staying mainly located in the colon and rectum. Inflammation occurs in the mucosal layer. Intermittent diarrhea and constipation is a symptom of irritable bowel syndrome; diarrhea can occur in both diseases. Both commonly occur during the teenage years and early adulthood, with a second peak in the sixth decade. Both are treated similarly in terms of bowel rest, diet, and drugs. Reference: 1023

Which is the best understanding of colon irrigation? A. It is taught to patients with ascending colostomies. B. The tip should be inside a cone to prevent perforation. C. Use cold water to promote peristalsis. D. Administer 2000 mL of sterile saline.

B Rationale The tip is inside a cone to control the depth of insertion, prevent water from leaking out, and prevent perforation. Irrigation is used only in the distal colon or rectum because the stool is solid there. The water should be warm; cold water can cause cramping. Approximately 500 to 1000 mL of tap water is administered. Reference: 1045

What screening test should you recommend to a white person with an average risk of colorectal cancer? A. Flexible sigmoidoscopy at age 21 B. Initial colonoscopy starting at age 50 C. Stool DNA every 10 years D. Carcinoembryonic antigen (CEA) yearly

B Rationale Whites with an average risk should have a colonoscopy every 10 years starting at age 50 (African Americans should have the first one at age 45). The use of a flexible sigmoidoscopy is decreasing because it evaluates only about 50% of the colon. When used, it is begun at age 50. Stool DNA is less favorable but acceptable. Stool tests must be done frequently since DNA shedding occur at intervals and may be easily missed. CEA is used to monitor disease recurrence after surgery or chemotherapy. Reference: 1036

A 61-year-old patient with suspected bowel obstruction had a nasogastric tube inserted at 4:00 AM. The nurse shares in the morning report that the day shift staff should check the tube for patency at what times? A. 7:00 AM, 10:00 AM, and 1:00 PM B. 8:00 AM, 12:00 PM, and 4:00 PM C. 9:00 AM and 3:00 PM D. 9:00 AM, 12:00 PM, and 3:00 PM

B A nasogastric tube should be checked for patency routinely at 4-hour intervals. Thus if the tube were inserted at 4:00 AM, it would be due to be checked at 8:00 AM, 12:00 PM, and 4:00 PM.

The client is diagnosed with hypothyroidism. Which s/s should the nurse expect the client to exhibit? a. complaints of extreme fatigue and hair loss b. exophthalmos and complaints of nervousness c. complaints of profused sweating and flushed skin d. tetany and complaints of stiffness of the hands

a. complaints of extreme fatigue and hair loss

What information would have the highest priority to be included in preoperative teaching for a 68-year-old patient scheduled for a colectomy? A. How to care for the wound B. How to deep breathe and cough C. The location and care of drains after surgery D. Which medications will be used during surgery

B Because anesthesia, an abdominal incision, and pain can impair the patient's respiratory status in the postoperative period, it is of high priority to teach the patient to cough and deep breathe. Otherwise, the patient could develop atelectasis and pneumonia, which would delay early recovery from surgery and hospital discharge. Care for the wound and location and care of the drains will be briefly discussed preoperatively, but done again with higher priority after surgery. Knowing which drugs will be used during surgery may not be meaningful to the patient and should be reviewed with the patient by the anesthesiologist.

A colectomy is scheduled for a 38-year-old woman with ulcerative colitis. The nurse should plan to include what prescribed measure in the preoperative preparation of this patient? A. Instruction on irrigating a colostomy B. Administration of a cleansing enema C. A high-fiber diet the day before surgery D. Administration of IV antibiotics for bowel preparation

B Preoperative preparation for bowel surgery typically includes bowel cleansing with antibiotics, such as oral neomycin and cleansing enemas, including Fleet enemas. Instructions to irrigate the colostomy will be done postoperatively. Oral antibiotics are given preoperatively, and an IV antibiotic may be used in the OR. A clear liquid diet will be used the day before surgery with the bowel cleansing.

A patient is taking high doses of prednisone to control the symptoms of an acute exacerbation of systemic lupus erythematosus. When teaching the pt about use of prednisone, which information is most important for the nurse to include? a. Call the doctor if you experience any mood alterations with the prednisone. b. Do not stop taking the prednisone suddenly; it should be decreased gradually. c. Weigh yourself daily to monitor for weight gain caused by water or increased fat. d. Check your temperature daily because prednisone can hide signs of infection.

B R: Acute adrenal insufficiency may occur if exogenous glucocorticoids are suddenly stopped. Mood alterations and weight gain are possible adverse effects of glucocorticoid use, but these are not life-threatening effects. Glucocorticoids do mask the signs of infection, but temperature elevation tends to be suppressed, so other signs of infection should be monitored. (Cognitive Level: Application Text Reference: p. 1314 NProcess: Implementation NCLEX: Physiological Integrity)

The first nursing action indicated when a patient returns to the surgical nursing unit following a thyroidectomy is to a. check the dressing for bleeding. b. assess respiratory rate and effort. c. support the patient's head with pillows. d. take the blood pressure and pulse.

B R: Airway obstruction is a possible complication after thyroidectomy because of swelling or bleeding at the site or tetany, and priority nursing action is to assess airway. The other actions are also part of the standard nursing care post-thyroidectomy but are not as high in priority.

A patient is hospitalized with acute adrenal insufficiency. The nurse determines that the pt is responding favorably to treatment upon finding a. decreasing serum sodium. b. decreasing serum potassium. c. decreasing blood glucose. d. increasing urinary output.

B R: CMs of Addison's disease include hyperkalemia and a decrease in potassium level indicates improvement. Decreasing serum sodium and decreasing blood glucose indicate that treatment has not been effective. Changes in urinary output are not an effective way of monitoring treatment for Addison's disease.

Following a thyroidectomy, a patient develops carpal spasm while the nurse is taking a blood pressure on the left arm. Which action by the nurse is appropriate? a. Administer the ordered muscle relaxant. b. Have the patient rebreathe using a paper bag. c. Start oxygen at 2 to 3 L/min per cannula. d. Give the ordered oral calcium supplement.

B R: Carpal spasm after a thyroidectomy suggests that pt has hypocalcaemia caused by damage to the parathyroid glands. The symptoms of hypocalcemia will be temporarily reduced by having the patient breath into a paper bag, which will raise the PaCO2 and create a more acidic pH. The muscle relaxant will not impact on ionized calcium level. There is no indication that the patient is experiencing laryngeal stridor or needs oxygen. IV calcium supplements will be given to normalize calcium level quickly.

When providing postoperative care for a patient who has had bilateral adrenalectomy, which assessment information obtained by the nurse is most important to communicate to HCP? a. The blood glucose is 156 mg/dl. b. The patient's blood pressure is 102/50. c. The patient has 5/10 incisional pain. d. The lungs have bibasilar crackles.

B R: During immediate postoperative period, marked fluctuation in cortisol levels may occur and the nurse must be alert for signs of acute adrenal insufficiency such as hypotension. nurse should also address elevated glucose, incisional pain, and crackles with appropriate collaborative or nursing actions, but prevention and treatment of acute adrenal insufficiency is the priority after adrenalectomy.

When developing a plan of care for a pt with SIADH, which interventions will the nurse include? a. Encourage fluids to 2000 ml/day. b. Offer patient hard candies to suck on. c. Monitor for increased peripheral edema. d. Keep head of bed elevated to 30 degrees.

B R: Sucking on hard candies decreases thirst for patient on a fluid restriction. Pts with SIADH are on fluid restrictions of 800-1000 ml/day. Peripheral edema isnt seen w SIADH. HOB is elevated no more than 10 degrees to increase left atrial filling pressure and decrease ADH release. (Cognitive Level: Application Text Reference: p. 1296 NProcess: Planning NCLEX: Physiological Integrity)

A patient is admitted to the hospital in addisonian crisis 1 month after a diagnosis of Addison's disease. The nurse identifies the nursing diagnosis of ineffective therapeutic regimen management related to lack of knowledge of management of condition when the patient says, a. "I double my dose of hydrocortisone on the days that I go for a run." b. "I had the stomach flu earlier this week and couldn't take the hydrocortisone." c. "I frequently eat at restaurants, and so my food has a lot of added salt." d. "I do yoga exercises almost every day to help me reduce stress and relax."

B R: The need for hydrocortisone replacement is increased with stressors such as illness, and the patient needs to be taught to call the health care provider because medication and IV fluids and electrolytes may need to be given. The other patient statements indicate appropriate management of the Addison's disease.

The nurse identifies a nursing dx of risk for injury: corneal ulceration related to inability to close the eyelids secondary to exophthalmos for a patient with Graves' disease. An appropriate nursing intervention for this problem is to a. teach the patient to blink every few seconds to lubricate the cornea. b. elevate the head of the patient's bed to reduce periorbital fluid. c. apply eye patches to protect the cornea from irritation. d. place cold packs on the eyes to relieve pain and swelling.

B R: The patient should sit upright as much as possible to promote fluid drainage from the periorbital area. With exophthalmos, the pt is unable to close eyes completely. Lubrication of the eyes, rather than eye patches, will protect the eyes from developing corneal scarring. The swelling of the eye is not caused by excessive blood flow to the eye, so cold packs will not be helpful.

A few hours after returning to the surgical nursing unit, a patient who has undergone a subtotal thyroidectomy develops laryngeal stridor and a cramp in the right hand. The nurse anticipates that intervention will include a. administration of IV morphine. b. administration of IV calcium gluconate. c. endotracheal intubation with mechanical ventilation. d. immediate tracheostomy and manual ventilation.

B R: The pt's CMs are consistent with tetany caused by hypocalcemia resulting from damage to the parathyroid glands during surgery. Tracheostomy may be needed if the calcium does not resolve the stridor. There is no indication that morphine is needed. Endotracheal intubation may be done, but only if calcium is not effective in correcting stridor

While hospitalized for a fractured femur, a 68-year-old pt is diagnosed with hypothyroidism. Which of these medications on the original admission orders will the nurse need to consult with the HCP about before it is administered? a. Docusate (Colace) b. Diazepam (Valium) c. Ibuprofen (Motrin) d. Cefoxitin (Mefoxin)

B R: Worsening of mental status and myxedema coma can be precipitated by the use of sedatives, especially in older adults. The nurse should discuss the diazepam with the health care provider before administration. The other medications may safely be given to the pt.

A newly diagnosed type 1 diabetic patient likes to run 3 miles several mornings a week. Which teaching will the nurse implement about exercise for this patient? a. "You should not take the morning NPH insulin before you run." b. "Plan to eat breakfast about an hour before your run." c. "Afternoon running is less likely to cause hypoglycemia." d. "You may want to run a little farther if your glucose is very high."

B Rationale: Blood sugar increases after meals, so this will be the best time to exercise. NPH insulin will not peak until mid-afternoon and is safe to take before a morning run. Running can be done in either the morning or afternoon. If the glucose is very elevated, the patient should postpone the run. Cognitive Level: Application Text Reference: p. 1269 Nursing Process: Implementation NCLEX: Physiological Integrity

A patient is suspected of having a pituitary tumor causing panhypopituitarism. During assessment of the pt, the nurse would expect to find a. elevated blood glucose. b. changes in secondary sex characteristics. c. high blood pressure. d. tachycardia and cardiac palpitations.

B Rationale: Changes in secondary sex characteristics are associated with decreases in FSH and LH. Fasting hypoglycemia and hypotension occur in panhypopituitarism as a result of decreases in ACTH and cortisol. Bradycardia is likely due to the decrease in TSH and thyroid hormones associated with panhypopituitarism. (Cognitive Level: Application Text Reference: p. 1294 NProcess: Assessment NCLEX: Physiological Integrity)

A 1200-calorie diet and exercise are prescribed for a patient with newly diagnosed type 2 diabetes. The patient tells the nurse, "I hate to exercise! Can't I just follow the diet to keep my glucose under control?" The nurse teaches the patient that the major purpose of exercise for diabetics is to a. increase energy and sense of well-being, which will help with body image. b. facilitate weight loss, which will decrease peripheral insulin resistance. c. improve cardiovascular endurance, which is important for diabetics. d. set a successful pattern, which will help in making other needed changes.

B Rationale: Exercise is essential to decrease insulin resistance and improve blood glucose control. Increased energy, improved cardiovascular endurance, and setting a pattern of success are secondary benefits of exercise, but they are not the major reason. Cognitive Level: Application Text Reference: p. 1269 Nursing Process: Implementation NCLEX: Physiological Integrity

A patient with type 1 diabetes has an unusually high morning glucose measurement, and the health care provider wants the patient evaluated for possible Somogyi effect. The nurse will plan to a. administer an increased dose of NPH insulin in the evening. b. obtain the patient's blood glucose at 3:00 in the morning. c. withhold the nighttime snack and check the glucose at 6:00 AM. d. check the patient for symptoms of hypoglycemia at 2:00 to 4:00 AM.

B Rationale: In the Somogyi effect, the patient's blood glucose drops in the early morning hours (in response to excess insulin administration), which causes the release of hormones that result in a rebound hyperglycemia. It is important to check the blood glucose in the early morning hours to detect the initial hypoglycemia. An increased evening NPH dose or holding the nighttime snack will further increase the risk for early morning hypoglycemia. Information about symptoms of hypoglycemia will not be as accurate as checking the patient's blood glucose in determining whether the patient has the Somogyi effect. Cognitive Level: Application Text Reference: pp. 1263-1264 Nursing Process: Planning NCLEX: Physiological Integrity

A patient with newly diagnosed type 2 diabetes mellitus asks the nurse what "type 2" means in relation to diabetes. The nurse explains to the patient that type 2 diabetes differs from type 1 diabetes primarily in that with type 2 diabetes a. the patient is totally dependent on an outside source of insulin. b. there is decreased insulin secretion and cellular resistance to insulin that is produced. c. the immune system destroys the pancreatic insulin-producing cells. d. the insulin precursor that is secreted by the pancreas is not activated by the liver.

B Rationale: In type 2 diabetes, the pancreas produces insulin, but the insulin is insufficient for the body's needs or the cells do not respond to the insulin appropriately. The other information describes the physiology of type 1 diabetes. Cognitive Level: Application Text Reference: p. 1255 Nursing Process: Implementation NCLEX: Physiological Integrity

A patient with type 1 diabetes who uses glargine (Lantus) and lispro (Humalog) insulin develops a sore throat, cough, and fever. When the patient calls the clinic to report the symptoms and a blood glucose level of 210 mg/dl, the nurse advises the patient to a. use only the lispro insulin until the symptoms of infection are resolved. b. monitor blood glucose every 4 hours and notify the clinic if it continues to rise. c. decrease intake of carbohydrates until glycosylated hemoglobin is less than 7%. d. limit intake to non-calorie-containing liquids until the glucose is within the usual range.

B Rationale: Infection and other stressors increase blood glucose levels and the patient will need to test blood glucose frequently, treat elevations appropriately with insulin, and call the health care provider if glucose levels continue to be elevated. Discontinuing the glargine will contribute to hyperglycemia and may lead to DKA. Decreasing carbohydrate or caloric intake is not appropriate as the patient will need more calories when ill. Glycosylated hemoglobins are not used to test for short-term alterations in blood glucose. Cognitive Level: Application Text Reference: p. 1272 Nursing Process: Implementation NCLEX: Health Promotion and Maintenance

During a diabetes screening program, a patient tells the nurse, "My mother died of complications of type 2 diabetes. Can I inherit diabetes?" The nurse explains that a. as long as the patient maintains normal weight and exercises, type 2 diabetes can be prevented. b. the patient is at a higher than normal risk for type 2 diabetes and should have periodic blood glucose level testing. c. there is a greater risk for children developing type 2 diabetes when the father has type 2 diabetes. d. although there is a tendency for children of people with type 2 diabetes to develop diabetes, the risk is higher for those with type 1 diabetes.

B Rationale: Offspring of people with type 2 diabetes are at higher risk for developing type 2 diabetes. The risk can be decreased, but not prevented, by maintenance of normal weight and exercising. The risk for children of a person with type 1 diabetes to develop diabetes is higher when it is the father who has the disease. Offspring of people with type 2 diabetes are more likely to develop diabetes than offspring of those with type 1 diabetes. Cognitive Level: Application Text Reference: p. 1256 Nursing Process: Implementation NCLEX: Physiological Integrity

The health care provider orders oral glucose tolerance testing for a patient seen in the clinic. Which information from the patient's health history is most important for the nurse to communicate to the health care provider? a. The patient had a viral illness 2 months ago. b. The patient uses oral contraceptives. c. The patient runs several days a week. d. The patient has a family history of diabetes.

B Rationale: Oral contraceptive use may falsely elevate oral glucose tolerance test (OGTT) values. A viral 2 months previously illness may be associated with the onset of type 1 diabetes but will not falsely impact the OGTT. Exercise and a family history of diabetes both can affect blood glucose but will not lead to misleading information from the OGTT. Cognitive Level: Application Text Reference: p. 1267 Nursing Process: Assessment NCLEX: Physiological Integrity

A diabetic patient is started on intensive insulin therapy. The nurse will plan to teach the patient about mealtime coverage using _____ insulin. a. NPH b. lispro c. detemir d. glargine

B Rationale: Rapid or short acting insulin is used for mealtime coverage for patients receiving intensive insulin therapy. NPH, glargine, or detemir will be used as the basal insulin. Cognitive Level: Application Text Reference: p. 1260 Nursing Process: Planning NCLEX: Physiological Integrity

A patient receives a daily injection of 70/30 NPH/regular insulin premix at 7:00 AM. The nurse expects that a hypoglycemic reaction is most likely to occur between a. 8:00 and 10:00 AM. b. 4:00 and 6:00 PM. c. 7:00 and 9:00 PM. d. 10:00 PM and 12:00 AM.

B Rationale: The greatest insulin effect with this combination occurs mid afternoon. The patient is not at a high risk at the other listed times, although hypoglycemia may occur. Cognitive Level: Comprehension Text Reference: p. 1260 Nursing Process: Evaluation NCLEX: Physiological Integrity

Which of these laboratory values noted by the nurse when reviewing the chart of a diabetic patient indicates the need for further assessment of the patient? a. Fasting blood glucose of 130 mg/dl b. Noon blood glucose of 52 mg/dl c. Glycosylated hemoglobin of 6.9% d. Hemoglobin A1C of 5.8%

B Rationale: The nurse should assess the patient with a blood glucose level of 52 mg/dl for symptoms of hypoglycemia, and give the patient some carbohydrate-containing beverage such as orange juice. The other values are within an acceptable range for a diabetic patient. Cognitive Level: Application Text Reference: pp. 1281-1282 Nursing Process: Assessment NCLEX: Physiological Integrity

A patient with type 2 diabetes has sensory neuropathy of the feet and legs and peripheral vascular disease evidenced by decreased peripheral pulses and dependent rubor. The nurse teaches the patient that a. the feet should be soaked in warm water on a daily basis. b. flat-soled leather shoes are the best choice to protect the feet from injury. c. heating pads should always be set at a very low temperature. d. over-the-counter (OTC) callus remover may be used to remove callus and prevent pressure.

B Rationale: The patient is taught to avoid high heels and that leather shoes are preferred. The feet should be washed, but not soaked, in warm water daily. Heating pad use should be avoided. Commercial callus and corn removers should be avoided; the patient should see a specialist to treat these problems. Cognitive Level: Application Text Reference: p. 1287 Nursing Process: Implementation NCLEX: Health Promotion and Maintenance

A patient with type 2 diabetes is scheduled for an outpatient coronary arteriogram. Which information obtained by the nurse when admitting the patient indicates a need for a change in the patient's regimen? a. The patient's most recent hemoglobin A1C was 6%. b. The patient takes metformin (Glucophage) every morning. c. The patient uses captopril (Capoten) for hypertension. d. The patient's admission blood glucose is 128 mg/dl.

B Rationale: To avoid lactic acidosis, metformin should not be used for 48 hours after IV contrast media are administered. The other patient data indicate that the patient is managing the diabetes appropriately. Cognitive Level: Application Text Reference: p. 1266 Nursing Process: Assessment NCLEX: Physiological Integrity

The nurse has taught a patient admitted with diabetes, cellulitis, and osteomyelitis about the principles of foot care. The nurse evaluates that the patient understands the principles of foot care if the patient makes what statement? A) "I should only walk barefoot in nice dry weather." B) "I should look at the condition of my feet every day." C) "I am lucky my shoes fit so nice and tight because they give me firm support." D) "When I am allowed up out of bed, I should check the shower water with my toes."

B) "I should look at the condition of my feet every day." Patients with diabetes mellitus need to inspect their feet daily for broken areas that are at risk for infection and delayed wound healing. Properly fitted (not tight) shoes should be worn at all times. Water temperature should be tested with the hands first.

A 54-year-old patient admitted with type 2 diabetes asks the nurse what "type 2" means. What is the most appropriate response by the nurse? A) "With type 2 diabetes, the body of the pancreas becomes inflamed." B) "With type 2 diabetes, insulin secretion is decreased, and insulin resistance is increased." C) "With type 2 diabetes, the patient is totally dependent on an outside source of insulin." D) "With type 2 diabetes, the body produces autoantibodies that destroy β-cells in the pancreas."

B) "With type 2 diabetes, insulin secretion is decreased, and insulin resistance is increased." In type 2 diabetes mellitus, the secretion of insulin by the pancreas is reduced, and/or the cells of the body become resistant to insulin. The pancreas becomes inflamed with pancreatitis. The patient is totally dependent on exogenous insulin and may have had autoantibodies destroy the β-cells in the pancreas with type 1 diabetes mellitus.

The nurse is caring for a 40-year-old man who has begun taking levothyroxine (Synthroid) for recently diagnosed hypothyroidism. What information reported by the patient is most important for the nurse to further assess? A. Weight gain or weight loss B. Chest pain and palpitations C. Muscle weakness and fatigue D. Decreased appetite and constipation

B. Chest pain and palpitations Levothyroxine (Synthroid) is used to treat hypothyroidism. Any chest pain or heart palpitations or heart rate greater than 100 beats/minute experienced by a patient starting thyroid replacement should be reported immediately, and an electrocardiogram (ECG) and serum cardiac enzyme tests should be performed.

The patient with sudden pain in the left upper quadrant radiating to the back and vomiting was diagnosed with acute pancreatitis. What intervention(s) should the nurse expect to include in the patient's plan of care? A) Immediately start enteral feeding to prevent malnutrition. B) Insert an NG and maintain NPO status to allow pancreas to rest. C) Initiate early prophylactic antibiotic therapy to prevent infection. D) Administer acetaminophen (Tylenol) every 4 hours for pain relief.

B) Initial treatment with acute pancreatitis will include an NG tube if there is vomiting and being NPO to decrease pancreatic enzyme stimulation and allow the pancreas to rest and heal. Fluid will be administered to treat or prevent shock. The pain will be treated with IV morphine because of the NPO status. Enteral feedings will only be used for the patient with severe acute pancreatitis in whom oral intake is not resumed. Antibiotic therapy is only needed with acute necrotizing pancreatitis and signs of infection.

The nurse is beginning to teach a diabetic patient about vascular complications of diabetes. What information is appropriate for the nurse to include? A) Macroangiopathy does not occur in type 1 diabetes but rather in type 2 diabetics who have severe disease. B) Microangiopathy is specific to diabetes and most commonly affects the capillary membranes of the eyes, kidneys, and skin. C) Renal damage resulting from changes in large- and medium-sized blood vessels can be prevented by careful glucose control. D) Macroangiopathy causes slowed gastric emptying and the sexual impotency experienced by a majority of patients with diabetes.

B) Microangiopathy is specific to diabetes and most commonly affects the capillary membranes of the eyes, kidneys, and skin. Microangiopathy occurs in diabetes mellitus. When it affects the eyes, it is called diabetic retinopathy. When the kidneys are affected, the patient has nephropathy. When the skin is affected, it can lead to diabetic foot ulcers. Macroangiopathy can occur in either type 1 or type 2 diabetes and contributes to cerebrovascular, cardiovascular, and peripheral vascular disease.

A 51-year-old patient with diabetes mellitus is scheduled for a fasting blood glucose level at 8:00 AM. The nurse instructs the patient to only drink water after what time? A) 6:00 PM on the evening before the test B) Midnight before the test C) 4:00 AM on the day of the test D)7:00 AM on the day of the test

B) Midnight before the test Typically, a patient is ordered to be NPO for 8 hours before a fasting blood glucose level. For this reason, the patient who has a lab draw at 8:00 AM should not have any food or beverages containing any calories after midnight.

A patient with diabetes mellitus who has multiple infections every year needs a mitral valve replacement. What is the most important preoperative teaching the nurse should provide to prevent a cardiac infection postoperatively? A) Avoid sick people and wash hands. B) Obtain comprehensive dental care. C) Maintain hemoglobin A1c below 7%. D) Coughing and deep breathing with splinting

B) Obtain comprehensive dental care. A person with diabetes is at high risk for postoperative infections. The most important preoperative teaching to prevent a postoperative infection in the heart is to have the patient obtain comprehensive dental care because the risk of septicemia and infective endocarditis increases with poor dental health. Avoiding sick people, hand washing, maintaining hemoglobin A1c below 7%, and coughing and deep breathing with splinting would be important for any type of surgery, but not the priority with mitral valve replacement for this patient.

The patient with right upper quadrant abdominal pain has an abdominal ultrasound that reveals cholelithiasis. What should the nurse expect to do for this patient? A) Prevent all oral intake. B) Control abdominal pain. C) Provide enteral feedings. D) Avoid dietary cholesterol.

B) Patients with cholelithiasis can have severe pain, so controlling pain is important until the problem can be treated. NPO status may be needed if the patient will have surgery but will not be used for all patients with cholelithiasis. Enteral feedings should not be needed, and avoiding dietary cholesterol is not used to treat cholelithiasis.

The nurse is providing discharge instructions to a patient with diabetes insipidus. Which instructions regarding desmopressin acetate (DDAVP) would be most appropriate? A) The patient can expect to experience weight loss resulting from increased diuresis. B) The patient should alternate nostrils during administration to prevent nasal irritation. C) The patient should monitor for symptoms of hypernatremia as a side effect of this drug. D) The patient should report any decrease in urinary elimination to the health care provider.

B) The patient should alternate nostrils during administration to prevent nasal irritation. DDAVP is used to treat diabetes insipidus by replacing the antidiuretic hormone that the patient is lacking. Inhaled DDAVP can cause nasal irritation, headache, nausea, and other signs of hyponatremia. Diuresis will be decreased and is expected, and hypernatremia should not occur.

The nurse identifies that which patient is at highest risk for developing colon cancer? A. A 28-year-old male who has a body mass index of 27 kg/m2 B. A 32-year-old female with a 12-year history of ulcerative colitis C. A 52-year-old male who has followed a vegetarian diet for 24 years D. A 58-year-old female taking prescribed estrogen replacement therapy

B. A 32-year-old female with a 12-year history of ulcerative colitis Risk for colon cancer includes personal history of inflammatory bowel disease (especially ulcerative colitis for longer than 10 years); obesity (body mass index ≥ 30 kg/m2); family (first-degree relative) or personal history of colorectal cancer, adenomatous polyposis, hereditary nonpolyposis colorectal cancer syndrome; red meat (=7 servings/week); cigarette use; and alcohol (=4 drinks/week).

Which patient should be taught preventive measures for CKD by the nurse because this patient is most likely to develop CKD? A. A 50-year-old white female with hypertension B. A 61-year-old Native American male with diabetes C. A 40-year-old Hispanic female with cardiovascular disease D. A 28-year-old African American female with a urinary tract infection

B. A 61-year-old Native American male with diabetes It is especially important for the nurse to teach CKD prevention to the 61-year-old Native American with diabetes. This patient is at highest risk because diabetes causes about 50% of CKD. This patient is the oldest, and Native Americans with diabetes develop CKD 6 times more frequently than other ethnic groups. Hypertension causes about 25% of CKD. Hispanics have CKD about 1.5 times more than non-Hispanics. African Americans have the highest rate of CKD because hypertension is significantly increased in African Americans. A UTI will not cause CKD unless it is not treated or UTIs occur recurrently.

A patient with a history of end-stage kidney disease secondary to diabetes mellitus has presented to the outpatient dialysis unit for his scheduled hemodialysis. Which assessments should the nurse prioritize before, during, and after his treatment? A. Level of consciousness B. Blood pressure and fluid balance C. Temperature, heart rate, and blood pressure D. Assessment for signs and symptoms of infection

B. Blood pressure and fluid balance Although all of the assessments are relevant to the care of a patient receiving hemodialysis, the nature of the procedure indicates a particular need to monitor the patient's blood pressure and fluid balance.

A 70-year-old male patient has sought care because of recent difficulties in establishing and maintaining a urine stream as well as pain that occasionally accompanies urination. How should the nurse document this abnormal assessment finding? A. Anuria B. Dysuria C. Oliguria D. Enuresis

B. Dysuria Painful and difficult urination is characterized as dysuria. Anuria is an absence of urine production, whereas oliguria is diminished urine production. Enuresis is involuntary nocturnal urination.

A 24-year-old female donated a kidney via a laparoscopic donor nephrectomy to a non-related recipient. The patient is experiencing a lot of pain and refuses to get up to walk. How should the nurse handle this situation? A. Have the transplant psychologist convince her to walk. B. Encourage even a short walk to avoid complications of surgery. C. Tell the patient that no other patients have ever refused to walk. D. Tell the patient she is lucky she did not have an open nephrectomy.

B. Encourage even a short walk to avoid complications of surgery. Because ambulating will improve bowel, lung, and kidney function with improved circulation, even a short walk with assistance should be encouraged after pain medication. The transplant psychologist or social worker's role is to determine if the patient is emotionally stable enough to handle donating a kidney, while postoperative care is the nurse's role. Trying to shame the patient into walking by telling her that other patients have not refused and telling the patient she is lucky she did not have an open nephrectomy (implying how much more pain she would be having if it had been open) will not be beneficial to the patient or her postoperative recovery.

A patient admitted to the hospital with hypertension is diagnosed with a pheochromocytoma. The nurse will plan to monitor the patient for A. flushing. B. headache. C. bradycardia. D. hypoglycemia.

B. Headache

A nurse is admitting a patient with the diagnosis of advanced renal carcinoma. Based upon this diagnosis, the nurse will expect to find what clinical manifestations as the "classic triad" occurring in patients with renal cancer? A. Fever, chills, flank pain B. Hematuria, flank pain, palpable mass C. Hematuria, proteinuria, palpable mass D. Flank pain, palpable abdominal mass, and proteinuria

B. Hematuria, flank pain, palpable mass There are no characteristic early symptoms of renal carcinoma. The classic manifestations of gross hematuria, flank pain, and a palpable mass are those of advanced disease.

Which assessment finding is a consequence of the oliguric phase of AKI? A. Hypovolemia B. Hyperkalemia C. Hypernatremia D. Thrombocytopenia

B. Hyperkalemia In AKI the serum potassium levels increase because the normal ability of the kidneys to excrete potassium is impaired. Sodium levels are typically normal or diminished, whereas fluid volume is normally increased because of decreased urine output. Thrombocytopenia is not a consequence of AKI, although altered platelet function may occur in AKI.

The nurse is caring for a group of older patients in a long-term care setting. Which physical changes in the patients should the nurse investigate as signs of possible endocrine dysfunction? A. Absent reflexes, diarrhea, and hearing loss B. Hypoglycemia, delirium, and incontinence C. Fatigue, constipation, and mental impairment D. Hypotension, heat intolerance, and bradycardia

B. Hypoglycemia, delirium, and incontinence Changes of aging often mimic clinical manifestations of endocrine disorders. Clinical manifestations of endocrine dysfunction such as fatigue, constipation, or mental impairment in the older adult are often missed because they are attributed solely to aging.

The patient is brought to the ED following a car accident and is wearing medical identification that says she has Addison's disease. What should the nurse expect to be included in the collaborative care of this patient? A. Low sodium diet B. Increased glucocorticoid replacement C. Suppression of pituitary ACTH synthesis D. Elimination of mineralocorticoid replacement

B. Increased glucocorticoid replacement The patient with Addison's disease needs lifelong glucocorticoid and mineralocorticoid replacement and has an increased need with illness, injury, or stress, as this patient is experiencing. The patient with Addison's may also need a high sodium diet. Suppression of pituitary ACTH synthesis is done for Cushing syndrome. Elimination of mineralocorticoid replacement cannot be done for Addison's disease.

Diffusion, osmosis, and ultrafiltration occur in both hemodialysis and peritoneal dialysis. The nurse should know that ultrafiltration in peritoneal dialysis is achieved by which method? A. Increasing the pressure gradient B. Increasing osmolality of the dialysate C. Decreasing the glucose in the dialysate D. Decreasing the concentration of the dialysate

B. Increasing osmolality of the dialysate Ultrafiltration in peritoneal dialysis is achieved by increasing the osmolality of the dialysate with additional glucose. In hemodialysis the increased pressure gradient from increased pressure in the blood compartment or decreased pressure in the dialysate compartment causes ultrafiltration. Decreasing the concentration of the dialysate in either peritoneal or hemodialysis will decrease the amount of fluid removed from the blood stream.

The patient with systemic lupus erythematosus had been diagnosed with syndrome of inappropriate antidiuretic hormone (SIADH). What should the nurse expect to include in this patient's plan of care (select all that apply)? A. Obtain weekly weights. B. Limit fluids to 1000 mL per day.* C. Monitor for signs of hypernatremia. D. Minimize turning and range of motion. E. Keep the head of the bed at 10 degrees or less elevation.

B. Limit fluids to 1000 mL per day.* E. Keep the head of the bed at 10 degrees or less elevation. The care for the patient with SIADH will include limiting fluids to 1000 mL per day or less to decrease weight, increase osmolality, and improve symptoms; and keeping the head of the bed elevated at 10 degrees or less to enhance venous return to the heart and increase left atrial filling pressure, thereby reducing the release of ADH. The weights should be done daily along with intake and output. Signs of hyponatremia should be monitored, and frequent turning, positioning, and range-of-motion exercises are important to maintain skin integrity and joint mobility.

When caring for a patient who has an adrenocortical adenoma, causing hyperaldosteronism, the nurse should A. provide a potassium-restricted diet. B. monitor the blood pressure every 4 hours. C. monitor blood glucose level every 4 hours. D. relieve edema by elevating the extremities.

B. Monitor the blood pressure every 4 hours.

The nurse interviews a 50-year-old man with a history of type 2 diabetes mellitus, chronic bronchitis, and osteoarthritis who has a fasting blood glucose of 154 mg/dL. Which medications, if taken by the patient, may raise blood glucose levels? A. Glargine (Lantus) B. Prednisone (Deltasone) C. Metformin (Glucophage) D. Acetaminophen (Tylenol)

B. Prednisone (Deltasone) Prednisone is a corticosteroid that may cause glucose intolerance in susceptible patients by increasing gluconeogenesis and insulin resistance. Insulin (e.g., glargine) and metformin (an oral hypoglycemic agent) decrease blood glucose levels. Acetaminophen has a glucose-lowering effect

The young adult female patient reports unilateral left lower abdominal pain. The last menstrual period was 4 weeks ago. What diagnostic test is essential? A. Orthostatic vital signs B. Hemoglobin and hematocrit C. Pregnancy test D. Appendix ultrasound

C Rationale A pregnancy test is performed in women of childbearing age with acute abdominal pain to rule out ectopic pregnancy, and it should be performed before the other tests. The appendix is on the right side. If the pregnancy test is negative, additional testing for other causes should be done. Reference: 1015

A 22-year-old patient's blood pressure at her physical done for her new job was 110/68. At the health fair two months later, her blood pressure is 154/96. What renal problem should the nurse be aware of that could contribute to this abrupt rise in blood pressure? A. Renal trauma B. Renal artery stenosis C. Renal vein thrombosis D. Benign nephrosclerosis

B. Renal artery stenosis Renal artery stenosis contributes to an abrupt rise in blood pressure, especially in people under 30 or over 50 years of age. Renal trauma usually has hematuria. Renal vein thrombosis causes flank pain, hematuria, fever, or nephrotic syndrome. Benign nephrosclerosis usually occurs in adults 30 to 50 years of age and is a result of vascular changes resulting from hypertension.

The nurse receives a phone call from a 36-year-old woman taking cyclophosphamide (Cytoxan) for treatment of non-Hodgkin's lymphoma. The patient tells the nurse that she has muscle cramps and weakness and very little urine output. Which response by the nurse is best? A. "Start taking supplemental potassium, calcium, and magnesium." B. "Stop taking the medication now and call your health care provider." C. "These symptoms will decrease with continued use of the medication." D. "Increase fluids to 3000 mL per 24 hours to improve your urine output."

B. Stop taking the medication now and call your health care provider." Cyclophosphamide may cause syndrome of inappropriate antidiuretic hormone (SIADH). Medications that stimulate the release of ADH should be avoided or discontinued. Treatment may include restriction of fluids to 800 to 1000 mL per day. If a loop diuretic such as furosemide (Lasix) is used to promote diuresis, supplements of potassium, calcium, and magnesium may be needed.

Which nursing intervention is most appropriate in providing care for an adult patient with newly diagnosed adult onset polycystic kidney disease (PKD)? A. Help the patient cope with the rapid progression of the disease. B. Suggest genetic counseling resources for the children of the patient. C. Expect the patient to have polyuria and poor concentration ability of the kidneys. D. Implement appropriate measures for the patient's deafness and blindness in addition to the renal problems.

B. Suggest genetic counseling resources for the children of the patient. PKD is one of the most common genetic diseases. The adult form of PKD may range from a relatively mild disease to one that progresses to chronic kidney disease. Polyuria, deafness, and blindness are not associated with PKD.

As a component of the head-to-toe assessment of a patient who has been recently transferred to the clinical unit, the nurse is preparing to palpate the patient's kidneys. How should the nurse position the patient for this assessment? A. Prone B. Supine C. Seated at the edge of the bed D. Standing, facing away from the nurse

B. Supine To palpate the right kidney, the patient is positioned supine, and the nurse's left hand is placed behind and supports the patient's right side between the rib cage and the iliac crest. The right flank is elevated with the left hand, and the right hand is used to palpate deeply for the right kidney. The normal-sized left kidney is rarely palpable because the spleen lies directly on top of it.

A patient with hyperthyroidism is treated with radioactive iodine (RAI) at a clinic. Before the patient is discharged, the nurse instructs the patient A. That symptoms of hyperthyroidism should be relieved in about a week. B. That symptoms of hypothyroidism may occur as the RAI therapy takes effect. C. To discontinue the antithyroid medications taken before the radioactive therapy. D. About radioactive precautions to take with urine, stool, and other body secretions.

B. That symptoms of hypothyroidism may occur as the RAI therapy takes effect. Feedback There is a high incidence of postradiation hypothyroidism after RAI, and the patient should be monitored for symptoms of hypothyroidism. RAI has a delayed response, with the maximum effect not seen for 2 to 3 months, and the patient will continue to take antithyroid medications during this time. The therapeutic dose of radioactive iodine is low enough that no radiation safety precautions are needed.

The nurse and a licensed practical/vocational nurse (LPN/LVN) are working together to care for a patient who had an esophagectomy 2 days ago. Which action by the LPN/LVN requires that the nurse intervene? a. The LPN/LVN uses soft swabs to provide for oral care. b. The LPN/LVN positions the head of the bed in the flat position. c. The LPN/LVN encourages the patient to use pain medications before coughing. d. The LPN/LVN includes the enteral feeding volume when calculating intake and output.

B. The LPN/LVN positions the head of the bed in the flat position

The nurse is caring for a patient with a nephrostomy tube. The tube has stopped draining. After receiving orders, what should the nurse do? A. Keep the patient on bed rest. B. Use 5 mL of sterile saline to irrigate. C. Use 30 mL of water to gently irrigate. D. Have the patient turn from side to side.

B. Use 5 mL of sterile saline to irrigate. With a nephrostomy tube, if the tube is occluded and irrigation is ordered, the nurse should use 5 mL or less of sterile saline to gently irrigate it. The patient with a ureteral catheter may be kept on bed rest after insertion, but this is unrelated to obstruction. Only sterile solutions are used to irrigate any type of urinary catheter. With a suprapubic catheter, the patient should be instructed to turn from side to side to ensure patency.

A patient with end-stage renal disease (ESRD) secondary to diabetes mellitus has arrived at the outpatient dialysis unit for hemodialysis. Which assessments should the nurse perform as a priority before, during, and after the treatment? Level of consciousness Blood pressure and fluid balance Temperature, heart rate, and blood pressure Assessment for signs and symptoms of infection

Blood pressure and fluid balance

A 71-year-old patient had an abdominal-perineal resection for colon cancer. Which nursing action is most important to include in the plan of care for the day after surgery? a. Teach about a low-residue diet. b. Monitor output from the stoma. c. Assess the perineal drainage and incision. d. Encourage acceptance of the colostomy stoma.

C

The nurse should advise women recovering from surgical treatment of an ectopic pregnancy that: A. she has increased risk for salpingitis B. bed rest must be maintained for 12 hours to assist healing C. having one ectopic pregnancy increases her risk for another D. intrauterine devices and infertility treatments should be avoided

C

A female patient has a sliding hiatal hernia. What nursing interventions will prevent the symptoms of heartburn and dyspepsia that she is experiencing? a) Keep the patient NPO. b) Put the bed in the Trendelenberg position. c) Have the patient eat 4 to 6 smaller meals each day. d) Give various antacids to determine which one works for the patient.

C Eating smaller meals during the day will decrease the gastric pressure and the symptoms of hiatal hernia. Keeping the patient NPO or in a Trendelenberg position are not safe or realistic for a long period of time for any patient. Varying antacids will only be done with the care provider's prescription, so this is not a nursing intervention.

Following administration of a dose of metoclopramide (Reglan) to the patient, the nurse determines that the medication has been effective when what is noted? a) Decreased blood pressure b) Absence of muscle tremors c) Relief of nausea and vomiting d) No further episodes of diarrhea

C Metoclopramide is classified as a prokinetic and antiemetic medication. If it is effective, the patient's nausea and vomiting should resolve. Metoclopramide does not affect blood pressure, muscle tremors, or diarrhea.

You are conducting a community education session. Which is the best information to provide? A. Women are at highest risk for inguinal hernia. B. Hernias are a result of a low-fiber diet. C. Hernia is not an emergency unless it cannot be reduced. D. Pulsation is commonly seen in abdominal hernias.

C Rationale A hernia is not a concern unless it cannot be reduced and an obstruction occurs in the intestine. Men are at highest risk for an inguinal hernia. Hernias are a result of an abnormal opening or a weakened area in the wall of the cavity. A pulsating abdominal mass is a symptom of an abdominal aortic aneurysm. Reference: 1048

What is cause of abdominal compartment syndrome? A. Uncontrolled systemic hypertension B. Wearing restrictive clothing, decreasing venous flow C. Internal bleeding that causes pressure on organs D. Deficiency of the clotting factors

C Rationale Abdominal compartment syndrome is organ dysfunction caused by intraabdominal hypertension. Abdominal or retroperitoneal bleeding places pressure on abdominal organs located within the abdominal cavity. The other options are not causes of acute compartment syndrome. Reference: 1018

What is the best indication that the intravenous (IV) fluid replacement is adequate during the treatment of a patient with intestinal obstruction? A. Serum sodium: 155 mEq/L B. Urine specific gravity: 1.050 C. Urine output: 0.5 ml/kg/ hour D. Bowel sounds: 4 times/minute

C Rationale Adequate fluid replacement results in urine output of 0.5 mL/kg/ hour. The first two options indicate dehydration. Bowel sounds (peristalsis) are not used to determine rehydration. Reference: 1033

Which action will assist in caring for the patient with fecal incontinence? A. Assess stool consistency using the Braden scale. B. Encourage the use of a rectal tube to prevent skin breakdown. C. Assist the patient to the bathroom at a regular time daily. D. Encourage morning coffee to promote stool evacuation.

C Rationale Assisting with bowel training can help regulate evacuation. This can include assisting the patient to the bathroom at a regular time daily. Thirty minutes after breakfast is the time often recommended. The Braden Scale measures skin breakdown; the Bristol Stool Scale assesses stool consistency. Rectal tubes should be avoided because they can decrease responsiveness of the rectal sphincter and cause ulceration of the rectal mucosa. Patients with stool incontinence are taught to avoid caffeine, which worsens symptoms. Reference: 1011-1012

An elderly hospitalized patient, who was recently on broad-spectrum antibiotics, develops foul-smelling diarrhea. What is the most likely cause? A. Giardia lamblia B. Salmonella C. Clostridium difficile D. Lactose intolerance

C Rationale C. difficile is often a health care-acquired infection. Spores can survive up to 70 days. People at risk include elderly hospitalized patients who had antibiotics in the past 2 months. G. lamblia is a parasite found in contaminated lakes or pools. Salmonella is found in undercooked eggs or chicken. It is unusual for the patient to suddenly develop lactose intolerance and symptoms. Reference: 1007-1008

A patient with metastatic colorectal cancer is scheduled for chemotherapy and radiation therapy. Patient teaching regarding these therapies should include which explanation? A. Chemotherapy can be used to cure colorectal cancer. B. Irradiation is routinely used as adjuvant therapy after surgery. C. Both chemotherapy and irradiation can be used as palliative treatments. D. The patient should expect few or no side effects from chemotherapeutic agents.

C Rationale Chemotherapy can be used to shrink the tumor before surgery, as an adjuvant therapy after colon resection, and as palliative therapy for nonresectable colorectal cancer. Radiation therapy may be used postoperatively as an adjuvant to surgery and chemotherapy or as a palliative measure for patients with metastatic cancer. Reference: 1038

The patient has nonresectable colorectal cancer. The primary provider has recommended chemotherapy. What is the best explanation of this treatment? A. It gives the patient a sense of hope that something is being done. B. It shrinks the tumor before surgery. C. It provides palliative treatment. D. It prevents metastasis to the liver.

C Rationale Palliative treatment is done for nonresectable colorectal cancer to shrink the tumor and prevent obstruction. Telling the patient that chemotherapy may cure the cancer is not realistic and provides false information. This patient's tumor is classified as nonresectable, indicating that surgical treatment is not an option. In this situation it probably means that the tumor has already metastasized. Reference: 1038

Which food is recommended for a patient 2 weeks after having a colostomy? A. Cabbage B. Popcorn C. Applesauce D. Dried fruit

C Rationale Patients are initially on low-residue and low-fiber diets until the intestine adjusts. The eventual goal is to return to a normal presurgical diet. The patient who had an ileostomy is susceptible to obstruction. Foods such as popcorn and dried fruits must be chewed extremely well so particles are small when swallowed. Patients with ostomies are encouraged to eliminate the cabbage family foods because they cause odor. Reference: 1045

What should you teach a patient with colorectal cancer who is receiving capecitabine (Xeloda) for chemotherapy? A. Obtain weekly CBC count. B. Monitor the stool for occult blood. C. Do not get any immunizations without physician's approval. D. Take your temperature daily.

C Rationale Patients should not receive immunizations during chemotherapy because resistance is low, and they are unable to build up immunity. The other options are not specific for this drug, although the general instruction to monitor for bleeding or infection can be done. Reference: 1038

A female college student goes to the university health clinic complaining of pain that started at the umbilicus and moved to the right lower quadrant over the last 12 hours. You notice muscle guarding on examination. What action should you take? A. Administer a PRN laxative per standing orders. B. Ask about the last menstrual period. C. Make the student NPO. D. Assess bowel sounds.

C Rationale This is a classic description of appendicitis. At the very least, it is an acute abdomen, and the student should be kept NPO until a need for surgery is ruled out. The student should be referred to an emergency department. A laxative should not be given because it can increase peristalsis and cause perforation. Asking about her last menstrual cycle (possibility of a ruptured ectopic pregnancy) is important but the symptoms suggest appendicitis. Bowel sounds should be assessed, but the NPO status is a priority. Reference: 1021

The patient had an ileostomy 4 days earlier and has a daily drainage of 1800 mL. What action should you take? A. Notify the primary provider. B. Send a specimen to the laboratory. C. Document the findings. D. Test the stool for occult blood.

C Rationale With an ileostomy, the volume of drainage is high (1000 to 1800 mL/day) after peristalsis returns because the adsorptive functions provided by the colon and the delay provided by the ileocecal valve have been altered. Eventually, the amount of drainage is reduced to 500 mL daily as the proximal small bowel adapts. If the small bowel has been shortened from resections, the drainage can be greater. There is no need to do the other options. Reference: 1045

A 74-year-old female patient with gastroesophageal reflux disease (GERD) takes over-the-counter medications. For which medication, if taken long-term, should the nurse teach about an increased risk of fractures? a) Sucralfate (Carafate) b) Cimetidine (Tagamet) c) Omeprazole (Prilosec) d) Metoclopramide (Reglan)

C There is a potential link between proton pump inhibitors (PPIs) (e.g., omeprazole) use and bone metabolism. Long-term use or high doses of PPIs may increase the risk of fractures of the hip, wrist, and spine. Lower doses or shorter duration of therapy should be considered. REGLAN--> TARDIVE DYSKINESIA

The patient with chronic gastritis is being put on a combination of medications to eradicate H. pylori. Which drugs does the nurse know will probably be used for this patient? a) Antibiotic(s), antacid, and corticosteroid b) Antibiotic(s), aspirin, and antiulcer/protectant c) Antibiotic(s), proton pump inhibitor, and bismuth d) Antibiotic(s) and nonsteroidal antiinflammatory drugs (NSAIDs)

C To eradicate H. pylori, a combination of antibiotics, a proton pump inhibitor, and possibly bismuth (for quadruple therapy) will be used. Corticosteroids, aspirin, and NSAIDs are drugs that can cause gastritis and do not affect H. pylori.

During the nursing assessment of a patient with Graves' disease, the nurse notes a bounding, rapid pulse and systolic hypertension. Based on these assessment data, which question is important for the nurse to ask the patient? a. "Do you have any problem with frequent constipation?" b. "Have you noticed any recent decrease in your appetite?" c. "Do you ever have any chest pain?" d. "Have you had recent muscle aches?"

C R: Angina is a possible complication of Graves' disease, especially for a patient with tachycardia and hypertension. The other CMs are associated with hypothyroidism.

The client diagnosed with type 1 diabetes is found lying unconscious on the floor of the bathroom. Which intervention should the nurse implement first? a. administer 50% dextrose IVP b. notify the HCP c. move the client to the ICU d. check the serum glucose level

a. administer 50% dextrose IVP dextrose is only given if the client is found unconscious and the nurse suspect hypoglycemia. This will arouse the patient immediately

A patient with Graves' disease is prepared for surgery with drug therapy consisting of 4 weeks of propylthiouracil (PTU) and 10 days of iodine before surgery. When teaching the patient about the drugs, the nurse explains that the drugs are given preoperatively to a. eliminate the risk for tetany during the postoperative period. b. decrease the risk of hypometabolism during and after the surgery. c. normalize metabolism and decrease the size and vascularity of the gland. d. assist in differentiating the thyroid and parathyroid glands during surgery.

C R: Antithyroid drugs and iodine decrease the levels of thyroid hormone and the vascularity of the thyroid gland prior to surgery and lower the risk for postoperative thyrotoxicosis and hemorrhage. Postoperative tetany might be caused by removal of the parathyroid gland during thyroidectomy. The medications will tend to decrease metabolic rate. The medications will not help in differentiating the tissues of the thyroid and parathyroid glands.

A patient with an antidiuretic hormone (ADH)-secreting small-cell cancer of the lung is treated with demeclocycline (Declomycin) to control the symptoms of syndrome of inappropriate secretion of antidiuretic hormone (SIADH). The nurse determines that the demeclocycline is effective upon finding that the a. patient's daily weight is stable. b. urine specific gravity is increased. c. patient's urinary output is increased. d. peripheral edema is decreased.

C R: Demeclocycline blocks the action of ADH on the renal tubules and increases urine output. A stable body weight and an increase in urine specific gravity indicate that the SIADH is not corrected. Peripheral edema does not occur with SIADH; a sudden weight gain without edema is a common clinical manifestation of this disorder.

A patient has an adrenocortical adenoma causing hyperaldosteronism and is scheduled for laparoscopic surgery to remove the tumor. During care before surgery, the nurse should a. monitor blood glucose level every 4 hours. b. provide a potassium-restricted diet. c. monitor the blood pressure every 4 hours. d. relieve edema by elevating the extremities.

C R: HTN caused by Na retention is a common complication of hyperaldosteronism. Hyperaldosteronism does not cause elevation in blood glucose. pt will be hypokalemic and require potassium supplementation prior to surgery. Edema does not usually occur with hyperaldosteronism. (Cognitive Level: Application Text : pp. 1319-1320 NProcess: Implementation NCLEX: Physiological Integrity)

A 72-year-old patient is diagnosed with hypothyroidism, and levothyroxine (Synthroid) is prescribed. During initiation of thyroid replacement for the patient, it is most important for the nurse to assess a. mental status. b. nutritional level. c. cardiac function. d. fluid balance.

C R: In older patients, initiation of levothyroxine therapy can increase myocardial oxygen demand and cause angina or dysrhythmias. The medication is also expected to improve mental status and fluid balance and will increase metabolic rate and nutritional needs, but these changes do not indicate a need to change the therapy.

A patient seen at clinic for an upper respiratory infection reports receiving subcutaneous somatotropin (Genotropin) when asked by the nurse about current medications. The nurse questions the pt further about a hx of a. adrenal disease. b. untreated acromegaly. c. a pituitary tumor. d. diabetes insipidus (DI).

C R: Somatotropin is a recombinant growth hormone product used for adults with growth hormone deficiency, such as that caused by a pituitary tumor. The med is not used in adrenal disease or DI. The patient with untreated acromegaly will have an excess of growth hormone.

A patient with a possible pheochromocytoma is admitted to the hospital for evaluation and diagnostic testing. During an attack, the nurse will monitor for hypertension and a. hypoglycemia. b. bradycardia. c. headache. d. flushing.

C R: The classic CMs of pheochromocytoma are hypertension, tachycardia, severe headache, diaphoresis, and abdominal or chest pain. Elevated blood glucose may also occur due to sympathetic nervous system stimulation. Bradycardia and flushing would not be expected. (Cognitive Level: Application Text Reference: p. 1320 NProcess: Assessment NCLEX: Physiological Integrity)

While assessing a patient who has just arrived in the postanesthesia recovery unit (PACU) after a thyroidectomy, the nurse obtains these data. Which information is most important to communicate to the surgeon? a. The pt is complaining of 7/10 incisional pain. b. The pt's cardiac monitor shows a HR of 112. c. The patient has increasing swelling of the neck. d. The pat's voice is weak and hoarse sounding.

C R: The neck swelling may lead to respiratory difficulty, and rapid intervention is needed to prevent airway obstruction. The incisional pain should be treated but is not unusual after surgery. A heart rate of 112 is not unusual in a pt who has been hyperthyroid and has just arrived in the PACU from surgery. Vocal hoarseness is expected after surgery due to edema.

A type 1 diabetic patient who was admitted with severe hypoglycemia and treated tells the nurse, "I did not have any of the usual symptoms of hypoglycemia." Which question by the nurse will help identify a possible reason for the patient's hypoglycemic unawareness? a. "Do you use any calcium-channel blocking drugs for blood pressure?" b. "Have you observed any recent skin changes?" c. "Do you notice any bloating feeling after eating?" d. "Have you noticed any painful new ulcerations or sores on your feet?"

C Rationale: Hypoglycemic unawareness is caused by autonomic neuropathy, which would also cause delayed gastric emptying. Calcium-channel blockers are not associated with hypoglycemic unawareness, although -adrenergic blockers can prevent patients from having symptoms of hypoglycemia. Skin changes can occur with diabetes, but these are not associated with autonomic neuropathy. If the patient can feel painful areas on the feet, neuropathy has not occurred. Cognitive Level: Application Text Reference: p. 1281 Nursing Process: Assessment NCLEX: Physiological Integrity

A patient is admitted to the hospital with severe renal colic. The nurse's first priority in management of the patient is to a. administer opioids as prescribed. b. obtain supplies for straining all urine c. encourage fluid intake of 3-4L/day d. keep the patient NPO in preparation for surgery

a. administer opioids as prescribed.

During a clinic visit 3 months following a diagnosis of type 2 diabetes, the patient reports following a reduced-calorie diet. The patient has not lost any weight and did not bring the glucose-monitoring record. The nurse will plan to obtain a(n) a. fasting blood glucose level. b. urine dipstick for glucose. c. glycosylated hemoglobin level. d. oral glucose tolerance test.

C Rationale: The glycosylated hemoglobin (Hb A1C) test shows the overall control of glucose over 90 to 120 days. A fasting blood level indicates only the glucose level at one time. Urine glucose testing is not an accurate reflection of blood glucose level and does not reflect the glucose over a prolonged time. Oral glucose tolerance testing is done to diagnose diabetes, but is not used for monitoring glucose control once diabetes has been diagnosed. Cognitive Level: Application Text Reference: pp. 1258-1259 Nursing Process: Planning NCLEX: Physiological Integrity

A patient with Cushing syndrome is admitted to the hospital to have laparoscopic adrenalectomy. During the admission assessment, the patient tells the nurse, "The worst thing about this disease is how terrible I look. I feel awful about it." best response by the nurse is a. "Let me show you how to dress so that the changes are not so noticeable." b. "I do not think you look bad. Your appearance is just altered by your disease." c. "Most of the physical and mental changes caused by the disease will gradually improve after surgery." d. "You really should not worry about how you look in the hospital. We see many worse things."

C Rationale: The most reassuring communication to the patient is that the physical and emotional changes caused by the Cushing syndrome will resolve after hormone levels return to normal postoperatively. The response beginning "Let me show you how to dress" indicates that the changes are permanent and that the patient's appearance needs disguising. The response beginning, "I do not think you look bad" does not acknowledge the patient's feelings and also fails to communicate that the changes will be resolved after surgery. And the response beginning "You really should not worry about how you look in the hospital" implies that the pt's appearance is not good.

A diabetic patient is admitted with ketoacidosis and the health care provider writes all of the following orders. Which order should the nurse implement first? a. Start an infusion of regular insulin at 50 U/hr. b. Give sodium bicarbonate 50 mEq IV push. c. Infuse 1 liter of normal saline per hour. d. Administer regular IV insulin 30 U.

C Rationale: The most urgent patient problem is the hypovolemia associated with DKA, and the priority is to infuse IV fluids. The other actions can be accomplished after the infusion of normal saline is initiated. Cognitive Level: Application Text Reference: p. 1280 Nursing Process: Implementation NCLEX: Physiological Integrity

The nurse receives an order for a parenteral dose of promethazine (Phenergan) and prepares to administer the medication to a 38-year-old male patient with nausea and repeated vomiting. Which action is most important for the nurse to take? a) Administer the medication subcutaneously for fast absorption. b) Administer the medication into an arterial line to prevent extravasation. c) Administer the medication deep into the muscle to prevent tissue damage. d) Administer the medication with 0.5 mL of lidocaine to decrease injection pain.

C Promethazine (Phenergan) is an antihistamine administered to relieve nausea and vomiting. Deep muscle injection is the preferred route of injection administration. This medication should not be administered into an artery or under the skin because of the risk of severe tissue injury, including gangrene. When administered IV, a risk factor is that it can leach out from the vein and cause serious damage to surrounding tissue.

A 29-year-old woman with systemic lupus erythematosus has been prescribed 2 weeks of high-dose prednisone therapy. Which information about the prednisone is most important for the nurse to include? a. "Weigh yourself daily to monitor for weight gain caused by increased appetite." b. "A weight-bearing exercise program will help minimize the risk for osteoporosis." c. "The prednisone dose should be decreased gradually rather than stopped suddenly." d. "Call the health care provider if you experience mood alterations with the prednisone."

C Acute adrenal insufficiency may occur if exogenous corticosteroids are suddenly stopped. Mood alterations and weight gain are possible adverse effects of corticosteroid use, but these are not life-threatening effects. Osteoporosis occurs when patients take corticosteroids for longer periods

The patient received regular insulin 10 units subcutaneously at 8:30 PM for a blood glucose level of 253 mg/dL. The nurse plans to monitor this patient for signs of hypoglycemia at which time related to the insulin's peak action? A) 8:40 PM to 9:00 PM B) 9:00 PM to 11:30 PM C) 10:30 PM to 1:30 AM D) 12:30 AM to 8:30 AM

C) 10:30 PM to 1:30 AM Regular insulin exerts peak action in 2 to 5 hours, making the patient most at risk for hypoglycemia between 10:30 PM and 1:30 AM. Rapid-acting insulin's onset is between 10-30 minutes with peak action and hypoglycemia most likely to occur between 9:00 PM and 11:30 PM. With intermediate acting insulin, hypoglycemia may occur from 12:30 AM to 8:30 AM.

A patient with cholelithiasis needs to have the gallbladder removed. Which patient assessment is a contraindication for a cholecystectomy? A) Low-grade fever of 100° F and dehydration B) Abscess in the right upper quadrant of the abdomen C) Activated partial thromboplastin time (aPTT) of 54 seconds D) Multiple obstructions in the cystic and common bile duct

C) An aPTT of 54 seconds is above normal and indicates insufficient clotting ability. If the patient had surgery, significant bleeding complications postoperatively are very likely. Fluids can be given to eliminate the dehydration; the abscess can be assessed, and the obstructions in the cystic and common bile duct would be relieved with the cholecystectomy.

A 54-year-old patient admitted with diabetes mellitus, malnutrition, osteomyelitis, and alcohol abuse has a serum amylase level of 280 U/L and a serum lipase level of 310 U/L. To which of the following diagnoses does the nurse attribute these findings? A. Malnutrition B. Osteomyelitis C. Alcohol abuse D. Diabetes mellitus

C) The patient with alcohol abuse could develop pancreatitis as a complication, which would increase the serum amylase (normal 30-122 U/L) and serum lipase (normal 31-186 U/L) levels as shown.

A 65-year-old patient with type 2 diabetes has a urinary tract infection (UTI). The unlicensed assistive personnel (UAP) reported to the nurse that the patient's blood glucose is 642 mg/dL and the patient is hard to arouse. When the nurse assesses the urine, there are no ketones present. What collaborative care should the nurse expect for this patient? A) Routine insulin therapy and exercise B) Administer a different antibiotic for the UTI. C) Cardiac monitoring to detect potassium changes D) Administer IV fluids rapidly to correct dehydration.

C) Cardiac monitoring to detect potassium changes This patient has manifestations of hyperosmolar hyperglycemic syndrome (HHS). Cardiac monitoring will be needed because of the changes in the potassium level related to fluid and insulin therapy and the osmotic diuresis from the elevated serum glucose level. Routine insulin would not be enough, and exercise could be dangerous for this patient. Extra insulin will be needed. The type of antibiotic will not affect HHS. There will be a large amount of IV fluid administered, but it will be given slowly because this patient is older and may have cardiac or renal compromise requiring hemodynamic monitoring to avoid fluid overload during fluid replacement.

A college student is newly diagnosed with type 1 diabetes. She now has a headache, changes in her vision, and is anxious, but does not have her portable blood glucose monitor with her. Which action should the campus nurse advise her to take? A) Eat a piece of pizza. B) Drink some diet pop. C) Eat 15 g of simple carbohydrates. D) Take an extra dose of rapid-acting insulin.

C) Eat 15 g of simple carbohydrates. When the patient with type 1 diabetes is unsure about the meaning of the symptoms she is experiencing, she should treat herself for hypoglycemia to prevent seizures and coma from occurring. She should also be advised to check her blood glucose as soon as possible. The fat in the pizza and the diet pop would not allow the blood glucose to increase to eliminate the symptoms. The extra dose of rapid-acting insulin would further decrease her blood glucose.

A patient, who is admitted with diabetes mellitus, has a glucose level of 380 mg/dL and a moderate level of ketones in the urine. As the nurse assesses for signs of ketoacidosis, which respiratory pattern would the nurse expect to find? A) Central apnea B) Hypoventilation C) Kussmaul respirations D) Cheyne-Stokes respirations

C) Kussmaul respirations In diabetic ketoacidosis, the lungs try to compensate for the acidosis by blowing off volatile acids and carbon dioxide. This leads to a pattern of Kussmaul respirations, which are deep and nonlabored.

The newly diagnosed patient with type 2 diabetes has been prescribed metformin (Glucophage). What should the nurse tell the patient to best explain how this medication works? A) Increases insulin production from the pancreas. B) Slows the absorption of carbohydrate in the small intestine. C) Reduces glucose production by the liver and enhances insulin sensitivity. D) Increases insulin release from the pancreas, inhibits glucagon secretion, and decreases gastric emptying.

C) Reduces glucose production by the liver and enhances insulin sensitivity. Metformin is a biguanide that reduces glucose production by the liver and enhances the tissue's insulin sensitivity. Sulfonylureas and meglitinides increase insulin production from the pancreas. α-glucosidase inhibitors slow the absorption of carbohydrate in the intestine. Glucagon-like peptide receptor agonists increase insulin synthesis and release from the pancreas, inhibit glucagon secretion, and decrease gastric emptying.

The patient called the clinic with manifestations of burning on urination, dysuria, and frequency. What is the best advice for the nurse to give the patient? A. "Drink less fluid so you don't have to void so often." B. "Take some acetaminophen to decrease the discomfort." C. "Come in so we can check a clean catch urine specimen." D. "Avoid caffeine and spicy food to decrease inflammation."

C. "Come in so we can check a clean catch urine specimen." The patient's symptoms are typical of a urinary tract infection (UTI). To verify this, a clean catch urine specimen must be obtained for a specimen of urine to culture. Drinking less fluid will not improve the symptoms. Acetaminophen would not decrease the discomfort; an antibiotic would be needed. Avoiding caffeine and spicy food may decrease bladder inflammation but will not affect these symptoms.

A patient is admitted to the hospital in Addisonian crisis. Which patient statement supports the nursing diagnosis of ineffective self-health management related to lack of knowledge about management of Addison's disease? A. "I double my dose of hydrocortisone on the days that I go for a run." B. "I frequently eat at restaurants, and so my food has a lot of added salt." C. "I had the stomach flu earlier this week and couldn't take the hydrocortisone." D. "I take twice as much hydrocortisone in the morning as I do in the afternoon."

C. "I had the stomach flu earlier this week and couldn't take the hydrocortisone."

A 58-year-old woman is being discharged home today after ostomy surgery for colon cancer. The nurse should assign the patient to which staff member? A. A nursing assistant on the unit who also has hospice experience B. A licensed practical nurse who has worked on the unit for 10 years C. A registered nurse with 6 months of experience on the surgical unit D. A registered nurse who has floated to the surgical unit from pediatrics

C. A registered nurse with 6 months of experience on the surgical unit The patient needs ostomy care directions/reinforcement at discharge and should be assigned to a registered nurse with experience in providing discharge teaching for ostomy care. Teaching should not be delegated to a licensed practical/vocational nurse or unlicensed assistive personnel.

An older male patient visits his primary care provider because of burning on urination and production of urine that he describes as "foul smelling." The health care provider should assess the patient for what factor that may put him at risk for a urinary tract infection (UTI)? A. High-purine diet B. Sedentary lifestyle C. Benign prostatic hyperplasia (BPH) D. Recent use of broad-spectrum antibiotics

C. Benign prostatic hyperplasia (BPH) BPH causes urinary stasis, which is a predisposing factor for UTIs. A sedentary lifestyle and recent antibiotic use are unlikely to contribute to UTIs, whereas a diet high in purines is associated with renal calculi.

The patient has scleroderma and is experiencing hypertension. The nurse should know that this could be related to which renal problem? A. Obstructive uropathy B. Goodpasture syndrome C. Chronic glomerulonephritis D. Calcium oxalate urinary calculi

C. Chronic glomerulonephritis Hypertension occurs with chronic glomerulonephritis that may be found in patients with scleroderma. Obstructive uropathy, Goodpasture syndrome, and calcium oxalate urinary calculi are not related to scleroderma and do not cause hypertension.

A patient asks the nurse about taking phentermine and topiramate (Qsymia) for weight loss. To avoid side effects, it is important for the nurse to determine whether the patient has a history of a. glaucoma. b. hypertension. c. valvular heart disease. d. irritable bowel disease.

Correct answer: a Rationale: Qsymia is a combination of phentermine and topiramate. It must not be used in patients with glaucoma or hyperthyroidism.

Which instruction should the nurse provide when teaching a patient to exercise the pelvic floor? A. Tighten both buttocks together. B. Squeeze thighs together tightly. C. Contract muscles around rectum. D. Lie on back and lift legs together.

C. Contract muscles around rectum. To teach pelvic floor exercises, or Kegel exercise, the nurse should instruct the patient (without contracting the legs, buttocks, or abdomen) to contract the muscles around the rectum (pelvic floor muscles) as if stopping a stool, which should result in a pelvic lifting sensation

A 20-year-old man is admitted to the emergency department after a motor vehicle crash with suspected abdominal trauma. What assessment finding by the nurse is of highest priority? A. Nausea and vomiting B. Hyperactive bowel sounds C. Firmly distended abdomen D. Abrasions on all extremities

C. Firmly distended abdomen Clinical manifestations of abdominal trauma are guarding and splinting of the abdominal wall; a hard, distended abdomen (indicating possible intraabdominal bleeding); decreased or absent bowel sounds; contusions, abrasions, or bruising over the abdomen; abdominal pain; pain over the scapula; hematemesis or hematuria; and signs of hypovolemic shock (tachycardia and decreased blood pressure).

A patient is recovering in the intensive care unit (ICU) after receiving a kidney transplant approximately 24 hours ago. What is an expected assessment finding for this patient during this early stage of recovery? A. Hypokalemia B. Hyponatremia C. Large urine output D. Leukocytosis with cloudy urine output

C. Large urine output Patients frequently experience diuresis in the hours and days immediately following a kidney transplant. Electrolyte imbalances and signs of infection are unexpected findings that warrant prompt intervention.

For a patient with Crohn's disease which assessment finding is most important for you to follow-up? A. Bloody diarrheal stool: 4 times/day B. Abdominal cramping C. Temperature: 100.4° F (38° C) D. Positive rebound tenderness

D Rationale Positive rebound tenderness is a classic sign of peritonitis and requires emergency follow-up. The other options are expected signs or symptoms with ulcerative colitis, which has intermittent exacerbations. Reference: 1021, 1023

The surgeon was unable to spare a patient's parathyroid gland during a thyroidectomy. Which assessments should the nurse prioritize when providing postoperative care for this patient? A. Assessing the patient's white blood cell levels and assessing for infection B. Monitoring the patient's hemoglobin, hematocrit, and red blood cell levels C. Monitoring the patient's serum calcium levels and assessing for signs of hypocalcemia D. Monitoring the patient's level of consciousness and assessing for acute delirium or agitation

C. Monitoring the patient's serum calcium levels and assessing for signs of hypocalcemia Loss of the parathyroid gland is associated with hypocalcemia. Infection and anemia are not associated with loss of the parathyroid gland, whereas cognitive changes are less pronounced than the signs and symptoms of hypocalcemia.

The nurse is caring for a 36-year-old woman with possible hypoparathyroidism after a thyroidectomy. It is most appropriate for the nurse to assess for which clinical manifestations? A. Polyuria, polydipsia, and weight loss B. Cardiac dysrhythmias and hypertension C. Muscle spasms and hyperactive deep tendon reflexes D. Hyperpigmentation, skin ulcers, and peripheral edema

C. Muscle spasms and hyperactive deep tendon reflexes Common assessment abnormalities associated with hypoparathyroidism include tetany (muscle spasms) and increased deep tendon reflexes. Hyperpigmentation is associated with Addison's disease. Skin ulcers occur in patient with diabetes. Edema is associated with hypothyroidism. Polyuria and polydipsia occur in patients with diabetes mellitus or diabetes insipidus. Weight loss occurs in hyperthyroidism or diabetic ketoacidosis. Hypertension and cardiac dysrhythmias may be caused by hyperthyroidism, hyperparathyroidism, or pheochromocytoma.

When caring for a patient with nephrotic syndrome, the nurse should know the patient understands dietary teaching when the patient selects which food item? A. Peanut butter and crackers B. One small grilled pork chop C. Salad made of fresh vegetables D. Spaghetti with canned spaghetti sauce

C. Salad made of fresh vegetables Of the options listed, only salad made with fresh vegetables would be acceptable for the diet that limits sodium and protein as well as saturated fat if hyperlipidemia is present. Peanut butter and crackers are processed so they contain significant sodium, and peanut butter contains some protein. A pork chop is a high-protein food with saturated fat. Canned spaghetti sauce is also high in sodium.

The patient has a low-grade carcinoma on the left lateral aspect of the prostate gland and has been on "watchful waiting" status for 5 years. Six months ago his last prostate-specific antigen (PSA) level was 5 ng/mL. Which manifestations now indicate that the prostate cancer may be growing and he needs a change in his care (select all that apply)? A. Casts in his urine B. Presence of α-fetoprotein C. Serum PSA level 10 ng/mL D. Onset of erectile dysfunction E. Nodularity of the prostate gland

C. Serum PSA level 10 ng/mL E. Nodularity of the prostate gland The manifestations of increased PSA level along with the new nodularity of the prostate gland potentially indicate that the tumor may be growing. Casts in the urine, presence of α-fetoprotein, and new onset of erectile dysfunction do not indicate prostate cancer growth.

The nurse is planning care for a client diagnosed with Addisons disease. Which intervention should be included? a. administer steroid medications b. place the client on fluid restriction c. provide frequent stimulation d. consult physical therapy for gait training

a. administer steroid medications

Which effect of aging on the urinary system is most likely to affect the action of bumetanide (Bumex)? A. Benign enlargement of prostatic tissues B. Decreased sensation of bladder capacity C. Decreased function of the loop of Henle D. Less absorption in the Bowman's capsule

C.Decreased function of the loop of Henle Bumetanide (Bumex) is a loop diuretic that acts in the loop of Henle to decrease reabsorption of sodium and chloride. Because the loop of Henle loses function with aging, the excretion of drugs becomes less and less efficient. Thus the circulating levels of drugs are increased and their effects prolonged. The benign enlargement of prostatic tissue, decreased sensation of bladder capacity, and loss of concentrating ability do not directly affect the action of loop diuretics.

The physician has decided to use renal replacement therapy to remove large volumes of fluid from a patient who is hemodynamically unstable in the intensive care unit. The nurse should expect which treatment to be used for this patient? Hemodialysis (HD) three times per week Automated peritoneal dialysis (APD) Continuous venovenous hemofiltration (CVVH) Continuous ambulatory peritoneal dialysis (CAPD)

Continuous venovenous hemofiltration (CVVH) *CVVH removes large volumes of water and solutes from the patient over a longer period of time by using ultrafiltration and convection. HD three times per week would not be used for this patient because fluid and solutes build up and then are rapidly removed. With APD (used at night instead of during the day) fluid and solutes build up during the day and would not benefit this patient as much. CAPD will not as rapidly remove large amounts of fluid as CVVH can do.

The patient with right upper quadrant abdominal pain has an abdominal ultrasound that reveals cholelithiasis. What should the nurse expect to do for this patient? Prevent all oral intake. Control abdominal pain. Provide enteral feedings. Avoid dietary cholesterol.

Control abdominal pain. Patients with cholelithiasis can have severe pain, so controlling pain is important until the problem can be treated. NPO status may be needed if the patient will have surgery but will not be used for all patients with cholelithiasis. Enteral feedings should not be needed, and avoiding dietary cholesterol is not used to treat cholelithiasis.

The nurse is caring for a patient who complains of abdominal pain and hematemesis. Which new assessment finding(s) would indicate the patient is experiencing a decline in condition? a. Pallor and diaphoresis b. Ecchymotic peripheral IV site c. Guaiac-positive diarrhea stools d. Heart rate 90, respiratory rate 20, BP 110/60

Correct Answer: a A patient with hematemesis has some degree of bleeding from an unknown source. Guaiac-positive diarrhea stools would be an expected finding. When monitoring the patient for stability, the nurse observes for signs of hypovolemic shock such as tachycardia, tachypnea, hypotension, altered level of consciousness, pallor, and cool and clammy skin. An ecchymotic peripheral IV site will require assessment to determine the need for reinsertion. Access would be critical in the immediate treatment of shock, but the ecchymotic site does not represent a decline in condition.

Which patient would be at highest risk for developing oral candidiasis? a. A 74-yr-old patient who has vitamin B and C deficiencies b. A 22-yr-old patient who smokes 2 packs of cigarettes per day c. A 32-yr-old patient who is receiving ciprofloxacin for 3 weeks d. A 58-yr-old patient who is receiving amphotericin B for 2 days

Correct answer: C Oral candidiasis is caused by prolonged antibiotic treatment (e.g., ciprofloxacin) or high doses of corticosteroids. Amphotericin B is used to treat candidiasis. Vitamin B and C deficiencies may lead to Vincent's infection. Use of tobacco products leads to stomatitis, not candidiasis.

Assessment findings suggestive of peritonitis include a. rebound abdominal pain. b. a soft, distended abdomen. c. dull, continuous abdominal pain. d. observing that the patient is restless.

Correct answer: a Rationale: With peritoneal irritation, the abdomen is hard, like a board, and the patient has severe abdominal pain that is worse with any sudden movement. The patient lies very still. Palpating the abdomen and releasing the hands suddenly causes sudden movement within the abdomen and severe pain. This is called rebound tenderness.

What problem should the nurse assess the patient for if the patient was on prolonged antibiotic therapy? a. Coagulation problems b. Elevated serum ammonia levels c. Impaired absorption of amino acids d. Increased mucus and bicarbonate secretion

Correct answer: a Rationale: Bacteria int he colon (1) synthesize vitamin K, which is needed for the production of prothrombin by the liver and (2) deaminate undigested or non absorbed proteins, producing ammonia, which is converted to urea by the liver. A reduction in normal flora bacteria by antibiotic therapy can lead to decreased vitamin K, resulting in decreased prothrombin and coagulation problems. Bowel bacteria do not influence protein absorption or the secretion of mucus.

A patient had a stomach resection for stomach cancer. The nurse should teach the patient about the loss of the hormone that stimulates gastric acid secretion and motility and maintains lower esophageal sphincter tone. Which hormone will be decreased with a gastric resection? a. Gastrin b. Secretin c. Cholecystokinin d. Gastric inhibitory peptide

Correct answer: a Rationale: Gastrin is the hormone activated in the stomach (and duodenal mucosa) by stomach distention that stimulates gastric acid secretion and motility and maintains lower esophageal sphincter tone. Secretin inhibits gastric motility and acid secretion and stimulates pancreatic bicarbonate secretion. Cholecystokinin allows increased flow of bile into the duodenum and release of pancreatic digestive enzymes. Gastric inhibitory peptide inhibits gastric acid secretion and motility.

The nurse is caring for a patient who is 5'6" tall and weighs 186 lb. The nurse has discussed reasonable weight loss goals and a low-calorie diet with the patient. Which statement made by the patient indicates a need for further teaching? a. "I will limit intake to 500 calories a day." b. "I will try to eat very slowly during mealtimes." c. "I'll try to pick foods from all of the basic food groups." d. "It's important for me to begin a regular exercise program."

Correct answer: a Rationale: Limiting intake to 500 calories per day is not indicated for this patient, and the severe calorie energy restriction would place this patient at risk for multiple nutrient deficiencies. Decreasing caloric intake at least 500 to 1000 calories a day is recommended for weight loss of one to two pounds per week. The other options show understanding of the teaching.

A 50-year-old African American woman has a BMI of 35 kg/m2, type 2 diabetes mellitus, hypercholesterolemia, and irritable bowel syndrome (IBS). She is seeking assistance in losing weight, because, "I have trouble stopping eating when I should, but I do not want to have bariatric surgery." Which drug therapy should the nurse question if it is prescribed for this patient? a. Orlistat (Xenical) b. Locaserin (Belviq) c. Phentermine (Adipex-P) d. Phentermine and topiramate (Qsymia)

Correct answer: a Rationale: Orlistat (Xenical), which blocks fat breakdown and absorption in the intestine, produces some unpleasant GI side effects. This drug would not be appropriate for someone with IBS. Locaserin (Belviq) suppresses the appetite and creates a sense of satiety that may be helpful for this patient. Phentermine (Adipex-P) needs to be used for a limited period of time (3 months or less). Qsymia is a combination of two drugs, phentermine and topiramate. Phentermine is a sympathomimetic agent that suppresses appetite and topiramate induces a sense of satiety.

A patient has been on a 1000-calorie diet with a daily exercise routine. In 2 months, the patient has lost 20 lb (9kg) toward a goal of 50 lb (23 kg) but is now discouraged that no weight has been lost in the last 2 weeks. What should the nurse tell the patient about this? a. Plateaus where no weight is lost normally occur during a weight-loss program. b. A weight considered by the body to most efficient for functioning has been reached. c. A return to former eating habits is the most common cause of not continuing to lose weight. d. A steady weight may be due to water gain from eating foods high in sodium.

Correct answer: a Rationale: Plateau periods during which no weight is lost are normal occurrences during weight reduction and may last for several days to several weeks but weight loss will resume if the prescribed weight reduction plan is continued. Weight loss may stop if former eating habits are resumed but this not the most common cause of plateaus.

The percentage of daily calories for a healthy individual consists of a. 50% carbohydrates, 25% protein, 25% fat, and <10% of fat from saturated fatty acids. b. 65% carbohydrates, 25% protein, 25% fat, and >10% of fat from saturated fatty acids. c. 50% carbohydrates, 40% protein, 10% fat, and <10% of fat from saturated fatty acids. d. 40% carbohydrates, 30% protein, 30% fat, and >10% of fat from saturated fatty acids.

Correct answer: a Rationale: The 2005 Dietary Guidelines for Americans recommend that 45% to 65% of total calories should come from carbohydrates. Ideally, 10% to 35% of daily caloric needs should come from protein. Individuals should limit their fat intake to 20% to 35% of total calories. Additional recommendations focus on the type of fat consumed because diets high in excess calories, usually in the form of fats, contribute to the development of obesity. Individuals should consume less than 10% of calories from saturated fatty acids, limit intake of fat and oils high in trans fatty acids, and should limit intake of dietary cholesterol to 300 mg/day.

A patient with hepatitis A is in the acute phase. The nurse plans care for the patient based on the knowledge that a. pruritus is a common problem with jaundice in this phase. b. the patient is most likely to transmit the disease during this phase. c. gastrointestinal symptoms are not as severe in hepatitis A as they are in hepatitis B. d. extrahepatic manifestations of glomerulonephritis and polyarteritis are common in this phase. (Lewis 1042)

Correct answer: a Rationale: The acute phase of jaundice may be icteric or anicteric. Jaundice results when bilirubin diffuses into the tissues. Pruritus sometimes accompanies jaundice. Pruritus is the result of an accumulation of bile salts beneath the skin.

During the initial postoperative period following bariatric surgery, the nurse recognizes the importance of monitoring obese patients for respiratory insufficiency based on what knowledge? a. The body stores anesthetics in adipose tissue. b. Postoperative pain may cause a decreased respiratory rate. c. Intubation may be difficult because of extra chin skinfolds. d. The patient's head must remain flat for a minimum of 2 hours postprocedure.

Correct answer: a Rationale: The body stores anesthetics in adipose tissue, placing patients with excess adipose tissue at risk for re-sedation. As adipose cells release anesthetics back into the bloodstream, the patient may become sedated after surgery, increasing the risk of hypoventilation and resultant respiratory insufficiency. Difficult intubation does not cause respiratory insufficiency. Pain usually increases respiratory rate. The patient's head should be elevated after bariatric surgery to decrease abdominal pressure and facilitate respirations.

The nurse cares for a 34-year-old woman after bariatric surgery. The nurse determines that discharge teaching related to diet is successful if the patient makes which statement? a. "A high protein diet that is low in carbohydrates and fat will prevent diarrhea." b. "Food should be high in fiber to prevent constipation from the pain medication." c. "Three meals a day with no snacks between meals will provide optimal nutrition." d. "Fluid intake should be at least 2000 mL per day with meals to avoid dehydration."

Correct answer: a Rationale: The diet generally prescribed is high in protein and low in carbohydrates, fat, and roughage and consists of six small feedings daily. Fluids should not be ingested with the meal, and in some cases, fluids should be restricted to less than 1000 mL per day. Fluids and foods high in carbohydrate tend to promote diarrhea and symptoms of the dumping syndrome. Generally, calorically dense foods (foods high in fat) should be avoided to permit more nutritionally sound food to be consumed.

Checking for the return of the gag reflex and monitoring for LUQ pain, nausea and vomiting are necessary nursing actions after which diagnostic procedure? a. ERCP b. Colonoscopy c. Barium swallow d. Esophagogastroduodenoscopy (EGD)

Correct answer: a Rationale: The left upper quadrant (LUQ) pain and nausea and vomiting could occur from perforation. The return of gag reflex is essential to prevent aspiration after an ERCP. The gag reflex is also assessed with an EGD. These are not relevant assessments for the colonoscopy and barium swallow.

The nurse explains to the patient undergoing ostomy surgery that the procedure that maintains the most normal functioning of the bowel is a. a sigmoid colostomy. b. a transverse colostomy. c. a descending colostomy. d. an ascending colostomy.

Correct answer: a Rationale: The more distal the ostomy is, the more the intestinal contents resemble feces eliminated from an intact colon and rectum. Output from a sigmoid colostomy resembles normally formed stool, and some patients are able to regulate emptying time so they do not need to wear a collection bag.

The nurse teaches a 50-year-old woman who has a body mass index (BMI) of 39 kg/m2 about weight loss. Which dietary change would be appropriate for the nurse to recommend to this patient? a. Decrease fat intake and control portion size b. Increase vegetables and decrease fluid intake c. Increase protein intake and avoid carbohydrates d. Decrease complex carbohydrates and limit fiber

Correct answer: a Rationale: The safest dietary guideline for weight loss is to decrease caloric intake by maintaining a balance of nutrients and adequate hydration while controlling portion size and decreasing fat intake.

During assessment of the patient with protein-calorie malnutrition, what should the nurse expect to find (select all that apply)? a. Frequent cold symptoms b. Decreased bowel sounds c. Cool, rough, dry, scaly skin d. A flat or concave abdomen e. Prominent bony structures f. Decreased reflexes and lack of attention

Correct answer: a, b, c, e, f Rationale: In malnutrition, metabolic processes are slowed, leading to increased sensitivity to cold, decreased heart rate (HR) and cardiac output (CO), and decreased neurologic function. Because of slowed GI motility and absorption, the abdomen becomes distended and protruding and bowel sounds are decreased. Skin is rough, dry, and scaly whereas bone structures protrude because of muscle loss. Because the immune system is weakened, susceptibility to respiratory infections is increased.

The stable patient has a gastrostomy tube for enteral feeding. Which care could the RN delegate to the LPN (select all that apply)? a. Administer bolus or continuous feedings. b. Evaluate the nutritional status of the patient. c. Administer medications through the gastrostomy tube. d. Monitor for complications related to the tube and enteral feeding. e. Teach the caregiver about feeding via the gastrostomy tube at home.

Correct answer: a, c Rationale: For the stable patient, the LPN can administer bolus or continuous feedings and administer medications through the gastrostomy. The RN must evaluate the nutritional status of the patient, monitor for complications related to the tube and the enteral feeding, and teach the caregiver about feeding via the gastrostomy tube at home.

The patient has parenteral nutrition infusing with amino acids and dextrose. In report, the oncoming nurse is told that the tubing, the bag, and the dressing were changed 22 hours ago. What care should the nurse coming on be prepared to do (select all that apply)? a. Give the patient insulin. b. Check amount of feeding left in the bag. c. Check that the next bag has been ordered. d. Check the insertion site and change the tubing. e. Check the label to ensure ingredients and solution are as ordered.

Correct answer: a,b,c,e Rationale: The nurse should check the amount of feeding left in the bag, and that the next bag has been ordered to be sure the solution will not run out before the next bag is available. Parenteral nutrition solutions are only good for 24 hours and usually take some time for the pharmacy to mix for each patient. The label on the bag should be checked to ensure that the ingredients and solution are what was ordered. The patient would only receive insulin if the patient is experiencing hyperglycemia and was receiving sliding scale insulin or had diabetes mellitus. The insertion site should be checked, but the tubing is only changed every 72 hours unless lipids are being used.

The nurse should administer an as-needed dose of magnesium hydroxide (MOM) after noting what information while reviewing a patient's medical record? a. Abdominal pain and bloating b. No bowel movement for 3 days c. A decrease in appetite by 50% over 24 hours d. Muscle tremors and other signs of hypomagnesemia

Correct answer: b Rationale: MOM is an osmotic laxative that produces a soft, semisolid stool usually within 15 minutes to 3 hours. This medication would benefit the patient who has not had a bowel movement for 3 days. MOM would not be given for abdominal pain and bloating, decreased appetite, or signs of hypomagnesemia.

The nurse is caring for a postoperative patient who has just vomited yellow green liquid and reports nausea. Which action would be an appropriate nursing intervention? a. Offer the patient an herbal supplement such as ginseng. b. Apply a cool washcloth to the forehead and provide mouth care. c, Take the patient for a walk in the hallway to promote peristalsis. d. Discontinue any medications that may cause nausea or vomiting. TERM

Correct answer: b Cleansing the face and hands with a cool washcloth and providing mouth care are appropriate comfort interventions for nausea and vomiting. Ginseng is not used to treat postoperative nausea and vomiting. Unnecessary activity should be avoided. The patient should rest in a quiet environment. Medications may be temporarily withheld until the acute phase is over, but the medications should not be discontinued without consultation with the health care provider.

The patient receiving chemotherapy rings the call bell and reports the onset of nausea. The nurse should prepare an as-needed dose of which medication? a. Zolpidem b. Ondansetron c. Dexamethasone d. Morphine sulfate

Correct answer: b Ondansetron is a 5-HT3 receptor antagonist antiemetic that is especially effective in reducing cancer chemotherapy-induced nausea and vomiting. Morphine sulfate may cause nausea and vomiting. Zolpidem does not relieve nausea and vomiting. Dexamethasone is usually used in combination with ondansetron for acute and chemotherapy-induced emesis.

The nurse is caring for a 45-year-old woman with a herniated lumbar disc. The patient realizes that weight loss is necessary to lessen back strain. The patient is 5'6" tall and weighs 186 lb (84.5 kg) with a body mass index (BMI) of 28 kg/m2. The nurse explains to the patient that this measurement places her in which of the following weight categories? a. Normal weight b. Overweight c. Obese d. Severely obese

Correct answer: b Rationale: A normal BMI is 18.5 to 24.9 kg/m2, whereas a BMI of 25 to 29.9 kg/m2 is considered overweight. A BMI of 30.0-39.9 is considered obese, and a BMI of 40 or greater is severely obese.

A woman is 5 ft, 6 in (166 cm) tall and weighs 200 lb (90.9 kg) with a waist-to-hip ratio of 0.7. The nurse counsels the patient with the knowledge that the patient is at greatest risk for a. heart disease. b. osteoporosis. c. diabetes mellitus. d. endometrial cancer.

Correct answer: b Rationale: A patient who is obese (BMI of 32.2) but has a waist-to-hip ratio of less than 0.8, indicating gynoid obesity, has an increased risk for osteoporosis. The other conditions are risks associated with android obesity.

When teaching the older adult about nutritional needs during aging, what does the nurse emphasize? a. Need for all nutrients decreases as one ages. b. Fewer calories, but the same or slightly increased amount of protein, are required as one ages. c. Fats, carbohydrates, and protein should be decreased, but vitamin and mineral intake should be increased. d. High-calorie oral supplements should be taken between meals to ensure that recommended nutrient needs are met.

Correct answer: b Rationale: Although calorie intake should be decreased in the older adult because of decreased activity and basal metabolic rate, the need for specific nutrients, such as proteins and vitamins, does not change.

The patient with advanced cirrhosis asks why his abdomen is so swollen. The nurse's response is based on the knowledge that a. a lack of clotting factors promotes the collection of blood in the abdominal cavity. b. portal hypertension and hypoalbuminemia cause a fluid shift into the peritoneal space. c. decreased peristalsis in the GI tract contributes to gas formation and distention of the bowel. d. bile salts in the blood irritate the peritoneal membranes, causing edema and pocketing of fluid. (Lewis 1042)

Correct answer: b Rationale: Ascites is the accumulation of serous fluid in the peritoneal or abdominal cavity and is a common manifestation of cirrhosis. With portal hypertension, proteins shift from the blood vessels through the larger pores of the sinusoids (capillaries) into the lymph space. When the lymphatic system is unable to carry off the excess proteins and water, those substances leak through the liver capsule into the peritoneal cavity. Osmotic pressure of the proteins pulls additional fluid into the peritoneal cavity. A second mechanism of ascites formation is hypoalbuminemia, which results from the inability of the liver to synthesize albumin. Hypoalbuminemia results in decreased colloidal oncotic pressure. A third mechanism is hyperaldosteronism, which occurs when aldosterone is not metabolized by damaged hepatocytes. The increased level of aldosterone causes increases in sodium reabsorption by the renal tubules. Sodium retention and an increase in antidiuretic hormone levels cause additional water retention.

A 80-year-old man states that, although be adds a lot of salt to his food, it still does not have much taste. The nurse's response is based on the knowledge that the older adult a. should not experience changes in taste. b. has a loss of taste buds, especially for sweet and salty. c. has some loss of taste but no difficulty chewing food. d. loses the sense of taste because the ability to smell is decreased.

Correct answer: b Rationale: Older adults have decreased numbers of taste buds and a decreased sense of smell. These age-related changes diminish the sense of taste (especially of salty and sweet substances).

A patient has an elevated blood level of indirect (unconjugated) bilirubin. One cause of this finding is that a. the gallbladder is unable to contract to release stored bile. b. bilirubin is not being conjugated and excreted into the bile by the liver. c. the Kupffer cells in the liver are unable to remove bilirubin from the blood. d. there is an obstruction in the biliary tract preventing flow of bile into the small intestine.

Correct answer: b Rationale: Bilirubin is a pigment derived from the breakdown of hemoglobin and is insoluble in water. Bilirubin is bound to albumin for transport to the liver and is referred to as unconjugated. An indirect bilirubin determination is a measurement of unconjugated bilirubin, and the level may be elevated in hepatocellular and hemolytic conditions.

A patient who has suffered severe burns in a motor vehicle accident will soon be started on parenteral nutrition (PN). Which principle should guide the nurse's administration of the patient's nutrition? a. Administration of PN requires clean technique. b. Central PN requires rapid dilution in a large volume of blood. c. Peripheral PN delivery is preferred over the use of a central line. d. Only water-soluble medications may be added to the PN by the nurse.

Correct answer: b Rationale: Central PN is hypertonic and requires rapid dilution in a large volume of blood. Because PN is an excellent medium for microbial growth, aseptic technique is necessary during administration. Administration through a central line is preferred over the use of peripheral PN, and the nurse may not add any medications to PN.

A patient with acute hepatitis B is being discharged in 2 days. In the discharge teaching plan the nurse should include instructions to a. avoid alcohol for the first 3 weeks. b. use a condom during sexual intercourse. c. have family members get an injection of immunoglobulin. d. follow a low-protein, moderate-carbohydrate, moderate-fat diet. (Lewis 1042)

Correct answer: b Rationale: Hepatitis B virus may be transmitted by mucosal exposure to infected blood, blood products, or other body fluids (e.g., semen, vaginal secretions, saliva). Hepatitis B is a sexually transmitted disease that is acquired through unprotected sex with an infected person. Condom use should be taught to patients to prevent transmission of hepatitis B.

During starvation, the order in which the body obtains substrate for energy is a. visceral protein, skeletal protein, fat, glycogen b. glycogen, skeletal protein, fat stores, visceral protein c. visceral protein, fat stores, glycogen, skeletal protein d. fat stores, skeletal protein, visceral protein, glycogen

Correct answer: b Rationale: Initially, the body selectively uses carbohydrates (e.g., glycogen) rather than fat and protein to meet metabolic needs. These carbohydrate stores, found in the liver and muscles, are minimal and may be totally depleted within 18 hours. After carbohydrate stores are depleted, skeletal protein begins to be converted to glucose for energy. Within 5 to 9 days, body fat is fully mobilized to supply much of the needed energy. In prolonged starvation, up to 97% of calories are provided by fat, and protein is conserved. Depletion of fat stores depends on the amount available, but fat stores typically are used up in 4 to 6 weeks. After fat stores are used, body or visceral proteins, including those in internal organs and plasma, can no longer be spared and rapidly decrease because they are the only remaining body source of energy available.

The health care provider prescribes levothyroxine (Synthroid) for a patient with hypothyroidism. After teaching regarding this drug, the nurse determines that further instruction is needed when the patient says a. "I can expect the medication dose may need to be adjusted." b. "I only need to take this drug until my symptoms are improved." c. "I can expect to return to normal function with the use of this drug." d. "I will report any chest pain or difficulty breathing to the doctor right away."

Correct answer: b Rationale: Levothyroxine (Synthroid) is the drug of choice to treat hypothyroidism. The need for thyroid replacement therapy is usually lifelong.

When medications are used in the treatment of obesity, what is most important for the nurse to teach the patient? a. Over-the-counter (OTC) diet aids are safer than other agents and con be useful in controlling appetite. b. Drugs should be used only as adjuncts to a diet and exercise program as treatment for a chronic condition. c. All drugs used for weight control are capable of altering central nervous system (CNS) function and should be used with caution. d. The primary effect of the medications is psychologic, controlling the urge to eat in response to stress or feelings of rejection.

Correct answer: b Rationale: Medications are used only as adjuncts to diet and exercise programs in the treatment of obesity. Drugs do not cure obesity; without changes in food intake and physical activity, weight gain will occur when the medications are discontinued. The medications used work in a variety of ways to control appetite but over-the-counter drugs are probably the least effective and most abused of these drugs.

A patient has been told that she has elevated liver enzymes caused by nonalcoholic fatty liver disease (NAFLD). The nursing teaching plan should include a. having genetic testing done. b. recommending a heart-healthy diet. c. the necessity to reduce weight rapidly. d. avoiding alcohol until liver enzymes return to normal. (Lewis 1042)

Correct answer: b Rationale: Nonalcoholic fatty liver disease (NAFLD) can progress to liver cirrhosis. There is no definitive treatment, and therapy is directed at reduction of risk factors, which include treatment of diabetes, reduction in body weight, and elimination of harmful medications. For patients who are overweight, weight reduction is important. Weight loss improves insulin sensitivity and reduces liver enzyme levels. No specific dietary therapy is recommended. However, a heart-healthy diet as recommended by the American Heart Association is appropriate.

The nurse admitting a patient for bariatric surgery obtains the following information from the patient. Which finding should be brought to the surgeon's attention before proceeding with further patient preparation? a. History of hypertension b. History of untreated depression c. History of multiple attempts at weight loss d. History of sleep apnea treated with continuous positive airway pressure (CPAP)

Correct answer: b Rationale: Patients with histories of untreated depression or psychosis are not good candidates for surgery. All other historical information includes medical complications of severe obesity that would help to qualify the patient for the surgery.

An older patient was admitted with a fractured hip after being found on the floor of her home. She was extremely malnourished and started on parenteral nutrition (PN) 3 days ago. Which assessment finding would be of most concern to the nurse? a. Blood glucose level of 125 mg/dL b. Serum phosphate level of 1.9 mg/dL c. White blood cell count of 10,500/µL d. Serum potassium level of 4.6 mEq/L

Correct answer: b Rationale: Refeeding syndrome can occur if a malnourished patient is started on aggressive nutritional support. Hypophosphatemia (serum phosphate level less than 2.4 mg/dL) is the hallmark of refeeding syndrome and could result in cardiac dysrhythmias, respiratory arrest, and neurologic problems. An increase in the blood glucose level is expected during the first few days after PN is started. The goal is to maintain a glucose range of 110 to 150 mg/dL. An elevated white blood cell count (greater than 11,000/µL) could indicate an infection. Normal serum potassium levels are between 3.5 and 5.0 mEq/L.

Which patient is at highest risk for complications of obesity? a. A 30-year-old woman who is 5 ft (151 cm) tall, weighs 140 lb (63 kg), and carries weight in her thighs. b. A 56-year-old woman with a BMI of 38 kg/m2, a waist measurement of 38 in (96 cm), and a hip measurement of 36 in (91 cm) c. A 42-year-old man with a waist measurement of 36 in (91 cm) and a hip measurement of 36 in (91 cm) who is 5 ft, 6 in (166 cm) tall and weighs 150 lb (68.2 kg) d. A 68-year-old man with a waist measurement of 38 in (96 cm) and a hip measurement of 42 in (76 cm) who is 5 ft, 11 in (179 cm) tall and weighs 200 lb (90.9 kg)

Correct answer: b Rationale: The 56-year-old woman has a body mass index (BMI) of 38 kg/m2 (obese, Class II) with a waist-to-hip ratio of 1.1 with android obesity and is more at risk (very high) than the other patients. The 30-year-old woman has the least risk with a BMI of 27.3 kg/m2 (overweight) and gynoid shape. The 42-year-old man has a BMI of 24.2 kg/m2 (normal weight) with one risk factor in the waist-to-hip ratio of 1.0 and the 68-year-old man has a BMI of 27.9 kg/m2 (overweight) with a waist-to-hip ratio of 0.9.

A 40-year-old severely obese female patient with type 2 diabetes wants to lose weight. After learning about the surgical procedures, she thinks a combination of restrictive and malabsorptive surgery would be best. Which procedure should the nurse teach her about? a. Lipectomy b. Roux-en-Y gastric bypass c. Adjustable gastric banding d. Vertical sleeve gastrectomy

Correct answer: b Rationale: The Roux-en-Y gastric bypass is a common combination of restrictive (limiting the size of the stomach) and malabsorptive (less food is absorbed) surgery. Lipectomy is used to remove unsightly flabby folds of adipose tissue. Adjustable gastric banding is the most common restrictive procedure. Vertical sleeve gastrectomy is a restrictive procedure that preserves stomach function.

The patient had a car accident and was "scared to death." The patient is now reporting constipation. What affecting the gastrointestinal (GI) tract does the nurse know could be contributing to the constipation? a. The patient is too nervous to eat or drink, so there is no stool. b. The sympathetic nervous system was activated, so the GI tract was slowed. c. The parasympathetic nervous system is now functioning to slow the GI tract. d. The circulation in the GI system has been increased, so less waste is removed.

Correct answer: b Rationale: The constipation is most likely related to the sympathetic nervous system activation from the stress related to the accident. SNS activation can decrease peristalsis. Even without oral intake for a short time, stool will be formed. The parasympathetic system stimulates peristalsis. The circulation to the GI system is decreased with stress.

As gastric contents move into the small intestine, the bowel is normally protected from the acidity of gastric contents by the a. inhibition of secretin release. b. release of bicarbonate by the pancreas. c. release of pancreatic digestive enzymes. d. release of gastrin by the duodenal mucosa.

Correct answer: b Rationale: The hormone secretin stimulates the pancreas to secrete fluid with a high concentration of bicarbonate. This alkaline secretion enters the duodenum and neutralizes acid in the chyme.

The nurse has completed initial instruction with a patient regarding a weight loss program. The nurse determines that the teaching has been effective when the patient makes which statement? a. "I plan to lose 4 lb a week until I have lost the 60-pound goal." b. "I will keep a diary of weekly weights to illustrate my weight loss." c. "I will restrict my carbohydrate intake to less than 30 g/day to maximize weight loss." d."I should not exercise more than my program requires since increased activity increases the appetite."

Correct answer: b Rationale: The patient should monitor and record weight once per week. This prevents frustration at the normal variations in daily weights and may help the patient to maintain motivation to stay on the prescribed diet. Weight loss should occur at a rate of 1 to 2 lb/week. The diet should be well balanced rather than lacking in specific components that may cause an initial weight loss but is not usually sustainable. Exercise is a necessary component of any successful weight loss program.

Which assessment should the nurse prioritize in the care of a patient who has recently begun receiving parenteral nutrition (PN)? a. Skin integrity and bowel sounds b. Electrolyte levels and daily weights c. Auscultation of the chest and tests of blood coagulability d. Peripheral vascular assessment and level of consciousness (LOC)

Correct answer: b Rationale: The use of PN necessitates frequent and thorough assessments. Key focuses of these assessments include daily weights and close monitoring of electrolyte levels. Assessments of bowel sounds, integument, peripheral vascular system, LOC, chest sounds, and blood coagulation may be variously performed, but close monitoring of fluid and electrolyte balance supersedes these in importance.

The nurse is caring for a patient admitted to the hospital for asthma who weighs 186 lb (84.5 kg). During dietary counseling, the patient asks the nurse how much protein he should ingest each day. How many grams of protein does the nurse recommend should be included in the diet based on the patient's current weight? a. 24 b. 41 c. 68 d. 93

Correct answer: c Rationale: The daily intake of protein should be between 0.8 and 1 g/kg of body weight. Thus this patient should take in between 68 and 84 g of protein per day in the diet.

Which female patient is most likely to have metabolic syndrome? a. BP 128/78 mm Hg, triglycerides 160 mg/dL, fasting blood glucose 102 mg/dL b. BP 142/90 mm Hg, high-density lipoproteins 45 mg/dL, fasting blood glucose 130 mg/dL c. Waist circumference 36 in, triglycerides 162 mg/dL, high-density lipoproteins 55 mg/dL d. Waist circumference 32 in, high-density lipoproteins 38 mg/dL, fasting blood glucose 122 mg/dL

Correct answer: b Rationale: Three of the following five measures are needed for a woman to be diagnosed with metabolic syndrome: waist circumference >35 in, triglycerides >150 mg/dL, high-density lipoproteins <50 mg/dL, BP >130 mm Hg systolic or >85 mm Hg diastolic, fasting blood glucose >110 mg/dL. Although the other options have some abnormal measures, none has all three measures in the diagnostic ranges. The criteria for metabolic syndrome for both women and men are listed in Table 41-10.

Which statement about obesity is explained by genetics? a. Older obese patients have exacerbated changes of aging. b. Android body shape and weight gain are influenced by genetics. c. White Americans have a higher incidence of obesity than African Americans. d. Men have a harder time losing weight, as they have more muscle mass than women.

Correct answer: b Rationale: Twin studies and studies with adopted children have shown that body shape and weight gain are influenced by genetics but more research is needed. Older obese people do have exacerbated aging problems related to declines in physical function. African Americans and Hispanics have a higher incidence of obesity than whites. Women have a higher incidence of obesity and more difficulty losing weight than men because women have a higher percentage of metabolically less-active fat.

Inspection of an older patient's mouth reveals the presence of white, curd-like lesions on the patient's tongue. What is the most likely etiology for this abnormal assessment finding? a. Herpesvirus b. Candida albicans c. Vitamin deficiency d. Irritation from ill-fitting dentures

Correct answer: b Rationale: White, curd-like lesions surrounded by erythematous mucosa are associated with oral candidiasis. Herpesvirus causes benign vesicular lesions in the mouth. Vitamin deficiencies may cause a reddened, ulcerated, swollen tongue. Irritation from ill-fitting dentures will cause friable, edematous, painful, bleeding gingivae.

Before administering a bolus of intermittent tube feeding to a patient with a percutaneous endoscopic gastrostomy (PEG), the nurse aspirates 220 mL of gastric contents. How should the nurse respond? a. Return the aspirate to the stomach and recheck the volume of aspirate in an hour. b. Return the aspirate to the stomach and continue with tube feeding as planned. c. Discard the aspirate to prevent over distending the stomach when the new feeding is given. d. Notify the health care provider that the feedings have been scheduled too frequently to allow for stomach emptying.

Correct answer: b Rationale: With intermittent feedings, less than 250 mL residual does not require further action. With continuous feedings and a residual of 250 mL or more after the second residual check, a pro motility agent should be considered.

Which digestive substances are active or activated in the stomach (select all that apply)? a. Bile b. Pepsin c. Gastrin d. Maltase e. Secretin f. Amylase

Correct answer: b, c Rationale: Pepsinogen is changed to pepsin by acidity of the stomach, where it begins to break down proteins. Gastrin stimulates gastric acid secretion and motility and maintains lower esophageal sphincter tone. The stomach also secretes lipase for fat digestion.Bile is secreted by the liver and stored in the gallbladder for emulsifying fats. Maltase is secreted in the small intestine and converts maltose to glucose. Secretin is secreted y the duodenal mucosa and inhibits gastric motility and acid secretion. Amylase is secured in the small intestine and by the pancreas for carbohydrate digestion.

The nurse should recognize that the liver performs which functions (select all that apply) a. Bile storage b. Detoxification c. Protein metabolism d. Steroid metabolism e. Red blood cell (RBC) destruction

Correct answer: b, c, d Rationale: The liver performs multiple major functions that aid in the maintenance of homeostasis. These include metabolism of proteins and steroids as well as detoxification of drugs and metabolic waste products. The Kupffer cells of the liver participate in the breakdown of old RBCs. The liver produces bile, but storage occurs in the gall bladder.

A patient with stage I colorectal cancer is scheduled for surgery. Patient teaching for this patient would include an explanation that a. chemotherapy will begin after the patient recovers from the surgery. b. both chemotherapy and radiation can be used as palliative treatments. c. follow-up colonoscopies will be needed to ensure that the cancer does not recur. d. a wound, ostomy, and continence nurse will visit the patient to identify an abdominal site for the ostomy.

Correct answer: c Rationale: Stage 1 colorectal cancer is treated with surgical removal of the tumor and reanastomosis, and so there is no ostomy. Chemotherapy is not recommended for stage I tumors. Follow-up colonoscopy is recommended because colorectal cancer can recur.

The patient tells the nurse she had a history of abdominal pain, so she had a surgery to make an opening into the common bile duct to remove stones. The nurse knows that this surgery is called a a. colectomy b. cholecystectomy c. choledocholithotomy d. choledochojejunostomy

Correct answer: c Rationale: A choledocholithotomy is an opening into the common bile duct for the removal of stones. A colectomy is the removal of the colon. The cholecystectomy is the removal of the gallbladder. The choledochojejunostomy is an opening between the common bile duct and the jejunum.

What is a normal finding during physical assessment of the mouth? a. A red, slick appearance of the tongue b. Uvular deviation to the side on saying "Ahh" c. A thin, white coating of the dorsum of the tongue d. Scattered red, smooth areas on the dorsum of the tongue

Correct answer: c Rationale: A thin white coating of the dorsum (top) of the tongue is normal. A red, slick appearance is characteristic of cobalamin deficiency and scattered red, smooth areas on the tongue are known as geographic tongue. The uvula should remain in the midline while the patient is saying "Ahh"

The nurse teaches the patient that the best time to take corticosteroids for replacement purposes is a. once a day at bedtime. b. every other day on awakening. c. on arising and in the late afternoon. d. at consistent intervals every 6 to 8 hours.

Correct answer: c Rationale: As replacement therapy, glucocorticoids are usually administered in divided doses: two thirds in the morning and one third in the afternoon. This dosage schedule reflects normal circadian rhythm in endogenous hormone secretion and decreases the side effects associated with corticosteroid replacement therapy.

An important preoperative nursing intervention before an adrenalectomy for hyperaldosteronism is to a. monitor blood glucose levels. b. restrict fluid and sodium intake. c. administer potassium-sparing diuretics. d. advise the patient to make postural changes slowly.

Correct answer: c Rationale: Before surgery, patients should be treated with potassium-sparing diuretics (spironolactone [Aldactone], eplerenone [Inspra]) to normalize serum potassium levels. Spironolactone and eplerenone block the binding of aldosterone to the mineralocorticoid receptor in the terminal distal tubules and collecting ducts of the kidney, thus increasing sodium excretion, water excretion, and potassium retention. Oral potassium supplements may also be necessary.

The nurse recognizes that the majority of patients' caloric needs should come from which source? a. Fats b. Proteins c. Polysaccharides d. Monosaccharides

Correct answer: c Rationale: Carbohydrates should constitute between 45% and 65% of caloric needs, compared with 20% to 35% from fats and 10% to 35% from proteins. Polysaccharides are the complex carbohydrates that are contained in breads and grains. Monosaccharides are simple sugars.

In developing an effective weight reduction plan for an overweight patient who states a willingness to try to lose weight, it is most important for the nurse to first assess which factor? a. The length of time the patient has been obese b. The patient's current level of physical activity c. The patient's social, emotional, and behavioral influences on obesity d. Anthropometric measurements, such as body mass index and skinfold thickness

Correct answer: c Rationale: Eating patterns are established early in life, and eating has many meanings for people. To establish a weight reduction plan that will be successful for the patient, the nurse should first explore the social, emotional, and behavioral influences on the patient's eating patterns. The duration of obesity, current physical activity level, and current anthropometric measurements are not as important for the weight reduction plan.

The health care team is assessing a male patient for acute pancreatitis after he presented to the emergency department with severe abdominal pain. Which laboratory value is the best diagnostic indicator of acute pancreatitis? a. Gastric pH b. Blood glucose c. Serum amylase d. Serum potassium

Correct answer: c Rationale: Elevated serum amylase levels indicate early pancreatic dysfunction and are used to diagnose acute pancreatitis. Serum lipase levels stay elevated longer than serum amylase in acute pancreatitis. Blood glucose, gastric pH, and potassium levels are not direct indicators of acute pancreatic dysfunction.

A patient with a head injury develops SIADH. Manifestations the nurse would expect to find include a. hypernatremia and edema. b. muscle spasticity and hypertension. c. low urine output and hyponatremia. d. weight gain and decreased glomerular filtration rate.

Correct answer: c Rationale: Excess ADH increases the permeability of the renal distal tubule and collecting ducts, which leads to the reabsorption of water into the circulation. Consequently, extracellular fluid volume expands, plasma osmolality declines, the glomerular filtration rate increases, and sodium levels decline (i.e., dilutional hyponatremia). Hyponatremia causes muscle cramping, pain, and weakness. Initially, the patient displays thirst, dyspnea on exertion, and fatigue. Patients with the syndrome of inappropriate antidiuretic hormone secretion (SIADH) experience low urinary output and increased body weight. As the serum sodium level falls (usually to less than 120 mEq/L), manifestations become more severe and include vomiting, abdominal cramps, muscle twitching, and seizures. As plasma osmolality and serum sodium levels continue to decline, cerebral edema may occur, leading to lethargy, anorexia, confusion, headache, seizures, and coma.

What is an indication for parenteral nutrition that is not an appropriate indication for enteral tube feedings? a. Head and neck cancer b. Hypermetabolic states c. Malabsorption syndrome d. Protein-calorie malnutrition

Correct answer: c Rationale: In malabsorption syndrome, foods that are ingested into the intestinal tract cannot be digested or absorbed and tube feedings infused into the intestinal tract would not be absorbed. All of the other conditions can be treated with enteral or parenteral nutrition, depending on the patient's needs.

A patient receives atropine, an anticholinergic drug, in preparation for surgery. The nurse expects this drug to affect the GI tract by doing what? a. Increasing gastric emptying b. Relaxing pyloric and ileocecal sphincters c. Decreasing secretions and peristaltic action d. Stimulation the nervous system of the GI tract

Correct answer: c Rationale: The parasympathetic nervous system stimulates activity of the gastrointestinal (GI) tract, increasing motility and secretions and relaxing sphincters to promote movement of contents. A drug that blocks this activity decreases secretions and peristalsis, slows gastric emptying, and contracts sphincters. The enteric nervous system of the GI tract is modulated by sympathetic and parasympathetic influence.

The nurse is evaluating the nutritional status of a 55-year-old man who is undergoing radiation treatment for oropharyngeal cancer. Which laboratory test would be the best indicator to determine if the patient has protein-calorie malnutrition? a. Serum transferrin b. C-reactive protein c. Serum prealbumin d. Alanine transaminase (ALT)

Correct answer: c Rationale: In the absence of an inflammatory condition, the best indicator of protein-calorie malnutrition (PCM) is prealbumin; prealbumin is a protein synthesized by the liver and indicates recent or current nutritional status. Decreased albumin and transferrin levels are other indicators that protein is deficient. C-reactive protein (CRP) is elevated during inflammation and is used to determine if prealbumin, albumin, and transferrin are decreased related to protein deficiency or an inflammatory process. Other indicators of protein deficiency include elevated serum potassium levels, low red blood cell counts and hemoglobin levels, decreased total lymphocyte count, elevated liver enzyme levels (ALT), and decreased levels of both fat-soluble and water-soluble vitamins.

What is the main underlying risk factor for metabolic syndrome? a. Age b. Heart disease c. Insulin resistance d. High cholesterol levels

Correct answer: c Rationale: Insulin resistance is the main underlying risk factor for metabolic syndrome. Aging is associated with metabolic syndrome. High cholesterol, hypertension, and increased clotting risk are characteristics of metabolic syndrome.

Which statement accurately describes vitamin deficiencies? a. The two nutrients most often lacking in the diet of a vegan are vitamin B6 and folic acid. b. Vitamin imbalances occur frequently in the United States because of excessive fat intake. c. Surgery on the GI tract may contribute to vitamin deficiencies because of impaired absorption. d. Vitamin deficiencies in adults most commonly are clinically manifested by disorders of the skin.

Correct answer: c Rationale: Patients who have surgery on the GI tract may be at risk for vitamin deficiencies because of inability to absorb or metabolize them. The strict vegan diet most often lacks cobalamin (vitamin B12) and iron. Although the high intake of fat is a major nutritional problem in the United States, vitamin deficiencies are rare in developed countries except in those with eating disorders or chronic alcohol abusers. Some vitamin deficiencies in adults have neurologic manifestations.

The nurse monitors the laboratory results of the patient with protein-calorie malnutrition during treatment. Which result is an indication of improvement in the patient's condition? a. Decreased lymphocytes b. Increased serum potassium c. Increased serum transferrin d. Decreased serum prealbumin

Correct answer: c Rationale: Serum transferrin is a protein that is synthesized by the liver and used for iron transport and decreases when there is protein deficiency. An increase in the protein would indicate a more positive nitrogen balance with amino acids available for synthesis. Decreased lymphocytes and serum prealbumin are indicators of protein depletion and increased serum potassium shows continuing failure of the sodium-potassium pump.

During care of the severely obese patient, what is most important for the nurse to do? a. Avoid reference to the patient's weight to avoid embarrassing the patient. b. Emphasize to the patient how important it is to lose weight to maintain health. c. Plan for necessary modifications in equipment and nursing techniques before initiating care. d. Recognize that a full assessment of each body system might not be possible because of numerous layers of skinfolds.

Correct answer: c Rationale: Special considerations are needed for the care of the severely obese patient because most hospital units are not prepared with beds, chairs, BP cuffs, and other equipment that will need to be used with the very obese patient. Consideration of all aspects of care should be made before implementing care for the patient, including extra time and perhaps assistance for positioning, physical assessment, and transferring the patient.

When caring for the patient with heart failure, the nurse knows that which gastrointestinal process is most dependent on cardiac output and may affect the patient's nutritional status? a. Ingestion b. Digestion c. Absorption d. Elimination

Correct answer: c Rationale: Substances that interface with the absorptive surfaces of the GI tract (primarily in the small intestine) diffuse across the intestinal membranes into intestinal capillaries and are then carried to other parts of the body for use in energy production. The cardiac output provides the blood flow for this absorption of nutrients to occur.

How will an obstruction at the ampulla of Vater affect the digestion of all nutrients? a. Bile is responsible for emulsification of all nutrients and vitamins. b. Intestinal digestive enzymes are released through the ampulla of Vater. c. Both bile and pancreatic enzymes enter the duodenum at the ampulla of Vater. d. Gastric contents can ply pass to the duodenum when the ampulla of Vater is open.

Correct answer: c Rationale: The ampulla of Vater is the site where the pancreatic duct and common bile duct enter the duodenum and the opening and closing of the ampulla is controlled by the sphincter of Oddi. Because bile from the common bile duct is needed for emulsification of fat to promote digestion and pancreatic enzymes from the pancreas are needed for digestion of all nutrients, a blockage at this point would affect the digestion of all nutrients. Gastric contents pass into the duodenum through the pylorus or pyloric valve.

A patient's serum liver enzyme tests reveal an elevated aspartate aminotransferase (AST). The nurse recognizes what about the elevated AST? a. It eliminates infection as a cause of liver damage. b. It is diagnostic for liver inflammation and damage. c. Tissue damage in organs other than the liver may be identified. d. Nervous system symptoms related to hepatic encephalopathy may be the cause.

Correct answer: c Rationale: The aspartate aminotransferase (AST) level is elevated in liver disease but it is important to note that it is also elevated in damage to the heart and lungs and is not a specific test for liver function. Measurements of most of the transaminases involves nonspecific tests unless isoenzyme fractions are determined. Hepatic encephalopathy is related to elevated ammonia levels.

The nurse is assessing a 50-year-old woman admitted with a possible bowel obstruction. Which assessment finding would be expected in this patient? a. Tympany to abdominal percussion b. Aortic pulsation visible in epigastric region c. High-pitched sounds on abdominal auscultation d. Liver border palpable 1 cm below the right costal margin

Correct answer: c Rationale: The bowel sounds are more high pitched (rushes and tinkling) when the intestines are under tension, as in intestinal obstruction. Bowel sounds may also be diminished or absent with an intestinal obstruction. Normal findings include aortic pulsations on inspection and tympany with percussion, and the liver may be palpable 1 to 2 cm along the right costal margin.

When the nurse is assessing the health perception-health maintenance pattern as related to GI function, an appropriate question to ask is a. "What is your usual bowel elimination pattern?" b. "What percentage of your income is spent on food?" c. "Have you traveled to a foreign country in the last year?" d. "Do you have diarrhea when you are under a lot of stress?"

Correct answer: c Rationale: When assessing gastrointestinal function in relation to the health perception-health management pattern, the nurse should ask the patient about recent foreign travel with possible exposure to hepatitis, parasitic infestation, or bacterial infection.

Which explanation about weight reduction should be included when teaching the obese patient and her obese husband? a. Weight gain is caused by psychologic factors. b. Daily weighing is recommended to monitor weight loss. c. Fat is not burned until the glycogen-water pool is depleted. d. Men lose weight less quickly than women because they have a higher percentage of metabolically less-active fat.

Correct answer: c Rationale: With reducing diets that severely restrict carbohydrates, the body's glycogen stores become depleted within a few days. The glycogen normally binds to water in fat cells and it is this water loss that causes weight loss in the first few days. Fat is not burned until the glycogen-water pool is depleted. Although psychosocial components (i.e., using food for comfort or reward and inability to buy high-nutritional quality food) may have an influence on weight gain, these factors along with lack of physical exercise, underestimation of portion size, and genetics contribute to weight gain. Weekly weighing is recommended as a more reliable indicator of weight loss because daily weighing shows frequent fluctuation from retained water (including urine) and elimination of feces. Men are able to lose weight more quickly than women because women have a higher percentage of metabolically less-active fat.

Which nursing actions are indicated for a liver biopsy (select all that apply)? a. Observe for white stools b. Monitor for rectal bleeding c. Monitor for internal bleeding d. Position to right side after test e. Ensure bowel preparation was done f. Check coagulation status before test

Correct answer: c, d, f Rationale: Because the liver is a vascular organ, vital signs are monitored to assess for internal bleeding. Prevention of bleeding is the reason for positioning on the right side for at least 2 hours and for splinting the puncture site. Again, because of the vasculature of the liver, coagulation status is checked before the biopsy is done. White stools occur with upper gastrointestinal (UGI) or barium swallow tests. No smoking is to be done after midnight before the study with an UGI. The bowel must be cleared before a lower GI or barium enema, a virtual colonoscopy, or a colonoscopy. Rectal bleeding may occur with a sigmoidoscopy or colonoscopy. A perforation may occur with an esophagogastroduodenoscopy (EGD), ERCP, or peritoneoscopy.

When teaching a patient about weight reduction diets, the nurse teaches the patient that an appropriate single serving of a food is a. a 6-inch bagel. b. 1 cup of chopped vegetables. c. a piece of cheese the size of three dice. d. a chicken breast the size of a deck of cards.

Correct answer: d Rationale: A chicken breast the size of a deck of cards is about 3 oz, a recommended portion size of meat. Other normal portions include a 3-inch bagel, 1/2 cup of chopped vegetables, and a piece of cheese the size of six dice.

Which patient is at highest risk for developing metabolic syndrome? a. A 62-year-old white man who has coronary artery disease with chronic stable angina b. A 54-year-old Hispanic woman who is sedentary and has nephrogenic diabetes insipidus c. A 27-year-old Asian American woman who has preeclampsia and gestational diabetes mellitus d. A 38-year-old Native American man who has diabetes mellitus and elevated hemoglobin A1C

Correct answer: d Rationale: African Americans, Hispanics, Native Americans, and Asians are at an increased risk for development of metabolic syndrome. Other risk factors include individuals who have diabetes that cannot maintain a normal glucose level, have hypertension, and secrete a large amount of insulin, or who have survived a heart attack and have hyperinsulinemia.

In the immediate postoperative period a nurse cares for a severely obese 72-year-old man who had surgery for repair of a lower leg fracture. Which assessment would be most important for the nurse to make? a. Cardiac rhythm b. Surgical dressing c. Postoperative pain d. Oxygen saturation

Correct answer: d Rationale: After surgery an older and/or severely obese patient should be closely monitored for oxygen desaturation. The body stores anesthetics in adipose tissue, placing patients with excess adipose tissue (e.g., obesity, older) at risk for resedation. As adipose cells release anesthetic back into the bloodstream, the patient may become sedated after surgery. This may depress the respiratory rate and result in a drop in oxygen saturation.

To evaluate the effect of nutritional interventions for a patient with protein-calorie malnutrition, what is the best indicator for the nurse to use? a. Height and weight b. Body mass index (BMI) c. Weight in relation to ideal body weight d. Mid-upper arm circumference and triceps skinfold

Correct answer: d Rationale: Anthropometric measurements, including mid-upper arm circumference and triceps skinfold measurements, are good indicators of lean body mass and skeletal protein reserves and are valuable in evaluating persons who may have been or are being treated for acute protein malnutrition. The other measurements do not specifically address muscle mass.

At the first visit to the clinic, the female patient with a BMI of 29 kg/m2 tells the nurse that she does not want to become obese. Which question used for assessing weight issues is the most important question for the nurse to ask? a. "What factors contributed to your current body weight?" b. "How is your overall health affected by your body weight?" c. "What is your history of gaining weight and losing weight?" d. "In what ways are you interested in managing your weight differently?"

Correct answer: d Rationale: Asking the patient about her desire to manage her weight in a different manner helps the nurse determine the patient's readiness for learning, degree of motivation, and willingness to change lifestyle habits. The nurse can help the patient set realistic goals. This question will also lead to discussing the patient's history of gaining and losing weight and factors that have contributed to the patient's current weight. The patient may be unaware of the overall health effects of her body weight, so this question is not helpful at this time.

After eating, a patient with an inflamed gallbladder experiences pain caused by contraction of the gallbladder. What is the mechanism responsible for this action? a. Production of bile by the liver b. Production of secretin by the duodenum c. Release of gastrin from the stomach antrum d. Production of cholecystokinin by the duodenum

Correct answer: d Rationale: Cholecystokinin is secreted by the duodenal mucosa when fats and amino acids enter the duodenum and stimulate the gallbladder to release bile to emulsify the fats for digestion. The bile is produced by the liver but stored in the gallbladder. Secretin is responsible for stimulating pancreatic bicarbonate secretion and gastrin increases gastric motility and acid secretion.

A severely obese patient has undergone Roux-en-Y gastric bypass surgery. In planning postoperative care, the nurse anticipates that the patient a. may have severe diarrhea early in the postoperative period. b. will not be allowed to ambulate for 1 to 2 days postoperatively. c. will require nasogastric suction until the incision heals. d. may have only liquids orally, and in very limited amounts, during the postoperative period.

Correct answer: d Rationale: During the immediate postoperative period, water and sugar-free clear liquids are given (30 mL every 2 hours while the patient is awake).

What is the most common cause of secondary protein-calorie malnutrition in the United States? a. The unavailability of foods high in protein b. A lack of knowledge about nutritional needs c. A lack of money to purchase high-protien foods d. An alteration in ingestion, digestion, absorption, or metabolism

Correct answer: d Rationale: In the United States, where rote in intake is high and of good quality, protein-calorie malnutrition most commonly results from problems of the GI system. In developing countries, adequate food sources might not exist, the inhabitants may not be well educated about nutritional needs, and economic conditions can prevent purchase of balanced diets.

The obesity classification that is most often associated with cardiovascular health problems is a. primary obesity. b. secondary obesity. c. gynoid fat distribution. d. android fat distribution.

Correct answer: d Rationale: Individuals with fat located primarily in the abdominal area (i.e., whose body is apple-shaped) are at greater risk for obesity-related complications (e.g., heart disease) than are those whose fat is primarily located in the upper legs (i.e., whose body is pear-shaped). Individuals whose fat is distributed over the abdomen and upper body (i.e., neck, arms, and shoulders) are classified as having android obesity.

The nurse is reviewing the home medication list for a 44-year-old man admitted with suspected hepatic failure. Which medication could cause hepatotoxicity? a. Nitroglycerin b. Digoxin (Lanoxin) c. Ciprofloxacin (Cipro) d. Acetaminophen (Tylenol)

Correct answer: d Rationale: Many chemicals and drugs are potentially hepatotoxic (see Table 39-6) and result in significant patient harm unless monitored closely. For example, chronic high doses of acetaminophen and nonsteroidal antiinflammatory drugs (NSAIDs) may be hepatotoxic.

After thyroid surgery, the nurse suspects damage or removal of the parathyroid glands when the patient develops a. muscle weakness and weight loss. b. hyperthermia and severe tachycardia. c. hypertension and difficulty swallowing. d. laryngospasms and tingling in the hands and feet.

Correct answer: d Rationale: Painful tonic spasms of smooth and skeletal muscles can cause laryngospasms that may compromise breathing. These spasms may be related to tetany, which occurs if the parathyroid glands are removed or damaged during surgery, which leads to hypocalcemia.

The nurse has completed initial instruction with a patient regarding a weight-loss program. Which patient comment indicates to the nurse that the teaching has been effective? a. "I will keep a diary of daily weight to illustrate my weight loss." b. "I plan to lose 4 lb a week until I have lost the 60 lb I want to lose." c. "I should not exercise more than what is required so I don't increase my appetite." d. "I plan to join a behavior modification group to help establish long-term behavior changes."

Correct answer: d Rationale: People who have undergone behavior therapy are more successful in maintaining weight losses over time because most programs deemphasize the diet, focus on how and when the person eats and education, and provide support from others. Weighing daily is not recommended and plateaus may not allow for consistent weight loss. A goal for weight loss must be set and 1 to 2 pounds a week is realistic. A more rapid loss often causes skin and underlying tissue to lose elasticity and become flabby folds of tissue. Exercising more often depresses appetite and exercise need not be limited.

A patient is admitted to the hospital with a diagnosis of diarrhea with dehydration. The nurse recognizes that increased peristalsis resulting in diarrhea can be related to a. sympathetic inhibition. b. mixing and propulsion. c. sympathetic stimulation. d. parasympathetic stimulation.

Correct answer: d Rationale: Peristalsis is increased by parasympathetic stimulation.

The nurse is reviewing the laboratory test results for a 71-year-old patient with metastatic lung cancer. The patient was admitted with a diagnosis of malnutrition. The serum albumin level is 4.0 g/dL, and prealbumin is 10 mg/dL. What should this indicate to the nurse? a. The albumin level is normal, and therefore the patient does not have protein malnutrition. b. The albumin level is increased, which is a common finding in patients with cancer who have malnutrition. c. Both the serum albumin and prealbumin levels are reduced, consistent with the admitting diagnosis of malnutrition. d. Although the serum albumin level is normal, the prealbumin level more accurately reflects the patient's nutritional status.

Correct answer: d Rationale: Prealbumin has a half-life of 2 days and is a better indicator of recent or current nutritional status. Serum albumin has a half-life of approximately 20 to 22 days. The serum level may lag behind actual protein changes by more than 2 weeks and is therefore not a good indicator of acute changes in nutritional status.

A patient with anorexia nervosa shows signs of malnutrition. During initial referring, the nurse carefully assesses the patient for a. hyperkalemia. b. hypoglycemia. c. hypercalcemia. d. hypophosphatemia.

Correct answer: d Rationale: Refeeding syndrome is characterized by fluid retention, electrolyte imbalances (e.g., hypophosphatemia, hypokalemia, hypomagnesemia), and hyperglycemia. Conditions that predispose patients to refeeding syndrome include long-standing malnutrition states such as those induced by chronic alcoholism, vomiting and diarrhea, chemotherapy, and major surgery. Refeeding syndrome can occur any time a malnourished patient is started on aggressive nutritional support. Hypophosphatemia is the hallmark of refeeding syndrome, and it is associated with serious outcomes, including cardiac dysrhythmias, respiratory arrest, and neurologic disturbances (e.g., paresthesias).

The best nutritional therapy plan for a person who is obese is a. the Zone diet. b. the Atkins diet. c. Sugar Busters. d. foods from the basic food groups.

Correct answer: d Rationale: Restricted food intake is a cornerstone for any weight loss or maintenance program. A good weight loss plan should include foods from the basic food groups.

Which patient has the highest morbidity risk? a. Male 6 ft. 1 in. tall, BMI 29 kg/m2 b. Female 5 ft. 6 in. tall, weight 150 lb. c. Male with waist circumference 46 in. d. Female 5 ft. 10 in. tall, obesity Class III

Correct answer: d Rationale: The patient in Class III obesity has the highest risk for disease because Class III denotes severe obesity or a BMI greater than 40 kg/m2. The patient with waist circumference 46 in. has a high risk for disease, but without the BMI or obesity class, a more precise determination cannot be made. The female who is 5 ft. 6 in. tall has a normal weight for her height. The male patient who is over 6 ft. tall is overweight, which increases his risk of disease, but a more precise determination cannot be made without the waist circumference.

What is a postoperative nursing intervention for the obese patient who has undergone bariatric surgery? a. Irrigating and repositioning the nasogastric (NG) tube as needed b. Delaying ambulation until the patient has enough strength to support self c. Keeping the patient positioned on the side to facilitate respiratory function d. Providing adequate support to the incision during coughing, deep breathing, and turning

Correct answer: d Rationale: Turning, coughing, and deep breathing are essential to prevent postoperative complications. Protecting the incision from strain is important since wound dehiscence is a problem for obese patients. If a nasogastric (NG) tube that is present following gastric surgery for severe obesity becomes blocked or needs repositioning, the health care provider should be notified. Ambulation is usually started on the evening of surgery and addition help will be needed to support the patient. Respiratory function is promoted by keeping the head of the bed elevated at an angle of 35 to 40 degrees.

A 68-year-old patient is in the office for a physical. She notes that she no longer has regular bowel movements. Which suggestion by the nurse would be most helpful to the patient? a. Take an additional laxative to stimulate defecation. b. Eat less acidic foods to enable the gastrointestinal system to increase peristalsis. c. Eat less food at each meal to prevent feces from backing up related to slowed peristalsis. d. Attempt defecation after breakfast because gastrocolic reflexes increase colon peristalsis at that time.

Correct answer: d Rationale: When food inters the stomach and duodenum, the gastrocolic and duodenocolic reflexes are initiated and are more active after the first daily meal. Additional laxatives or laxative abuse contribute to constipation in older adults. Decreasing food intake is not recommended, as many older adults have a decreased appetite. Fibre and fluids should be increased.

The ED nurse has inspected, auscultated, and palpated the abdomen with no obvious abnormalities, except pain. When the nurse palpates the abdomen for rebound tenderness, there is severe pain. The nurse should know that this could indicate what problem? a. Hepatic cirrhosis b. Hypersplenomegaly c. Gall bladder distention d. Peritoneal inflammation

Correct answer: d Rationale: When palpating for rebound tenderness, the problem area of the abdomen will produce pain and severe muscle spasm when there is peritoneal inflammation. Hepatic cirrhosis, hypersplenomegaly, and gall bladder distention do not manifest with rebound tenderness.

This bariatric surgical procedure involves creating a stoma and gastric pouch that is reversible, and no malabsorption occurs. What surgical procedure is this? a. Vertical gastric banding b. Biliopancreatic diversion c. Roux-en-Y gastric bypass d. Adjustable gastric banding

Correct answer: d Rationale: With adjustable gastric banding (AGB), the stomach size is limited by an inflatable band placed around the fundus of the stomach. The band is connected to a subcutaneous port and can be inflated or deflated to change the stoma size to meet the patient's needs as weight is lost. The procedure is performed laparoscopically and, if necessary, can be modified or reversed after the initial procedure.

Wich of the following criteria must be met for a diagnosis of metabolic syndrome (select all that apply)? a. Hypertension b. Elevated triglycerides c. Elevated plasma glucose d. Increased waist circumference e. Decreased low-density lipoproteins

Correct answers: a, b, c, d Rationale: Three of the following five criteria must be met for a diagnosis of metabolic syndrome: • Waist circumference of 40 inches or more in men and 35 inches or more in women • Triglyceride levels higher than 150 mg/dL, or need for drug treatment for elevated triglyceride levels • High-density lipoprotein (HDL) cholesterol levels lower than 40 mg/dL in men and lower than 50 mg/dL in women, or need for drug treatment for reduced HDL cholesterol levels • Blood pressure: 130 mm Hg or higher systolic or 85 mm Hg or higher diastolic, or need for drug treatment for hypertension • Fasting blood glucose level of 110 mg/dL or higher, or need for drug treatment for elevated glucose levels

When a 35-year-old female patient is admitted to the emergency department with acute abdominal pain, which possible diagnosis should you consider that may be the cause of her pain (select all that apply)? a. Gastroenteritis b. Ectopic pregnancy c. Gastrointestinal bleeding d. Irritable bowel syndrome e. Inflammatory bowel disease

Correct answers: a, b, c, d, e Rationale: All these conditions could cause acute abdominal pain.

Normally, which hormones and peptides affect appetite (select all that apply)? a. Leptin b. Insulin c. Ghrelin d. Peptide YY e. Neuropeptide Y f. Cholecystokinin

Correct answers: a, b, c, d, e, f Rationale: Normally ghrelin and neuropeptide Y stimulate appetite. Leptin suppresses appetite and hunger. Insulin decreases appetite. Peptide YY and cholecystokinin inhibit appetite by slowing gastric emptying and sending satiety signals to the hypothalamus.

A normal physical assessment finding of the GI system is/are (select all that apply) a. nonpalpable liver and spleen. b. borborygmi in upper right quadrant. c. tympany on percussion of the abdomen. d. liver edge 2 to 4 cm below the costal margin. e. finding of a firm, nodular edge on the rectal examination.

Correct answers: a, c Rationale: Normal assessment findings for the gastrointestinal system include a nonpalpable liver and spleen and generalized tympany on percussion. Normally, bowel sounds are high pitched and gurgling; loud gurgles indicate hyperperistalsis and are called borborygmi (stomach growling). If the patient has chronic obstructive pulmonary disease, large lungs, or a low-set diaphragm, the liver may be palpated 0.4 to 0.8 inch (1 to 2 cm) below the right costal margin. On palpation, the rectal wall should be soft and smooth and should have no nodules.

Health risks associated with obesity include (select all that apply) a. colorectal cancer. b. rheumatoid arthritis. c. polycystic ovary syndrome. d. nonalcoholic steatohepatitis. e. systemic lupus erythematosus.

Correct answers: a, c, d Rationale: Health risks associated with obesity include cardiovascular disease (related to increased low-density lipoprotein levels, increased triglyceride levels, and decreased high-density lipoprotein levels), hypertension, sleep apnea, obesity hypoventilation syndrome, reduced chest wall compliance, increased work of breathing, decreased total lung capacity and functional residual capacity, type 2 diabetes mellitus (i.e., hyperinsulinemia and insulin resistance), osteoarthritis, hyperuricemia, gout, gastroesophageal reflux disease, gallstones, nonalcoholic steatohepatitis, fatty liver and cirrhosis, cancer (mainly breast, endometrial, kidney, colorectal, pancreas, esophagus, and gallbladder), psychosocial problems (employment, education, and health care), low self-esteem, withdrawal from social interactions, and major depression.

Which teaching points are important when providing information to a patient with metabolic syndrome (select all that apply)? a. Stop smoking. b. Monitor weight daily. c. Increase level of activity. d. Decrease saturated fat intake. e. Reduce weight and maintain lower weight. f. Check blood glucose each morning prior to eating.

Correct answers: a, c, d, e Rationale: Patients with metabolic syndrome need to lower their risk factors by reducing and maintaining weight, increasing physical activity, establishing healthy diet habits, and smoking cessation. Some patients with metabolic syndrome are diabetic and would need to monitor glucose levels frequently. When monitoring weight reduction, it is recommended to check weight weekly, not daily.

A community health nurse is conducting an initial assessment of a new patient. Which assessments should the nurse include when screening the patient for metabolic syndrome (select all that apply)? a. Blood pressure b. Resting heart rate c. Physical endurance d. Waist circumference e. Fasting blood glucose

Correct answers: a, d, e Rationale: The diagnostic criteria for metabolic syndrome include elevated blood pressure, fasting blood glucose, waist circumference, triglycerides, and HDL cholesterol. Resting heart rate and physical endurance are not part of the diagnostic criteria.

Important nursing intervention(s) when caring for a patient with Cushing syndrome include (select all that apply) a. restricting protein intake. b. monitoring blood glucose levels. c. observing for signs of hypotension. d. administering medication in equal doses. e. protecting patient from exposure to infection.

Correct answers: b, e Rationale: Hyperglycemia occurs with Cushing disease because of glucose intolerance (associated with cortisol-induced insulin resistance) and increased gluconeogenesis by the liver. High levels of corticosteroids increase susceptibility to infection and delay wound healing.

What characteristics describe adjustable gastric banding (select all that apply)? a. 85% of the stomach is removed. b. Stomach restriction can be reversed. c. Eliminates hormones that stimulate hunger. d. Malabsorption of fat-soluble vitamins occurs. e. Inflatable band allows for modification of gastric stoma size. f. Stomach with a gastric pouch surgically anastomosed to the jejunum.

Correct answers: b, e Rationale: The adjustable gastric banding procedure is reversible and allows a change in gastric stoma size by inflation or deflation of the band around the funds of the stomach. The vertical sleeve gastrectomy removes 85% of the stomach and eliminates the hormones produced in the stomach that stimulate hunger. The biliopancreatic diversion is a maladaptive surgery that prevents absorption of nutrients, including fat-soluble vitamins. The Roux-en-Y gastric bypass reduces the stomach size with a gastric pouch anastomosed to the small intestine, so it is both restrictive and malabsorptive.

Which clinical manifestations of inflammatory bowel disease are common to both patients with ulcerative colitis (UC) and Crohn's disease (select all that apply)? a. Restricted to rectum b. Strictures are common. c. Bloody, diarrhea stools d. Cramping abdominal pain e. Lesions penetrate intestine.

Correct answers: c, d Rationale: Clinical manifestations of UC and Crohn's disease include bloody diarrhea, cramping abdominal pain, and nutritional disorders. Intestinal lesions associated with UC are usually restricted to the rectum before moving into the colon. Lesions that penetrate the intestine or cause strictures are characteristic of Crohn's disease.

In preparing to care for the obese patient with cancer, what physiologic problems is this patient at a greater risk for having (select all that apply)? a. Tinnitus b. Fractures c. Sleep apnea d. Trousseau's sign e. Type 2 diabetes mellitus f. Gastroesophageal reflux disease (GERD)

Correct answers: c, e, f Rationale: Obese patients are at a higher risk for cancer, sleep apnea and sleep deprivation, type 2 diabetes mellitus, gastroesophageal reflux disease (GERD), nonalcoholic steatohepatits, osteoarthritis, and cardiovascular problems. The other options are not related to obesity.

An appropriate question to ask the patient with painful menstruation to differentiate primary from secondary dysmenorrhea is: A. "does your pain become worse with activity or overexcertion?" B. "have you had a recent personal crisis or change in your lifestyle?" C. "is your pain relieved by NSAIDS?" D. "when in your menstrual history did the pain with your period begin?"

D

A 72-year-old patient was admitted with epigastric pain due to a gastric ulcer. Which patient assessment warrants an urgent change in the nursing plan of care? a) Chest pain relieved with eating or drinking water b) Back pain 3 or 4 hours after eating a meal c) Burning epigastric pain 90 minutes after breakfast d) Rigid abdomen and vomiting following indigestion

D A rigid abdomen with vomiting in a patient who has a gastric ulcer indicates a perforation of the ulcer, especially if the manifestations of perforation appear suddenly. Midepigastric pain is relieved by eating, drinking water, or antacids with duodenal ulcers, not gastric ulcers. Back pain 3-4 hours after a meal is more likely to occur with a duodenal ulcer. Burning epigastric pain 1-2 hours after a meal is an expected manifestation of a gastric ulcer related to increased gastric secretions and does not cause an urgent change in the nursing plan of care.

The patient has an obstruction high in the small intestine. What patient assessment do you anticipate finding? A. No bowel sounds B. Metabolic acidosis C. Flank pain D. Vomiting

D Rationale A patient with a high small intestinal obstruction is likely to have vomiting, which can be profuse. Lower intestinal obstruction is associated with a greater risk of metabolic acidosis. In small intestinal obstructions bowel sounds can still be heard in the large intestine. Flank pain is typically related to renal calculi. Reference: 1032

Why are adenomatous polyps removed during a colonoscopy? A. They eventually cause intestinal obstruction. B. They are prone to bleeding and lead to anemia. C. They lead to familial adenomatous polyposis (FAP). D. They are closely linked to colorectal cancer.

D Rationale Adenomatous polyps are neoplastic; 85% of colorectal adenocarcinomas arise from them. Removing adenomatous polyps decreases the occurrence of colorectal cancer. All polyps are considered abnormal and should be removed. FAP is a genetic disorder that is autosomal dominant and not related to the typical appearance of adenomatous polyps. Reference: 1034

You are conducting a community education session. Which option is correct information related to diverticulosis? A. It is commonly seen in young people. B. The classic presentation is right-sided abdominal pain. C. Adequate protein can prevent its occurrence. D. It is a result of aging and decreased stool size.

D Rationale Although the exact cause is unknown, diverticulosis is thought to result from high intraluminal pressure on weakened areas of the bowel, often resulting from inadequate fiber. The decreased stool raises intraluminal pressure. Diverticular disease is common, and the incidence increases with age. In Western civilization, the classic anatomic location of diverticulum is in the sigmoid colon or left-sided abdomen. It usually is asymptomatic unless inflammation (diverticulitis) is present. Preventive measures include increasing the bulk in diet (fresh fruits and vegetables) and decreasing intake of red meats and fats. Reference: 1046

Which is the best method for evaluation and treatment of large intestine polyps? A. Sigmoidoscopy B. Barium enema C. Digital examination D. Colonoscopy

D Rationale Colonoscopy is preferred because it allows evaluation of the total colon and polyps can be immediately removed. Only polyps in the distal colon and rectum can be detected and removed during sigmoidoscopy. Polyps can be detected but not removed during barium enema and radiography. Digital examination is used for prostate evaluation but not for the diagnosis of colon polyps because it only assesses the rectal area and not the colon. Reference: 1034

You are caring for a 68-year-old patient admitted with abdominal pain, nausea, and vomiting. The patient has an abdominal mass and a bowel obstruction is suspected. You are auscultating the abdomen listening for which types of bowel sounds that are consistent with the patient's clinical picture? A. Low pitched and rumbling above the area of obstruction B. High pitched and hypoactive below the area of obstruction C. Low pitched and hyperactive below the area of obstruction D. High pitched and hyperactive above the area of obstruction

D Rationale Early in intestinal obstruction, the patient's bowel sounds are hyperactive and high pitched, sometimes referred to as "tinkling" above the level of the obstruction. This occurs because peristaltic action increases to push past the area of obstruction. As the obstruction becomes complete, bowel sounds decrease and finally become absent. Reference: 1033

Which finding is most important to monitor in a patient with Crohn's disease? A. Elevated WBC level B. Frequent diarrhea C. Abdominal cramping D. Brown discharge in urine

D Rationale Fistulas can develop between bowel and bladder. Stool in the urine and urinary tract infections (UTIs) are the signs of a fistula. The other options are expected findings during an exacerbation of the condition. Reference: 1023, 1025

During the assessment of a patient with acute abdominal pain, what should you do? A. Perform deep palpation before auscultation. B. Obtain blood pressure and pulse rate to determine hypervolemic changes. C. Auscultate bowel sounds because hyperactive bowel sounds suggest paralytic ileus. D. Measure body temperature because an elevated temperature may indicate an inflammatory or infectious process.

D Rationale For the patient complaining of acute abdominal pain, you should take vital signs immediately. Increased pulse and decreasing blood pressure are indicative of hypovolemia. An elevated temperature suggests an inflammatory or infectious process. Intake and output measurements provide essential information about the adequacy of vascular volume. Inspect the abdomen first, and then auscultate bowel sounds. Palpation is performed next and should be gentle. Reference: 1016

Which discharge teaching should you provide to a patient with a herniorrhaphy for an inguinal hernia repair? A. Wear a truss continually. B. Call the primary provider if scrotal edema occurs. C. Cough frequently to prevent atelectasis. D. Do not do heavy lifting for 6 to 8 weeks.

D Rationale Heavy lifting should be avoided for 6 to 8 weeks after repair. Preoperatively a truss is used by some people to prevent a hernia from protruding. It is not worn after surgery. Scrotal edema is a painful but not serious complication after an inguinal hernia repair. Coughing is not encouraged because it increases pressure. The patient should be encouraged to do deep breathing and turning instead. Reference: 1048

What is the most common symptom of an acute abdominal problem? A. Nausea B. Flatulence C. Fever D. Pain

D Rationale Pain is the most common symptoms of an acute abdominal problem. The other symptoms may be present, along with vomiting, diarrhea, constipation, flatulence, fatigue, or increased abdominal girth, but pain is most common. Reference: 1015

Which items are included in the Rome III symptom-based criteria for inflammatory bowel syndrome (IBS)? A. Positive occult blood stool specimen B. Unilateral abdominal pain C. Diarrhea 10 or more times/day D. Abdominal pain for at least 3 months

D Rationale The Rome III criteria include abdominal discomfort or pain for at least 3 months, with onset at least 6 months before that has at least two characteristics: (1) relieved with defecation, (2) onset associated with a change in stool frequency, and (3) onset associated with change in stool appearance. Positive occult blood in the stool can be an indication of colorectal cancer and that must be ruled out. Pain tends to be across the lower abdomen. While some people can have frequent diarrhea, others have constipation or intermittent diarrhea and constipation. Reference: 1018

Which alterations occur in IBS? A. Ulceration in the gastric mucosa B. Viral infection C. Protozoal infestation D. Altered bowel motility

D Rationale The cause of IBS is unknown, but altered bowel motility, heightened visceral sensitivity, inflammation, and psychological distress are likely to be involved. Ulceration is associated with inflammatory bowel disease (IBD). IBS is not associated with infection or protozoal infestation. Reference: 1018

After inserting a nasogastric tube for a 68-year-old patient with suspected bowel obstruction, you should write which priority nursing diagnosis on the patient's problem list? A. Anxiety related to nasogastric tube placement B. Abdominal pain related to nasogastric tube placement C. Risk of deficient knowledge related to nasogastric tube placement D. Altered oral mucous membrane related to nasogastric tube placement

D Rationale With nasogastric tube placement, the patient is likely to breathe through the mouth and may experience irritation in the affected nares. For this reason, you should plan preventive measures based on this nursing diagnosis. Reference: 1034

When teaching the patient about the diet for diverticular disease, which foods should the nurse recommend? A. White bread, cheese, and green beans B. Fresh tomatoes, pears, and corn flakes C. Oranges, baked potatoes, and raw carrots D. Dried beans, All Bran (100%) cereal, and raspberries

D A high fiber diet is recommended for diverticular disease. Dried beans, All Bran (100%) cereal, and raspberries all have higher amounts of fiber than white bread, cheese, green beans, fresh tomatoes, pears, corn flakes, oranges, baked potatoes, and raw carrots.

The nurse would question the use of which cathartic agent in a patient with renal insufficiency? A. Bisacodyl (Dulcolax) B. Lubiprostone (Amitiza) C. Cascara sagrada (Senekot) D. Magnesium hydroxide (Milk of Magnesia)

D Milk of Magnesia may cause hypermagnesemia in patients with renal insufficiency. The nurse should question this order with the health care provider. Bisacodyl, lubiprostone, and cascara sagrada are safe to use in patients with renal insufficiency as long as the patient is not currently dehydrated.

A patient who uses every-other-day prednisone therapy for rheumatoid arthritis complains of not feeling as well on the non-prednisone days and asks nurse about taking prednisone daily instead. The best response to the pt is that a. an every-other-day schedule mimics the normal pattern of cortisol secretion from the adrenal gland. b. glucocorticoids are taken on a daily basis only when theyre being used for replacement therapy. c. if it improves the symptoms, it would be acceptable to take half the usual dose every day. d. there is less effect on normal adrenal function when prednisone is taken every other day.

D R: An alternate-day regimen is given to minimize the impact of exogenous glucocorticoids on adrenal gland function. The normal pattern of cortisol secretion is diurnal. Glucocorticoids are taken daily when being used for replacement therapy, but this is not the only indication for daily use. Taking half the usual dose would not achieve the goal of minimizing adrenal gland suppression.

Which information obtained when caring for a pt who has just been admitted for evaluation of DI will be of greatest concern to the nurse? a. The patient has a urine output of 800 ml/hr. b. The patient's urine specific gravity is 1.003. c. The patient had a recent head injury. d. The patient is confused and lethargic.

D R: Pts with diabetes insipidus compensate for fluid losses by drinking copious amounts of fluids, but a patient who is lethargic will be unable to drink enough fluids and will become hypovolemic. A high urine output, low urine specific gravity, and history of a recent head injury are consistent with DI, but they do not require immediate nursing action to avoid life-threatening complications.

A patient with hypoparathyroidism receives instructions from the nurse regarding symptoms of hypocalcemia and hypercalcemia. The nurse teaches the patient that if mild symptoms of hypocalcemia occur, the patient should a. increase daily fluid intake to twice usual amount b. self-administer IM calcium before calling doctor. c. call an ambulance because the symptoms will progress to seizures. d. rebreathe with a paper bag and then seek medical assistance.

D R: Rebreathing may help alleviate mild sx, but it will only temporarily increase ionized calcium level, so the pt should call HCP. There is no need to increase fluid intake. Calcium is not given IM but given slowly through IV route. Mild hypocalcemia is unlikely to progress to seizures.

A patient with primary hyperparathyroidism has a serum calcium level of 14 mg/dl (3.5 mmol/L), phosphorus of 1.7 mg/dl (0.55 mmol/L), serum creatinine of 2.2 mg/dl (194 mmol/L), and a high urine calcium. While the patient awaits surgery, the nurse should a. institute seizure precautions such as padded siderails. b. assist the patient to perform range-of-motion exercises QID. c. monitor the patient for positive Chvostek's or Trousseau's sign. d. encourage the pt to drink 4000 ml of fluid daily.

D R: The pt with hypercalcemia is at risk for kidney stones, which may be prevented by a high fluid intake. Seizure precautions and monitoring for Chvostek's or Trousseau's sign are appropriate for hypocalcemic patients. The pt should engage in weight-bearing exercise rather than range-of-motion because weight-bearing decreases calcium loss from bone.

A patient recovering from DKA asks the nurse how acidosis occurs. The best response by the nurse is that a. insufficient insulin leads to cellular starvation, and as cells rupture they release organic acids into the blood. b. when an insulin deficit causes hyperglycemia, then proteins are deaminated by the liver, causing acidic by-products. c. excess glucose in the blood is metabolized by the liver into acetone, which is acidic. d. an insulin deficit promotes metabolism of fat stores, which produces large amounts of acidic ketones.

D Rationale: Ketoacidosis is caused by the breakdown of fat stores when glucose is not available for intracellular metabolism. The other responses are inaccurate. Cognitive Level: Application Text Reference: pp. 1278-1279 Nursing Process: Implementation NCLEX: Physiological Integrity

A patient with type 1 diabetes has received diet instruction as part of the treatment plan. The nurse determines a need for additional instruction when the patient says, a. "I may have an occasional alcoholic drink if I include it in my meal plan." b. "I will need a bedtime snack because I take an evening dose of NPH insulin." c. "I will eat meals as scheduled, even if I am not hungry, to prevent hypoglycemia." d. "I may eat whatever I want, as long as I use enough insulin to cover the calories."

D Rationale: Most patients with type 1 diabetes need to plan diet choices very carefully. Patients who are using intensified insulin therapy have considerable flexibility in diet choices but still should restrict dietary intake of items such as fat, protein, and alcohol. The other patient statements are correct and indicate good understanding of the diet instruction. Cognitive Level: Application Text Reference: p. 1268 Nursing Process: Evaluation NCLEX: Physiological Integrity

A college student who has type 1 diabetes normally walks each evening as part of an exercise regimen. The student now plans to take a swimming class every day at 1:00 PM. The clinic nurse teaches the patient to a. delay eating the noon meal until after the swimming class. b. increase the morning dose of neutral protamine Hagedorn (NPH) insulin on days of the swimming class. c. time the morning insulin injection so that the peak occurs while swimming. d. check glucose level before, during, and after swimming.

D Rationale: The change in exercise will affect blood glucose, and the patient will need to monitor glucose carefully to determine the need for changes in diet and insulin administration. Because exercise tends to decrease blood glucose, patients are advised to eat before exercising. Increasing the morning NPH or timing the insulin to peak during exercise may lead to hypoglycemia, especially with the increased exercise. Cognitive Level: Application Text Reference: p. 1269 Nursing Process: Implementation NCLEX: Health Promotion and Maintenance

A program of weight loss and exercise is recommended for a patient with impaired fasting glucose (IFG). When teaching the patient about the reason for these lifestyle changes, the nurse will tell the patient that a. the high insulin levels associated with this syndrome damage the lining of blood vessels, leading to vascular disease. b. although the fasting plasma glucose levels do not currently indicate diabetes, the glycosylated hemoglobin will be elevated. c. the liver is producing excessive glucose, which will eventually exhaust the ability of the pancreas to produce insulin, and exercise will normalize glucose production. d. the onset of diabetes and the associated cardiovascular risks can be delayed or prevented by weight loss and exercise.

D Rationale: The patient with IFG is at risk for developing type 2 diabetes, but this risk can be decreased with lifestyle changes. Glycosylated hemoglobin levels will not be elevated in IFG and the Hb A1C test is not included in prediabetes testing. Elevated insulin levels do not cause the damage to blood vessels that can occur with IFG. The liver does not produce increased levels of glucose in IFG. Cognitive Level: Application Text Reference: p. 1255 Nursing Process: Implementation NCLEX: Physiological Integrity

A patient screened for diabetes at a clinic has a fasting plasma glucose level of 120 mg/dl (6.7 mmol/L). The nurse will plan to teach the patient about a. use of low doses of regular insulin. b. self-monitoring of blood glucose. c. oral hypoglycemic medications. d. maintenance of a healthy weight.

D Rationale: The patient's impaired fasting glucose indicates prediabetes and the patient should be counseled about lifestyle changes to prevent the development of type 2 diabetes. The patient with prediabetes does not require insulin or the oral hypoglycemics for glucose control and does not need to self-monitor blood glucose. Cognitive Level: Application Text Reference: p. 1255 Nursing Process: Planning NCLEX: Physiological Integrity

The nurse caring for a patient hospitalized with diabetes mellitus would look for which laboratory test result to obtain information on the patient's past glucose control? A) Prealbumin level B) Urine ketone level C) Fasting glucose level D) Glycosylated hemoglobin level

D) Glycosylated hemoglobin level A glycosylated hemoglobin level detects the amount of glucose that is bound to red blood cells (RBCs). When circulating glucose levels are high, glucose attaches to the RBCs and remains there for the life of the blood cell, which is approximately 120 days. Thus the test can give an indication of glycemic control over approximately 2 to 3 months. The prealbumin level is used to establish nutritional status and is unrelated to past glucose control. The urine ketone level will only show that hyperglycemia or starvation is probably currently occurring. The fasting glucose level only indicates current glucose control.

A pt reports having dry mouth and asks for some liquid to drink. The nurse reasons that this symptom can most likely be attributed to a common adverse effect of which of the following medications? A) Digoxin (Lanoxin) B) Cefotetan (Cefotan) C) Famotidine (Pepcid) D) Promethazine (Phenergan)

D) Promethazine (Phenergan) A common adverse effect of promethazine, an antihistamine antiemetic agent, is dry mouth; another is blurred vision.

When teaching the patient with acute hepatitis C (HCV), the patient demonstrates understanding when the patient makes which statement? A) "I will use care when kissing my wife to prevent giving it to her." B) "I will need to take adofevir (Hepsera) to prevent chronic HCV." C) "Now that I have had HCV, I will have immunity and not get it again." D) "I will need to be checked for chronic HCV and other liver problems."

D) The majority of patients who acquire HCV usually develop chronic infection, which may lead to cirrhosis or liver cancer. HCV is not transmitted via saliva, but percutaneously and via high-risk sexual activity exposure. The treatment for acute viral hepatitis focuses on resting the body and adequate nutrition for liver regeneration. Adofevir (Hepsera) is taken for severe hepatitis B (HBV) with liver failure. Chronic HCV is treated with pegylated interferon with ribavirin. Immunity with HCV does not occur as it does with HAV and HBV, so the patient may be reinfected with another type of HCV.

The nurse is teaching a patient with type 2 diabetes mellitus about exercise to help control his blood glucose. The nurse knows the patient understands when the patient elicits which exercise plan? A) "I want to go fishing for 30 minutes each day; I will drink fluids and wear sunscreen." B) "I will go running each day when my blood sugar is too high to bring it back to normal." C) "I will plan to keep my job as a teacher because I get a lot of exercise every school day." D)"I will take a brisk 30-minute walk 5 days per week and do resistance training 3 times a week."

D)"I will take a brisk 30-minute walk 5 days per week and do resistance training 3 times a week." The best exercise plan for the person with type 2 diabetes is for 30 minutes of moderate activity 5 days per week and resistance training 3 times a week. Brisk walking is moderate activity. Fishing and teaching are light activity, and running is considered vigorous activity.

A patient with a severe pounding headache has been diagnosed with hypertension. However, the hypertension is not responding to traditional treatment. What should the nurse expect as the next step in management of this patient? A. Administration of β-blocker medications B. Abdominal palpation to search for a tumor C. Administration of potassium-sparing diuretics D. A 24-hour urine collection for fractionated metanephrines

D. A 24-hour urine collection for fractionated metanephrines Pheochromocytoma should be suspected when hypertension does not respond to traditional treatment. The 24-hour urine collection for fractionated metanephrines is simple and reliable with elevated values in 95% of people with pheochromocytoma. In a patient with pheochromocytoma preoperatively an α-adrenergic receptor blocker is used to reduce BP. Abdominal palpation is avoided to avoid a sudden release of catecholamines and severe hypertension. Potassium-sparing diuretics are not needed. Most likely they would be used for hyperaldosteronism, which is another cause of hypertension.

The patient has had type 1 diabetes mellitus for 25 years and is now reporting fatigue, edema, and an irregular heartbeat. On assessment, the nurse finds that the patient has newly developed hypertension and difficulty with blood glucose control. The nurse should know that which diagnostic study will be most indicative of chronic kidney disease (CKD) in this patient? A. Serum creatinine B. Serum potassium C. Microalbuminuria D. Calculated glomerular filtration rate (GFR)

D. Calculated glomerular filtration rate (GFR) The best study to determine kidney function or chronic kidney disease (CKD) that would be expected in the patient with diabetes is the calculated GFR that is obtained from the patient's age, gender, race, and serum creatinine. It would need to be abnormal for 3 months to establish a diagnosis of CKD. A creatinine clearance test done with a blood sample and a 24-hour urine collection is also important. Serum creatinine is not the best test for CKD because the level varies with different patients. Serum potassium levels could explain why the patient has an irregular heartbeat. The finding of microalbuminuria can alert the patient with diabetes about potential renal involvement and potentially failing kidneys. However, urine albumin levels are not used for diagnosis of CKD.

The nurse is admitting a 68-year-old man with severe dehydration and frequent watery diarrhea. He just completed a 10-day outpatient course of antibiotic therapy for bacterial pneumonia. It is most important for the nurse to take which action? A. Wear a mask to prevent transmission of infection. B. Wipe equipment with ammonia-based disinfectant. C. Instruct visitors to use the alcohol-based hand sanitizer. D. Don gloves and gown before entering the patient's room.

D. Don gloves and gown before entering the patient's room. Clostridium difficile is an antibiotic-associated diarrhea transmitted by contact, and the spores are extremely difficult to kill. Patients with suspected or confirmed infection with C. difficile should be placed in a private room and gloves and gowns should be worn by visitors and health care providers. Alcohol-based hand cleaners and ammonia-based disinfectants are ineffective and do not kill all of the spores. Equipment cannot be shared with other patients, and a disposable stethoscope and individual patient thermometer are kept in the room. Objects should be disinfected with a 10% solution of household bleach.

Following a parathyroidectomy, a patient develops tingling of the lips and a positive Trousseau's sign. Which action should the nurse take first? A. Administer the ordered muscle relaxant. B. Give the ordered oral calcium supplement. C. Start the PRN oxygen at 2 L/min per cannula. D. Have the patient rebreathe using a paper bag.

D. Have the patient rebreathe using a paper bag.

The nurse is caring for a patient admitted with suspected hyperparathyroidism. Because of the potential effects of this disease on electrolyte balance, the nurse should assess this patient for what manifestation? A. Neurologic irritability B. Declining urine output D. Lethargy and weakness C. Hyperactive bowel sounds

D. Lethargy and weakness Hyperparathyroidism can cause hypercalcemia. Signs of hypercalcemia include muscle weakness, polyuria, constipation, nausea and vomiting, lethargy, and memory impairment. Neurologic irritability, declining urine output, and hyperactive bowel sounds do not occur with hypercalcemia.

After an abdominal hysterectomy, a 45-year-old woman complains of severe gas pains. Her abdomen is distended. It is most appropriate for the nurse to administer which prescribed medication? A. Morphine sulfate B. Ondansetron (Zofran) C. Acetaminophen (Tylenol) D. Metoclopramide (Reglan)

D. Metoclopramide (Reglan) Swallowed air and reduced peristalsis after surgery can result in abdominal distention and gas pains. Early ambulation helps restore peristalsis and eliminate flatus and gas pain. Medications used to reduce gas pain include metoclopramide (Reglan) or alvimopan (Entereg) to stimulate peristalsis.

Which task can the nurse delegate to an unlicensed assistive personnel (UAP) in the care of a patient who has recently undergone prostatectomy? A. Assessing the patient's incision B. Irrigating the patient's Foley catheter C. Assessing the patient's pain and selecting analgesia D. Performing cleansing of the meatus and perineal region

D. Performing cleansing of the meatus and perineal region Performing perineal care is an appropriate task for delegation. Selecting analgesia, irrigating the patient's catheter, and assessing his incision are not appropriate skills or tasks for unlicensed personnel.

The nurse preparing to administer a dose of calcium acetate (PhosLo) to a patient with chronic kidney disease (CKD) should know that this medication should have a beneficial effect on which laboratory value? A. Sodium B. Potassium C. Magnesium D. Phosphorus

D. Phosphorus Phosphorus and calcium have inverse or reciprocal relationships, meaning that when phosphorus levels are high, calcium levels tend to be low. Therefore administration of calcium should help to reduce a patient's abnormally high phosphorus level, as seen with CKD. PhosLo will not have an effect on sodium, potassium, or magnesium levels.

The patient with an adrenal hyperplasia is returning from surgery for an adrenalectomy. For what immediate postoperative risk should the nurse plan to monitor the patient? A. Vomiting B. Infection C. Thomboembolism D. Rapid BP changes

D. Rapid BP changes The risk of hemorrhage is increased with surgery on the adrenal glands as well as large amounts of hormones being released in the circulation, which may produce hypertension and cause fluid and electrolyte imbalances to occur for the first 24 to 48 hours after surgery. Vomiting, infection, and thromboembolism may occur postoperatively with any surgery.

When caring for a patient during the oliguric phase of acute kidney injury (AKI), what is an appropriate nursing intervention? A. Weigh patient three times weekly. B. Increase dietary sodium and potassium. C. Provide a low-protein, high-carbohydrate diet. D. Restrict fluids according to previous daily loss.

D. Restrict fluids according to previous daily loss. Patients in the oliguric phase of acute kidney injury will have fluid volume excess with potassium and sodium retention. Therefore they will need to have dietary sodium, potassium, and fluids restricted. Daily fluid intake is based on the previous 24-hour fluid loss (measured output plus 600 ml for insensible loss). The diet also needs to provide adequate, not low, protein intake to prevent catabolism. The patient should also be weighed daily, not just three times each week

A 50-year-old female patient smokes, is getting a divorce, and is reporting eye problems. On assessment of this patient, the nurse notes exophthalmos. What other abnormal assessments should the nurse expect to find in this patient? A. Puffy face, decreased sweating, and dry hair B. Muscle aches and pains and slow movements C. Decreased appetite, increased thirst, and pallor D. Systolic hypertension and increased heart rate

D. Systolic hypertension and increased heart rate The patient's manifestations point to Graves' disease or hyperthyroidism, which would also include systolic hypertension and increased heart rate and increased thirst. Puffy face, decreased sweating; dry, coarse hair; muscle aches and pains and slow movements; decreased appetite and pallor are all manifestations of hypothyroidism.

A nurse is caring for a client who will perform fecal occult blood testing at home. Which of the following should the nurse include when explaining the procedure to the client? A. Eating more protein is recommended prior to testing. B. One stool specimen is sufficient for testing. C. A red color change indicates a positive test. D. The specimen cannot be contaminated with urine.

D. The specimen cannot be contaminated with urine. For fecal occult blood testing at home, the stool specimens cannot be contaminated with water or urine; three specimens from three different bowel movements are required; some proteins such as red meat, fish, and poultry can alter the test results; and a blue color indicates a positive guaiac or presence of fecal occult blood not red. Chapter 45; Page 1046

When the patient reports acute, severe, renal colic pain in the lower abdomen, the nurse knows that the patient is most likely to have an obstruction at which area? A.Kidney B. Urethra C. Bladder D. Ureterovesical junction

D. Ureterovesical junction The ureterovesical junction (UVJ) is the narrowest part of the urethra and easily obstructed by urinary calculi. With a stone in the kidney or at the ureteropelvic junction (UPJ), the pain may be dull costovertebral flank pain. Stones in the bladder do not cause obstruction or symptoms unless they are staghorn stones. The urethra seldom has obstruction related to stones.

Which urinalysis result should the nurse recognize as an abnormal finding? A. pH 6.0 B. Amber yellow color C. Specific gravity 1.025 D. White blood cells (WBCs) 9/hpf

D. White blood cells (WBCs) 9/hpf Correct Normal WBC levels in urine are below 5/hpf, with levels exceeding this indicative of inflammation or urinary tract infection. A urine pH of 6.0 is average; amber yellow is normal coloration, and the reference ranges for specific gravity are 1.003 to 1.030.

The patient is wondering why anesthesia is needed when the lithotripsy being done is noninvasive. The nurse explains that the anesthesia is required to ensure the patient's position is maintained during the procedure. The nurse knows that this type of lithotripsy is called A. laser lithotripsy. B. electrohydraulic lithotripsy. C. percutaneous ultrasonic lithotripsy. D. extracorporeal shock-wave lithotripsy (ESWL).

D. extracorporeal shock-wave lithotripsy (ESWL). ESWL is noninvasive, but anesthesia is used to maintain the patient's position. The other types of lithotripsy are invasive. Laser lithotripsy uses an ureteroscope and small fiber to reach the stone. Electrohydraulic lithotripsy positions a probe directly on the stone; then continuous saline irrigation flushes are used to rinse the stone out. Percutaneous ultrasonic lithotripsy places an ultrasonic probe in the renal pelvis via a percutaneous nephroscope inserted through an incision in the flank.

A 18-year-old male patient is undergoing a growth hormone stimulation test. The nurse should monitor the patient for A. hypothermia. B. hypertension. C. hyperreflexia. D. hypoglycemia.

D. hypoglycemia Insulin or arginine (agent that stimulates insulin secretion) is administered for a growth hormone stimulation test. The nurse should monitor the patient closely for hypoglycemia. Hypothermia and hypertension are not expected in response to insulin or arginine. Hyperreflexia is an autonomic complication of spinal cord injury.

The nurse identifies the client problem "risk for imbalance body temp" for a client diagnosed with hypothyroidism. which intervention should be included in the plan of care? a. discourage the use of an electric blanket b. assess the clients temp q2h c. keep the room temp cool d. space activities to promote rest

a. discourage the use of an electric blanket

A patient is admitted with possible syndrome of inappropriate antidiuretic hormone (SIADH). Which information obtained by the nurse is most important to communicate rapidly to the health care provider? A. The patient complains of dyspnea with activity. B. The patient has a urine specific gravity of 1.025. C. The patient has a recent weight gain of 8 lb. D. The patient has a serum sodium level of 119 mEq/L.

D. the pt has a serum sodium level of 119 mEq/L R: A serum sodium of less than 120 mEq/L increases risk for complications such as seizures and needs rapid correction. The other data are not unusual for a pt with SIADH and do not indicate the need for rapid action.

During hemodialysis, the patient develops light-headedness and nausea. What should the nurse do first? Administer hypertonic saline. Administer a blood transfusion. Decrease the rate of fluid removal. Administer antiemetic medications.

Decrease the rate of fluid removal. *The patient is experiencing hypotension from a rapid removal of vascular volume. The rate and volume of fluid removal will be decreased, and 0.9% saline solution may be infused. Hypertonic saline is not used because of the high sodium load. A blood transfusion is not indicated. Antiemetic medications may help the nausea but would not help the hypovolemia.

The health care provider orders lactulose for a patient with hepatic encephalopathy. The nurse will monitor for effectiveness of this medication for this patient by assessing what? Relief of constipation Relief of abdominal pain Decreased liver enzymes Decreased ammonia levels

Decreased ammonia levels Hepatic encephalopathy is a complication of liver disease and is associated with elevated serum ammonia levels. Lactulose traps ammonia in the intestinal tract. Its laxative effect then expels the ammonia from the colon, resulting in decreased serum ammonia levels and correction of hepatic encephalopathy.

Which assessment findings would alert the nurse that the patient has entered the diuretic phase of acute kidney injury (AKI) (select all that apply.)? Dehydration Hypokalemia Hypernatremia BUN increases Urine output increases Serum creatinine increases

Dehydration Hypokalemia Urine output increases *The hallmark of entering the diuretic phase is the production of copious amounts of urine. Dehydration, hypokalemia, and hyponatremia occur in the diuretic phase of AKI because the nephrons can excrete wastes but not concentrate urine. Serum BUN and serum creatinine levels begin to decrease.

A 78-year-old patient in a long-term care facility has these medications prescribed. After the patient is diagnosed with hypothyroidism, the nurse will need to consult with the health care provider before administration of A. Docusate (Colace). B. Diazepam (Valium) C. . ibuprofen (Motrin). D. Cefoxitin (Mefoxin).

Diazepam (Valium) Feedback Worsening of mental status and myxedema coma can be precipitated by the use of sedatives, especially in older adults. The nurse should discuss the diazepam with the health care provider before administration. The other medications may be given safely to the patient.

Which patient diagnosis or treatment is most consistent with prerenal acute kidney injury (AKI)? IV tobramycin Incompatible blood transfusion Poststreptococcal glomerulonephritis Dissecting abdominal aortic aneurysm

Dissecting abdominal aortic aneurysm

A patient with suspected acromegaly is seen at the clinic. To assist in making the diagnosis, which question should the nurse ask? A. Have you had a recent head injury? B. Do you have to wear larger shoes now? C. Are you experiencing tremors or anxiety? D. Is there any family history of acromegaly?

Do you have to wear larger shoes now? Acromegaly causes an enlargement of the hands and feet. Head injury and family history are not risk factors for acromegaly. Tremors and anxiety are not clinical manifestations of acromegaly

A patient with Graves' disease has exophthalmos. Which nursing action will be included in the plan of care? A. Apply eye patches to protect the cornea from irritation. B. Place cold packs on the eyes to relieve pain and swelling. C. Elevate the head of the patient's bed to reduce periorbital fluid. D. Teach the patient to blink every few seconds to lubricate the cornea.

Elevate the head of the patient's bed to reduce periorbital fluid. Feedback The patient should sit upright as much as possible to promote fluid drainage from the periorbital area. With exophthalmos the patient is unable to close the eyes completely. Lubrication of the eyes, rather than eye patches, will protect the eyes from developing corneal scarring. The swelling of the eye is not caused by excessive blood flow to the eye, so cold packs will not be helpful.

A patient with Cushing syndrome returns to the surgical unit following an adrenalectomy. During the initial postoperative period, the nurse gives the highest priority to A. Monitoring for infection. B. Protecting the patient's skin. C. Maintaining fluid and electrolyte status. D. Preventing severe emotional disturbances.

Feedback ANS: C After adrenalectomy, the patient is at risk for circulatory instability caused by fluctuating hormone levels, and the focus of care is to assess and maintain fluid and electrolyte status through the use of IV fluids and corticosteroids. The other goals also are important for the patient but are not as immediately life-threatening as the circulatory collapse that can occur with fluid and electrolyte disturbances.

A patient who has hepatitis B surface antigen (HBsAg) in the serum is being discharged with pain medication after knee surgery. Which medication order should the nurse question because it is most likely to cause hepatic complications? Tramadol (Ultram) Hydromorphone (Dilaudid) Oxycodone with aspirin (Percodan) Hydrocodone with acetaminophen (Vicodin)

Hydrocodone with acetaminophen (Vicodin) The analgesic with acetaminophen should be questioned because this patient is a chronic carrier of hepatitis B and is likely to have impaired liver function. Acetaminophen is not suitable for this patient because it is converted to a toxic metabolite in the liver after absorption, increasing the risk of hepatocellular damage.

A nurse is caring for a client who has had diarrhea for the past 4 days. When assessing the client, the nurse should expect which of the following findings? (Select all that apply.) ___Bradycardia ___Hypotension ___Fever ___Poor skin turgor ___Peripheral edema ___Abdominal cramping

Hypotension Fever Poor skin turgor Abdominal cramping Prolonged diarrhea leads to dehydration, which is characterized by tachycardia, hypotension, fever, lethargy, poor skin turgor, and abdominal cramping. Peripheral edema is more likely to be caused by a fluid overload rather than a fluid deficit. Chapter 45; Page 1096, Box 46-3

Which assessment finding for a patient who takes levothyroxine (Synthroid) to treat hypothyroidism indicates that the nurse should contact the health care provider before administering the medication? A. Increased thyroxine (T4) level B. Blood pressure 102/62 mm Hg C. Distant and difficult to hear heart sounds D. Elevated thyroid stimulating hormone level

Increased thyroxine (T4) level Feedback An increased thyroxine level indicates the levothyroxine dose needs to be decreased. The other data are consistent with hypothyroidism and the nurse should administer the Synthroid.

A few hours after returning to the surgical nursing unit, a patient who has undergone a subtotal thyroidectomy develops laryngeal stridor and a cramp in the right hand. Which action will the nurse anticipate taking next? A. Infuse IV calcium gluconate. B. Suction the patient's airway. C. Prepare for endotracheal intubation. D. Assist with emergency tracheostomy.

Infuse IV calcium gluconate. Feedback The patient's clinical manifestations of stridor and cramping are consistent with tetany caused by hypocalcemia resulting from damage to the parathyroid glands during surgery. Endotracheal intubation or tracheostomy may be needed if the calcium does not resolve the stridor. Suctioning will not correct the stridor.

An older male patient states that he is having problems starting and stopping his stream of urine and he feels the urgency to void. The best way to assist this patient is to: A) Help him stand to void. B) Place a condom catheter. C) Have him practice Credé's method. D) Initiate Kegel exercises.

Initiate Kegel exercises.

The patient with sudden pain in the left upper quadrant radiating to the back and vomiting was diagnosed with acute pancreatitis. What intervention(s) should the nurse expect to include in the patient's plan of care? a. Immediately start enteral feeding to prevent malnutrition. b. Insert an NG and maintain NPO status to allow pancreas to rest. c. Initiate early prophylactic antibiotic therapy to prevent infection. d. Administer acetaminophen (Tylenol) every 4 hours for pain relief.

Insert an NG and maintain NPO status to allow pancreas to rest. Correct Initial treatment with acute pancreatitis will include an NG tube if there is vomiting and being NPO to decrease pancreatic enzyme stimulation and allow the pancreas to rest and heal. Fluid will be administered to treat or prevent shock. The pain will be treated with IV morphine because of the NPO status. Enteral feedings will only be used for the patient with severe acute pancreatitis in whom oral intake is not resumed. Antibiotic therapy is only needed with acute necrotizing pancreatitis and signs of infection.

The nurse is caring for a 55-year-old man patient with acute pancreatitis resulting from gallstones. Which clinical manifestation would the nurse expect the patient to exhibit? Hematochezia Left upper abdominal pain Ascites and peripheral edema Temperature over 102o F (38.9o C)

Left upper abdominal pain Abdominal pain (usually in the left upper quadrant) is the predominant manifestation of acute pancreatitis. Other manifestations of acute pancreatitis include nausea and vomiting, low-grade fever, leukocytosis, hypotension, tachycardia, and jaundice. Abdominal tenderness with muscle guarding is common. Bowel sounds may be decreased or absent. Ileus may occur and causes marked abdominal distention. Areas of cyanosis or greenish to yellow-brown discoloration of the abdominal wall may occur. Other areas of ecchymoses are the flanks (Grey Turner's spots or sign, a bluish flank discoloration) and the periumbilical area (Cullen's sign, a bluish periumbilical discoloration).

To minimize the patient experiencing nocturia, the nurse would teach him or her to: A) Perform perineal hygiene after urinating. B) Set up a toileting schedule. C) Double void. D) Limit fluids before bedtime.

Limit fluids before bedtime

The patient has rapidly progressing glomerular inflammation. Weight has increased and urine output is steadily declining. What is the priority nursing intervention? Monitor the patient's cardiac status. Teach the patient about hand washing. Obtain a serum specimen for electrolytes. Increase direct observation of the patient.

Monitor the patient's cardiac status. *due to hypervolemia, hyperkalemia, and metabolic acidosis

Besides being mixed with struvite or oxalate stones, what characteristic is associated with calcium phosphate calculi? a. assocaited with alkaline urine b. genetic autosomal recessive defect c. threee times as common in women as in men d. defective GI and kidney absorption

a. assocaited with alkaline urine

The nurse assesses that the patient has a full bladder, and the patient states that he or she is having difficulty voiding. The nurse would teach the patient to: A) Use the double-voiding technique. B) Perform Kegel exercises. C) Use Credé's method. D) Keep a voiding diary.

Use Credé's method.

When caring for the patient with a traumatic brain injury (TBI), the nurse knows that damage to which endocrine gland can affect the hormone secretion from some of the other endocrine glands? ANTERIOR PITUITARY ADRENAL HYPOTHALAMUS PARATHYROID

With a TBI, the anterior pituitary is likely to be damaged. The anterior pituitary gland secrets tropic hormones that control the secretion of hormones by other endocrine glands (the thyroid, adrenal cortex, and reproductive organs). The parathyroids secrete parathyroid hormone that regulates serum calcium level by acting on bone, the kidneys, and indirectly the gastrointestinal tract. The pineal gland secretes melatonin that helps regulate circadian rhythm and reproduction. The thyroid glands secrete thyroxine (T4), triiodothyronine (T3) that regulates the cell processes of cell growth and tissue differentiation, and calcitonin that affects bone tissue to regulate serum calcium and phosphorus levels.

Which is the best explanation of the pathophysiology that occurs in celiac or gluten-sensitive enteropathy? A. Intestinal inflammation, especially in the duodenum B. Full-thickness ulcers throughout the small and large intestines C. Lack of enzyme necessary to digest gluten D. Irregular peristalsis with heightened sensitivity

a Rationale Tissue destruction occurs as a result of chronic inflammation, activated by gluten. Damage is most severe in the duodenum, probably because it is the site of the highest concentration of gluten. Intermittent, full-thickness ulcers are characteristic of IBD. Lack of the necessary enzyme for digestion of milk is characteristic of lactase deficiency. Irregular peristalsis with enhanced sensitivity is characteristic of irritable bowel syndrome (IBS). Reference: 1050

The nurse performs a detailed assessment of the abdomen of a patient with a possible bowel obstruction, knowing that manifestations of a obstruction in the large intestine are (select all that apply): a. persistent abdominal pain b. marked abdominal distention c. diarrhea that is loose or liquid d. colicky, severe, intermittent pain e. profuse committing that relieves abdominal pain

a & b Rationale: With lower intestinal obstructions, abdominal distention is markedly increased and pain is persistent. Onset of a large intestine obstruction is gradual, vomiting is rare, and there is usually absolute constipation, not diarrhea.

Prevention of AKI is important because of the high mortality rate. Which patients are at increased risk for AKI (select all that apply)? a. An 86-year-old woman scheduled for a cardiac catheterization b. A 48-year-old man with multiple injuries from a motor vehicle accident c. A 32-year-old woman following a C-section delivery for abruptio placentae d. A 64-year-old woman with chronic heart failure admitted with bloody stools e. A 58-year-old man with prostate cancer undergoing preoperative workup for prostatectomy

a, b, c, d, e. High-risk patients include those exposed to nephrotoxic agents and advanced age (a), massive trauma (b), prolonged hypovolemia or hypotension (possibly b and c), obstetric complications (c), cardiac failure (d), preexisting chronic kidney disease, extensive burns, or sepsis. Patients with prostate cancer may have obstruction of the outflow tract, which increases risk of postrenal AKI (e).

Which drugs will be used to treat the patient with CKD for mineral and bone disorder (select all that apply)? a. Cinacalcet (Sensipar) b. Sevelamer (Renagel) c. IV glucose and insulin d. Calcium acetate (PhosLo) e. IV 10% calcium gluconate

a, b, d. Cinacalcet (Sensipar), a calcimimetic agent to control secondary hyperparathyroidism; sevelamer (Renagel), a noncalcium phosphate binder; and calcium acetate (PhosLo), a calcium-based phosphate binder are used to treat mineral and bone disorder in CKD. IV glucose and insulin and IV 10% calcium gluconate along with sodium polystyrene sulfonate (Kayexalate) are used to treat the hyperkalemia of CKD.

The patient with CKD is receiving dialysis, and the nurse observes excoriations on the patient's skin. What pathophysiologic changes in CKD can contribute to this finding (select all that apply)? a. Dry skin b. Sensory neuropathy c. Vascular calcifications d. Calcium-phosphate skin deposits e. Uremic crystallization from high BUN

a, b, d. Pruritus is common in patients receiving dialysis. It causes scratching from dry skin, sensory neuropathy, and calcium-phosphate deposition in the skin. Vascular calcifications contribute to cardiovascular disease, not to itching skin. Uremic frost rarely occurs without BUN levels greater than 200 mg/dL, which should not occur in a patient on dialysis; urea crystallizes on the skin and also causes pruritis.

For the patient hospitalized with inflammatory bowel disease (IBD), which treatments would be used to rest the bowel (select all that apply)? a. NPO d. Sedatives b. IV fluids e. Nasogastric suction c. Bed rest f. Parenteral nutrition

a, b, e, f. With an acute exacerbation of inflammatory bowel disease (IBD), to rest the bowel the patient will be NPO, receive IV fluids and parenteral nutrition, and have nasogastric suction. Sedatives would be used to alleviate stress. Enteral nutrition will be used as soon as possible

A 20-year old patient with a history of Crohn's disease comes to the clinic with persistent diarrhea. What are characteristics of Crohn's disease (select all that apply)? a. Weight loss b. Rectal bleeding c. Abdominal pain d. Toxic megacolon e. Has segmented distribution f. Involves the entire thickness of the bowel wall

a, c, e, f. Crohn's disease may have severe weight loss, segmented distribution through the entire wall of the bowel, and crampy abdominal pain. Rectal bleeding and toxic megacolon are more often seen with ulcerative colitis.

Which characteristics describe the use of RAI (select all that apply)? a. Often causes hypothyroidism over time b. Decreases release of thyroid hormones c. Blocks peripheral conversion of T4 to T3 d. Treatment of choice in nonpregnant adults e. Decreases thyroid secretion by damaging thyroid gland f. Often used with iodine to produce euthyroid before surgery

a, d, e. RAI causes hypothyroidism over time by damaging thyroid tissue and is the treatment of choice for nonpregnant adults. Potassium iodide decreases the release of thyroidhormones and decreases the size of the thyroid gland preoperatively. Propylthiouracil (PTU) blocks peripheral conversion of T4 to T3 and may be used with iodine to produce a euthyroid state before surgery.

The patient calls the clinic and describes a bump at the site of a previous incision that disappears when he lies down. The nurse suspects that this is which type of hernia (select all that apply)? a. Ventral d. Reducible b. Inguinal e. Incarcerated c. Femoral f. Strangulated

a, d. The ventral or incisional hernia is due to a weakness of the abdominal wall at the site of a previous incision. It is reducible when it returns to the abdominal cavity. Inguinal hernias are at the weak area of the abdominal wall where the spermatic cord in men or the round ligament in women emerges. A femoral hernia is a protrusion through the femoral ring into the femoral canal. Incarcerated hernias do not reduce.

Nursing management of the patient with acute pancreatitis includes (select all that apply) a. checking for signs of hypocalcemia. b. providing a diet low in carbohydrates. c. giving insulin based on a sliding scale. d. observing stools for signs of steatorrhea. e. monitoring for infection, particularly respiratory tract infection. (Lewis 1042)

a, e Rationale: During the acute phase, it is important to monitor vital signs. Hemodynamic stability may be compromised by hypotension, fever, and tachypnea. Intravenous fluids are ordered, and the response to therapy is monitored. Fluid and electrolyte balances are closely monitored. Frequent vomiting, along with gastric suction, may result in decreased levels of chloride, sodium, and potassium. Because hypocalcemia can occur in acute pancreatitis, the nurse should observe for symptoms of tetany, such as jerking, irritability, and muscular twitching. Numbness or tingling around the lips and in the fingers is an early indicator of hypocalcemia. The patient should be assessed for Chvostek's sign or Trousseau's sign. A patient with acute pancreatitis should be observed for fever and other manifestations of infection. Respiratory infections are common because the retroperitoneal fluid raises the diaphragm, which causes the patient to take shallow, guarded abdominal breaths.

Which statement by the patient with type 2 diabetes is accurate? a. "I am supposed to have a meal or snack if I drink alcohol." b. "I am not allowed to eat any sweets because of my diabetes." c. "I do not need to watch what I eat because my diabetes is not the bad kind." d. "The amount of fat in my diet is not important; it is just the carbohydrates that raise my blood sugar."

a. "I am supposed to have a meal or snack if I drink alcohol."

When evaluating the patient's understanding about the care of the ileostomy, what statement by the patient indicates the patient needs more teaching? a. "I will be able to regulate when I have stools." b. "I will be able to wear the pouch until it leaks." c. "Dried fruit and popcorn must be chewed very well." d. "The drainage from my stoma can damage my skin."

a. "I will be able to regulate when I have stools." The ileostomy is in the ileum and drains liquid stool frequently, unlike the colostomy which has more formed stool the further distal the ostomy is in the colon. The ileostomy pouch is usually worn 4-7 days or until it leaks. It must be changed immediately if it leaks because the drainage is very irritating to the skin. To avoid obstruction, popcorn, dried fruit, coconut, mushrooms, olives, stringy vegetables, food with skin, and meats with casings must be chewed extremely well before swallowing because of the narrow diameter of the ileostomy lumen.

Number in sequence the following ascending pathologic changes that occur in the urinary tract in the presence of a bladder outlet obstruction. a. Hydronephrosis b. Reflux of urine into ureter c. Bladder detrusor muscle hypertrophy d. Ureteral dilation e. Renal atrophy f. Vesicoureteral reflux g. Large residual urine in bladder h. Chronic pyelonephritis

a. 6; b. 3; c. 1; d. 4; e. 8; f. 5; g. 2; h. 7

When obtaining a nursing history from the patient with colorectal cancer, the nurse should specifically ask the patient about a. dietary intake. b. sports involvement. c. environmental exposure to carcinogens. d. long-term use of nonsteroidal antiinflammatory drugs (NSAIDs).

a. A diet high in red meat and low fruit and vegetable intake is associated with development of colorectal cancer (CRC), as are alcohol intake and smoking. Family and personal history of CRC also increases the risk. Other environmental agents are not known to be related to colorectal cancer. Long-term use of nonsteroidal antiinflammatory drugs (NSAIDs) is associated with reduced CRC risk.

When the nurse assesses the patient that has pancreatitis, what function may be altered related to the endocrine function of the pancreas? a. Blood glucose regulation b. Increased response to stress c. Fluid and electrolyte regulation d. Regulates metabolic rate of cells

a. Blood glucose regulation The endocrine functions of the pancreas are regulated by α cells that produce and secrete glucagon, β cells that produce and secrete insulin and amylin, delta cells that produce and secrete somatostatin, and F cells that secrete pancreatic polypeptide. Glucagon, insulin, and amylin, and somatostatin all affect blood glucose. Pancreatic polypeptide regulates appetite. Increased response to stress occurs from epinephrine secreted by the adrenal medulla. Fluid and electrolyte regulation occurs in response to several hormones (mineralocorticoids, antidiuretic hormone, parathyroid hormone, calcitonin) from several organs (adrenal cortex, posterior pituitary, parathyroid, thyroid). The metabolic rate of cells is regulated by triiodothyronine (T3) from the thyroid

The surgeon was unable to save a patient's parathyroid gland during a radical thyroidectomy. The nurse should consequently pay particular attention to which laboratory value? a. Calcium levels b. Potassium levels c. Blood glucose levels d. Sodium and chloride levels

a. Calcium levels The parathyroid gland plays a key role in maintaining calcium levels. Potassium, sodium, glucose, and chloride are not directly influenced by the loss of the parathyroid gland.

Besides being mixed with struvite or oxalate stones, what characteristic is associated with calcium phosphate calculi? a. Associated with alkaline urine c. Three times as common in women as in men b. Genetic autosomal recessive defect d. Defective gastrointestinal (GI) and kidney absorption

a. Calcium phosphate calculi are typically mixed with struvite or oxalate stones and related to alkaline urine. Cystine calculi are associated with a genetic autosomal recessive defect and defective GI and kidney absorption of cystine. Struvite calculi are three to four times more common in women than in men.

A patient on a medical unit has a potassium level of 6.8 mEq/L. What is the priority action that the nurse should take? a. Place the patient on a cardiac monitor. b. Check the patient's blood pressure (BP). c. Instruct the patient to avoid high-potassium foods. d. Call the lab and request a redraw of the lab to verify results.

a. Dysrhythmias may occur with an elevated potassium level and are potentially lethal. Monitor the rhythm while contacting the physician or calling the rapid response team. Vital signs should be checked. Depending on the patient's history and cause of increased potassium, instruct the patient about dietary sources of potassium; however, this would not help at this point. The nurse may want to recheck the value but until then the heart rhythm needs to be monitored.

The nurse plans teaching for the patient with a colostomy but the patient refuses to look at the nurse or the stoma, stating, "I just can't see myself with this thing." What is the best nursing intervention for this patient? a. Encourage the patient to share concerns and ask questions. b. Refer the patient to a chaplain to help cope with this situation. c. Explain that there is nothing the patient can do about it and must take care of it. d. Tell the patient that learning about it will prevent stool leaking and the sounds of flatus.

a. Encouraging the patient to share concerns and ask questions will help the patient to begin to adapt to living with the colostomy. The other options do not support the patient and do not portray the nurse's focus on helping the patient or treating the patient as an individual.

Which method is preferred for immediate treatment of an acute episode of constipation? a. An enema c. Stool softeners b. Increased fluid d. Bulk-forming medication

a. Enemas are fast acting and beneficial in the immediate treatment of acute constipation but should be limited in their use. Bulk-forming medication stimulates peristalsis but takes 24 hours to act. Stool softeners have a prolonged action, taking up to 72 hours for an effect, and fluids can help to decrease the incidence of constipation

Which complication of chronic kidney disease is treated with erythropoietin (EPO)? a. Anemia b. Hypertension c. Hyperkalemia d. Mineral and bone disorder

a. Erythropoietin is used to treat anemia, as it stimulates the bone marrow to produce red blood cells.

Which statement accurately describes Graves' disease? a. Exophthalmos occurs in Graves' disease. b. It is an uncommon form of hyperthyroidism. c. Manifestations of hyperthyroidism occur from tissue desensitization to the sympathetic nervous system. d. Diagnostic testing in the patient with Graves' disease will reveal an increased thyroid-stimulating hormone (TSH) level.

a. Exophthalmos or protrusion of the eyeballs may occur in Graves' disease from increased fat deposits and fluid in the orbital tissues and ocular muscles, forcing the eyeballs outward. Graves' disease is the most common form of hyperthyroidism. Increased metabolic rate and sensitivity of the sympathetic nervous system lead to the clinical manifestations. Thyroid-stimulating hormone (TSH) level is decreased in Graves' disease.

Which characteristic is more likely with acute pyelonephritis than with a lower UTI a. Fever b. Dysuria c. Urgency d. frequency

a. Fever

Priority Decision: During the immediate postoperative care of a recipient of a kidney transplant, what should the nurse expect to do? a. Regulate fluid intake hourly based on urine output. b. Monitor urine-tinged drainage on abdominal dressing. c. Medicate the patient frequently for incisional flank pain. d. Remove the urinary catheter to evaluate the ureteral implant.

a. Fluid and electrolyte balance is critical in the transplant recipient patient, especially because diuresis often begins soon after surgery. Fluid replacement is adjusted hourly based on kidney function and urine output. Urine-tinged drainage on the abdominal dressing may indicate leakage from the ureter implanted into the bladder and the health care provider should be notified. The donor patient may have a flank or laparoscopic incision(s) where the kidney was removed. The recipient has an abdominal incision where the kidney was placed in the iliac fossa. The urinary catheter is usually used for 2 to 3 days to monitor urine output and kidney function.

Nutritional support and management are essential across the entire continuum of chronic kidney disease. Which statements would be considered true related to nutritional therapy (select all that apply)? a. Fluid is not usually restricted for patients receiving peritoneal dialysis. b. Sodium and potassium may be restricted in someone with advanced CKD. c. Decreased fluid intake and a low potassium diet are hallmarks of the diet for a patient receiving hemodialysis. d. Decreased fluid intake and a low potassium diet are hallmarks of the diet for a patient receiving peritoneal dialysis. e. Increased fluid intake and a diet with potassium-rich foods are hallmarks of a diet for a patient receiving hemodialysis.

a. Fluid is not usually restricted for patients receiving peritoneal dialysis. b. Sodium and potassium may be restricted in someone with advanced CKD. c. Decreased fluid intake and a low potassium diet are hallmarks of the diet for a patient receiving hemodialysis.

The patient has been feeling tired lately and has gained weight; reports thickened, dry skin and increased cold sensitivity even though it is now summer. Which endocrine diagnostic test should be done first? a. Free thyroxine (FT4) b. Serum growth hormone (GH) c. Follicle stimulating hormone (FSH) d. Magnetic resonance imaging (MRI) of the head

a. Free thyroxine (FT4) The manifestations the patient is experiencing could be related to hypothyroidism. Free thyroxine (FT4) is considered a better indicator of thyroid function than total T4 and could be done to evaluate the patient for hypothyroidism. Growth hormone excess could cause thick, leathery, oily skin but does not demonstrate the other manifestations. FSH is manifest with menstrual irregularity and would be useful in distinguishing primary gonadal problems from pituitary insufficiency. MRI is the examination of choice for radiologic evaluation of the pituitary gland and the hypothalamus but would not be the first diagnostic study to further explore the basis of these manifestations.

During discharge teaching of a patient with Addisons disease, which statement by the patient indicates that the nurse needs to do additional teaching? a. I should always call the doctor if I develop vomiting or diarrhea b. if my weight goes down, my dosage of steroid is probably too high c. i should double or trip my steroid dose if i undergo rigorous physical activity d. i need to carry an emergency kit with injectable hydrocortisone in case i cant take my medication by mouth

a. I should always call the doctor if I develop vomiting or diarrhea

A patient received a kidney transplant last month. Because of the effects of immunosuppressive drugs and CKD, what complication of transplantation should the nurse be assessing the patient for to decrease the risk of mortality? a. Infection b. Rejection c. Malignancy d. Cardiovascular disease

a. Infection is a significant cause of morbidity and mortality after transplantation because the surgery, the immunosuppressive drugs, and the effects of CKD all suppress the body's normal defense mechanisms, thus increasing the risk of infection. The nurse must assess the patient as well as use aseptic technique to prevent infections. Rejection may occur but for other reasons. Malignancy occurrence increases later due to immunosuppressive therapy. Cardiovascular disease is the leading cause of death after renal transplantation but this would not be expected to cause death within the first month after transplantation.

A patient with ulcerative colitis has a total proctocolectomy with formation of a terminal ileum stoma. What is the most important nursing intervention for this patient postoperatively? a. Measure the ileostomy output to determine the status of the patient's fluid balance. b. Change the ileostomy appliance every 3 to 4 hours to prevent leakage of drainage onto the skin. c. Emphasize that the ostomy is temporary and the ileum will be reconnected when the large bowel heals. d. Teach the patient about the high-fiber, low-carbohydrate diet required to maintain normal ileostomy drainage.

a. Initial output from a newly formed ileostomy may be as high as 1500 to 2000 mL daily and intake and output must be accurately monitored for fluid and electrolyte imbalance. Ileostomy bags may need to be emptied every 3 to 4 hours but the appliance should not be changed for several days unless there is leakage onto the skin. A terminal ileum stoma is permanent and the entire colon has been removed. A return to a normal, presurgical diet is the goal for the patient with an ileostomy, with restrictions based only on the patient's individual tolerances.

A patient with a gunshot wound to the abdomen complains of increasing abdominal pain several hours after surgery to repair the bowel. What action should the nurse take first? a. Take the patient's vital signs. b. Notify the health care provider. c. Position the patient with the knees flexed. d. Determine the patient's IV intake since the end of surgery.

a. It is likely that the patient could be developing a peritonitis, which could be life-threatening, and assessment of vital signs for hypovolemic shock should be done to report to the health care provider. If an IV line is not in place, it should be inserted and pain may be eased by flexing the knees.

When caring for a patient with primary hyperaldosteronism, the nurse would question a physician's order for the use of a. Lasix b. amiloride (midamor) c. spironolactone (aldactone) d. aminoglutethimide (cytadren)

a. Lasix (R- hyperaldosteronism is an excess of aldosterone, which is manifested by sodium and water retention and potassium excretion. Lasix is a potassium-wasting diuretic that would increase the potassium deficiency. Aminoglutethimide blocks aldosterone synthesis; amiloride is apotassium-sparing diuretic; and spironolactone blocks mineralocorticoid receptors in the kidney, increasing secretion of sodium and water and retention of potassium.)

Metabolic acidosis occurs in the oliguric phase of AKI as a result of impairment of a. ammonia synthesis. b. excretion of sodium. c. excretion of bicarbonate. d. conservation of potassium.

a. Metabolic acidosis occurs in AKI because the kidneys cannot synthesize ammonia or excrete acid products of metabolism, resulting in an increased acid load. Sodium is lost in urine because the kidneys cannot conserve sodium. Impaired excretion of potassium results in hyperkalemia. Bicarbonate is normally generated and reabsorbed by the functioning kidney to maintain acidbase balance.

The nurse plans care for the patient with APSGN based on what knowledge? a. Most patients with APSGN recover completely or rapidly improve with conservative management. b. Chronic glomerulonephritis leading to renal failure is a common sequela to acute glomerulonephritis. c. Pulmonary hemorrhage may occur as a result of antibodies also attacking the alveolar basement membrane. d. A large percentage of patients with APSGN develop rapidly progressive glomerulonephritis, resulting in kidney failure.

a. Most patients recover completely from acute poststreptococcal glomerulonephritis (APSGN) with supportive treatment. Chronic glomerulonephritis that progresses insidiously over years and rapidly progressive glomerulonephritis that results in renal failure within weeks or months occur in only a few patients with APSGN. In Goodpasture syndrome, antibodies are present against both the GBM and the alveolar basement membrane of the lungs and dysfunction of both renal and pulmonary are present.

Which descriptions characterize acute kidney injury (select all that apply)?

a. Primary cause of death is infection. c. Disease course is potentially reversible.

A 72-year-old woman is diagnosed with diabetes. The nurse recognizes that management of diabetes in the older adult a. does not require as tight glucose control as in younger diabetics. b. is usually not treated unless the patient becomes severely hyperglycemic. c. does not include treatment with insulin because of limited dexterity and vision. d. usually requires that a younger family member be responsible for care of the patient.

a. Rationale: Because the clinical manifestations of long-term complications of diabetes take 10 to 20 years to develop, and because tight glucose control in the older patient is associated with an increase frequency of hypoglycemia, the goals for glycemic control are not as rigid as in the younger population. Treatment is indicated, and insulin may be used if the patient does not respond to oral agents. The patient's needs, rather than age, determine the responsibility of other in care.

Two days following a self-managed hypoglycemic episode at home, the patient tells the nurse that his blood glucose levels since the episode have been between 80 and 90mg/dL. The best response by the nurse is, a. "That is a good range for your glucose levels." b. "You should call your health care provider because you need to have your insulin increased." c. "That level is too low in view of your recent hypoglycemia, and you should increase your food intake." d. "You should only take half your insulin dosage for the next few days to get your glucose level back to normal."

a. Rationale: Blood glucose levels of 80 to 90mg/dL (4.5-5 mmol/L) are within the normal range and are desired in the patient with diabetes, even following a recent hypoglycemic episode. Hypoglycemia is often caused by a single event, such as skipping a meal or taking too much insulin or vigorous exercise; once corrected, normal control should be maintained.

A patient with diabetes calls the clinic because she is experiencing nausea and flulike symptoms. The nurse advises the patient to a. administer the usual insulin dosage. b. hold fluid intake until the nausea subsides. c. come to the clinic immediately for evaluation and treatment. d. monitor the blood glucose every 1 to 2 hours and call if the glucose rises over 150 mg/dL (8.3 mmol/L)

a. Rationale: During minor illnesses, the patient with diabetes should continue drug therapy and food intake. Insulin is important because counter regulatory hormones may raise blood glucose during the stress of illness, and food or a carbohydrate liquid substitution is important because during illness the body requires extra energy to deal with the stress of the illness. Blood glucose monitoring should be done every 4 hour, and the health care provider should be notified if the level is >240mg/dL (13.9 mmol/L) or if fever, ketonuria, or nausea and vomiting occur.

The major advantage of using an insulin pump is that a. tight glycemic control can be maintained. b. errors in insulin dosing are less likely to happen. c. complications of insulin therapy are prevented. d. frequent blood glucose monitoring is unnecessary.

a. Rationale: Insulin pumps provide tight glycemic control by continuous subcutaneous insulin infusion based on the patient's basal profile, with bolus doses at mealtime at the patient's discretion. Errors in insulin dosing and complications of insulin therapy are still potential risks with insulin pumps.

When caring for a patient with metabolic syndrome, the nurse gives the highest priority to teaching the patient about a. maintaining a normal weight. b. performing daily aerobic exercise. c. eliminating red mean from the diet. d. monitoring the blood glucose periodically.

a. Rationale: Metabolic syndrome is a cluster of abnormalities that include elevated insulin levels, elevated triglycerides and low-density lipoproteins (LDL), and decreased high-density lipoproteins (HDL). These abnormalities greatly increase the risk for cardiovascular disease associated with diabetes that can be prevented or delayed with weight loss. Exercise is also important, but normal weight is most important.

In nutritional management of all types of diabetes, it is important for the patient to a. eat regular meals at regular times. b. restrict calories to promote moderate weight loss. c. eliminate sucrose and other simple sugars from the diet. d. limit saturated fat intake to 30% of dietary intake.

a. Rationale: The body requires food at regularly spaced intervals throughout the day, and omission or delay of meals can result in hypoglycemia, especially for the patient taking insulin or oral hypoglycemic agents. Weight loss may be recommended in type 2 diabetes if the individual is overweight, but many patients with type 1 diabetes are thin and require an increase in caloric intake. Fewer than 7% of total calories should be from saturated fats, and simple sugars should be limited, but moderate amounts can used if counted as a part of total carbohydrate intake.

A diabetic patient is learning to mix regular insulin and NPH insulin in the same syringe. The nurse determines that additional teaching is needed when the patient a. withdraws the NPH dose in the syringe first b. injects air equal to the NPH dose into the NPH vial first. c. removes any air bubbles after withdrawing the first insulin. d. adds air equal to the insulin dose into the regular vial and withdraws the dose.

a. Rationale: When mixing regular with a longer-acting insulin, regular insulin should always be drawn in the syringe first to prevent contamination of the regular insulin vial with longer-acting insulin additives. Air is added in the neutral protamine Hagedorn (NPH) vial; then air is added to the regular vial, and the regular insulin is withdrawn, bubbles are removed, and the dose of NPH is withdrawn.

Which information should be included in health teaching for clients taking insulin? (Select all that apply.) a. Recognize signs of hypoglycemic reaction. b. Adhere to the prescribed diet. c. Take insulin as prescribed. d. Monitor blood glucose level. e. Be sure to exercise. f. Keep appointments with health care provider. g. Alter insulin dose based on how you're feeling.

a. Recognize signs of hypoglycemic reaction. b. Adhere to the prescribed diet. c. Take insulin as prescribed. d. Monitor blood glucose level.

The patient has a thoracic spinal cord lesion and incontinence that occurs equally during the day and night. What type of incontinence is this patient experiencing? a. Reflex incontinence c. Functional incontinence b. Overflow incontinence d. Incontinence after trauma

a. Reflex incontinence occurs with no warning, equally during the day and night, and with spinal cord lesions above S2. Overflow incontinence is when the pressure of urine in the overfull bladder overcomes sphincter control and is caused by bladder or urethral outlet obstruction. Functional incontinence is loss of urine resulting from cognitive, functional, or environmental factors. Incontinence after trauma or surgery occurs when fistulas have occurred or after a prostatectomy.

A patient with inflammatory bowel disease has a nursing diagnosis of imbalanced nutrition: less than body requirements related to decreased nutritional intake and decreased intestinal absorption. Which assessment data support this nursing diagnosis? a. Pallor and hair loss c. Anorectal excoriation and pain b. Frequent diarrhea stools d. Hypotension and urine output below 30 mL/hr

a. Signs of malnutrition include pallor from anemia, hair loss, bleeding, cracked gingivae, and muscle weakness, which support a nursing diagnosis that identifies impaired nutrition. Diarrhea may contribute to malnutrition but is not a defining characteristic. Anorectal excoriation and pain relate to problems with skin integrity. Hypotension relates to problems with fluid deficit.

Which characteristic is more likely with acute pyelonephritis than with a lower UTI? a. Fever c. Urgency b. Dysuria d. Frequency

a. Systemic manifestations of fever and chills with leukocytosis and nausea and vomiting are more common in pyelonephritis than in a lower UTI. Dysuria, frequency, and urgency can be present with both.

A postoperative patient has a nursing diagnosis of pain related to effects of medication and decreased GI motility as evidenced by abdominal pain and distention and inability to pass flatus. Which nursing intervention is most appropriate for this patient? a. Ambulate the patient more frequently. b. Assess the abdomen for bowel sounds. c. Place the patient in high Fowler's position. d. Withhold opioids because they decrease bowel motility.

a. The abdominal pain and distention that occur from the decreased motility of the bowel should be treated with increased ambulation and frequent position changes to increase peristalsis. If the pain is severe, cholinergic drugs, rectal tubes, or application of heat to the abdomen may be prescribed. Assessment of bowel sounds is not an intervention to relieve the pain and a high Fowler's position is not indicated. Opioids may still be necessary for pain control and motility can be increased by other means.

A patient is admitted to the hospital with a diagnosis of Cushing syndrome. On physical assessment of the patient, what should the nurse expect to find? a. Hypertension, peripheral edema, and petechiae b. Weight loss, buffalo hump, and moon face with acne c. Abdominal and buttock striae, truncal obesity, and hypotension d. Anorexia, signs of dehydration, and hyperpigmentation of the skin

a. The effects of adrenocortical hormone excess, especially glucocorticoid excess, include weight gain from accumulation and redistribution of adipose tissue, sodium and water retention, glucose intolerance, protein wasting,loss of bone structure, loss of collagen, and capillary fragility leading to petechiae. Clinical manifestations of adrenocortical hormone deficiency include hypotension, dehydration, weight loss, and hyperpigmentation of the skin.

A 82-year-old man is admitted with an acute attack of diverticulitis. What should the nurse include in his care? a. Monitor for signs of peritonitis. b. Treat with daily medicated enemas. c. Prepare for surgery to resect the involved colon. d. Provide a heating pad to apply to the left lower quadrant.

a. The inflammation and infection of diverticula cause small perforations with spread of the inflammation to the surrounding area in the intestines. Abscesses may form or complete perforation with peritonitis may occur. Systemic antibiotic therapy is often used but medicated enemas would increase intestinal motility and increase the possibility of perforation, as would the application of heat. Surgery is only necessary to drain abscesses or to resect an obstructing inflammatory mass.

The wound, ostomy, and continence (WOC) nurse selects the site where the ostomy will be placed. What should be included in the consideration for the site? a. The patient must be able to see the site. b. Outside the rectus muscle area is the best site. c. It is easier to seal the drainage bag to a protruding area. d. The ostomy will need irrigation, so area should not be tender.

a. The patient must be able to see the site. In selection of the ostomy site, the WOC nurse will want a site visible to the patient so the patient can take care of it, within the rectus muscle to avoid hernias, and on a flat surface to more easily create a good seal with the drainage bag.

A patient is admitted to the emergency department with acute abdominal pain. What nursing intervention should the nurse implement first? a. Measurement of vital signs b. Administration of prescribed analgesics c. Assessment of the onset, location, intensity, duration, and character of the pain d. Physical assessment of the abdomen for distention, bowel sounds, and pigmentation changes

a. The patient with an acute abdomen may have significant fluid or blood loss into the abdomen and evaluation of blood pressure (BP) and heart rate (HR) should be the first intervention, followed by assessment of the abdomen and the nature of the pain. Analgesics should be used cautiously until a diagnosis can be determined so that symptoms are not masked.

A patient has sought care because of a loss of 25 lb over the past 6 months, during which the patient claims to have made no significant dietary changes. What potential problem should the nurse assess the patient for? a. Thyroid disorders b. Diabetes insipidus c. Pituitary dysfunction d. Parathyroid dysfunction

a. Thyroid disorders Hyperthyroidism is associated with weight loss. Alterations in pituitary function, such as diabetes insipidus, and parathyroid dysfunction are not commonly associated with this phenomenon.

What preoperative instruction should the nurse give to the patient scheduled for a subtotal thyroidectomy? a. How to support the head with the hands when turning in bed b. Coughing should be avoided to prevent pressure on the incision c. Head and neck will need to remain immobile until the incision heals d. Any tingling around the lips or in the fingers after surgery is expected and temporary

a. To prevent strain on the suture line postoperatively, the patient's head must be manually supported while turning and moving in bed but range-of-motion exercises for the head and neck are also taught preoperatively to be gradually implemented after surgery. There is no contraindication for coughing and deep breathing and these should be carried out postoperatively. Tingling around the lips or fingers is a sign of hypocalcemia, which may occur if the parathyroid glands are inadvertently removed during surgery. This sign should be reported immediately.

Which are appropriate therapies for patients with diabetes mellitus (select all that apply)? a. Use of statins to treat dyslipidemia b. Use of diuretics to treat nephropathy c. Use of ACE inhibitors to treat nephropathy d. Use of laser photocoagulation to treat retinopathy

a. Use of statins to treat dylipidemia c. Use of ACE inhibitors to treat nephropathy d. Use of laser photocoagulation to treat retinopathy

A man with end-stage kidney disease is scheduled for hemodialysis following healing of an arteriovenous fistula (AVF). What should the nurse explain to him that will occur during dialysis? a. He will be able to visit, read, sleep, or watch TV while reclining in a chair. b. He will be placed on a cardiac monitor to detect any adverse effects that might occur. c. The dialyzer will remove and hold part of his blood for 20 to 30 minutes to remove the waste products. d. A large catheter with two lumens will be inserted into the fistula to send blood to and return it from the dialyzer.

a. While patients are undergoing hemodialysis, they can perform quiet activities that do not require the limb that has the vascular access. Blood pressure is monitored frequently and the dialyzer monitors dialysis function but cardiac monitoring is not usually indicated. The hemodialysis machine continuously circulates both the blood and the dialysate past the semipermeable membrane in the machine. Graft and fistula access involve the insertion of two needles into the site: one to remove blood from and the other to return blood to the dialyzer.

individualized nutrition therapy for patients using conventional, fixed insulin regimens should include teaching the patient to a. eat regular meals at regular times b. restrict calories to promote moderate weight loss c. eliminate sucrose and other simple sugars from the diet d. limit saturated fat intake to 30% of dietary calorie intake

a. eat regular meals at regular times

The nurses adminstered 28 units of Humulin N, an intermediate-acting insulin, to a client diagnosed with type 1 diabetes at 1600. Which intervention should the nurse implement? a. ensure the client eats the bedtime snack b. determine how much food the client ate at lunch c. perform a glucometer reading at 0700 d. offer the client protein after administering insulin

a. ensure the client eats the bedtime snack

A patient with acromegaly is treated w a transphenoidal hypophysectomy. Postoperatively, the nurse a. ensures that any clear nasal drainage is tested for glucose b. maintains the patient flat in bed to prevent cerebrospinal fluid leak c. assists the patient with toothbrushing Q4H to keep the surgical area clean d. encourages deep breathing and coughing to prevent respiratory complications

a. ensures any clear nasal drainage is tested for glucose (R- a transphenoidal hypophysectomy involves entry into the sella turcica through an incision in the upper lip and gingiva into the floor of the nose and the sphenoid sinuses. Postoperative clear nasal drainage with glucose content indicates CSF leakage from an open connection to the brain, putting the pt at risk for meningitis. After surgery, the pt is positioned with head elevated to avoid pressure on the sella turcica, coughing and straining are avoided to prevent increased ICP and CSF leakage, and although mouth care is required Q4H toothbrushing should not be performed for 7-10post sx.)

the 68 year old client diagnosed with hyperthyroidism is being treated with radioactive iodine therapy. Which intervention should the nurse discuss with the client? a. explain it will take up to a month for symptoms of hyperthyroidism to subside b. teach the iodine therapy will have to be tapered slowing over one week c. discuss the client will have to be hospitalized during the radioactive therapy d. inform the client after therapy the client will not have to take any meds

a. explain it will take up to a month for symptoms of hyperthyroidism to subside

Which clinical manifestations may be seen in a client experiencing a hypoglycemic (insulin) reaction? (Select all that apply.) a. headache b. nervousness c. tremor d. excessive perspiration e. tachycardia f. abdominal pain

a. headache b. nervousness c. tremor d. excessive perspiration e. tachycardia

the patient with diabetes has a blood glucose level of 248 mg/dL. Which manifestations in the patient would the nurse understand as being related to this blood glucose level? (select all that apply) a. headache b. unsteady gait c. abdominal cramps d. emotional changes e. increase in urination f. weakness and fatigue

a. headache c. abdominal cramps e. increase in urination f. weakness and fatigue

the client is admitted to the ICU diagnosed with DKA. Which intervention should the nurse implement? select all that apply a. maintain adequate ventilation b. assess fluid volume status c. administer IV potassium d. check for urinary ketones e. monitor intake and output

a. maintain adequate ventilation b. assess fluid volume status c. administer IV potassium d. check for urinary ketones e. monitor intake and output

Preoperative instructions for the patient scheduled for a subtotal thyroidectomy includes teaching the patient a. how to support the head w hands when moving b. that coughing should due avoided to prevent pressure on the incision c. that the head and neck will need to remain immobile until the incision heals d. that any tingling around the lips or in the fingers after surgery is expected and temporary

a. how to support the head with the hands when moving (R- to prevent strain on suture line postoperatively, head must be manually supported while turning and moving in bed, but range-of-motion exercise for the head and neck are also taught preoperatively to be gradually implemented after surgery. There is no contraindication for coughing and deep breathing, and they should be carrier out postoperatively. Tingling around the lips or fingers is a sign of hypocalcemia, which may occur if the parathyroid glands are inadvertently removed during surgery, and should be reported immediately.)

Patients with chronic kidney disease experience an increased incidence of cardiovascular disease related to (select all that apply) a. hypertension. b. vascular calcifications. c. a genetic predisposition. d. hyperinsulinemia causing dyslipidemia. e. increased high-density lipoprotein levels.

a. hypertension. b. vascular calcifications. d. hyperinsulinemia causing dyslipidemia.

The nurse is performing discharge teaching for a client diagnosed with cushings disease. Which statement by the client demonstrates an understanding of the instructions? a. i will be sure to notify my HCP if i start to run a fever b. before i stop taking the prednisone, i will be taught how to taper it off c. if i get weak and shaky, i need to eat some hard candy or drink some juice d. it is fine if i continue to participate in weekend games of tackle football

a. i will be sure to notify my HCP if i start to run a fever

Two days following a colectomy for an abdominal mass, a patient reports gas pains and abdominal distention. The nurse plans care for the patient based on the knowledge that the symptoms are occurring as a result of a. impaired peristalsis. b. irritation of the bowel. c. nasogastric suctioning. d. inflammation of the incision site.

a. impaired peristalsis. Until peristalsis returns to normal following anesthesia, the patient may experience slowed gastrointestinal motility leading to gas pains and abdominal distention. Irritation of the bowel, nasogastric suctioning, and inflammation of the surgical site do not cause gas pains or abdominal distention.

The elderly client is admitted to the intensive care department diagnosed with severe HHNS. which collaborative intervention should the nurse include in the plan of care? a. infuse 0.9% normal saline IV b. administer intermediate-acting insulin c. perform a blood glucometer cheeks daily d. monitor arterial blood gas results

a. infuse 0.9% normal saline IV

A 72 year old woman is diagnosed with diabetes. What does the nurse recognize about the management of diabetes in the older adult? a. it is more difficult to achieve strict glucose control than in younger patients b. it usually is not treated unless the patient becomes severely hyperglycemic c. it does not include treatment with insulin because of limited dexterity and vision d. it usually requires that a younger family member be responsible for care of the patient

a. it is more difficult to achieve strict glucose control than in younger patients

The nurse is teaching the patient with prediabetes ways to prevent or delay the development of type two diabetes. What information should be included (select all that apply). a. maintain a healthy weight b. exercise for 60 minutes each day c. have BP checked regularly d. assess for visual changes on monthly basis e. monitor for polyuria, polyphagia, and polydipsia

a. maintain a healthy weight e. monitor for polyuria, polyphagia, and polydipsia

During care of a patient with syndrome of inappropriate ADH (SIADH), the nurse should a. monitor neurologic status Q2H or more often if needed b. keep the HOB elevated to prevent ADH release c. teach the patient receiving treatment with diuretics to restrict sodium intake d. notify the physician if the p's blood pressure decreases more than 20mmHg from baseline

a. monitor neurologic status Q2H or more often if needed R- the pt with SIADH has marked dilution hyponatremia and should be monitored for decreased neurologic function and convulsions every 2 hours. ADH release is reduced by keeping the HOB flat to increase left atrial filling pressure, and sodium intake is supplemented because of hyponatremia and sodium loss caused by diuretics. A reduction in BP indicates a reduction in total fluid vo and is an expected outcome of treatment.)

the nurse is teaching the client diagnosed with hyperthyroidism. which information should be taught to the client? select all that apply a. notify the HCP if a 3 pound weight loss occurs in 2 weeks b. discuss ways to cope with the emotional liability c. notify the HCP if taking OTC meds d. carry a medical information card or bracelet e. teach how to take thyroid meds correctly

a. notify the HCP if a 3 pound weight loss occurs in 2 weeks b. discuss ways to cope with the emotional liability c. notify the HCP if taking OTC meds d. carry a medical information card or bracelet

What are the major symptoms that characterize diabetes? (Select all that apply.) a. polyuria b. polyphagia c. polyposia d. polydipsia e. polyrrhea

a. polyuria b. polyphagia d. polydipsia

Which drugs may cause hyperglycemia? (Select all that apply.) a. prednisone b. epinephrine c. levothyroxine d. hydrochlorothiazide

a. prednisone b. epinephrine d. hydrocholorothiazide

One of the nruse's most important roles in relation to acute poststreptococcal golmerulonephritis is to* a. promote early diagnosis and treatment of sore throats and skin lesions b.encourage patients to obtain antibiotic therapy for upper respiratory tract infections c.teach patients with APSGN that long term prophylactic antibiotic therapy is necessary to prevent recurrence d.monitor patients for respiratory symptoms that indicate the disease is affecting the alveolar basement membrane

a. promote early diagnosis and treatment of sore throats and skin lesions

The client diagnosed with addisons disease is admitted to the ED after a day at the lake. The client is lethargic, forgetful, and weak. Which intervention should the nurse implement? a. start an IV with an 18 gauge needle and infuse NS rapidly b. have the client wait in the waiting room until a bed is available c. obtain a permit for the client to receive a blood transfusion d. collect urinalysis and blood samples for a CBC and calcium level

a. start an IV with an 18 gauge needle and infuse NS rapidly

The patient with type 2 diabetes is being put on acarbose (precose) and wants to know why she is taking it. What should the nurse include in this patients teaching (select all that apply) a. take it with the first bite of each meal b. it is not used in patients with HF c. endogenous glucose production is decreased d. effectiveness is measured by 2 hour postprandial glucose e. it delays glucose absorption from the GI tract

a. take it with the first bite of each meal d. effectiveness is measured by 2 hour postprandial glucose e. it delays glucose absorption from the GI tract

the diabetic educator is teach a class on diabetes type 1 and is discussing sick-day rules. Which intervention should the diabetes educator include in the discussion? select all that apply a. take the diabetic medication even if unable to eat the clients normal diabetic diet b. if unable to eat, drink liquids equal to the clients normal caloric diet c. if is not necessary to notify the HCP if ketones are in the urine d. test blood glucose levels and test urine ketones once a day and keep a record e. call the HCP if glucose levels are higher than 180 mg/dL

a. take the diabetic medication even if unable to eat the clients normal diabetic diet b. if unable to eat, drink liquids equal to the clients normal caloric diet e. call the HCP if glucose levels are higher than 180 mg/dL

what accurately describes prostate cancer detection and/or treatment (select all that apply) a. the symptoms of pelvic or perineal pain, fatigue, and malaise be may be present b. palpation of the prostate reveals hard and asymmetric enlargment with areas of induration or nodules c. orchiectomy is a treatment option for all patients with prostatic cancer except those with stage IV tumors d. the preferred hormonal therapy for treament of prostate cancer includes estrogen and androgen receptor blockers e. early detection of cancer of the prostate is incrased with annual rectal examination and serum prostatic acid phosphatase (PAP) measurememts f. an annul prostate examination is recommended starting at age 45 for African American men because of the increased mortality rate from prostatic cancer in this population

a. the symptoms of pelvic or perineal pain, fatigue, and malaise be may be present b. palpation of the prostate reveals hard and asymmetric enlargment with areas of induration or nodules f. an annul prostate examination is recommended starting at age 45 for African American men because of the increased mortality rate from prostatic cancer in this population

What are manifestations of DKA (select all that apply) a. thirst b. ketonuria c. dehydration d. metabolic acidosis e. Kussmaul respirations f. sweet, fruity breath odor

a. thirst b. ketonuria c. dehydration d. metabolic acidosis e. Kussmaul respirations f. sweet, fruity breath odor

Which clinical manifestations may be seen in a client experiencing diabetic ketoacidosis (hyperglycemia)? (Select all that apply.) a. thirst b. polyuria c. bradycardia d. Kussmaul's sign e. dry mucous membranes f. fruity breath odor

a. thirst b. polyuria d. Kussmaul's sign e. dry mucous membranes f. fruity breath odor

when teaching the patient with Type 1 diabetes, what should the nurse emphasize as the major disadvantage of using an insulin pump? a. tight glycemic control can be maintained b. errors in insulin dosing are less likely to occur c. complications of insulin therapy are prevented d. frequent blood glucose monitoring is unnecessary

a. tight glycemic control can be maintained

A patient with diabetes is learning to mix regular insulin and NPH insulin in the same syringe. The nurse determines that additional teaching is needed when the patient does what? a. withdraws the NPH dose into the syringe first b. infects air equal to the NPH dose into the NPH vial first c. removes any air bubbles after withdrawing the first insulin d. adds air equal to the insulin dose into the regular vial and withdraws the dose

a. withdraws the NPH dose into the syringe first

medication used to treat polynephritis

antispasmodics MOA: inhibit the action of acetylcholine and relaxes smooth muscle of the ureters and bladder S.E.: dizziness, drowsiness, blurred vision, dry mouth, constipation, increased heart rate, delirium NRSG CONSIDER: do not give to pts with closed-angle glaucoma or hypotension Rx: oxybutynin chloride(Ditropan); flavoxate(Urispas); belladonna and opium suppositories

The patient has not voided for 6 hours after a herniorrhaphy. What action should you take? A. Document the findings. B. Have patient sit on toilet and listen to running water. C. Obtain a residual urine sample. D. Apply pressure on the suprapubic region to promote urge.

b Rationale After a hernia repair, the patient may have difficulty voiding. Voiding is expected by 6 hours after the operation. Measures to help the patient void include using the normal sitting position, pouring warm water over the perineum, and listening to running water. Because it has been 6 hours, action beyond documentation is required. The residual urine is the amount remaining after the patient has voided. The bladder can be palpated for distention, but the first nursing action for someone who does not have an atonic bladder is not manual pressure. Reference: 1048

Which is correct information about an anal fissure? A. Classic symptom is black, tarry stool. B. Defecating is described as "passing broken glass." C. Avoid bulk to present large stool that irritates the fissure. D. Typical treatment includes packing the rectum to absorb drainage.

b Rationale An anal fissure is a crack in the lining of the anal wall, producing the key symptoms of bright red rectal bleeding and severe anal pain on defecation. Black, tarry stool indicates bleeding higher in the gastrointestinal tract. Treatment of an anal fissure includes fiber supplements, adequate fluid intake, sitz baths, and topical analgesics. Rectal packing is used after a hemorrhoidectomy. Reference: 1053

Which is correct information regarding gastrointestinal stromal tumors (GISTs)? A. It is cancer of the villa cells that affect absorption of nutrients. B. It is cancer of cells controlling movement of food through stomach. C. They are usually detected by occult blood stool screening. D. Screening looks for elevation of the carcinogenic embryonic antigen (CEA) level.

b Rationale GISTs are a rare form of cancer that originates in cells found in the gastrointestinal tract wall. These cells (interstitial cells of Cajal) are part of the autonomic nervous system. They help control the movement of food and liquid through the stomach and intestines. About 65% are found in the stomach. Because initial symptoms are usually vague and similar to many other gastrointestinal problems, it is difficult to detect early. Colon cancer is usually detected by occult blood stool screening or by screening for elevation of the level of CEA. Reference: 1052

The patient with lactase deficiency is at risk for which nutritional deficiency? A. Vitamin K B. Calcium C. Folic acid D. Cobalamin (vitamin B12)

b Rationale Milk products are the main source of calcium in the American diet, and a lactose-intolerant person can acquire a calcium deficiency. The other nutrients are not a concern. Reference: 1051

The patient has cystic fibrosis. Which finding indicates malabsorption? A. Small stature and thin B. Greasy, foul-smelling, mushy stool C. Urine specific gravity of 1.031 D. Current jelly-like stool

b Rationale People with cystic fibrosis have malabsorption because of pancreatic dysfunction. Steatorrhea is a symptom of failure to properly digest fats. Children with cystic fibrosis typically have growth issues, but small stature and being thin does not necessarily indicate malabsorption. The urine specific gravity is slightly high and indicates dehydration, not malabsorption. Current jelly-like stool is a classic symptom of intussusception. Reference: 1049

Which behavior can help people with lactase deficiency to tolerate milk? A. Drink the milk on an empty stomach 30 minutes before meals. B. Add Lactaid to the consumed milk. C. Consume the milk with calcium supplement. D. Use on buttermilk.

b Rationale The lactase enzyme (Lactaid) is available as an over-the-counter product to mix with milk and break down the lactose before the milk is ingested. Some can tolerate the lactose better if lactase is taken with meals. Although lack of milk consumption can lead to a calcium deficiency, consuming it with calcium will not make a difference. The symptoms result from an inability to digest lactose, not a calcium deficiency. Buttermilk has additional fat, but it also has lactose. Reference: 1051

Teaching in relation to home management after a laparoscopic cholecystectomy should include a. keeping the bandages on the puncture sites for 48 hours. b. reporting any bile-colored drainage or pus from any incision. c. using over-the-counter antiemetics if nausea and vomiting occur. d. emptying and measuring the contents of the bile bag from the T tube every day. (Lewis 1042)

b Rationale: The following discharge instructions are taught to the patient and caregiver after a laparoscopic cholecystectomy: First, remove the bandages on the puncture site the day after surgery and shower. Second, notify the surgeon if any of the following signs and symptoms occur: redness, swelling, bile-colored drainage or pus from any incision; and severe abdominal pain, nausea, vomiting, fever, or chills. Third, gradually resume normal activities. Fourth, return to work within 1 week of surgery. Fifth, resume a usual diet, but a low-fat diet is usually better tolerated for several weeks after surgery.

The male patient is Jewish, has a history of gout, and has been diagnosed with renal calculi. Which treatment will be used with this patient (select all that apply)? a. Reduce dietary oxalate b. Administer allopurinol c. Administer α-penicillamine d. Administer thiazide diuretics e. Reduce animal protein intake f. Reduce intake of milk products

b, e. This patient is most likely to have uric acid calculi, which have a high incidence in Jewish men, and gout is a predisposing factor. The treatment will include allopurinol and reducing animal protein intake to reduce purine, as uric acid is a waste product from purine metabolism. Reducing oxalate and using thiazide diuretics help to treat calcium oxalate calculi. Administration of α-penicillamine and tiopronin prevent cystine crystallization for cystine calculi. Reducing intake of milk products to reduce calcium intake may be used with calcium calculi.

During routine health screening, a patient is found to have a fasting plasma glucose (FPG) of 132 mg/dL (7.33. mmol/L). At a follow-up visit, a diagnostic of diabetes would be made based on (select all that apply) a. glucosuria of 3+. b. an A1C of 7.5%. c. a FPG of ≥126mg/dL (6.9 mmol/L). d. random blood glucose of 126 mg/dL (7.0 mmol/L). e. a 2-hour oral glucose tolerance test (OGTT) of 190 mg/dL (10.5 mmol/L).

b,c. Rationale: The patient has one prior test result that meets criteria for a diagnosis of diabetes, but on a subsequent day must again have results from on of the three tests that meet the criteria for diabetes diagnosis. These criteria include a fasting plasma glucose level of ≥126mg/dL (7.0 mmol/L), or A1C ≥6.5% or a 2 hour OGTT level ≥200mg/dL (11.1 mmol/L). Both the fasting plasma glucose (FPG) and A1C would confirm a diagnosis of diabetes in this patient.

A patient who is given a bisacodyl (Dulcolax) suppository asks the nurse how long it will take to work. The nurse replies that the patient will probably need to use the bedpan or commode within which time frame after administration? a. 2-5 minutes b. 15-60 minutes c. 2-4 hours d. 6-8 hours

b. 15-60 minutes Bisacodyl suppositories usually are effective within 15 to 60 minutes of administration, so the nurse should plan accordingly to assist the patient to use the bedpan or commode.

What is an appropriate nursing intervention for the patient with hyperparathyroidism? a. Pad side rails as a seizure precaution. b. Increase fluid intake to 3000 to 4000 mL daily. c. Maintain bed rest to prevent pathologic fractures. d. Monitor the patient for Trousseau's and Chvostek's signs.

b. A high fluid intake is indicated in hyperparathyroidism to dilute the hypercalcemia and flush the kidneys so that calcium stone formation is reduced. Seizures are not associated with hyperparathyroidism. Impending tetany of hypoparathyroidism after parathyroidectomy can be noted with Trousseau's and Chvostek's signs. The patient with hyperparathyroidism is at risk for pathologic fractures resulting from decreased bone density but mobility is encouraged to promote bone calcification.

following electrohydraulic lithotripsy for treatment of renal calculi, the patient has a nursing diagnosis of risk for infection related to the introduction of bacteria following manipulation of the urinary tract. What is the most appropriate nursing intervention for this patient? a. monitor for hematuria b. encourage fluid intake of 3L/day c. apply moist heat to the flank area d. strain all urine through gauze or a special strainer

b. A high fluid intake maintains dilute urine, which decreases bacterial concentration in addition to washing stone fragments and expected blood through the urinary system following lithotripsy. High urine output also prevents supersaturation of minerals. Moist heat to the flank may be helpful to relieve muscle spasms during renal colic and all urine should be strained in patients with renal stones to collect and identify stone composition but these are not related to infection.

During assessment of the patient who has a nephrectomy, what should the nurse expect to find? a. Shallow, slow respirations b. Clear breath sounds in all lung fields c. Decreased breath sounds in the lower left lobe d. Decreased breath sounds in the right and left lower lobes

b. A nephrectomy incision is usually in the flank, just below the diaphragm or in the abdominal area. Although the patient is reluctant to breathe deeply because of incisional pain, the lungs should be clear. Decreased sounds and shallow respirations are abnormal and would require intervention.

On examining a patient 8 hours after having surgery to create a colostomy, what should the nurse expect to find? a. Hyperactive, high-pitched bowel sounds b. A brick-red, puffy stoma that oozes blood c. A purplish stoma, shiny and moist with mucus d. A small amount of liquid fecal drainage from the stoma

b. A normal new colostomy stoma should appear bright red, have mild to moderate edema, and have a small amount of bleeding or oozing of blood when touched. A purplish stoma indicates inadequate blood supply and should be reported. The colostomy will not have any fecal drainage for 2 to 4 days but there may be some earlier mucus or serosanguineous drainage. Bowel sounds after extensive bowel surgery will be diminished or absent

What is the primary way that a nurse will evaluate the patency of an AVF? a. Palpate for pulses distal to the graft site. b. Auscultate for the presence of a bruit at the site. c. Evaluate the color and temperature of the extremity. d. Assess for the presence of numbness and tingling distal to the site.

b. A patent arteriovenous fistula (AVF) creates turbulent blood flow that can be assessed by listening for a bruit or palpated for a thrill as the blood passes through the graft. Assessment of neurovascular status in the extremity distal to the graft site is important to determine that the graft does not impair circulation to the extremity but the neurovascular status does not indicate whether the graft is open.

What medication is used with thyrotoxicosis to block the effects of the sympathetic nervous stimulation of the thyroid hormones? a. Potassium iodide b. Atenolol (Tenormin) c. Propylthiouracil (PTU) d. Radioactive iodine (RAI)

b. The β-adrenergic blocker atenolol is used to block the sympathetic nervous system stimulation by thyroid hormones. Potassium iodide is used to prepare the patient for thyroidectomy or for treatment of thyrotoxic crisis to inhibit the synthesis of thyroid hormones. Antithyroid medications inhibit the synthesis of thyroid hormones. Radioactive iodine (RAI) therapy destroys thyroid tissue, which limits thyroid hormone secretion.

In providing care for the patient with adult-onset polycystic kidney disease, what should the nurse do? a. Help the patient to cope with the rapid progression of the disease b. Suggest genetic counseling resources for the children of the patient c. Expect the patient to have polyuria and poor concentration ability of the kidneys d. Implement measures for the patient's deafness and blindness in addition to the renal problems

b. Adult-onset polycystic kidney disease is an inherited autosomal dominant disorder that often manifests after the patient has children but the children should receive genetic counseling regarding their life choices. The disease progresses slowly, eventually causing progressive renal failure. Hereditary medullary cystic disease causes poor concentration ability of the kidneys and classic Alport syndrome is a hereditary nephritis that is associated with deafness and deformities of the optic lens.

The patient asks the nurse to explain what the physician meant when he said the patient had an anorectal abscess. Which description should the nurse use to explain this to the patient? a. Ulcer in anal wall c. Sacrococcygeal hairy tract b. Collection of perianal pus d. Tunnel leading from the anus or rectum

b. An anorectal abscess is a collection of perianal pus. An ulcer in the anal wall is an anal fissure. Sacrococcygeal hairy tract describes a pilonidal sinus. A tunnel leading from the anus or rectum is an anorectal fistula.

In which type of dialysis does the patient dialyze during sleep and leave the fluid in the abdomen during the day? a. Long nocturnal hemodialysis b. Automated peritoneal dialysis (APD) c. Continuous venovenous hemofiltration (CVVH) d. Continuous ambulatory peritoneal dialysis (CAPD)

b. Automated peritoneal dialysis (APD) is the type of dialysis in which the patient dialyzes during sleep and leaves the fluid in the abdomen during the day. Long nocturnal hemodialysis occurs while the patient is sleeping and is done up to six times per week. Continuous venovenous hemofiltration (CVVH) is a type of continuous renal replacement therapy used to treat AKI. Continuous ambulatory peritoneal dialysis (CAPD) is dialysis that is done with exchanges of 1.5 to 3 L of dialysate at least four times daily.

A teaching plan developed by the nurse for the patient with a new ileal conduit includes instructions to do what? a. Clean the skin around the stoma with alcohol every day. b. Use a wick to keep the skin dry during appliance changes. c. Use sterile supplies and technique during care of the stoma. d. Change the appliance every day and wash it with soap and warm water.

b. Because the stoma continuously drains urine, a wick formed of a rolled-up 4 × 4 gauze or a tampon is held against the stoma to absorb the urine while the skin is cleaned and a new appliance is attached. The skin is cleaned with warm water only because soap and other agents cause drying and irritation and clean, not sterile, technique is used. The appliance should be left in place for as long as possible before it loosens and allows leakage onto the skin, perhaps up to 14 days.

The nurse has identified the nursing diagnosis of fatigue for a patient who is hypothyroid. What should the nurse do while caring for this patient? a. Monitor for changes in orientation, cognition, and behavior. b. Monitor for vital signs and cardiac rhythm response to activity. c. Monitor bowel movement frequency, consistency, shape, volume, and color. d. Assist in developing well-balanced meal plans consistent with level of energy expenditure.

b. Cardiorespiratory response to activity is important to monitor in this patient to determine the effect of activities and plan activity increases. Monitoring changes in orientation, cognition, and behavior are interventions for impaired memory. Monitoring bowels is needed to plan care for the patient with constipation. Assisting with meal planning will help the patient with imbalanced nutrition: more than body requirements to lose weight if needed.

What information should be included when the nurse teaches a patient about colostomy irrigation? a. Infuse 1500 to 2000 mL of warm tap water as irrigation fluid. b. Allow 30 to 45 minutes for the solution and feces to be expelled. c. Insert a firm plastic catheter 3 to 4 inches into the stoma opening. d. Hang the irrigation bag on a hook about 36 inches above the stoma.

b. Following infusion of the fluid into the stoma, the solution and feces will take about 30 to 45 minutes to return and the patient can plan to read or perform other quiet activities during the wait time. Between 500 and 1000 mL of warm tap water should be used. A cone tip on the end of the tubing prevents bowel damage that could occur if a stiff plastic catheter is used. Fluid should be elevated about 18 to 24 inches above the stoma, or to about shoulder level, to prevent too rapid infusion of the solution and cramping.

The nurse is planning care for a 68-year-old patient with an abdominal mass and suspected bowel obstruction. Which factor in the patient's history increases the patient's risk for colorectal cancer? a. Osteoarthritis b. History of colorectal polyps c. History of lactose intolerance d. Use of herbs as dietary supplements

b. History of colorectal polyps A history of colorectal polyps places this patient at risk for colorectal cancer. This tissue can degenerate over time and become malignant. Osteoarthritis, lactose intolerance, and the use of herbs do not pose additional risk to the patient.

The patient has persistent and continuous pain at McBurney's point. The nursing assessment reveals rebound tenderness and muscle guarding with the patient preferring to lie still with the right leg flexed. What should the nursing interventions for this patient include? a. Laxatives to move the constipated bowel b. NPO status in preparation for possible appendectomy c. Parenteral fluids and antibiotic therapy for 6 hours before surgery d. NG tube inserted to decompress the stomach and prevent aspiration

b. The patient's manifestations are characteristic of appendicitis. After laboratory test and CT scan confirmation, the patient will have surgery. Laxatives are not used. The 6 hours of fluids and antibiotics preoperatively would be used only if the appendix was ruptured. The NG tube is more likely to be used with abdominal trauma.

Which of the following would be the highest priority information to include in preoperative teaching for a 68-year-old patient scheduled for a colectomy? a. How to care for the wound b. How to deep breathe and cough c. The location and care of drains after surgery d. What medications will be used during surgery

b. How to deep breathe and cough Because anesthesia, an abdominal incision, and pain can impair the patient's respiratory status in the postoperative period, it is of high priority to teach the patient to cough and deep breathe. Otherwise, the patient could develop atelectasis and pneumonia, which would delay early recovery from surgery and hospital discharge. Care for the wound and location and care of the drains will be briefly discussed preoperatively, but done again with higher priority after surgery. Knowing which drugs will be used during surgery may not be meaningful to the patient and should be reviewed with the patient by the anesthesiologist.

A client is newly diagnosed with type 1 diabetes mellitus and requires daily insulin injections. Which instruction should the nurse include in the teaching of insulin administration? a. Teach the family members to administer glucagon by injection if the client has a hyperglycemic reaction. b. Instruct the client about the necessity for compliance with prescribed insulin therapy. c. Teach the client that hypoglycemic reactions more likely to occur at the onset of action time. d. Instruct the client in the care of insulin container and syringe handling.

b. Instruct the client about the necessity for compliance with prescribed insulin therapy.

An important nursing intervention for a patient with a small intestinal obstruction who has an NG tube is to a. offer ice chips to suck PRN. b. provide mouth care every 1 to 2 hours. c. irrigate the tube with normal saline every 8 hours. d. keep the patient supine with the head of the bed elevated 30 degrees.

b. Mouth care should be done frequently for the patient with a small intestinal obstruction who has an NG tube because of vomiting, fecal taste and odor, and mouth breathing. No ice chips are allowed when a patient is NPO because of a bowel obstruction. The NG tube should be checked for patency and irrigated as ordered. The position of the patient should be one of comfort.

A patient has a right ureteral catheter placed following a lithotripsy for a stone in the ureter. In caring for the patient after the procedure, what is an appropriate nursing action? a. Milk or strip the catheter every 2 hours. b. Measure ureteral urinary drainage every 1 to 2 hours. c. Irrigate the catheter with 30-mL sterile saline every 4 hours. d. Encourage ambulation to promote urinary peristaltic action.

b. Output from ureteral catheters must be monitored every 1 to 2 hours because an obstruction will cause overdistention of the renal pelvis and renal damage. The renal pelvis has a capacity of only 3 to 5 mL and if irrigation is ordered, no more than 5 mL of sterile saline is used. The patient with a ureteral catheter is usually kept on bed rest until specific orders for ambulation are given. Suprapubic tubes may be milked to prevent obstruction of the catheter by sediment and clots.

Which assessment parameter is of highest priority when caring for a patient undergoing a water deprivation test? a. Serum glucose b. Patient weight c. Arterial blood gases d. Patient temperature

b. Patient weight A patient is at risk for severe dehydration during a water deprivation test. The test should be discontinued and the patient rehydrated if the patient's weight drops more than 2 kg at any time. The other assessment parameters do not assess fluid balance.

To prevent the most common serious complication of PD, what is important for the nurse to do? a. Infuse the dialysate slowly. b. Use strict aseptic technique in the dialysis procedures. c. Have the patient empty the bowel before the inflow phase. d. Reposition the patient frequently and promote deep breathing.

b. Peritonitis is a common complication of peritoneal dialysis (PD) and may require catheter removal and termination of dialysis. Infection occurs from contamination of the dialysate or tubing or from progression of exit-site or tunnel infections and strict sterile technique must be used by health professionals as well as the patient to prevent contamination. Too-rapid infusion may cause shoulder pain and pain may be caused if the catheter tip touches the bowel. Difficulty breathing, atelectasis, and pneumonia may occur from pressure of the fluid on the diaphragm, which may be prevented by elevating the head of the bed and promoting repositioning and deep breathing.

A patient with type 1 diabetes uses 20U of 70/30 neutral protamine Hagedorn (NPH/regular) in the morning and at 6:00pm. When teaching the patient about this regimen, the nurse stresses that a. hypoglycemia is most likely to occur before the noon meal. b. a set meal pattern with a bedtime snack is necessary to prevent hypoglycemia. c. flexibility in food intake is possible because insulin is available 24 hours/day. d. pre-meal glucose checks are required to determine needed changes in daily dosing.

b. Rationale: A split-mixed dose of insulin requires that the patient adhere to a set meal pattern to provide glucose for the action of the insulins, and a bedtime snack is usually required when patients take a long-acting insulin late in the day to prevent nocturnal hypoglycemia. Hypoglycemia is most likely to occur with this dose late in the afternoon and during the night. When premixed formulas are used, flexible dosing based on glucose levels is not recommended.

To prevent hyperglycemia or hypoglycemia with exercise, the nurse teaches the patient using glucose-lowering agents that exercise should be undertaken a. only after a 10- to 15-g carbohydrate snack is eaten. b. about 1 hour after a eating, when blood glucose levels are rising. c. when glucose monitoring reveals that the blood glucose is in the normal range. d. when blood glucose levels are high because exercise always has a hypoglycemic effect.

b. Rationale: During exercise, a diabetic person needs both adequate glucose to prevent exercise-induced hypoglycemia and adequate insulin because conterregulatory hormones are produced during the stress of exercise and may cause hyperglycemia. Exercise after meals is best, but a 10- to 15-g carbohydrate snack may be taken if exercise is performed before meals or is prolonged. Blood glucose levels should be monitored before, during, and after exercise to determine the effect of exercise on the levels.

Goals of nutritional therapy for the patient with type 2 diabetes include maintenance of a. ideal body weight. b. normal serum glucose and lipid levels. c. a special diet using diabetic foods. d. five small meals per day with a bedtime snack.

b. Rationale: Maintenance of as near-normal blood glucose levels as possible and a achievement of optimal serum lipid levels with dietary modification are believed to be the most important factors in preventing both short- and long-term complications of diabetes. There is no specific "diabetic diet," and use of dietetic foods is not necessary for diabetes control. Most diabetics eat three meals a day, and some require a bedtime snack for control of nighttime hypoglycemia. A reasonable weight, which may or may not be an ideal body weight, is also a goal of nutritional therapy.

A nurse working in an outpatient clinic plans a screening program for diabetes. Recommendations for screening would include a. OGTT fro all minority populations every year. b. FPG for all individuals at age 45 and then every 3 years. c. testing all people under the age of 21 for islet cell antibodies. d. testing for type 2 diabetes only in overweight or obese individuals.

b. Rationale: The American Diabetes Association recommends that testing for type 2 diabetes with a FPG should be considered for all individuals at the age of 45 and above and, if normal, repeated every 3 years. Testing for immune markers of type 1 diabetes is not recommended. Testing of a younger age or more frequently should be done for members of a high-risk ethnic population, including African Americans, Hispanics, Native Americans, Asians Americans, and Pacific Islanders.

In diabets, atherosclerotic disease affecting the cerebrovascular, cardiovascular, and peripheral vascular systems a. can be prevent by tight glucose control. b. occurs with a higher frequency and earlier onset than in the nondiabetic population. c. is cause by the hyperinsulinemia related to insulin resistance common in type 2 diabetes. d. cannot be modified by reduction of risk factors such as smoking, obesity, and high fat intake.

b. Rationale: The development of atherosclerotic vessel disease seems to be promoted by the altered lipid metabolism common to diabetes, and although tight glucose control may help delay the process, it does not prevent it completely. Atherosclerosis in diabetic patients does respond somewhat to a reduction in general risk factors, as it does in nondiabetics, and reduction in fat intake, control of hypertension, abstention from smoking, maintenance of normal weight, and regular exercise should be carried out by diabetic patients.

What is a nursing intervention that is indicated for a male patient following an inguinal herniorrhaphy? a. Applying heat to the inguinal area c. Applying a truss to support the operative site b. Elevating the scrotum with a scrotal support d. Encouraging the patient to cough and deep breathe

b. Scrotal edema is a common and painful complication after an inguinal hernia repair and can be relieved in part by application of ice and elevation of the scrotum with a scrotal support. Heat would increase the edema and the discomfort and a truss is used to keep unrepaired hernias from protruding. Coughing is discouraged postoperatively because it increases intraabdominal pressure and stress on the repair site.

In replying to a patient's questions about the seriousness of her chronic kidney disease (CKD), the nurse knows that the stage of CKD is based on what? a. Total daily urine output b. Glomerular filtration rate c. Degree of altered mental status d. Serum creatinine and urea levels

b. Stages of chronic kidney disease are based on the GFR. No specific markers of urinary output, mental status, or azotemia classify the degree of chronic kidney disease (CKD).

The female patient with a UTI also has renal calculi. The nurse knows that these are most likely which type of stone? a. Cystine c. Uric acid b. Struvite d. Calcium phosphate

b. Struvite calculi are most common in women and always occur with UTIs. They are also usually large staghorn type.

Which therapies for BPH are done on an outpatient basis? (select all that apply) a. intraprostatic urethral stents b. TUNA c. TUIP d. TUMT e. VLAP

b. TUNA c. TUIP d. TUMT

When the client is prescribed glipizide (Glucotrol), the nurse knows that which side effects/adverse effects may be expected? (Select all that apply.) a. Tachypnea b. Tachycardia c. Increase alertness d. Increased weight gain e. Visual disturbances f. Hunger

b. Tachycardia e. Visual disturbances f. Hunger

A female patient with a UTI has a nursing diagnosis of risk for infection related to lack of knowledge regarding prevention of recurrence. What should the nurse include in the teaching plan instructions for this patient? a. Empty the bladder at least 4 times a day. b. Drink at least 2 quarts of water every day. c. Wait to urinate until the urge is very intense. d. Clean the urinary meatus with an antiinfective agent after voiding.

b. The bladder should be emptied at least every 3 to 4 hours. Fluid intake should be increased to about 2000 mL/ day without caffeine, alcohol, citrus juices, and chocolate drinks, because they are potential bladder irritants. Cleaning the urinary meatus with an antiinfective agent after voiding will irritate the meatus but the perineal area should be wiped from front to back after urination and defecation to prevent fecal contamination of the meatus.

A patient with ulcerative colitis undergoes the first phase of a total proctocolectomy with ileal pouch and anal anastomosis. On postoperative assessment of the patient, what should the nurse expect to find? a. A rectal tube set to low continuous suction b. A loop ileostomy with a plastic rod to hold it in place c. A colostomy stoma with an NG tube in place to provide pouch irrigations d. A permanent ileostomy stoma in the right lower quadrant of the abdomen

b. The initial procedure for a total proctocolectomy with ileal pouch and anal anastomosis includes a colectomy, rectal mucosectomy, ileal reservoir construction, ileoanal anastomosis, and a temporary ileostomy. A loop ileostomy is the most common temporary ileostomy and it may be held in place with a plastic rod for the first week. A rectal tube to suction is not indicated in any of the surgical procedures for ulcerative colitis. A colostomy is not used and an NG tube would not be used to irrigate the pouch. A permanent ileostomy stoma would be expected following a total proctocolectomy with a permanent ileostomy.

How is the most common form of malabsorption syndrome treated? a. Administration of antibiotics b. Avoidance of milk and milk products c. Supplementation with pancreatic enzymes d. Avoidance of gluten found in wheat, barley, oats, and rye

b. The most common type of malabsorption syndrome is lactose intolerance and it is managed by restricting the intake of milk and milk products. Antibiotics are used in cases of bacterial infections that cause malabsorption, pancreatic enzyme supplementation is used for pancreatic insufficiency, and restriction of gluten is necessary for control of adult celiac disease (celiac sprue, gluten-induced enteropathy).

A 68-year-old man with a history of heart failure resulting from hypertension has AKI as a result of the effects of nephrotoxic diuretics. Currently his serum potassium is 6.2 mEq/L (6.2 mmol/L) with cardiac changes, his BUN is 108 mg/dL (38.6 mmol/L), his serum creatinine is 4.1 mg/dL (362 mmol/L), and his serum HCO3 − is 14 mEq/L (14 mmol/L). He is somnolent and disoriented. Which treatment should the nurse expect to be used for him? a. Loop diuretics b. Renal replacement therapy c. Insulin and sodium bicarbonate d. Sodium polystyrene sulfonate (Kayexalate)

b. This patient has at least three of the six common indications for renal replacement therapy (RRT), including (1) high potassium level, (2) metabolic acidosis, and (3) changed mental status. The other indications are (4) volume overload, resulting in compromised cardiac status (this patient has a history of hypertension), (5) BUN greater than 120 mg/dL, and (6) pericarditis, pericardial effusion, or cardiac tamponade. Although the other treatments may be used, they will not be as effective as RRT for this older patient. Loop diuretics and increased fluid are used if the patient is dehydrated. Insulin and sodium bicarbonate can be used to temporarily drive the potassium into the cells. Sodium polystyrene sulfonate (Kayexalate) is used to actually decrease the amount of potassium in the body.

A physician just told a patient that she has a volvulus. When the patient asks the nurse what this is, what is the best description for the nurse to give her? a. Bowel folding on itself c. Emboli of arterial supply to the bowel b. Twisting of bowel on itself d. Protrusion of bowel in weak or abnormal opening

b. Volvulus is the bowel twisting on itself. The bowel folding on itself is intussusception. Emboli of arterial blood supply to the bowel is vascular obstruction. Protrusion of bowel in a weak or abnormal opening is a hernia

A patient with continuous bladder irrigation following a prostatectomy tells the nurse that he has bladder spasms and leaking of urine around the catheter. What should the nurse do first? a. slow the rate of the irrigation b. assess the patency of the catheter c. encourage the patient to try and urinate around the catheter d. administer a belladonna and opium (B&O) suppository as prescribed

b. assess the patency of the catheter

What medication is used with thyrotoxicosis to block the effects of the sympathetic nervous stimulation of the thyroid hormones? a. potassium iodine b. atenolol (tenormin) c. Propylthiouracil (PTU) d. radioactive iodine (RAI)

b. atenolol (tenormin)

When the patient with parathyroid disease experiences symptoms of hypocalcemia, a measure that can be used to temporarily raise serum calcium levels is to a. administer IV normal saline b. have the patient rebreathe in a paper bag c. administer Lasix as ordered d. administer oral phosphorous supplements

b. have the patient rebreathe in a paper bag (R- rebreathing in a paper bag promotes CO2 retention in blood, which lowers pH and creates an acidosis. An academia enhances solubility and ionization of calcium, increasing the proportion of total body Ca available in physiologically active form and relieving the sx of hypocalcemia. Saline promotes calcium excretion, as does Lasix. Phosphate levels in blood are reciprocal to calcium and an increase in phosphate promotes calcium excretion.)

Which s/s should make the nurse suspect the client is experiencing a thyroid storm? a. obstipation and hypoactive bowel sounds b. hyperpyrexia and extreme tachycardia c. hypotension and bradycarida d. decreased respirations and hypoxia

b. hyperpyrexia and extreme tachycardia

A patient with SIADH is treated with water restriction and administration of IV fluids. The nurses evaluates that treatment has been effective when the patient experiences a. increased urine output, decreased serum sodium, and increased urine specific gravity b. increased urine output, increased serum sodium, and decreased urine specific gravity c. decreased urine output, increased serum sodium, and decreased urine specific gravity d. decreased urine output, decreased serum sodium, and increased urine specific gravity

b. increased urine output, increased serum sodium, and decreased urine specific gravity (rationale- the patient with SIADH has water retention with hyponatremia, decreased urine output and concentrated urine with high specific gravity. improvement in the patient's condition reflected by increased urine output, normalization of serum sodium, and more water in the urine, decreasing the specific gravity.)

A client is to receive insulin before breakfast, and the time of breakfast tray delivery is variable. The nurse knows that which insulin should not be administered until the breakfast tray has arrived and the client is ready to eat? a. Humulin N b. lispro (Humalog) c. glargine (Lantus) d. Humulin R

b. lispro (Humalog)

A patient has a right urethral catheter placed following a lithotripsy for a stone in the ureter. IN caring for the patient after the procedure, what is an appropriate nursing action a. milk or strip the catheter every 2 hours b. measure uretheral urinary drainage every 1 to 2 hours c. irrigate the catheter with 30 mL sterile saline every 4 hours d. encourage ambulation to promote urinary peristaltic action

b. measure uretheral urinary drainage every 1 to 2 hours

To prevent complications in the patient with Cushing syndrome, the nurse monitors the patient for a. hypotension b. hypoglycemia c. cardiac arrhythmias d. decreased cardiac output

c. cardiac arrhythmias rationale- electrolyte changes that occur in Cushing syndrome include sodium retention and potassium excretion by the kidney, resulting in hypokalemia

The UAP on the medical floor tells the nurse the client diagnosed with DKA wants something else to eat for lunch. Which intervention should the nurse implement? a. instruct the UAP to get the client additional food b. notify the dietician about the clients request c. request the HCP increase the clients caloric intake d. tell the UAP the client cannot have anything else

b. notify the dietician about the clients request

In type 1 diabetes there is an osmotic effect of glucose when insulin deficiency prevents the use of glucose for energy. Which classic symptom is caused by the osmotic effect of glucose? a. fatigue b. polydipsia c. polyphagia d. recurrent infections

b. polydipsia

Which electrolyte replacement should the nurse anticipate being ordered by the HCP in the client diagnosed with DKA who has been admitted to the ICU? a. glucose b. potassium c. calcium d. sodium

b. potassium

A patient with hypothyroidism is treated h Synthroid. When teaching the pt about the therapy, the nurse a. explains that caloric intake must be reduced when drug therapy is started b. provides written instruction for all information related to the medication therapy c. assures the patient that a return to normal function will occur with replacement therapy d. informs the patient that medications must be taken until hormone balance is reestablished

b. provides written instruction for all information related to the medication therapy (R- bc of mental sluggishness, inattentiveness, and memory loss that occur with hypothyroidism, it is important to provide written instructions and repeat information when teaching pt. Caloric intake can be increased when drug therapy is started, because of an increased metabolic rate, and replacement therapy must be taken for life. Although most pts return to a normal state w treatment, cardiovascular conditions and psychoses may persist.)

Which infection is asymptomatic in the male patient at first then progresses to cystitis, frequent urination, buring on voiding, and epididymitis? a. urosepsis b. renal tb c. urethral diverticula d. goodpasture syndrome

b. renal tb

why are the hormones cortisol, glucagon, epinephrine, and growth hormone referred to as counter regulatory hormones? a. decrease glucose production b. stimulate glucose output by the liver c. increase glucose transport into the cells d. independently regulate glucose level in the blood

b. stimulate glucose output by the liver

The client has developed cushings disease. What statement is the scientific rationale for the development of this diagnosis? a. the client has an autoimmune problem causing the destruction of the adrenal cortex b. the client has been taking steroid meds for an extended period for another diseases process c. the client has a pituitary gland tumor causing the adrenal glands to produce too much cortisol d. the client has developed an adrenal gland problem for which the HCP does not have an explanation

b. the client has been taking steroid meds for an extended period for another diseases process

A patient with ureterolithotomy returns from surgery with a nephrostomy tube in place. Postoperative nursing care of the patient includes:* a.encourage the patient to drink fruit juices and milk b.encouraging fluids of at least 2-3 L/day after nausea has subsided c. irrigating the nephrostomy tube with 10ml of NS solution as needed d. notifying the physician if nephrostomy tube drainage is more than 30ml/hr

b.encouraging fluids of at least 2-3 L/day after nausea has subsided

Polydipsia and polyuria related to diabetes mellitus are primarily due to a.the release of ketones from cells during fat metabolism. b.fluid shifts resulting from the osmotic effect of hyperglycemia. c.damage to the kidneys from exposure to high levels of glucose. d.changes in RBCs resulting from attachment of excessive glucose to hemoglobin.

b.fluid shifts resulting from the osmotic effect of hyperglycemia.

The nurse identifies a risk factor for kidney and bladder cancer in a patient who relates a history of a.aspirin use b.tobacco use c.chronic alcohol abuse d.use of artificial sweeteners

b.tobacco use

The nurse teaches the female paitent who has frequent UTIs that she should* a. take tub baths with bubble bath b.urinate before and after sexual intercourse c.take prophylactic sufonamides for the rest of her life d. restrict fluid intake to prevent the need for frequent voiding

b.urinate before and after sexual intercourse

Which food item should a patient with celiac disease avoid? A. Steamed rice B. Yogurt C. Pancakes D. Raw pineapple

c Rationale A patient with celiac disease cannot digest wheat, rye, or barley. Pancakes are made with flour, which is made with wheat. Reference: 1049-1050

Which is a risk for developing hemorrhoids? A. Body mass index (BMI) of 17 kg/m2 B. Younger than 30 years C. Chronic constipation D. History of lactase deficiency

c Rationale Chronic constipation increases intraluminal pressure, which can predispose to hemorrhoids. Overweight is a risk, not underweight (normal BMI is 18.5 to 24.9 kg/m2). Older adults are at higher risk. Lactase deficiency is not related to hemorrhoids. Reference: 1052

The patient reports that she experiences cramping, flatulence, and abdominal cramping about an hour after consuming milk. What action should you pursue? A. Encourage patient to ensure she is not drinking milk past the expiration date. B. Assess the patient for the presence of dumping syndrome. C. Suggest a hydrogen breath test. D. Ask about recent stressors in the patient's life that could cause gastrointestinal upset.

c Rationale In lactase deficiency, the enzyme to digest lactose in milk is absent. Common symptoms include bloating, flatulence, cramping abdominal pain, and diarrhea 30 minutes to several hours after ingestion. It can be diagnosed with a lactose tolerance test, a hydrogen breath test to assess intolerance, or genetic testing using DNA. The most likely cause should be explored before assuming the food is spoiled or the reaction is a result of emotional stress. Dumping syndrome occurs when the stomach storage area has decreased from surgery and concentrated carbohydrates are consumed. Reference: 1051

Which is the best food for the patient with lactase deficiency? A. Skim milk B. Low-fat ice cream C. Live-culture yogurt D. Cheddar cheese

c Rationale Live-culture yogurt can be consumed if no milk products have been added. The other options all contain lactose. Reference: 1051

Which is the most common causative organism of an anorectal abscess? A. Giardia lamblia B. Clostridium difficile C. Escherichia coli D. The organism for tuberculosis

c Rationale The most common organisms causing perianal pus are E. coli, staphylococci, and streptococci. Reference: 1054

The patient had a colon resection and a colostomy today. You should call the primary provider after noticing which finding during the patient assessment? A. No drainage in the colostomy bag B. Moderate stoma swelling C. Dusky blue stoma D. Stoma that bleeds when touched

c Rationale The stoma should be pink or red to show adequate circulation. Drainage is not expected the first 24 hours because peristalsis has not returned. Mild to moderate swelling of the stoma is expected the first 2 to 3 weeks after surgery. The raw surgical mucosa bleeds with trauma during the first postoperative day. Reference: 1041

A patient with pancreatic cancer is admitted to the hospital for evaluation of possible treatment options. The patient asks the nurse to explain the Whipple procedure that the surgeon has described. The explanation includes the information that a Whipple procedure involves a. creating a bypass around the obstruction caused by the tumor by joining the gallbladder to the jejunum. b. resection of the entire pancreas and the distal portion of the stomach, with anastomosis of the common bile duct and the stomach into the duodenum. c. removal of part of the pancreas, part of the stomach, the duodenum, and the gallbladder, with joining of the pancreatic duct, the common bile duct, and the stomach into the jejunum. d. radical removal of the pancreas, the duodenum, and the spleen, and attachment of the stomach to the jejunum, which requires oral supplementation of pancreatic digestive enzymes and insulin replacement therapy. (Lewis 1042)

c Rationale: The classic operation for pancreatic cancer is a radical pancreaticoduodenectomy, or Whipple procedure. This entails resection of the proximal pancreas (i.e., proximal pancreatectomy), the adjoining duodenum (i.e., duodenectomy), the distal portion of the stomach (i.e., partial gastrectomy), and the distal segment of the common bile duct. The pancreatic duct, common bile duct, and stomach are anastomosed to the jejunum.

The patient is having an esophagoenterostomy with anastomosis of a segment of the colon to replace the resected portion. What initial postoperative care should the nurse expect when this patient returns to the nursing unit? a Turn, deep breathe, cough, and use spirometer every 4 hours. b Maintain an upright position for at least 2 hours after eating. c NG will have bloody drainage and it should not be repositioned. d Keep in a supine position to prevent movement of the anastomosis.

c The patient will have bloody drainage from the nasogastric (NG) tube for 8 to 12 hours, and it should not be repositioned or reinserted without contacting the surgeon. Turning and deep breathing will be done every 2 hours, and the spirometer will be used more often than every 4 hours. Coughing would put too much pressure in the area and should not be done. Because the patient will have the NG tube, the patient will not be eating yet. The patient should be kept in a semi-Fowler's or Fowler's position, not supine, to prevent reflux and aspiration of secretions.

A 83-year-old female patient was found lying on the bathroom floor. She said she fell 2 days ago and has not been able to take her heart medicine or eat or drink anything since then. What conditions could be causing prerenal AKI in this patient (select all that apply)? a. Anaphylaxis b. Renal calculi c. Hypovolemia d. Nephrotoxic drugs e. Decreased cardiac output

c, e. Because the patient has had nothing to eat or drink for 2 days, she is probably dehydrated and hypovolemic. Decreased cardiac output (CO) is most likely because she is older and takes heart medicine, which is probably for heart failure or hypertension. Both hypovolemia and decreased CO cause prerenal AKI. Anaphylaxis is also a cause of prerenal AKI but is not likely in this situation. Nephrotoxic drugs would contribute to intrarenal causes of AKI and renal calculi would be a postrenal cause of AKI.

On assessment of the patient with a renal calculus passing down the ureter, what should the nurse expect the patient to report? a. A history of chronic UTIs b. Dull, costovertebral flank pain c. Severe, colicky back pain radiating to the groin d. A feeling of bladder fullness with urgency and frequency

c. A classic sign of the passage of a calculus down the ureter is intense, colicky back pain that may radiate into the testicles, labia, or groin and may be accompanied by mild shock with cool, moist skin. Many patients with renal stones do not have a history of chronic UTIs. Stones obstructing a calyx or at the ureteropelvic junction may produce dull costovertebral flank pain and large bladder stones may cause bladder fullness and lower obstructive symptoms.

A patient on hemodialysis develops a thrombus of a subcutaneous arteriovenous (AV) graft, requiring its removal. While waiting for a replacement graft or fistula, the patient is most likely to have what done for treatment? a. Peritoneal dialysis b. Peripheral vascular access using radial artery c. Silastic catheter tunneled subcutaneously to the jugular vein d. Peripherally inserted central catheter (PICC) line inserted into subclavian vein

c. A more permanent, soft, flexible Silastic double-lumen catheter is used for long-term access when other forms of vascular access have failed. These catheters are tunneled subcutaneously and have Dacron cuffs that prevent infection from tracking along the catheter.

The patient asks the nurse why she needs to have surgery for a femoral, strangulated hernia. What is the best explanation the nurse can give the patient? a. The surgery will relieve her constipation. b. The abnormal hernia must be replaced into the abdomen. c. The surgery is needed to allow intestinal flow and prevent necrosis. d. The hernia is because the umbilical opening did not close after birth as it should have.

c. A strangulated femoral hernia obstructs intestinal flow and blood supply, thus requiring emergency surgery. The other options are incorrect.

What is included in nursing care that applies to the management of all urinary catheters in hospitalized patients? a. Measuring urine output every 1 to 2 hours to ensure patency b. Turning the patient frequently from side to side to promote drainage c. Using strict sterile technique during irrigation and obtaining culture specimens d. Daily cleaning of the catheter insertion site with soap and water and application of lotion

c. All urinary catheters in hospitalized patients pose a very high risk for infection, especially antibiotic-resistant, health care-associated infections, and scrupulous aseptic technique is essential in the insertion and maintenance of all catheters. Routine irrigations are not performed. Turning the patient to promote drainage is recommended only for suprapubic catheters. Cleaning the insertion site with soap and water should be performed for urethral and suprapubic catheters but lotion or powder should be avoided and site care for other catheters may require special interventions.

During a routine screening colonoscopy on a 56-year-old patient, a rectosigmoidal polyp was identified and removed. The patient asks the nurse if his risk for colon cancer is increased because of the polyp. What is the best response by the nurse? a. "It is very rare for polyps to become malignant but you should continue to have routine colonoscopies." b. "Individuals with polyps have a 100% lifetime risk of developing colorectal cancer and at an earlier age than those without polyps." c. "All polyps are abnormal and should be removed but the risk for cancer depends on the type and if malignant changes are present." d. "All polyps are premalignant and a source of most colon cancer. You will need to have a colonoscopy every 6 months to check for new polyps."

c. Although all polyps are abnormal growths, the most common type of polyp (hyperplastic) is non-neoplastic, as are inflammatory, lipomas, and juvenile polyps. However, adenomatous polyps are characterized by neoplastic changes in the epithelium and most colorectal cancers appear to arise from these polyps. Only patients with a family history of familial adenomatous polyposis (FAP) have close to a 100% lifetime risk of developing colorectal cancer.

A patient returns to the surgical unit with a nasogastric (NG) tube to low intermittent suction, IV fluids, and a Jackson-Pratt drain at the surgical site following an exploratory laparotomy and repair of a bowel perforation. Four hours after admission, the patient experiences nausea and vomiting. What is a priority nursing intervention for the patient? a. Assess the abdomen for distention and bowel sounds. b. Inspect the surgical site and drainage in the Jackson-Pratt. c. Check the amount and character of gastric drainage and the patency of the NG tube. d. Administer prescribed prochlorperazine (Compazine) to control the nausea and vomiting.

c. An adequately functioning nasogastric (NG) tube should prevent nausea and vomiting because stomach contents are continuously being removed. The first intervention in this case is to check the amount and character of the recent drainage and check the tube for patency. Decreased or absent bowel sounds are expected after a laparotomy and the Jackson-Pratt drains only fluid from the tissue of the surgical site. Antiemetics may be given if the NG tube is patent because anesthetic agents may cause nausea.

The patient is receiving the following medications. Which one is prescribed to relieve symptoms rather than treat a disease? a. Corticosteroids c. Antidiarrheal agents b. 6-Mercaptopurine d. Sulfasalazine (Azulfidine)

c. Antidiarrheal agents only relieve symptoms. Corticosteroids, 6-mercaptopurine, and sulfasalazine (Azulfidine) are used to treat and control inflammation with various diseases.

The nurse identifies a need for additional teaching when a patient with acute infectious diarrhea makes which statement? a. "I can use A&D ointment or Vaseline jelly around the anal area to protect my skin." b. "Gatorade is a good liquid to drink because it replaces the fluid and salts I have lost." c. "I may use over-the-counter Imodium or Parepectolin when I need to control the diarrhea." d. "I must wash my hands after every bowel movement to prevent spreading the diarrhea to my family."

c. Antiperistaltic agents, such as loperamide (Imodium) and paregoric, should not be used in infectious diarrhea because of the potential of prolonging exposure to the infectious agent. Demulcent agents may be used to coat and protect mucous membranes in these cases. The other options are all appropriate measures to use in cases of infectious diarrhea.

What is the most common cause of acute pyelonephritis resulting from an ascending infection from the lower urinary tract? a. The kidney is scarred and fibrotic. b. The organism is resistant to antibiotics. c. There is a preexisting abnormality of the urinary tract. d. The patient does not take all of the antibiotics for treatment of a UTI.

c. Ascending infections from the bladder to the kidney are prevented by the normal anatomy and physiology of the urinary tract unless a preexisting condition, such as vesicoureteral reflux or lower urinary tract dysfunction (bladder tumors, prostatic hyperplasia, strictures, or stones), is present. Resistance to antibiotics and failure to take a full prescription of antibiotics for a UTI usually result in relapse or reinfection of the lower urinary tract.

What can patients at risk for renal lithiasis do to prevent the stones in many cases? a. Lead an active lifestyle c. Drink enough fluids to produce dilute urine b. Limit protein and acidic foods in the diet d. Take prophylactic antibiotics to control UTIs

c. Because crystallization of stone constituents can precipitate and unite to form a stone when in supersaturated concentrations, one of the best ways to prevent stones of any type is by drinking adequate fluids to keep the urine dilute and flowing (e.g., an output of about 2 L of urine a day). Sedentary lifestyle is a risk factor for renal stones but exercise also causes fluid loss and a need for additional fluids. Protein foods high in purine should be restricted only for the small percentage of patients with uric acid stones and although UTIs contribute to stone formation, prophylactic antibiotics are not indicated.

What is the priority action for the nurse to take if the patient with type 2 diabetes complains of blurred vision and irritability? a. Call the physician. b. Administer insulin as ordered. c. Check the patients blood glucose level. d. Assess for other neurologic symptoms.

c. Check the patients blood glucose level.

A patient with AKI is a candidate for continuous renal replacement therapy (CRRT). What is the most common indication for use of CRRT? a. Azotemia b. Pericarditis c. Fluid overload d. Hyperkalemia

c. Continuous renal replacement therapy (CRRT) is indicated for the patient with AKI as an alternative or adjunct to hemodialysis to slowly remove solutes and fluid in the hemodynamically unstable patient. It is especially useful for treatment of fluid overload, but hemodialysis is indicated for treatment of hyperkalemia, pericarditis, or other serious effects of uremia.

What does the dialysate for PD routinely contain? a. Calcium in a lower concentration than in the blood b. Sodium in a higher concentration than in the blood c. Dextrose in a higher concentration than in the blood d. Electrolytes in an equal concentration to that of the blood

c. Dextrose or icodextrin or amino acid is added to dialysate fluid to create an osmotic gradient across the membrane to remove excess fluid from the blood. The dialysate fluid has no potassium so that potassium will diffuse into the dialysate from the blood. Dialysate also usually contains higher calcium to promote its movement into the blood. Dialysate sodium is usually less than or equal to that of blood to prevent sodium and fluid retention.

Glomerulonephritis is characterized by glomerular damage caused by a. growth of microorganisms in the glomeruli. b. release of bacterial substances toxic to the glomeruli. c. accumulation of immune complexes in the glomeruli. d. hemolysis of red blood cells circulating in the glomeruli.

c. Glomerulonephritis is not an infection but rather an antibody-induced injury to the glomerulus, where either autoantibodies against the glomerular basement membrane (GBM) directly damage the tissue or antibodies reacting with nonglomerular antigens are randomly deposited as immune complexes along the GBM. Prior infection by bacteria or viruses may stimulate the antibody production but is not present or active at the time of glomerular damage.

What describes the primary difference in treatment of DKA and hyperosmolar hyperglycemia syndrome (HHS) a. DKA requires administration of bicarbonate to correct acidosis b. potassium replacement is not necessary in management of HHS c. HHS requires greater fluid replacement to correct the dehydration d. administration of glucose is withheld in HHS until the blood glucose reaches a normal level

c. HHS requires greater fluid replacement to correct the dehydration

A client is diagnosed with type 2 diabetes mellitus. The nurse is aware that which statement is true? a. Client is most likely a teenager. b. Client is most likely a child younger than 10 years. c. Heredity is a major causative factor. d. Viral infections contribute most to disease development.

c. Heredity is a major causative factor.

A patient with Addison's disease comes to the emergency department with complaints of N/V/D, and fever. The nurse would expect collaborative care to include a. parenteral injections of ACTH b. IV administration of vasopressors c. IV administration of hydrocortisone d. IV administration of D5W with 20mEq of KCl

c. IV administration of hydrocortisone (R- vom and dia are early indicators of addisonian crisis and fever indicates an infection, which s causing additional stress for pt. trtmt of crisis requires immediate glucocorticoid replacement, and IV hydrocortisone, fluids, Na and glucose are necessary for 24hours. Addison's disease is a primary insufficiency of adrenal gland, and ACTH is not effective, nor would vasopressors be effective w fluid deficiency of Addison's. Potassium levels are incd in Addison's dz, and KCl would be contraindicated.)

The patient has peritonitis, which is a major complication of appendicitis. What treatment will the nurse plan to include? a. Peritoneal lavage c. IV fluid replacement b. Peritoneal dialysis d. Increased oral fluid intake

c. IV fluid replacement along with antibiotics, NG suction, analgesics, and surgery would be expected. Peritoneal lavage may be used to determine abdominal trauma. Peritoneal dialysis would not be performed. Oral fluids would be avoided with peritonitis.

The patient comes to the emergency department with intermittent crampy abdominal pain, nausea, projectile vomiting, and dehydration. The nurse suspects a GI obstruction. Based on the manifestations, what area of the bowel should the nurse suspect is obstructed? a. Large intestine c. Upper small intestine b. Esophageal sphincter d. Lower small intestine

c. Intermittent crampy abdominal pain, nausea, projectile vomiting, and dehydration are characteristics of mechanical upper small intestinal obstruction. With continued vomiting, metabolic alkalosis may occur. Large bowel obstruction is characterized by constipation, low-grade abdominal pain, and abdominal distention. Fecal vomiting is seen with lower small intestinal obstruction.

The patient with CKD is brought to the emergency department with Kussmaul respirations. What does the nurse know about CKD that could cause this patient's Kussmaul respirations? a. Uremic pleuritis is occurring. b. There is decreased pulmonary macrophage activity. c. They are caused by respiratory compensation for metabolic acidosis. d. Pulmonary edema from heart failure and fluid overload is occurring.

c. Kussmaul respirations occur with severe metabolic acidosis when the respiratory system is attempting to compensate by removing carbon dioxide with exhalations. Uremic pleuritis would cause a pleural friction rub. Decreased pulmonary macrophage activity increases the risk of pulmonary infection. Dyspnea would occur with pulmonary edema.

The nurse should teach the patient with chronic constipation that which food has the highest dietary fiber? a. Peach c. Dried beans b. Popcorn d. Shredded wheat

c. Of the foods listed, dried beans contain the highest amount of dietary fiber and are an excellent source of soluble fiber. Bran and berries also have large amounts of fiber.

The nurse assesses the diabetic patient's technique of self-monitoring of blood glucose (SMBG) 3 months after initial instruction. An error in the performance of SMBG noted by the nurse that requires intervention is a. doing the SMBG before and after exercising. b. puncturing the finger on the side of the finger pad. c. cleaning the puncture site with alcohol before the puncture. d. holding the hand down for a few minutes before the puncture.

c. Rationale: Cleaning the puncture site with alcohol is not necessary and may interfere with test results and lead to drying and splitting of the fingertips. Washing the hands with warm water is adequate cleaning and promotes blood flow to the fingers. Blood flow is also increase by holding the hand down. Puncture on the side of the finger pad are less painful. Self-monitored blood glucose (SMBG) should be performed before and after exercise.

The nurse determines that a patient with a 2-hour OGTT of 152mg/dL has a. diabetes. b. impaired fasting glucose. c. impaired glucose tolerance. d. elevated glycosylated hemoglobin (Hb)

c. Rationale: Impaired glucose tolerance exists when a 2-hour plasma glucose level is higher than normal but lower than the level diagnostic for diabetes (i.e., 140-199mg/dL). Impaired fasting glucose exists when fasting glucose levels are greater than the normal of 100mg/dL but less than the 126mg/dL diagnostic of diabetes. Both conditions represent a condition known as pre diabetes.

The nurse is assessing a newly admitted diabetic patient. Which of these observations should be addressed as a priority by the nurse? a. Bilateral numbness of both hands b. Stage II pressure ulcer on the right heel c. Rapid respirations with deep inspiration d. Areas of lumps and dents on the abdomen.

c. Rationale: Rapid deep respirations are symptoms of diabetic ketoacidosis (DKA). Stage II ulcers and bilateral numbness are chronic complications of diabetes. The lumps and dents on the abdomen indicate a need to teach the patient about site rotation.

The treatment for DKA and HHS differs primarily in that a. DKA requires administration of bicarbonate to correct acidosis. b. potassium replacement is not necessary in management of HHS. c. HHS require greater fluid replacement to correct the dehydration. d. administration of glucose is withheld in HHS until the blood glucose reaches a normal level.

c. Rationale: The management of DKA is similar to that of HHS except HHS requires greater fluid replacement because of the severe hyperosmolar state. Bicarbonate is not usually given in DKA to correct acidosis unless the pH is <7.0 because administration of insulin will reverse the abnormal fat metabolism. Total body potassium deficit is high in both conditions, requiring potassium administration, and, in both conditions, glucose is added to IV fluids when blood glucose levels fall to 250mg/dL (13.9 mmol/L).

A patient taking insulin has recorded fasting glucose levels above 200mg/dL (11.1 mmol/L) on awakening for the last five mornings. The nurse advises the patient to a. increase the evening insulin dose to prevent the dawn phenomenon. b. use a single dose insulin regimen with an intermediate-acting insulin. c. monitor the glucose level at bedtime, between 2:00 and 4:00 am, and on arising. d. decrease the evening insulin dosage to prevent night hypoglycemia and the Somogyi effect.

c. Rationale: The patient's elevated glucose on arising may be the result of either dawn phenomenon or Somogyi effect, and the best way to determine whether the patient needs more or less insulin is by monitoring the glucose at bedtime, between 2:00 and 4:00 am, and on arising. If predawn levels are below 60mg/dL, the insulin dose should be reduced, but if the 2:00 to 4:00 am blood glucose is high, the insulin should be increased.

When teaching the patient with diabetes about insulin administration, the nurse instructs the patient to a. pull back on the plunger after inserting the needle to check for blood. b. clean the skin at the injection site with an alcohol swab before each injection. c. consistently use the same size of the appropriate strength insulin syringe to avoid dosing errors. d. rotate injection sites from arms to thighs to abdomen with each injection to prevent lipodystrophies.

c. Rationale: U100 insulin must be used with a U100 syringe, but for those using low doses of insulin, syringes are available that have increments of 1 unit instead of 2 units. Errors can be made in dosing if patients switch back and forth between different sizes of syringes. Aspiration before injection of the insulin is not recommended, nor is the use of alcohol to clean the skin. Because the rate of peak serum concentrations varies with the site selected for injection, injections should be rotated within a particular area, such as the abdomen.

Ketoacidosis occurs as a complication of diabetes when a. illnesses causing nausea and vomiting lead to bicarbonate loss with body fluids. b. the glucose level becomes so high that osmotic diuresis promotes fluid and electrolyte loss. c. an insulin deficit causes the body to metabolize large amounts of fatty acids rather than glucose for energy. d. the patient skips meals after taking insulin, leading to rapid metabolism of glucose and breakdown of fats for energy.

c. Rationale: When insulin is insufficient and glucose cannot be used for cellular energy, the body releasees and breaks down stored fats and protein to meet energy needs. Free fatty acids from stored triglycerides are released and metabolized in the liver in such large quantities that ketones are formed. Ketones are acidic and alter the pH of the blood, causing acidosis. Osmotic diuresis occurs as a result of loss of both glucose and ketones in the urine.

Following bowel resection, a patient has a nasogastric (NG) tube to suction, but complains of nausea and abdominal distention. The nurse irrigates the tube as necessary as ordered, but the irrigating fluid does not return. What should be the priority action by the nurse? a. Notify the physician. b. Auscultate for bowel sounds. c. Reposition the tube and check for placement. d. Remove the tube and replace it with a new one.

c. Reposition the tube and check for placement. The tube may be resting against the stomach wall. The first action by the nurse (since this is intestinal surgery and not gastric surgery) is to reposition the tube and check it again for placement. The physician does not need to be notified unless the tube function cannot be restored by the nurse. The patient does not have bowel sounds, which is why the NG tube is in place. The NG tube would not be removed and replaced unless it was no longer in the stomach or the obstruction of the tube could not be relieved.

When providing discharge instructions to a patient following a subtotal thyroidectomy, the nurse advises the patient to a. never miss a daily dose of thyroid replacement therapy b. avoid regular exercise until thyroid function is normalized c. avoid eating foods such as soybeans, turnips, and rutabagas d. use warm salt water gargles several times a day to relieve throat pain

c. avoid eating foods such as soybeans, turnips, and rutabagas Rationale- the patient should avoid goitrogens, foods that inhibit thyroid, such as soybeans, turnips, rutabagas, and peanut skins.

A patient is diagnosed with celiac disease following a workup for iron-deficiency anemia and decreased bone density. The nurse identifies that additional teaching about disease management is needed when the patient makes which statement? a. "I should ask my close relatives to be screened for celiac disease." b. "If I do not follow the gluten-free diet, I might develop a lymphoma." c. "I don't need to restrict gluten intake because I don't have diarrhea or bowel symptoms." d. "It is going to be difficult to follow a gluten-free diet because it is found in so many foods."

c. The autoimmune process associated with celiac disease continues as long as the body is exposed to gluten, regardless of the symptoms it produces, and a lifelong gluten-free diet is necessary. The other statements regarding celiac disease are all true

For a patient with CKD the nurse identifies a nursing diagnosis of risk for injury: fracture related to alterations in calcium and phosphorus metabolism. What is the pathologic process directly related to the increased risk for fractures? a. Loss of aluminum through the impaired kidneys b. Deposition of calcium phosphate in soft tissues of the body c. Impaired vitamin D activation resulting in decreased GI absorption of calcium d. Increased release of parathyroid hormone in response to decreased calcium levels

c. The calcium-phosphorus imbalances that occur in CKD result in hypocalcemia, from a deficiency of active vitamin D and increased phosphorus levels. This leads to an increased rate of bone remodeling with a weakened bone matrix. Aluminum accumulation is also believed to contribute to the osteomalacia. Osteitis fibrosa involves replacement of calcium in the bone with fibrous tissue and is primarily a result of elevated levels of parathyroid hormone resulting from hypocalcemia.

A patient with hypoparathyroidism resulting from surgical treatment of hyperparathyroidism is preparing for discharge. What should the nurse teach the patient? a. Milk and milk products should be increased in the diet. b. Parenteral replacement of parathyroid hormone will be required for life. c. Calcium supplements with vitamin D can effectively maintain calcium balance. d. Bran and whole-grain foods should be used to prevent GI effects of replacement therapy.

c. The hypocalcemia that results from PTH deficiency is controlled with calcium and vitamin D supplementation and possibly oral phosphate binders. Replacement with PTH is not used because of antibody formation to PTH, the need for parenteral administration, and cost. Milk products, although good sources of calcium, also have high levels of phosphate, which reduce calcium absorption. Whole grains and foods containing oxalic acid also impair calcium absorption.

What results in the edema associated with nephrotic syndrome? a. Hypercoagulability c. Decreased plasma oncotic pressure b. Hyperalbuminemia d. Decreased glomerular filtration rate

c. The massive proteinuria that results from increased glomerular membrane permeability in nephrotic syndrome leaves the blood without adequate proteins (hypoalbuminemia) to create an oncotic colloidal pressure to hold fluid in the vessels. Without oncotic pressure, fluid moves into the interstitium, causing severe edema. Hypercoagulability occurs in nephrotic syndrome but is not a factor in edema formation and glomerular filtration rate (GFR) is not necessarily affected in nephrotic syndrome.

While caring for a 77-year-old woman who has a urinary catheter, the nurse monitors the patient for the development of a UTI. What clinical manifestations is the patient most likely to experience? a. Cloudy urine and fever b. Urethral burning and bloody urine c. Vague abdominal discomfort and disorientation d. Suprapubic pain and slight decline in body temperature

c. The usual classic manifestations of UTI are often absent in older adults, who tend to experience nonlocalized abdominal discomfort and cognitive impairment characterized by confusion or decreased level of consciousness rather than dysuria and suprapubic pain.

Thirty percent of patients with kidney cancer have metastasis at the time of diagnosis. Why does this occur? a. The only treatment modalities for the disease are palliative. b. Diagnostic tests are not available to detect tumors before they metastasize. c. Classic symptoms of hematuria and palpable mass do not occur until the disease is advanced. d. Early metastasis to the brain impairs the patient's ability to recognize the seriousness of symptoms.

c. There are no early characteristic symptoms of cancer of the kidney and gross hematuria, flank pain, and a palpable mass do not occur until the disease is advanced. The treatment of choice is a partial or radical nephrectomy, which can be successful in early disease. Many kidney cancers are diagnosed as incidental imaging findings. Targeted therapy is the preferred treatment for metastatic kidney cancer. Radiation is palliative. The most common sites of metastases are the lungs, liver, and long bones.

When instructing a patient regarding a urine study for free cortisol, what is most important for the nurse to tell the patient? a. Save the first voided urine in the morning. b. Maintain a high-sodium diet 3 days before collection. c. Try to avoid stressful situations during the collection period. d. Complete at least 30 minutes of exercise before collecting the urine sample.

c. Try to avoid stressful situations during the collection period. A urine study for free cortisol requires a 24-hour urine collection. The patient should be instructed to avoid stressful situations and excessive physical exercise that could unduly increase cortisol levels. The patient should also maintain a low-sodium diet before and during the urine collection period.

A woman with no history of UTIs who is experiencing urgency, frequency, and dysuria comes to the clinic, where a dipstick and microscopic urinalysis indicate bacteriuria. What should the nurse anticipate for this patient? a. Obtaining a clean-catch midstream urine specimen for culture and sensitivity b. No treatment with medication unless she develops fever, chills, and flank pain c. Empirical treatment with trimethoprim-sulfamethoxazole (TMP-SMX, Bactrim) for 3 days d. Need to have a blood specimen drawn for a complete blood count (CBC) and kidney function tests

c. Unless a patient has a history of recurrent UTIs or a complicated UTI, trimethoprim-sulfamethoxazole (TMPSMX) or nitrofurantoin (Macrodantin) is usually used to empirically treat an initial UTI without a culture and sensitivity or other testing. Asymptomatic bacteriuria does not justify treatment but symptomatic UTIs should always be treated.

What causes the gastrointestinal (GI) manifestation of stomatitis in the patient with CKD? a. High serum sodium levels b. Irritation of the GI tract from creatinine c. Increased ammonia from bacterial breakdown of urea d. Iron salts, calcium-containing phosphate binders, and limited fluid intake

c. Uremic fetor, or the urine odor of the breath, is caused by high urea content in the blood. Increased ammonia from bacterial breakdown of urea leads to stomatitis and mucosal ulcerations. Irritation of the gastrointestinal (GI) tract from urea in CKD contributes to anorexia, nausea, and vomiting. Ingestion of iron salts and calcium-containing phosphate binders, limited fluid intake, and limited activity cause constipation.

Which patient should the nurse plan to teach how to prevent or delay the development of diabetes? a. an obese 50 year old hispanic woman b. a child whose father has type 1 diabetes c. a 34 year old woman whose parents both have type 2 diabetes d. a 12 year old boy whose father has maturity onset diabetes of the young (MODY)

c. a 34 year old woman whose parents both have type 2 diabetes

A patient seen at the clinic for an upper respiratory infection reports receiving subcutaneous somatotropin (Genotropin) when asked by the nurse about his current medications. The nurse questions the patient further about a history of 1. adrenal disease. 2. a pituitary tumor. 3. diabetes insipidus. 4. untreated acromegaly

c. a pituitary tumor.

What tissues require insulin to enable movement of glucose into the tissue cells (select all that apply) a. liver b. brain c. adipose d. blood cells e. skeletal muscles

c. adipose e. skeletal muscles

The nurse should observe the patient for symptoms of ketoacidosis when a. illness causing nausea and vomiting lead to bicarbonate loss with body fluids b. glucose levels become so high that osmotic diuresis promotes fluid and electrolyte loss c. an insulin deficit causes the body to metabolize large amounts of fatty acids rather than glucose for energy d. the patient skips meals after taking insulin, leading to rapid metabolism of glucose and breakdown of fats for energy

c. an insulin deficit causes the body to metabolize large amounts of fatty acids rather than glucose for energy

The client diagnosed with type 2 diabetes is admitted to the intensive care unit with hyperosmolar hyperglycemia nonketonic syndrome (HHNS) coma. Which assessment data should the nurse expect the client to exhibit? a. Kussmauls respiration b. kiarrhea and epigastric pain c. dry mucous membranes d. ketones breath odor

c. dry mucous membranes

A patient is admitted to the hospital in thyrotoxic crisis. On physical assessment of the patient, the nurse would expect to find a. hoarseness and laryngeal stridor b. bulging eyeballs and arrhythmias c. elevated temperature and signs of heart failure d. lethargy progressing suddenly to impairment of consciousness

c. elevated temperature and signs of heart failure (R- a hyperthyroid crisis results in marked manifs of hyperthyroidism, w fever tachycardia, heart failure, shock, hyperthermia, agitation, N/V/D, delirium, and coma. Although exophthalmos may be present in pt w Gravs', it is not a signif factor in hyperthyroid crisis. Hoarsness and laryngeal stridor are ch of tetany of hypoparathyroidsm, and lethargy progressing to coma is characteristic of myxedema coma, a complication of hypothyroidism.

The client with cushings disease has undergone a unilateral adrenalectomy. Which discharge instructions should the nurse discuss with the client? a. instruct the client to take the glucocorticoid and mineralocorticoid meds as prescribed b. teach the client regarding sexual functioning and androgen replacement therapy c. explain the s/s of infection and when to call the HCP d. demonstrate turn, cough, and deep breathing exercises the client should perform q2h

c. explain the s/s of infection and when to call the HCP

The client received 10 units of Humulin R, a fast actign insulin, at 0700. At 1030 the UAP tells the nurse the client has a headache and is really acting "funny." Which intervention should the nurse implement first? a. Instruct the UAP to obtain the blood glucose level b. Have the client drink 8 oz of orange juice c. go to the clients room and assess the client for hypoglycemia d. prepare to administer 1 ampule 50% dextrose IV

c. go to the clients room and assess the client for hypoglycemia

The emergency department nurse is caring for a client diagnosed with HHNS who has a blood glucose of 680 mg.dL. Which question should the nurse ask the client to determine the cause of this acute complication a. when is the last time you took your insulin b. when did you have your last meal c. have you had some type of infection lately d. how long have you had diabetes

c. have you had some type of infection lately

on admission to the ambulatory surgical center, a patient with BPH informs the nurse that he is going to have a laser treatment of his enlarged prostate. The nurse plans patient teaching with the knowledge that the patient will need to know what? a. the effects of general anesthesia b. the possibility of short term incontenence c. home management of an indwelling catheter d. monitoring for postop urinary retention

c. home management of an indwelling catheter

The patient with newly diagnosed diabetes is displaying shakiness, confusion, irritability, and slurred speech. What should the nurse suspect is happening a. DKA b. HHS c. hypoglycemia d. hyperglycemia

c. hypoglycemia

A patient is scheduled for bilateral adrenalectomy. During the postoperative period, the nurse would expect administration of corticosteroids to be a. reduced to promote wound healing b. withheld until sx of hypocortisolism appear c. increased to promote an adequate response to the stress of surgery d. reduced bc excessive hormones are released during surgical manipulation of the glands

c. increased to promote an adequate response to the stress of surgery (R- although pt with Cushing syndrome has excess corticosteroids, removal of the glands and the stress of surgery require that high doses of cortisone be administered postoperatively for several days. The nurse should monitor the pt postoperatively to detect whether large amounts of hormones were released during surgical manipulation and to ensure the healing is satisfactory.)

A patient with mild iatrogenic cushing syndrome is on an alternate-day regimen of corticosteroid therapy. What does the nurse explain to the patient about this regimen? a. it maintains normal adrenal hormone balance b. it prevents ACTH release from the pituitary gland c. it minimizes hypothalmic pituitary adrenal supression d. it provides a more effective therapeutic effect of the drug

c. it minimizes hypothalmic pituitary adrenal supression

In planning care for the patient with Crohn's disease, the nurse recognizes that a major difference between ulcerative colitis and Crohn's disease is that Crohn's disease: a. frequently results in toxic megacolon b. causes fever nutritional deficiencies than ulcerative colitis c. often recurs after surgery, whereas ulcerative colitis is curable with a colectomy d. is manifested by rectal bleeding and anemia more frequently than is ulcerative colitis

c. often recurs after surgery, whereas ulcerative colitis is curable with a colectomy Rationale: Ulcerative colitis affects only the colon and rectum; it can cause megacolon and rectal bleeding, but not nutrient malabsorption. Surgical removal of the colon and rectum cures it. Crohn's disease usually involves the ileum, where bile salts and vitamin cobalamin are absorbed. After surgical treatment, disease recurrence at the site is common.

when caring for a patient following a radical prostectomy with a perineal approach, what is the priority nursing intervention the nurse should use to prevent complications? a. use the chemotherapeutic agents to prevent metastasis b. administer sildenafil (viagra) as needed for erectile dysfunction c. provide wound care after each bowel movement to prevent infection d. insert a small indwelling urinary catheter to prevent urinary retention

c. provide wound care after each bowel movement to prevent infection

A diabetic patient has a serum glucose level of 824 mg/dL (45.7 mmol/L) and is unresponsive. Following assessment of the patient, the nurse suspects diabetic ketoacidosis rather than hyperosmolar hyperglycemic syndrome based on the findings of a. polyuria. b. severe dehydration. c. rapid, deep respirations. d. decreased serum potassium.

c. rapid, deep respirations.

A 55 year old man with a hx of prostate cancer in his family asks the nurse what he can do to decrease the risk of prostate cancer. What should the nurse teach him about prostate cancer risks? a. nothing can decrease the risk because prostate cancer is primarily a disease of aging. b. treatment of any enlargment of the prostate gland will help prevent prostate cancer. c. substituting fresh fruits and veggies for high fat foods in the diet may lower the risk of prostate cancer d. using a natural herb, saw palmetto, has been found to be an effective protection against prostate cancer

c. substituting fresh fruits and veggies for high fat foods in the diet may lower the risk of prostate cancer

when taking a nursing history from a patient with BPH, the nurse would expect the patient to report a. nocturia, dysuria, and bladder spasms b. urinary frequency, hematuria, and perineal pain c. urinary hesitancy, postvoid dribbling, and weak urinary stream d, urinary urgency with a forceful urinary stream and cloudy urine

c. urinary hesitancy, postvoid dribbling, and weak urinary stream

the nurse is discussing ways to prevent DKA with the client diagnosed with type 1 diabetes. Which instruction is the most import to discuss with the client? a. refer the client to the American Diabetes Association b. do not take an OTC meds c. take the prescribed insulin even when unable to eat because of illness d. explain the need to get the annual flu and pneumonia vaccines

c. take the prescribed insulin even when unable to eat because of illness

The nurse is assessing the feet of a client with long term type 2 diabetes. Which assessment data warrant immediate intervention by the nurse? a. the client has crumbling toe nails b. the client has athletes foot c. the client has a necrotic big toe d.the client has thickened toenails

c. the client has a necrotic big toe

The charge nurse is making client assignments in the ICU. Which client should be assigned to the most experienced nurse? a. the client with type 2 diabetes who has a blood glucose level of 348 mg/dL b. the client diagnosed with type 1 diabetes who is experiencing hypoglycemia c. the client with DKA who has multifocal premature ventricular contractions d. the client with HHNS who has a plasma osmolarity of 290 mOsm/L

c. the client with DKA who has multifocal premature ventricular contractions

The nurse is preparing to administer the following meds. Which med should she question a. the thyroid hormone to the client who does not have a T3 or T4 level b. the regular insulin to the client with a blood glucose level of 210 mg/dL c. the loop diuretic with a potassium level of 3.3 mEq/L d. the cardiac glycoside to the client who has a digoxin level of 1.4 mg/dL

c. the loop diuretic with a potassium level of 3.3 mEq/L

The client diagnosed with Type 1 diabetes has a glycosylated hemoglobin (A1C) of 8.1%. which interpretation should the nurse make based on this result? a. this result is below normal levels b. this result is within acceptable levels c. this result is above recommended levels d. this result is dangerously high

c. this result is above recommended levels

During assessment of the pt with acromegaly, the nurse would expect the patient to report a. infertility b. dry, irritated skin c. undesirable changes in appearance d. an increase in height of 2 to 3 inches per year

c. undesirable changes in appearance (R- the increased production of GH in acromegaly causes an increase in thickness and width of bones and enlargement of soft tissues, resulting in marked changes in facial features, oily and coarse skin, and speech difficulties. Height is not increased in adults w GH excess bc the epiphyses of the bones are closed, and infertility is not a common finding because growth hormone is usually the only pituitary hormone involved in acromegaly.)

the extent of urinary obstruction caused by BPH can be determined by which diagnostic study? a. a cystometrgram b. transrectal ultrasound c. urodynamic flow studies d. postvoiding catheterization

c. urodynamic flow studies

The nurse writes a problem of "altered body image" for a 34 year old client diagnosed with cushings disease. Which intervention should be implemented? a. monitor blood glucose levels prior to meals and at bedtime b. perform a head to toe assessment on the client every shift c. use therapeutic communication to allow the client to discus feelings d. assess bowel sounds and temp q4h

c. use therapeutic communication to allow the client to discus feelings

A patient with oral cancer is not eating. A small-bore feeding tube was inserted and the patient started on enteral feedings. Which patient goal would indicate improvement? a Weight gain of 1 kg in 1 week b Administer tube feeding at 25 mL/hr. c Consume 50% of clear liquid tray this shift. d Monitor for tube for placement and gastrointestinal residual

correct answer a The goal for a patient with oral cancer that is not eating would be to note weight gain rather than loss. Consuming 50% of the clear liquid tray is not a realistic goal. Administering feedings, monitoring tube placement, and tolerance are interventions used to achieve the goal.

A patient was admitted with epigastric pain because of a gastric ulcer. Which patient assessment warrants an urgent change in the nursing plan of care? a Back pain 3 or 4 hours after eating a meal b Chest pain relieved with eating or drinking water c Burning epigastric pain 90 minutes after breakfast d Rigid abdomen and vomiting following indigestion

correct answer d A rigid abdomen with vomiting in a patient who has a gastric ulcer indicates a perforation of the ulcer, especially if the manifestations of perforation appear suddenly. Midepigastric pain is relieved by eating, drinking water, or antacids with duodenal ulcers, not gastric ulcers. Back pain 3 to 4 hours after a meal is more likely to occur with a duodenal ulcer. Burning epigastric pain 1 to 2 hours after a meal is an expected manifestation of a gastric ulcer related to increased gastric secretions and does not cause an urgent change in the nursing plan of care.

A patient is seeking emergency care after choking on a piece of steak. The nursing assessment reveals a history of alcoholism, cigarette smoking, and hemoptysis. Which diagnostic study is most likely to be performed on this patient? a Barium swallow b Endoscopic biopsy c Capsule endoscopy d Endoscopic ultrasonography

correct answer:b Because of this patient's history of excessive alcohol intake, smoking, and hemoptysis and the current choking episode, cancer may be present. A biopsy is necessary to make a definitive diagnosis of carcinoma, so an endoscope will be used to obtain a biopsy and observe other abnormalities as well. A barium swallow may show narrowing of the esophagus, but it is more diagnostic for achalasia. An endoscopic ultrasonography may be used to stage esophageal cancer. Capsule endoscopy can show alterations in the esophagus but is more often used for small intestine problems. A barium swallow, capsule endoscopy, and endoscopic ultrasonography cannot provide a definitive diagnosis for

The patient who is admitted with a diagnosis of diverticulitis and a history of irritable bowel disease and gastroesophageal reflux disease (GERD) has received a dose of Mylanta 30 mL PO. The nurse will determine the medication was effective when which symptom has been resolved? a Diarrhea b Heartburn c Constipation d Lower abdominal pain

correct answer:b Mylanta is an antacid that contains both aluminum and magnesium. It is indicated for the relief of gastrointestinal discomfort, such as heartburn associated with GERD. Mylanta can cause both diarrhea and constipation as a side effect. Mylanta does not affect lower abdominal pain.

What should a patient be taught after a hemorrhoidectomy? A. Do not use the Valsalva maneuver. B. Eat a low-fiber diet to rest the colon. C. Administer oil-retention enema to empty the colon. D. Use prescribed pain medication before a bowel movement.

d Rationale After a hemorrhoidectomy, the patient usually dreads the first bowel movement and often resists the urge to defecate. Pain medication may be given before the bowel movement to reduce discomfort. The patient should avoid constipation and straining. A high-fiber diet can reduce constipation. A stool softener such as docusate (Colace) is usually ordered for the first few postoperative days. If the patient does not have a bowel movement within 2 to 3 days, an oil-retention enema may be given. Reference: 1053

The person with lactase deficiency is at risk for which condition? A. Colorectal cancer B. Pancreatitis C. Osteoarthritis D. Osteoporosis

d Rationale Avoidance of milk and milk products can lead to calcium deficiency, which can lead to osteoporosis. Lactase deficiency is not linked to the other options. Reference: 1051

After a fistulectomy, the patient is prescribed sitz baths. What is the main rationale for a sitz bath? A. To prevent hemorrhoids B. To promote defecation C. To relieve pressure on the area D. To provide comfort

d Rationale Sitz baths are started 1 to 2 days after surgery to provide comfort and keep the anal area clean. A sponge ring may be used to relieve pressure. Reference: 1054, 1053

The nursing management of the patient with cholecystitis associated with cholelithiasis is based on the knowledge that a. shock-wave therapy should be tried initially. b. once gallstones are removed, they tend not to recur. c. the disorder can be successfully treated with oral bile salts that dissolve gallstones. d. laparoscopic cholecystectomy is the treatment of choice in most patients who are symptomatic. (Lewis 1042)

d Rationale: Laparoscopic cholecystectomy is the treatment of choice for symptomatic cholelithiasis.

What are intrarenal causes of acute kidney injury (AKI) (select all that apply)? a. Anaphylaxis b. Renal stones c. Bladder cancer d. Nephrotoxic drugs e. Acute glomerulonephritis f. Tubular obstruction by myoglobin

d, e, f. Intrarenal causes of acute kidney injury (AKI) include conditions that cause direct damage to the kidney tissue, including nephrotoxic drugs, acute glomerulonephritis, and tubular obstruction by myoglobin, or prolonged ischemia. Anaphylaxis and other prerenal problems are frequently the initial cause of AKI. Renal stones and bladder cancer are among the postrenal causes of AKI.

Which characteristics are associated with urge incontinence (select all that apply)? a. Treated with Kegel exercises b. Found following prostatectomy c. Common in postmenopausal women d. Involuntary urination preceded by urgency e. Caused by overactivity of the detrusor muscle f. Bladder contracts by reflex, overriding central inhibition

d, e, f. Urge incontinence is involuntary urination preceded by urgency caused by overactivity of the detrusor muscle when the bladder contracts by reflex, which overrides central inhibition. Treatment includes treating the underlying cause and retraining the bladder with urge suppression, anticholinergic drugs, or containment devices. The other options are characteristic of stress incontinence. Patients may have a combination of urge and stress incontinence.

The RN coordinating the care for a patient who is 2 days postoperative following an anterior- posterior resection with colostomy may delegate which interventions to the licensed practical nurse (LPN) (select all that apply)? a. Irrigate the colostomy. b. Teach ostomy and skin care. c. Assess and document stoma appearance. d. Monitor and record the volume, color, and odor of the drainage. e. Empty the ostomy bag and measure and record the amount of drainage.

d, e. The licensed practical nurse (LPN) can monitor and record observations related to the drainage and can measure and record the amount. The LPN could also monitor the skin around the stoma for breakdown. LPNs can irrigate a colostomy in a stable patient but this patient is only 2 days postoperative. The other actions are responsibilities of the RN (teaching, assessing stoma, and developing a care plan).

A patient with Grave's dz asks the nurse what caused the disorder. The best response by the nurse is a. "The cause of Grave's disease is not known, although it is thought to be genetic." b. "It is usually associated with goiter formation from an iodine deficiency over a long period of time." c. "Antibodies develop against thyroid tissue and destroy it, causing a deficiency of thyroid hormones" d. "In genetically susceptible persons antibodies form that attack thyroid tissue and stimulate overproduction of thyroid hormones."

d. "In genetically susceptible persons antibodies form that attack thyroid tissue and stimulate overproduction of thyroid hormones." (R- The antibodies present in Graves' disease that attack thyroid tissue cause hyperplasia of the gland and stimulate TSH receptors on the thyroid and activate the production of thyroid hormones, creating hyperthyroidism. disease is not directly genetic, but individuals appear to have a genetic susceptibility to become sensitized to develop autoimmune antibodies. Goiter formation from insufficient iodine intake is usually associated with hypothyroidism.)

The nurse is caring for a postoperative patient with a colostomy. The nurse is preparing to administer a dose of famotidine (Pepcid) when the patient asks why the medication was ordered since the patient does not have a history of heartburn or gastroesophageal reflux disease (GERD). What response by the nurse would be the most appropriate? a. "This will prevent air from accumulating in the stomach, causing gas pains." b. "This will prevent the heartburn that occurs as a side effect of general anesthesia." c. "The stress of surgery is likely to cause stomach bleeding if you do not receive it." d. "This will reduce the amount of HCl in the stomach until the nasogastric tube is removed and you can eat a regular diet again."

d. "This will reduce the amount of HCl in the stomach until the nasogastric tube is removed and you can eat a regular diet again." Famotidine is an H2-receptor antagonist that inhibits gastric HCl secretion and thus minimizes damage to gastric mucosa while the patient is not eating a regular diet after surgery. Famotidine does not prevent air from accumulating in the stomach or stop the stomach from bleeding. Heartburn is not a side effect of general anesthesia.

Priority Decision: What is the most appropriate snack for the nurse to offer a patient with stage 4 CKD? a. Raisins b. Ice cream c. Dill pickles d. Hard candy

d. A patient with CKD may have unlimited intake of sugars and starches (unless the patient is diabetic) and hard candy is an appropriate snack and may help to relieve the metallic and urine taste that is common in the mouth. Raisins are a high-potassium food. Ice cream contains protein and phosphate and counts as fluid. Pickled foods have high sodium content.

Which test is required for a diagnosis of pyelonephritis? a. Renal biopsy c. Intravenous pyelogram (IVP) b. Blood culture d. Urine for culture and sensitivity

d. A urine specimen specifically obtained for culture and sensitivity is required to diagnose pyelonephritis because it will show pyuria, the specific bacteriuria, and what drug the bacteria is sensitive to for treatment. The renal biopsy is used to diagnose chronic pyelonephritis or cancer. Blood cultures would be done if bacteremia is suspected. Intravenous pyelogram (IVP) would increase renal irritation, but CT urograms may be used to assess for signs of infection in the kidney and complications of pyelonephritis.

Acute tubular necrosis (ATN) is the most common cause of intrarenal AKI. Which patient is most likely to develop ATN? a. Patient with diabetes mellitus b. Patient with hypertensive crisis c. Patient who tried to overdose on acetaminophen d. Patient with major surgery who required a blood transfusion

d. Acute tubular necrosis (ATN) is primarily the result of ischemia, nephrotoxins, or sepsis. Major surgery is most likely to cause severe kidney ischemia in the patient requiring a blood transfusion. A blood transfusion hemolytic reaction produces nephrotoxic injury if it occurs. Diabetes mellitus, hypertension, and acetaminophen overdose will not contribute to ATN.

What manifestation in the patient will indicate the need for restriction of dietary protein in management of acute poststreptococcal glomerulonephritis (APSGN)? a. Hematuria b. Proteinuria c. Hypertension d. Elevated blood urea nitrogen (BUN)

d. An elevated blood urea nitrogen (BUN) indicates that the kidneys are not clearing nitrogenous wastes from the blood and protein may be restricted until the kidney recovers. Proteinuria indicates loss of protein from the blood and possibly a need for increased protein intake. Hypertension is treated with sodium and fluid restriction, diuretics, and antihypertensive drugs. The hematuria is not specifically treated.

The hypothalamus secretes releasing hormones and inhibiting hormones. What is the target tissue of these releasing hormones and inhibiting hormones? a. Pineal b. Adrenal cortex c. Posterior pituitary d. Anterior pituitary

d. Anterior pituitary The anterior pituitary is the target tissue of the releasing hormones (corticotropin releasing hormone, thyrotropin releasing hormone, growth hormone releasing factor, gonadotropin releasing hormone, prolactin releasing factor) and the inhibiting hormones (somatostatin, prolactin inhibiting factor). These hormones release or inhibit other hormones that affect the thyroid, adrenal cortex, pancreas, reproductive organs, and all body cells. The pineal gland is not directly affected by the releasing and inhibiting hormones from the hypothalamus. The posterior pituitary releases antidiuretic hormone (ADH) in response to plasma osmolality changes that is not directly affected by the hypothalamus hormones.

Which disease causes connective tissue changes that cause glomerulonephritis? a. Gout c. Diabetes mellitus b. Amyloidosis d. Systemic lupus erythematosus

d. Systemic lupus erythematosus causes connective tissue damage that affects the glomerulus. Gout deposits uric acid crystals in the kidney. Amyloidosis deposits hyaline bodies in the kidney. Diabetes mellitus causes microvascular damage affecting the kidney.

What is a cause of primary hypothyroidism in adults? a. Malignant or benign thyroid nodules b. Surgical removal or failure of the pituitary gland c. Surgical removal or radiation of the thyroid gland d. Autoimmune-induced atrophy of the thyroid gland

d. Both Graves' disease and Hashimoto's thyroiditis are autoimmune disorders that eventually destroy the thyroid gland, leading to primary hypothyroidism. Thyroid tumors most often result in hyperthyroidism. Secondary hypothyroidism occurs as a result of pituitary failure and iatrogenic hypothyroidism results from thyroidectomy or radiation of the thyroid gland.

When caring for the patient with interstitial cystitis, what can the nurse teach the patient to do? a. Avoid foods that make the urine more alkaline. b. Use high-potency vitamin therapy to decrease the autoimmune effects of the disorder. c. Always keep a voiding diary to document pain, voiding frequency, and patterns of nocturia. d. Use the dietary supplement calcium glycerophosphate (Prelief) to decrease bladder irritation.

d. Calcium glycerophosphate (Prelief) alkalinizes the urine and can help to relieve the irritation from acidic foods. A diet low in acidic foods is recommended and if a multivitamin is used, high-potency vitamins should be avoided because these products may irritate the bladder. A voiding diary is useful in diagnosis but does not need to be kept indefinitely.

A patient rapidly progressing toward end-stage kidney disease asks about the possibility of a kidney transplant. In responding to the patient, the nurse knows that what is a contraindication to kidney transplantation? a. Hepatitis C infection b. Coronary artery disease c. Refractory hypertension d. Extensive vascular disease

d. Extensive vascular disease is a contraindication for renal transplantation, primarily because adequate blood supply is essential for the health of the new kidney. Other contraindications include disseminated malignancies, refractory or untreated cardiac disease, chronic respiratory failure, chronic infection, or unresolved psychosocial disorders. Coronary artery disease (CAD) may be treated with bypass surgery before transplantation and transplantation can relieve hypertension. Hepatitis B or C infection is not a contraindication.

The nurse is caring for a 68-year-old patient admitted with abdominal pain, nausea, and vomiting. The patient has an abdominal mass and a bowel obstruction is suspected. The nurse auscultating the abdomen listens for which of the following types of bowel sounds that is consistent with the patient's clinical picture? a. Low pitched and rumbling above the area of obstruction b. High pitched and hypoactive below the area of obstruction c. Low pitched and hyperactive below the area of obstruction d. High pitched and hyperactive above the area of obstruction

d. High pitched and hyperactive above the area of obstruction Early in intestinal obstruction, the patient's bowel sounds are hyperactive and high-pitched, sometimes referred to as "tinkling" above the level of the obstruction. This occurs because peristaltic action increases to "push past" the area of obstruction. As the obstruction becomes complete, bowel sounds decrease and finally become absent.

While caring for the patient in the oliguric phase of AKI, the nurse monitors the patient for associated collaborative problems. When should the nurse notify the health care provider? a. Urine output is 300 mL/day. b. Edema occurs in the feet, legs, and sacral area. c. Cardiac monitor reveals a depressed T wave and elevated ST segment. d. The patient experiences increasing muscle weakness and abdominal cramping.

d. Hyperkalemia is a potentially life-threatening complication of AKI in the oliguric phase. Muscle weakness and abdominal cramping are signs of the neuromuscular impairment that occurs with hyperkalemia. In addition, hyperkalemia can cause the cardiac conduction abnormalities of peaked T wave, prolonged PR interval, prolonged QRS interval, and depressed ST segment. Urine output of 300 mL/day is expected during the oliguric phase, as is the development of peripheral edema.

A nurse is doing a nursing assessment on a patient with chronic constipation. What data obtained during the interview may be a factor contributing to the constipation? a. Taking methylcellulose (Citrucel) daily c. History of hemorrhoids and hypertension b. High dietary fiber with high fluid intake d. Suppressing the urge to defecate while at work

d. Ignoring the urge to defecate causes the muscles and mucosa in the rectal area to become insensitive to the presence of feces and drying of the stool occurs. The urge to defecate is decreased and stool becomes more difficult to expel. Taking a bulk-forming agent with fluids or highfiber diet with fluids prevent constipation. Hemorrhoids are the most common complication of chronic constipation, caused by straining to pass hardened stool. The straining may cause problems in patients with hypertension but these do not cause constipation. Other things that may cause constipation are a history of diverticulosis, which is seen in individuals with low fiber intake, small stool mass, and hard stools. Chronic laxative use and chronic dilation and loss of colonic tone may also cause chronic constipation.

What characteristic is related to Hashimoto's thyroiditis? a. Enlarged thyroid gland b. Viral-induced hyperthyroidism c. Bacterial or fungal infection of thyroid gland d. Chronic autoimmune thyroiditis with antibody destruction of thyroid tissue

d. In Hashimoto's thyroiditis, thyroid tissue is destroyed by autoimmune antibodies. An enlarged thyroid gland is a goiter. Viral-induced hyperthyroidism is subacute granulomatous thyroiditis. Acute thyroiditis is caused by bacterial or fungal infection.

What indicates to the nurse that a patient with oliguria has prerenal oliguria? a. Urine testing reveals a low specific gravity. b. Causative factor is malignant hypertension. c. Urine testing reveals a high sodium concentration. d. Reversal of oliguria occurs with fluid replacement.

d. In prerenal oliguria, the oliguria is caused by a decrease in circulating blood volume and there is no damage yet to the renal tissue. It can be reversed by correcting the precipitating factor, such as fluid replacement for hypovolemia. Prerenal oliguria is characterized by urine with a high specific gravity and a low sodium concentration, whereas oliguria of intrarenal failure is characterized by urine with a low specific gravity and a high sodium concentration. Malignant hypertension causes damage to renal tissue and intrarenal oliguria.

What accurately describes the care of the patient with CKD? a. A nutrient that is commonly supplemented for the patient on dialysis because it is dialyzable is iron. b. The syndrome that includes all of the signs and symptoms seen in the various body systems in CKD is azotemia. c. The use of morphine is contraindicated in the patient with CKD because accumulation of its metabolites may cause seizures. d. The use of calcium-based phosphate binders in the patient with CKD is contraindicated when serum calcium levels are increased.

d. In the patient with CKD, when serum calcium levels are increased, calcium-based phosphate binders are not used. The nutrient supplemented for patients on dialysis is folic acid. The various body system manifestations occur with uremia, which includes azotemia. Meperidine is contraindicated in patients with CKD related to possible seizures.

The nurse is preparing to administer a dose of bisacodyl (Dulcolax). In explaining the medication to the patient, the nurse would explain that it acts in what way? a. Increases bulk in the stool b. Lubricates the intestinal tract to soften feces c. Increases fluid retention in the intestinal tract d. Increases peristalsis by stimulating nerves in the colon wall

d. Increases peristalsis by stimulating nerves in the colon wall Bisacodyl is a stimulant laxative that aids in producing a bowel movement by irritating the colon wall and stimulating enteric nerves. It is available in oral and suppository forms. Fiber and bulk forming drugs increase bulk in the stool; water and stool softeners soften feces, and saline and osmotic solutions cause fluid retention in the intestinal tract.

In caring for the patient with AKI, what should the nurse be aware of? a. The most common cause of death in AKI is irreversible metabolic acidosis. b. During the oliguric phase of AKI, daily fluid intake is limited to 1000 mL plus the prior day's measured fluid loss. c. Dietary sodium and potassium during the oliguric phase of AKI are managed according to the patient's urinary output. d. One of the most important nursing measures in managing fluid balance in the patient with AKI is taking accurate daily weights.

d. Measuring daily weights with the same scale at the same time each day allows for the evaluation and detection of excessive body fluid gains or losses. Infection is the leading cause of death in AKI, so meticulous aseptic technique is critical. The fluid limitation in the oliguric phase is 600 mL plus the prior day's measured fluid loss. Dietary sodium and potassium intake are managed according to the plasma levels.

Prevention of calcium oxalate stones would include dietary restriction of which foods or drinks? a. Milk and milk products b. Dried beans and dried fruits c. Liver, kidney, and sweetbreads d. Spinach, cabbage, and tomatoes

d. Oxalate-rich foods should be limited to reduce oxalate excretion. Foods high in oxalate include spinach, rhubarb, asparagus, cabbage, and tomatoes, in addition to chocolate, coffee, and cocoa. Currently, it is believed that high dietary calcium intake may actually lower the risk for renal stones by reducing the intestinal oxalate absorption and therefore the urinary excretion of oxalate. Milk, milk products, dried beans, and dried fruits are high sources of calcium. Organ meats are high in purine, which contributes to uric acid lithiasis.

A diabetic patient is found unconscious at home, and a family member calls the clinic. After determining that no glucometer is available, the nurse advise the family member to a. try to around the patient to drink some orange juice. b administer 10 U or regular insulin subcutaneously. c. call for an ambulance to transport the patient to a medical facility. d. administer glucagon 1mg intramuscularly (IM) or subcutaneously.

d. Rationale: If a diabetic patient is unconscious, immediate treatment for hypoglycemia must be given to prevent brain damage, and IM or subcutaneous administration of 1mg of glucagon should be done. If that unconsciousness has another cause, such as ketosis, the rise in glucose cause by the glucagon is not as dangerous as the low glucose level. Following administration of the glucagon, the patient should be transported to a medical facility for further treatment and evaluation. Insulin is contraindicated without knowledge of the patient's glucose level, and oral carbohydrate cannot be given when patients are unconscious.

The home care nurse should intervene to correct a patient whose insulin administration includes a. warming a pre filled refrigerated syringe in the hands before administration. b. storing syringes pre filled with NPH and regular insulin needle-up in the refrigerator. c. placing the insulin bottle currently in use in a small container on the bathroom countertop. d. mixing an evening dose of regular insulin with insulin glargine in one syringe for administration.

d. Rationale: Insulin glargine (Lantus), a long-acting insulin that is continuously released with no peak of action, cannot be diluted or mixed with any other insulin or solution. Mixed insulins should be stored needle-up in the refrigerator and warmed before administration. Currently used bottles of insulin can be kept at room temperature.

In addition to promoting the transport of glucose from the blood into the cell, insulin also a. enhances the breakdown of adipose tissue for energy. b. stimulates hepatic glycogenolysis and gluconeogenesis. c. prevents the transport of triglycerides into adipose tissue. d. accelerates the transport of amino acids into cells and their synthesis into protein.

d. Rationale: Insulin is an anabolic hormone, responsible for growth, repair, and storage, and it facilitates movement of amino acids into cells, synthesis of protein, storage of glucose as glycogen, and deposition of triglycerides and lipids as fat into adipose tissue. Glucagon is responsible for hepatic glycogenolysis and gluconeogenesis, and fat is used for energy when glucose levels are depleted.

Lispro insulin (Humalog) with NPH insulin is ordered for a patient with newly diagnosed type 1 diabetes. The nurse knows that when lispro insulin is used, it should be administered a. only once a day b. 1 hour before meals c. 30-45 minutes before meals d. at mealtime or within 15 minutes of meals

d. Rationale: Lispro is a rapid-acting insulin that has an onset of action of 5 to 15 minutes and should be injected at the time of the meal to within 15 minutes of eating. Regular insulin is short acting with an onset of action in 30 to 60 minutes following administration and should be given 30 to 45 minutes before meals.

Which of the following patients would a nurse plan to teach how to prevent or delay the development of diabetes? a. A 62 year-old obese white woman. b. An obese 50 year-old Hispanic woman. c. A child whose father has type 1 diabetes. d. A 34 year-old woman whose parents both have type 2 diabetes.

d. Rationale: Type 2 diabetes has a strong genetic influence, and offspring of parents who both have type 2 diabetes have an increased chance of developing it. Whereas type 1 diabetes is associated with genetic susceptibility related to human leukocyte antigens (HLAs), offspring of parents who both have type 1 diabetes have only a 6% to 10% chance of developing the disease. Lower risk factors for type 2 diabetes include obesity; being a Native American, Hispanic, or African-American; and being 55 years or older.

A male patient who has undergone an anterior-posterior repair is worried about his sexuality. What is an appropriate nursing intervention for this patient? a. Have the patient's sexual partner reassure the patient that he is still desirable. b. Reassure the patient that sexual function will return when healing is complete. c. Remind the patient that affection can be expressed in ways other than through sexual intercourse. d. Explain that physical and emotional factors can affect sexual function but not necessarily the patient's sexuality.

d. Sexual dysfunction may result from an anterior-posterior repair but the nurse should discuss with the patient that different nerve pathways affect erection, ejaculation, and orgasm and that a dysfunction of one does not mean total sexual dysfunction and also that an alteration in sexual activity does not have to alter sexuality. Simple reassurance of desirability and ignoring concerns about sexual function do not help the patient to regain positive feelings of sexuality.

What indicates to the nurse that a patient with AKI is in the recovery phase? a. A return to normal weight b. A urine output of 3700 mL/day c. Decreasing sodium and potassium levels d. Decreasing blood urea nitrogen (BUN) and creatinine levels

d. The blood urea nitrogen (BUN) and creatinine levels remain high during the oliguric and diuretic phases of AKI. The recovery phase begins when the glomerular filtration returns to a rate at which BUN and creatinine stabilize and then decrease. Urinary output of 3 to 5 L/ day, decreasing sodium and potassium levels, and fluid weight loss are characteristic of the diuretic phase of AKI.

The patient with a new ileostomy needs discharge teaching. What should the nurse plan to include in this teaching? a. The pouch can be worn for up to 2 weeks before changing it. b. Decrease the amount of fluid intake to decrease the amount of drainage. c. The pouch can be removed when bowel movements have been regulated. d. If leakage occurs, promptly remove the pouch, clean the skin, and apply a new pouch

d. The ileostomy drainage is extremely irritating to the skin, so the skin must be cleaned and a new solid skin barrier and pouch applied as soon as a leak occurs to prevent skin damage. The pouch is usually worn for 4 to 7 days unless there is a leak. Because the initial drainage from the ileostomy is high, the fluid intake must not be decreased. The pouch must always be worn, as the liquid drainage, not formed bowel movements, is frequent.

A 22-year-old patient calls the outpatient clinic complaining of nausea and vomiting and right lower abdominal pain. What should the nurse advise the patient to do? a. Use a heating pad to relax the muscles at the site of the pain. b. Drink at least 2 quarts of juice to replace the fluid lost in vomiting. c. Take a laxative to empty the bowel before examination at the clinic. d. Have the symptoms evaluated by a health care provider right away.

d. The patient is having symptoms of an acute abdomen and should be evaluated by a health care provider immediately. The patient's age, location of pain, and other symptoms are characteristic of appendicitis. Heat application and laxatives should not be used in patients with undiagnosed abdominal pain because they may cause perforation of the appendix or other inflammations. Fluids should not be taken until vomiting is controlled, nor should they be taken in the event that surgery may be performed.

In report, the nurse learns that the patient has a transverse colostomy. What should the nurse expect when providing care for this patient? a. Semiliquid stools with increased fluid requirements b. Liquid stools in a pouch and increased fluid requirements c. Formed stools with a pouch, needing irrigation, but no fluid needs d. Semiformed stools in a pouch with the need to monitor fluid balance

d. The patient with a transverse colostomy has semiliquid to semiformed stools needing a pouch and needs to have fluid balance monitored. The ascending colostomy has semiliquid stools needing a pouch and increased fluid. The ileostomy has liquid to semiliquid stools needing a pouch and increased fluid. The sigmoid colostomy has formed stools and may or may not need a pouch but will need irrigation.

A patient with suprapubic pain and symptoms of urinary frequency and urgency has two negative urine cultures. What is one assessment finding that would indicate interstitial cystitis? a. Residual urine greater than 200 mL b. A large, atonic bladder on urodynamic testing c. A voiding pattern that indicates psychogenic urinary retention d. Pain with bladder filling that is transiently relieved by urination

d. The symptoms of interstitial cystitis (IC) imitate those of an infection of the bladder but the urine is free of infectious agents. Unlike a bladder infection, the pain with IC increases as urine collects in the bladder and is temporarily relieved by urination. Acidic urine is very irritating to the bladder in IC and the bladder is small but urinary retention is not common.

What extraintestinal manifestations are seen in both ulcerative colitis and Crohn's disease? a. Celiac disease and gallstones c. Conjunctivitis and colonic dilation b. Peptic ulcer disease and uveitis d. Erythema nodosum and osteoporosis

d. Ulcerative colitis and Crohn's disease have many of the same extraintestinal symptoms, including erythema nodosum and osteoporosis, as well as gallstones, uveitis, and conjunctivitis. Colonic dilation and celiac disease are not extraintestinal.

A patient with bladder cancer undergoes cystectomy with formation of an ileal conduit. During the patient's first postoperative day, what should the nurse plan to do? a. Measure and fit the stoma for a permanent appliance. b. Encourage high oral intake to flush mucus from the conduit. c. Teach the patient to self-catheterize the stoma every 4 to 6 hours. d. Empty the drainage bag every 2 to 3 hours and measure the urinary output.

d. Urine drains continuously from an ileal conduit and the drainage bag must be emptied every 2 to 3 hours and measured to ensure adequate urinary output. Fitting for a permanent appliance is not done until the stoma shrinks to its normal size in a few weeks. With an ileal conduit, mucus is present in the urine because it is secreted by the ileal segment as a result of the irritating effect of the urine but the surgery causes paralytic ileus and the patient will be NPO for several days postoperatively. Self-catheterization is performed when patients have formation of a continent Kock pouch.

Following a hemorrhoidectomy, what should the nurse advise the patient to do? a. Use daily laxatives to facilitate bowel emptying. b. Use ice packs to the perineum to prevent swelling. c. Avoid having a bowel movement for several days until healing occurs. d. Take warm sitz baths several times a day to promote comfort and cleaning.

d. Warm sitz baths provide comfort, healing, and cleansing of the area following all anorectal surgery and may be done three or four times a day for 1 to 2 weeks. Stool softeners may be prescribed for several days postoperatively to help keep stools soft for passage but laxatives may cause irritation and trauma to the anorectal area and are not used postoperatively. Early passage of a bowel movement, although painful, is encouraged to prevent drying and hardening of stool, which would result in an even more painful bowel movement.

A patient's recent medical history is indicative of diabetes insipidus. The nurse would perform patient teaching related to which diagnostic test? a. Thyroid scan b. Fasting glucose test c. Oral glucose tolerance d. Water deprivation test

d. Water deprivation test A water deprivation test is used to diagnose the polyuria that accompanies diabetes insipidus. Glucose tests and thyroid tests are not directly related to the diagnosis of diabetes insipidus.

When providing discharge instructions to a patient who had a subtotal thyroidectomy for hyperthyroidism, what should the nurse teach the patient? a. Never miss a daily dose of thyroid replacement therapy. b. Avoid regular exercise until thyroid function is normalized. c. Use warm saltwater gargles several times a day to relieve throat pain. d. Substantially reduce caloric intake compared to what was eaten before surgery.

d. With the decrease in thyroid hormone postoperatively, calories need to be reduced substantially to prevent weight gain. When a patient has had a subtotal thyroidectomy, thyroid replacement therapy is not given because exogenous hormone inhibits pituitary production of TSH and delays or prevents the restoration of thyroid tissue regeneration. Regular exercise stimulates the thyroid gland and is encouraged. Saltwater gargles are used for dryness and irritation of the mouth and throat following radioactive iodine therapy.

A patient suspected of having acromegaly has an elevated plasma growth hormone level. In acromegaly, the nurse would also expect the pt's diagnostic results to include a. hyperinsulinemia b. a plasma glucose of less than 70 c. decreased growth hormone levels with an oral glucose challenge test d. a serum sometomedin C (insulin-like growth-factor) of more than 300

d. a serum somatomedin C (Insulin-like-growth-factor) of more than 300 (R- a normal response to growth hormone secretion is stimulation of the liver to produce somatomedin C which stimulates growth of bones and soft tissue. The increased levels of somatomedin C normally inhibit growth hormone, but in acromegaly the pituitary gland secretes GH despite elevated somatomedin C levels.)

In teaching a patient with pyelonephritis about the disorder, the nurse informs the paitent that the organisms that cause pyelonephritis most commonly reach the kidneys through* a. the bloodstream b.the lymphatic system c. a descending infection d. an ascending infection

d. an ascending infection

A nursing intervention that is most appropriate to decrease post-operative edema and pain after an inguinal herniorrhaphy is: a. applying a truss to the hernia site b. allowing the patient to stand to void c. supporting the incision during coughing d. applying a scrotal support with ice bag

d. applying a scrotal support with ice bag Rationale: Scrotal edema is a painful complication after an inguinal hernia repair. Scrotal support with application of an ice bag may help relieve pain and edema.

Causes of primary hypothyroidism in adults include a. malignant or benign thyroid nodules b. surgical removal or failure of the pituitary gland c. surgical removal or radiation of thyroid gland d. autoimmune-induced atrophy of the gland

d. autoimmune-induced atrophy of the gland (R- both Graves disease and Hasimotos thyroiditis are autoimmune disorders that eventually destroy thyroid gland, leading to primary hypothyroidism. Thyroid tumors most often result in hyperthyroidism. Secondary hypothyroidism occurs as a result of pituitary failure, and iatrogenic hypothyroidism results from thyroidectomy or radiation of the thyroid gland.)

what is a cause of primary hypothyroidism in adults? a. malignant or benign thyroid modules b. surgical removal or failure of the pituitary gland c. surgical removal or radiation of the thyroid gland d. autoimmune-induced atrophy of the thyroid gland

d. autoimmune-induced atrophy of the thyroid gland

A patient has has a cystectomy and ileal conduit diversion performed. Four days postoperatively, mucous shred are seen in the drainage bag. The nurse should.* a. notify the physician b.notify the charge nurse c. irrigate the drainage tube d. chart it as a normal observation

d. chart it as a normal observation

Physical changes of hypothyroidism that must be monitored when replacement therapy is started include a. achlorhydria and constipation b. slowed mental processes and lethargy c. anemia and increased capillary fragility d. decreased cardiac contractility and coronary atherosclerosis

d. decreased cardiac contractility and coronary atherosclerosis (R- hypothyroidism affects the heart in many ways, causing cardiomyopathy, coronary atherosclerosis, bradycardia, pericardial effusions, and weakened cardiac contractility. when thyroid replacement therapy is started, myocardial oxygen consumption is increased and resultant oxygen demand may cause angina, cardiac arrhythmias, and HFs. It is important to monitor pts with compromised cardiac status when starting replacement therapy.)

what is hte most common screening intervention for detecting BPH in men over age 50 a. PSA level b. urinalysis c. cystoscopy d. digital rectal examination

d. digital rectal examination

which lab results would indicate the patient has prediabetes? a. glucose tolerance result of 132mg/dL b. glucose tolerance result of 240 mg/dL c. fasting blood glucose result of 80 mg/dL d. fasting blood glucose result of 120 mg/dL

d. fasting blood glucose result of 120 mg/dL

Which statement made by the client makes the nurse suspect the client is experiencing hyperthyroidism a. i just don't seem to have any appetite anymore b. i have a BM about every 3-4 hours c. my skin is really becoming dry and coarse d. i have noticed all my collars are getting tigher

d. i have noticed all my collars are getting tigher

In a patient with central diabetes insipidus, administration of aqueous vasopressin during a water deprivation test will result in a a. decrease in body weight b. increase in urinary output c. decrease in blood pressure d. increase in urine osmolality

d. increase in urine osmolality (R- pt with DI has a deficiency of ADH with excessive loss of water from the kidney, hypovolemia, hypernatreamia, and dilute urine with a low specific gravity. When vasopressin is administered, the symptoms are reversed, with water retention, decreased urinary output that increases urine osmolality, and an increase in BP.)

The client diagnosed with HHNS was admitted yesterday with a blood glucose level of 780 mg/dL. The clients blood glucose level is now 300 mg/dL. Which intervention should the nurse implement? a. increase the regular insulin IV drip b. check the patients urine for ketones c. provide the client with a therapeutic diabetic meal d. notify the HCP to obtain an order to decrease insulin

d. notify the HCP to obtain an order to decrease insulin

which arterial blood gas results should.d the nurse expect in the client diagnosed with DKA a. pH 7.34, PaO2 99, PaCO2 48, HCO3 24 b. pH 7.38, PaO2 95, PaCO2 40, HCO3 22 c. pH 7.46, PaO2 85, PaCO2 30, HCO3 18 d. pH 7.30, PaO2 90, PaCO2 30, HCO3 18

d. pH 7.30, PaO2 90, PaCO2 30, HCO3 18

the nurse is discussing the importance of exercising with a client diagnosed with type 2 diabetes who diabetes is well controlled with diet and exercise. Which information should the nurse include in the teaching about diabetes? a. eat a simple carb snack before exercising b. carry peanut butter crackers when exercising c. encourage the client to walk 20 minutes three times a week d. perform warm up and cool down exercises

d. perform warm up and cool down exercises

Which nursing intervention should be included in the plan of care for the client diagnosed with hyperthyrodism a. increase the amount of fiber in the diet b. encourage low cal, low protein diet c. decrease the clients fluid intake to 1000 mL/day d. provide 6 small, well balanced meals per day

d. provide 6 small, well balanced meals per day

Which statement would be correct for a patient with type 2 diabetes who was admitted to the hospital with pneumonia? a.The patient must receive insulin therapy to prevent ketoacidosis. b.The patient has islet cell antibodies that have destroyed the pancreas's ability to produce insulin. c.The patient has minimal or absent endogenous insulin secretion and requires daily insulin injections. d.The patient may have sufficient endogenous insulin to prevent ketosis but is at risk for hyperosmolar hyperglycemic syndrome.

d.The patient may have sufficient endogenous insulin to prevent ketosis but is at risk for hyperosmolar hyperglycemic syndrome.

The edema that occurs in nephrotic syndrome is due to* a. increased hydrostatic pressure caused by sodium retention. b. decreased aldosterone secretion from adrenal insufficiency. c. increased fluid retention caused by decreased glomerular filtration d.decreased colloidal osmotic pressure caused by loss of serum albumin

d.decreased colloidal osmotic pressure caused by loss of serum albumin

Delegation Decision: Which nursing interventions could be delegated to unlicensed assistive personnel (UAP) (select all that apply)? a. Assess the need for catheterization. b. Use bladder scanner to estimate residual urine. c. Teach patient pelvic floor muscle (Kegel) exercises. d. Insert indwelling catheter for uncomplicated patient. e. Assist incontinent patient to commode at regular intervals. f. Provide perineal care with soap and water around a urinary catheter.

e, f. The unlicensed assistive personnel (UAP) may assist the incontinent patient to void at regular intervals and provide perineal care. An RN should perform the assessments and teaching. In long-term care and rehabilitation facilities, UAP may use bladder scanners after they are trained.

medication used to treat polynephritis

oral antibiotics MOA: inhibit bacterial growth and destroys microorganisms SE: same as antispasmodics. sulfa drugs may leave a metal aftertaste in the mouth. MRSG CONSIDER: pts should complete the entire course of drug therapy and report any further symptoms, continuing or worsening. drugs can be used for up to 14 days. pts don not require hospitalization unless nausea, vomiting, or signs of septicemia develop.


Kaugnay na mga set ng pag-aaral

The US Constitution: Articles and Amendments

View Set

AEM 264 - Dynamics Final Exam study Questions

View Set

2181 Chapter 10, 2181 Chapter 9, 2181 Chapter 8, 2181 Chapter 7, 2181 Chapter 6

View Set

PrepU's Ch 12: Oncologic Management

View Set

Теорія держави та права

View Set

The History of Earth's Climate GEOS 342 Third Exam Study Guide

View Set

Biological Psychology Chapter 5 (Kalat)

View Set

Essentials of Business Communications 2, 3, 4

View Set